Você está na página 1de 208

Professor Jefferson Celestino da Costa

1.500 2012/2013 Index


Top Questio Pages
ics ns
Adjectives and Adverbs 120 002

Pronouns 150 015

Quantifiers and Intensifiers 035 033

Verbs 180 037

Modal Auxiliaries 075 058

Active and Passive Voice 100 067

Direct and Indirect Speech 035 079

Conditionals 044 084

Question Tags 020 090

Rejoinders 006 092

Articles 024 093

Plural of the Nouns 020 096

Genitive Case 018 099

Numbers 008 101

Prepositions 090 102

Conjunctions 125 113

Subjunctive, Imperative, Infinitive and Gerund 020 129

Phrasal Verbs 018 131

False Cognate Words 018 133

Mixed Topics 025 135

Idioms and Vocabulary 030 139

Synonyms and Antonyms 035 143

Reading Skills and General Review 124 148

Translations 180 177

Answers 1.500 190

1
Professor Jefferson Celestino da Costa

1ST Part | Grammar Skills 5 | FATEC 2007


Assinale a alternativa que apresenta a forma correta do
adjetivo + sufixo 'IBLE' como em 'Edible Cotton':
Adjectives and Adverbs
a) Managible
1 | UNITAU 1995
b) Gullible
Assinale a alternativa que corresponde à tradução mais
c) Drinkible
adequada da seguinte sentença:
d) Lovible
e) Thinkible
The man gave a five-pound note to the shoe-repairer.
6 | UNITAU 1995
a) O homem deu uma nota de cinco libras para reparar o
Assinale a alternativa que corresponde à tradução mais
sapato.
adequada da forma adjetiva destacada a seguir:
b) O homem deu cinco potes de notas para reparar o sapato.
c) O homem deu um maço de cinco notas para consertar o
Literally thousands of parts are used to make up even THE
sapato.
SMALLEST family saloon.
d) O homem deu uma nota de cinco libras para o sapateiro.
e) O sapateiro recebeu uma nota de cinco libras do bom
a) o menor
homem.
b) o maior
c) o pequeno
2 | MACKENZIE 1999
d) o grande
Indicate the alternative that best completes the following
e) o amplo
sentence:
7 | UNITAU 1995
They finally decided to buy a . Assinale a alternativa que corresponde à tradução mais
adequada da expressão, em destaque, a seguir:
a) four-doors car
b) four doors car
Você é a pessoa MAIS INTERESSANTE que já conheci.
c) four-door car
d) four-door-car
a) the more interesting
e) four-car
b) the interestinger
c) the most interesting
3 | MACKENZIE 1999
d) the much interesting
Indicate the alternative that best completes the following
e) the best interesting
sentence:
8 | CESGRANRIO 1994
The building has nine stories. It is .
"The new generation of MT programs is less ambitious" is an
example of comparative form. Mark the item which also
a) a nine-stories-building contains a comparative form:
b) a nine-story building
c) a nine's-story building a) Of the four translations, I like this one best.
d) a nine-stories b) That young boy behaves the most carelessly of all.
e) a building's nine stories's c) This is the worst ice cream I've had in a long time.
d) This is the least expensive computer that we have.
4 | MACKENZIE 1999 e) The airport is farther than the university.
Indicate the alternative that best completes the following
sentence: 009 | FEI 1995
Indique a palavra que significa "mais forte":
The flight lasted two hours. It was .
a) larger
a) a flight's two-hours's b) clearer
b) a two-hours-flight c) higher
c) a two'-hours's flight d) better
d) a two-hours e) stronger
e) a two-hour flight
10 | FUVEST 1978
14 | PUCCAMP 1994
Assinale a alternativa que completa corretamente a
Assinale a letra correspondente à alternativa que preenche
sentença: corretamente as lacunas da frase apresentada a seguir:
Of all the movies I have seen lately, the one I saw yesterday - "Did Jerry come to work yesterday?"
was . - "Yes, he did. He arrived than his colleagues,
but worked the so that he got as much done as
a) worse. the others."
b) worst. - "Good. He's a very responsible fellow."
c) the worse.
d) the worst. a) late – harder
e) the most worse. b) later – hardest
c) earlier – hard
11 | UDESC 1996 d) early – hardest
Choose the correct grammatical answer: e) sooner – harder

I have been studying I can to learn English. 015 | EN 1983


Fill in the gap:
a) as hard as
b) so hard as - “You look fatter”.
c) as harder as - “That’s because I’ve been eating than I used
d) so harder so to”.
e) so hard so
a) many more
12 | MACKENZIE 1996 b) much more
Indicate the alternative that best completes the following c) a few
sentences: d) more than
e) any more
I. Which city is the from São Paulo?
II. My brother works at Mackenzie. 16 | UNESP 1985
III. Do you need any data on that matter? Assinale a alternativa correta:
IV. Which is the building in São Paulo?
V. Ribeirão Preto is the city that has the Our next examination may be the last one.
problems with pollution in Brazil.
a) more bad than
a) I. furthest; II. elder; III. more; IV. farthest; V. oldest b) more worse than
b) I. further; II. older; III. farther; IV. eldest; V. less c) much bad than
c) I. nearer; II. oldest; III. farthest; IV. longest; V. least d) worse than
d) I. nearest; II. elder; III. furthest; IV. eldest; V. biggest e) more badly than
e) I. farthest; II. eldest; III. further; IV. oldest; V. fewest
17 | UNESP 1987
13 | PUCCAMP 1992 Peter's house is mine.
Assinale a letra correspondente à alternativa que preenche
corretamente as lacunas da frase apresentada: a) larger as
b) most larger than
Mr. Smith: I'm sorry, Mr. Johnson. I believe the candidate c) larger than
you sent us will not suit our purposes. We need somebody d) so large than
than he. e) more large than
Mr. Johnson: In that case I would suggest Miss Cary. She's
definitely the person in our group. 18 | UNESP 1988
He is boy in town.
a) smarter – most intelligent
b) smart – intelligent a) so rich
c) smartest – more intelligent b) richer
d) as smart – as intelligent c) the richest
e) as smart – as intelligent as d) richest
e) the most rich
019 | UFRS 1997
024 | UEL 1994
The word that does not form the comparative in the same
Assinale a alternativa que preenche corretamente a lacuna
way as large or short is: da frase a seguir:
a) bad
The more time people spend at an exhibit, they
b) sad learn.
c) glad
d) great
a) more
e) late b) most
c) much
20 | EFOMM 2007 d) the more
‘In London there are lots of streets with the same name and
e) the most
it’s very if you are a tourist. Another problem is
that it’s a huge place. We walked everywhere on our last trip
025 | UEL 1997
and we were at the end of each day. But it’s an
Assinale a alternativa que preenche corretamente a lacuna
city, with so much to do.’ da frase a seguir:
a) confused – exhaust – excited I firmly believe that the tougher the laws, the
b) confusing – exhausted – exciting criminal rate.
c) confuse – exhausting – exciting
d) confusing – exhaust – excited
a) the lowest
e) confused – exhausted – excited
b) lowest
c) the lower
21 | EFOMM 2008
d) lower
The lecture we’ve attended was not good. It was quite
e) low
, and the audience was .
026 | UNESP 1989
a) amusing – amused The sooner a man begins to work .
b) bored – boring
c) amused – amusing a) the bad
d) boring – bored
b) the best
e) interesting – interested
c) the worst
d) the better
22 | EFOMM 2008 e) the good
Caren has a strange look. She seems to live on
frozen meals.
027 | PUC 1975
Fill in the blank of the following sentence correctly:
a) bad-prepare
b) recent-cooking The hole in front of his garage is becoming .
c) old-make
d) new-preparing a) deep and deep
e) ready-made b) deeper and deeper
c) deep and deeper
23 | UNESP 1983 d) deeper and deepest
Assinale a alternativa correta: e) deepest and deepest
Life in New York City is sometimes harder than
28 | UNESP 1997
in any other town in the world.
A lion is than a dog.
a) very
a) strong
b) much b) strongest
c) many c) more strong
d) so d) stronger
e) as e) most strong
29 | UNESP 1998
34 | PUCPR 1998
Assinale a alternativa que preenche corretamente a lacuna Choose the only correct alternative to complete the spaces:
da frase adiante:
I. The United States is not as Brazil.
This hill is than I thought it was.
II. The Everest is mountain in the world.
III. Chimpanzees are than dogs.
a) more lower
IV. Aids is disease of human being.
b) lowest
V. Mike Tyson is as Evander Holyfield.
c) lower
d) more low a) as beautiful – the higher – so intelligent – the bad – more
e) more high strong
b) more beautiful – the most high – as intelligent – the
30 | MACKENZIE 1997 baddest – so strong
Indicate the alternative that best completes the following
c) so beautiful – the high – most intelligent – the badder –
sentence:
stronger
d) so beautiful – the highest – more intelligent – the worst –
His salary as a pilot is much higher . as strong
e) as beautiful – as highest – more intelligent – the worst –
a) in comparison of teachers strongest
b) to compare as a teacher's
c) than that of a teacher
35 | UNESP 2000
d) than of teachers'
Assinale a alternativa que preenche corretamente a lacuna
e) than a teacher da frase apresentada:
031 | FEI 1997 This is the day I have ever had.
Complete:
a) worse
John is than the other students in his classroom, b) bad
but he is the .
c) worst
d) less good
a) younger – most intelligent
e) very bad
b) younger – more intelligent
c) more young – intelligentest
36 | UNESP 2001
d) most young – more intelligent Assinale a alternativa que preenche corretamente a lacuna
e) more young – most intelligent da frase apresentada:
32 | MACKENZIE 1999 Depending on the situation, having a computer may be
Which one is correct? having a telephone.
a) More have they, more want them.
a) easier
b) As more they have, as more they want. b) more useful
c) More they have, more they want. c) more convenient than
d) The more they have, the more they want.
d) the most difficult
e) Do more they have, do more they want. e) useful than
33 | UNESP 1999 37 | PUCRS 2001
Assinale a alternativa que preenche corretamente a lacuna
Which of the following words DOES NOT form the
da frase adiante:
comparative the same way as friendly in friendlier?
Paris is than Rome.
a) high
b) healthy
a) the most beautiful c) sexy
b) small d) costly
c) more rich e) ugly
d) more beautiful
e) largest
038 | UFPE 2002
042 | FEI 2000
"The caveman had a much harder life" means that his life WORST é superlativo de:
was:
a) well
a) much easier.
b) good
b) more difficult. c) worn
c) less dangerous.
d) bad
d) just as thrilling.
e) wealthy
e) not as hard as ours.
43 | PUCPR 1999
39 | FATEC 2002
Which is the option that completes the sentences
Interactivity makes life , yet and
CORRECTLY?
consumer-oriented. Plus, it's turning the world into a teeny,
homogenized global village and that is ultimately boring!
I. English is not as Portuguese.
II. Australia is island in the world.
– A alternativa que preenche correta e respectivamente os
III. In my opinion, Frank Sinatra was singer of
espaços em branco é:
this century.
IV. São Paulo is as New York.
a) more easier – hurrier
V. Europe is from Brazil than EUA.
b) more easy – more hurried
c) easyer – hurrieder
a) so difficult – bigger – the best – more noisier – as farther
d) easier – hurrieder
b) difficultest – biggest – the good – as noisier – farther
e) easier – more hurried
c) as difficult – the biggest – the best – as noisy – farther
d) as difficult – the bigger – the best – so noisy – farther
40 | PUCPR 2001
e) difficulter – most big – better – noisiest – so farther
About adjectives, choose the alternative that completes the
sentences correctly:
44 | PUCMG 1999
The capital expression in "The moment of discovery was NO
I. Is this | 1 one you have? LESS MAGICAL THAN the day in 1922" means that the
II. Celine Dion sings | 2 Madonna, but Madonna dances | 3. moment:
III. I wish I was | 4 Sean Connery.
IV. The church is | 5 monument of the town. a) was as magical as
b) was more magical
a) 1. the better; 2. best than; 3. well; 4. so handsome as; 5.
c) was less magical
the more ancient
d) wasn't magical
b) 1. the best; 2. so well as; 3. best; 4. more handsome than;
e) was most magical
5. the ancienter
c) 1. the worst; 2. worse than; 3. better; 4. most handsome 45 | PUCRS 1999
than; 5. the most ancient
The expression "as many as 80" expresses a capacity of:
d) 1. the best; 2. as well as; 3. better; 4. as handsome as; 5.
the most ancient
a) not more than 80.
e) 1. the worse; 2. better than; 3. best; 4. as handsome than;
b) at least 80.
5. the least ancient
c) over 80.
d) less than 80.
41 | MACKENZIE 2001
e) close to 80.
Indicate the alternative that best completes the following
sentence:
046 | UNIT 1999
"Less intimate and more isolated" are comparatives. Check
The more I read this book, .
the item that contains the correct superlative from of the
two adjectives:
a) the less I understand it
b) most is what he doesn't know
a) The intimatest and the more isolated.
c) the least do I understand it
b) The intimatest and the least isolated.
d) more I like
c) The most intimate and the less isolated.
e) the most I can understand
d) The least intimate and the less isolated.
e) The least intimate and the most isolated.
047 | FEI 2000
52 | UFPE 2003 – ADAPTED
The sidewalks are getting NARROWER. Indique o antônimo
The expression more and more, in "more and more
de NARROWER: complex" and in "more and more intelligent":
a) broader
( ) indicates emphasis.
b) thinner ( ) carries the meaning of even more.
c) more limited ( ) means greater in degree.
d) shorter
( ) is used as an intensifier.
e) higher ( ) signifies equal or identical in degree.
048 | UEL 2000 – ADAPTED a) V V V V F
O adjetivo GREATEST, no fragmento abaixo, está no grau:
b) V V V V V
c) V V F V F
One of the GREATEST meteor showers of OUR lifetime may d) V F V V F
(…) soon light up the night sky.
e) F V F V F
a) comparativo de igualdade. 53 | PUCRIO 2004
b) superlativo de superioridade. The only item that contains an adjective used in the
c) comparativo de superioridade. superlative form is:
d) comparativo de inferioridade.
e) normal.
a) "The widespread destruction (...) is happening before we
even know the most basic facts about what we are losing."
049 | UFRS 2001 b) "Covering only 6 percent of the Earth's surface, tropical
Os antônimos corretos das partes destacadas da expressão moist forests contain at least half of all species."
"THE BEST and THE NEWEST facilities" são, respectivamente:
c) "Scientists estimate that (...) as much as 20 or 25 percent
of the world's plant species will soon be extinct."
a) the most good – the youngest
d) "However, the chemical structures of most natural drugs
b) the baddest – the oldest
(...) simple extraction is usually less expensive than
c) the worst – the most old
synthesis."
d) the poorest – the most recent
e) "Tropical forests offer hope for safer contraceptives for
e) the worst – the oldest both women and men."
050 | UFRS 2002
054 | UFRS 2005
As palavras a seguir seguem o mesmo processo de formação
Considere a frase:
de WEARER, à exceção de:
This program teaches FASTER THAN any other language
a) computer.
program.
b) player.
c) weaker. – A alternativa que apresenta o significado antônimo da
d) murderer. expressão destacada é:
e) writer.
a) less faster than.
051 | UFV 2003
b) more slowly than.
All the alternatives below are examples of comparatives,
c) as slow as.
EXCEPT:
d) as fast as.
e) the slowest.
a) "...children as young as two..."
b) "...scored higher than people who..."
055 | UFRRJ 2003
c) "...listening longer results in staying smarter longer". The sequence that has the same formation of the words
d) "...Mozart was | ... a quick thinker". “newer” and “higher” is:
e) "Mozart makes you smarter!"
a) good – dark – late.
b) tough – tight – clear.
c) great – far – easy.
d) distant – tidy – thick.
e) big – wide – dangerous.
056 | UERJ 2004
061 | UFRS 2007
“If it's noisy, call back from somewhere quieter.”
Select the correct alternative to complete the sentence
below:
The suffix -er in quieter is semantically equivalent to the
suffix in:
The opposite of SMALLER and LIGHTER is respectively
and .
a) manners.
b) users.
a) littler – easier
c) caller.
b) larger – heavier
d) louder.
c) larger – easier
d) little – heavier
057 | UFV 2004
e) bigger – greater
All the options below are examples of comparatives, except:
62 | FATEC 2008
a) younger. Assinale a alternativa que apresenta o uso correto do termo
b) controller. fewer como no exemplo "fewer than one in five owners of a
c) rarer. mobile-wallet handset":
d) harder.
e) smaller. a) Mary has fewer money than John.
b) My teacher has fewer patience than the other teachers.
058 | UFV 2005 c) This company has fewer equipments than the others.
Choose the alternative in which BOTH words are examples d) My brother plays fewer musics than I do.
of the comparative form: e) Sue has fewer friends than her sister does.
a) lower / friendlier 63 | UNESP 2008
b) good-natured / miners Indique a alternativa que expresse o mesmo significado de
c) well-regulated / friendlier “Imitation is the sincerest form of flattery".
d) lower / miners
e) well-regulated / miners a) Imitation is the best form of provocation.
b) Imitation is a true form of irritation.
059 | UFSM 2005 c) Imitation is a real form of harassment.
A palavra "wider" apresenta uma marca de comparação. A d) Imitation is the most accurate form of exasperation.
mesma marca de comparação é apresentada na palavra: e) Imitation is the most genuine form of adulation.
a) speaker. 64 | MACKENZIE 1976
b) easier. Mark the correct item:
c) greatly.
d) learners. She is beautiful, but she is her brother.
e) together.
a) most beautiful of
060 | UFPE 2007 b) less beautiful
Select the phrase that is in the comparative degree of c) as beautiful
superiority: d) not so beautiful
e) not as beautiful as
a) An enormous man.
b) Dinner on the top floor. 65 | FATEC 2003
c) On the opposite corner. Assinale a alternativa em que o adjetivo é composto por dois
d) Earlier this evening. substantivos, como na palavra "weight-loss":
e) São Paulo's tallest building.
a) low-cholesterol meals.
b) high-fat intake.
c) western-style boots.
d) well-known people.
e) ice-cream flavors.
066 | FGV 2007
70 | UFSC 1997 – ADAPTED
In the sentence "Brazil's strong currency will likely also lead
Select the proposition(s) in which the capital letters are
to a loosening of foreign exchange restrictions", the word
ADJECTIVES:
LIKELY indicates a:
(1) The sun's rays are very POWERFUL.
a) comparison. (2) Don't take your VALUABLES on the beach.
b) conclusion. (04) COCONUTS are delicious.
c) probability. (08) The coral reef is BEAUTIFUL.
d) preference. (16) Monkeys can be DANGEROUS.
e) certainty. (32) The hotel is not RESPONSIBLE for your valuables.

67 | UNESP 1984 a) 01 + 02 + 08 + 16 + 32 = 59
Assinale a alternativa correta: b) 02 + 04 + 16 = 22
c) 02 + 08 + 16 + 32 = 58
a) That is a five-storey building. d) 01 + 08 + 16 + 32 = 57
b) That is a building five storeys. e) 01 + 02 + 04 + 08 + 16 + 32 = 63
c) That is a five-storeys building.
d) That storey building is five. 71 | PUCSP 2007 – ADAPTED
e) That building storey is five. Na sentença “Although American youth are more likely to
use the Internet every day”, a palavra LIKELY indica:
68 | ITA 1996 – ADAPTED
Assinale a opção cujo adjetivo possa substituir, de maneira a) preferência.
bastante aproximada, os adjetivos different e appealing, em b) desejo.
destaque no texto a seguir: c) similaridade.
d) probabilidade.
Who are these Blur blokes who, after a shaky start, have e) superioridade.
shaken the world? And what makes them so different, so
appealing as Pop Artist Richard Hamilton once asked in a 72 | MACKENZIE 2007 – ADAPTED
collage that they doubtless studied at art school. Is it art The opposite of "overall" in the text below is:
school itself (they all attended Goldsmith') that sets apart?
(…) These skills are regarded as essential components of a
diagnostic test which measures overall linguistic proficiency.
(Q. March, 1995)
a) specific.
a) outstanding b) challenging.
b) fancy c) regular.
c) lousy d) forbidden.
d) nice e) refreshing.
e) awful
073 | ITA 1999
69 | ITA 1998 – ADAPTED Determine a função gramatical de impaired em "visually
Morfologicamente, as palavras KOBE e JAPAN, na primeira impaired people" e de mouse em "mouse pad":
linha do texto a seguir, devem ser classificadas como:
a) adjetivo – adjetivo
The cause of the magnitude 7.2 Kobe, Japan, earthquake in b) verbo – substantivo
January 1995 is unknown. c) adjetivo – substantivo
d) substantivo – adjetivo
a) adjetivo. e) verbo – adjetivo
b) substantivo.
c) advérbio. 74 | MACKENZIE 1997
d) vocativo. The same as 'They hardly ever go to the movies' is:

a) Hardly ever they go to the movies.


b) Hardly they ever go to the movies.
c) Ever they do hardly go to the movies.
d) Hardly ever do they go to the movies.
e) They go to the movies hardly ever.
75 | MACKENZIE 1997
080 | JFS 2008
The same as 'Mr. Burton hardly talked to me.' is:
BARELY is used in "Mike and Josh have barely enough to pay
the rent this month."
a) Hardly did Mr. Burton talked to me.
b) Hardly Mr. Burton talked to me.
– Mark the option in which it must also be used to complete
c) Hardly did Mr. Burton talk to me.
the sentence meaningfully.
d) Did Mr. Burton hardly talk to me.
e) Mr. Burton talked to me hardly.
a) Although she had been ill for a long time, it still came as a
shock when she died.
76 | MACKENZIE 1997
b) If you miss this train you can catch the next
A sentença "Mal sabia ele que ela era casada", em inglês,
one.
seria:
c) He almost never washes the dishes and he rarely, if
a) He didn't little know that she married. , does any cleaning.
b) Did he little know that she was married. d) She was fifteen when she won her first
c) Badly knew he that she married. championship.
d) Little did he know that she was married. e) I'll have a piece of chocolate after the meals,
but it's quite rare.
e) Little knew he that she was married.

077 | AFA 2004 81 | FUVEST 1979


“An elderly German decided to commit suicide. Took a lot of Assinale a alternativa que preenche corretamente as
pills, tied a briefcase full of stones around his neck, rowed lacunas:
out into the middle of the Rhine and was found sound
asleep in his boat.” It's difficult to find a good wine.

(Buffalo News) a) so – such


b) such – so
In the first sentence “An elderly German decided to commit c) such – such
suicide” the word elderly is used as: d) so – so
e) such a – so
a) a more polite form for old.
b) a synonym for eldest. 82 | FUVEST 1979
c) the comparative form of the adjective elder. Assinale a alternativa de significado equivalente palavra
d) the comparative form of the adjective old usually used entre aspas:
when we compare members of a family.
He was 'fast' asleep.
078 | UFRS 2005
The word that could be placed between HAD and MET in the a) almost
sentence I HAD MET HER BY CHANCE ONLY A SHORT TIME b) quickly
BEFORE is: c) sound
d) very
a) still. e) nearly
b) ever.
c) yet. 83 | UEL 1996 – ADAPTED
d) though. No texto a seguir, 'very' significa:
e) already.
The seven-room 84th Street cooperative on Central Park
079 | UNESP 1995 West in a solid if decidedly uncharismatic building came on
This boat is small that we can't all get in. the market that 'very' morning.

a) very a) pouco mais que.


b) so b) mais que.
c) many c) demasiado.
d) much d) muito.
e) then e) mesma.
84 | MACKENZIE 1997
089 | UFPE 2000
Indicate the alternative that best completes the following Allen: You're pretty dirty.
sentence:
Helen: I'm even prettier when I'm clean.
We're having beautiful weather everybody
– In "pretty dirty", pretty is equivalent to:
.
1) very
a) so – can relax
2) somewhat
b) such a – wants to go out on the weekends
3) more
c) so – likes it
4) beautiful
d) such a – have been having fun
5) ugly
e) such – feels good
– The correct choices are:
85 | MACKENZIE 1999
I have been studying I can English.
a) 2 – 3
b) 4 – 5
a) as hard as – to learn
c) 3 – 4
b) so hard as – learning
d) 1 – 2
c) more hard as – to be learning
e) 1 – 5
d) harder than – to have learned
e) as much hard – to learn
90 | FATEC 2005 – ADAPTED
O advérbio SO na frase "he did so efficiently and discreetly"
86 | MACKENZIE 1999 pode ser substituído de forma adequada e sem prejuízo de
Please turn off the lights. I have to develop this film and it's
significado por:
here.
a) very.
a) bright enough b) too.
b) much bright
c) enough.
c) too bright
d) less.
d) brighter e) a little.
e) enough bright
91 | MACKENZIE 1996
87 | UECE 1998 – ADAPTED
Choose the correct alternative to complete the following
O vocábulo quite na sentença a seguir exerce a função de: sentence:
She sat with her head thrown back upon the cushion of the
Mr. Myers told me he will leave .
chair, quite motionless.
a) by train; for Paris; at 8 o'clock; next week.
a) substantivo.
b) for Paris; at 8 o'clock; next week; by train.
b) adjetivo.
c) next week; at 8 o'clock; by train; for Paris.
c) verbo. d) at 8 o'clock; next week; for Paris; by train.
d) advérbio.
e) for Paris; by train; at 8 o'clock; next week.
88 | UEL 1998 – ADAPTED 92 | EFOMM 2010
A lacuna é corretamente preenchida pela alternativa: In the sentence "There was a lengthy pursuit, over seven
hours", there is a word formed by the suffix "y". In which
We've been working to ensure all wood comes option below the word is formed by the same suffix?
from well managed forests.
a) lately
a) lot. b) mostly
b) hardly. c) fury
c) hard. d) ally
d) very. e) healthy
e) many.
93 | MACKENZIE 1998
098 | UFV 2000
Indicate the alternative that best completes the following
In the sentence "What is the BEST way to live?", the capital
sentence: word is the superlative form of the adjective:
he studies, he seems to know.
a) far.
b) bad.
a) More – least c) fun.
b) As much – as much
d) fair.
c) How much – more
e) good.
d) The more – the less
e) The least – the more 099 | UFV 2001
The adjective forms "bad" and "better" have as their
94 | FATEC 1999 superlative forms, respectively:
Indique a alternativa que corresponde ao sentido oposto de
LESS GUILTY empregado em "It enables them to feel less
a) worse and the best.
guilty":
b) the worst and the best.
c) the best and worse.
a) Guiltier. d) good and better.
b) Guiltiest.
e) better and the best.
c) More guilty.
d) Most guilty.
100 | JFS 2000
e) Much guilty.
After reading the following sentences attentively, mark the
alternative which contains the correct sequence of the
95 | UFPE 1998 – ADAPTED
adjectives:
"Learn and live" is the motto of Britain's and
most innovative university – the Open University. a) My brother bought a comfortable big American car.
b) Patty has a Colombian leather beautiful new jacket.
a) larger c) I like tall Brazilian stout charming women.
b) largest
d) She has two chubby 3-year-old mischievous children.
c) the largest e) Marion has a terrific book in English up-to-date language.
d) the larger
e) large 101 | JFS 2000
Qual a alternativa que possui a sequência correta dos
96 | UFRRJ 1999
adjetivos?
The word below that forms its superlative like "cheapest" is:
a) A leather light brown new suitcase.
a) unusual.
b) Two long stainless steel practical zips.
b) intelligent.
c) A small Egyptian copper jar.
c) good.
d) A square silk French red scarf.
d) small.
e) A plastic small red doll.
e) Parisian.
102 | FURG 1999
97 | UEL 1999 – ADAPTED
As expressões THE POOREST, THE MOST POPULOUS e THE
Preenche corretamente a lacuna na sentença a seguir a
MOST NOTORIOUS equivalem respectivamente a:
alternativa:
a) os mais pobres, as menos populosas, os mais notórios.
Movie star Paul Newman has unveiled his
b) os menos pobres, as menos populosas, os menos
project.
notórios.
c) os menos pobres, as mais populosas, os menos notórios.
a) the latest
d) os mais pobres, as menos populosas, os menos notórios.
b) latest
e) os mais pobres, as mais populosas, os mais notórios.
c) latter
d) later
e) late
103 | OSEC 1977
108 | EEAR 2008
Complete the sentence meaningfully:
In “It was a fair game”, we conclude that the game was
played:
Everybody feels in spring time.
a) well.
a) gooder b) badly.
b) more good c) honestly.
c) so good
d) brilliantly.
d) better
e) as good 109 | ITA 1995
O termo seldom, entre aspas no trecho adiante, poderia ser
104 | FAAP 1975 substituído por:
Marque a alternativa correta:
As an American Express Card member, you will enjoy a
Your classroom is not so as mine. Mine is the relationship with us that goes beyond the ordinary. You will
in the school. be treated as a MEMBER, not a number. And you will receive
the respect and recognition 'seldom' found today.
a) bigger – biggest
b) greater – greatest a) occasionally.
c) great – greater b) rarely.
d) large – largest c) often.
e) large – more large d) usually.
e) always.
105 | EFOMM 1997
Take it easy, John! You need not work so . 110 | UEL 1996
In the text bellow, the word nearly means:
a) hardly
b) harder After 20 years of scientific advances, 'nearly' three out of
c) hard four infertile couples seeking medical assistance to have a
d) hardest child still go home to an empty crib.
e) hardy
a) almost.
106 | EEAR 2008 b) hardly.
“You can fold most umbrellas” means that we can: c) close.
d) far.
a) enlarge them. e) over.
b) open them easily.
c) hardly carry them. 111 | UNESP 1990
d) make them smaller. The sun rises in the west.

107 | EEAR 2008 a) always


Read the sentence below: b) never
c) often
“In Mexico, 60% of men and 40% of women are d) sometimes
overweight.” e) usually

– According to the sentence above, we can conclude that 112 | UFRS 1996
the percentage of overweight men is: Gradually and powerfully are adverbs formed from the
adjectives gradual + ly and powerful + ly, respectively. Other
a) lower. adjectives can take the same suffix to form adverbs, in the
b) highest. same way, except:
c) higher.
d) lowest. a) historic.
b) usual.
c) wild.
d) abrupt.
e) intelligent.
113 | FUVEST 1977
119 | JFS 2012
Qual destas alternativas só contém expressões que indicam Which word is not an adverb?
tempo?
a) Wholly.
a) suddenly, at the same moment, through, just in time.
b) Weekly.
b) just, suddenly, apparently, all her life.
c) Earthly.
c) just, after, all, then.
d) Sadly.
d) any more, apparently, at the same moment.
e) Proudly.
e) right now, all her life, at the same moment, then.
120 | ITA 2010 – ADAPTED
114 | MACKENZIE 1976
Leia o seguinte fragmento:
Complete:

- "Have you finished your book yet?" With Japan about to hold an election that could end 55
- "Yes, I’ve done it." years of almost uninterrupted one-party rule, the
(I) ordinary citizens can fill the power vacuum
a) still by taking part in public life, the (II).
b) yet
c) already The Economist, de 8/8/2009.
d) ever
e) não sei Assinale a opção que preenche corretamente as lacunas I e
II, no excerto acima.
115 | FEI 1994
Qual das palavras a seguir significa “brevemente”? I II
a) most best
a) Now. b) least better
b) So. c) more best
c) Also. d) more better
d) However. e) less best
e) Soon.

116 | UNESP 1998 “The secret of getting things done is to act!”


They are going to work again . Dante Alighieri

a) yesterday
b) later
c) last year
d) last month
e) last week

117 | MACKENZIE 1996


The same as “She little realizes how smart she looks” is:

a) How smart does she realize she looks.


b) How smart she looks she doesn't realizes.
c) Little she realizes how smart she looks.
d) Does she realizes how smart she looks little.
e) Little does she realize how smart she looks.

118 | JFS 2012


Which word is not an adjective?

a) Beautiful.
b) Harmful.
c) Colourful.
d) Roomful.
e) Sorrowful.
Pronouns 5 | PUCMG 2001 – ADAPTED
I had just participated in a project that was to determine the
1 | ITA 1995 – ADAPTED minimum size of forest fragment necessary to save native
The defenders of Normandy were not the best of Hitler's species of animals and plants from extinction. With this
army. Those were in Russia and Italy, as well as in France, information, scientists could then work to form preservation
but on the other side of the Seine, the Pas-de-Calais, which areas in the forest fragments left behind by cattle ranchers.
the Germans thought the more likely invasion target.
– The word THIS refers to the:
(Extracted from Time – June 6, 1994)
a) utilization of many valuable native species
– O pronome demonstrativo "those" faz referência aos: b) necessity of urgently saving birds from extinction
c) size of the forest needed for wildlife reserves
a) Soldados escalados para a defesa da Normandia. d) destruction of thousands of native species
b) Soldados soviéticos da Ásia Central.
c) Soldados mais adestrados do exército de Hitler. 6 | UNIFESP 2002 – ADAPTED
d) Soldados das divisões estacionárias. The rise of molecular biology since the late 1950s has had
e) Soldados russos, italianos e franceses. the gradual and quite unforeseen effect of turning the eyes
of medical scientists increasingly toward the basic
2 | UNIRIO 1995 – ADAPTED mechanisms of life, rather than disease and death. Of
Researchers at Ohio State have developed a way to speed up course, this has always been the orientation of all non-
the growth of native shade trees – and the local utility plans medical biologists, studying growth, reproduction, nutrition
to help promote these saplings to homeowners. or any of the other characteristics shared by all living things.

(Popular Science – October, 1994, p.39) – A palavra "this" refere-se a:

– A palavra THESE em "…and the local utility plans to help a) research in molecular biology.
promote these saplings to homeowners" refere-se a: b) gradual and unforeseen effect.
c) medical scientists.
a) plans. d) study of basic mechanisms of life.
b) trees. e) study of disease and death.
c) researchers.
d) bills. 7 | MACKENZIE 1998
e) costs. Choose the correct alternative:

3 | UFMG 1995 – ADAPTED "Aquele é o Tim perto da porta?"


PISCES 20 Feb. – 20 March "Não, Tim é o que está na janela."
Every 200-odd years your fate becomes closely linked, for a
while, to your neighboring sign Aquarius. This is one of those a) "Is that Tim on the door?"
times – so read their horoscope as well! "No, Tim is the one on the window."
b) "Is that Tim near the door?"
– THIS in refers to: "No, Tim is the one on in the window."
c) "Is that Tim next to the door?”
a) Aquarius. "No, Tim is that one through the window."
b) confusion. d) "Is that Tim over the door?"
c) horoscope. "No, Tim is that one across the window."
d) travelling. e) "Is that Tim by the door?"
e) urging. "No, Tim is the one at the window."

4 | UFAL 2000 – ADAPTED 8 | UNESP 1984


If you're planning a trip abroad summer, don't I know he'll tell a different story.
let phrasebooks leave you tongue-tied.
a) they
a) a b) his
b) an c) your
c) the d) we
d) this e) us
e) that
009 | UFMG 1995
013 | UFSC 1996 – ADAPTED
Choose the GRAMMATICALLY CORRECT propositions to
Love Among the Laundry complete the blanks in the following sentence:

When Sally found a man's striped sock curled among her


clothes at the launderette she returned it to the tall dark were working, when she .
young man with a shy smile. They met there every week for
several months, then were seen no more. One of their 1. They – arrived.
wedding presents had been a washing machine. 2. He – Arrives.
04. We – left.
(Molly Burnett) 08. Mary – is writing.
16. You – called.
– The word IT in “she returned it to the tall dark young man” 32. David and Gregoire – came in.
refers to:
– Now, mark the correct sequence:
a) a smile.
b) a sock. a) 01 + 04 + 16 + 32 = 53
c) the launderette. b) 01 + 02 + 04 + 08 = 15
d) the laundry c) 01 + 04 + 08 + 16 = 29
e) the machine. d) 02 + 04 + 08 + 16 + 32 = 62
e) 02 + 08 + 32 = 42
010 | UNITAU 1995
Assinale a alternativa que corresponde à sequência de 014 | UFV 1996
pronomes que mais adequadamente completam a sentença The word THEY in the sentence “Personality questionnaires
a seguir: were sent out to more than 2000 men and women without
prior selection; when THEY were returned, the birth dates
cat is sick because ate were noted and the results were put through a computer”,
spoiled food over there. refers to:

a) Its; he; that a) results.


b) Its; he; this b) men.
c) His; its; this c) questionnaires.
d) Its; it; that d) birth dates.
e) His; it; that e) women.

011 | UEL 1996 15 | CESGRANRIO 1991


Mexicans can thank the peso crash for one thing: IT has The pronoun IT in the sentence “When we eat something
forced them to confront the country's deep-seated political with sugar in it, particularly refined sugar, enzymes in the
problems. Disappointed with the ruling party, the PRI, they saliva in the mouth begin to work immediately to change
are demanding a truly First World government. that sugar into a type of carbohydrate” refers to the word:

– In the above text, IT refers to: a) saliva.


b) sugar.
a) Mexicans. c) mouth.
b) peso crash. d) something.
c) PRI. e) refined sugar.
d) Mexico.
e) political problems. 16 | UNESP 1998
Assinale a alternativa que preenche corretamente cada
012 | UNESP 1989 lacuna da frase adiante:
work in the field of engineering.
Do you think is as experienced as ?
a) She
b) They a) her – I
c) He b) him – she
d) Them c) she – I
e) It d) myself – we
e) they – him
17 | PUCPR 1997
21 | UNESP 2001
Fill in the blanks of the text below with the appropriate
As a tool, the computer assists to perform a lot
pronouns: of activities.
Dear Debbie, a) we
How are you? Lisa and I are having a marvelous holiday. We b) us
are really enjoying . We brought three tubes of c) ourselves
suntan cream with and we've used d) they
all up already. Lisa is a bit annoyed because her suntan isn't e) to us
as good as .
22 | UEL 2001 – ADAPTED
a) ourselves – we – them – mine "A Chinese employee at Motorola complained that the
b) ourselves – us – them – mine company had been cheated when it bought numbers
c) us – us – they – my wholesale for its own staff, because IT was given numbers
d) us – we – themselves – mine that all ended in 4 (e.g. 54-7424), which means death."
e) ourselves – ourselves – they – my
– The capital word IT in the sentence above refers to:
18 | CESGRANRIO 1993
Mark the option which completes the following sentences a) wholesale.
with the adequate pronouns: b) a Chinese employee.
c) the Motorola company.
I. Businessmen have own priorities. d) the number 54-7424.
II. Everyone must feel happy with working e) death.
habits.
III. Working from home allows a mother to spend more time 23 | PUCSP 2002 – ADAPTED
with children. Na frase "Women now become doctors at nearly the same
IV. If you have never tried to work at home, you cannot rate as men, but they become physicians, not surgeons", o
discuss disadvantages. pronome they refere-se a:

a) I. his, II. their, III. her, IV. their a) doctors.


b) I. their, II. its, III. their, IV. its b) physicians.
c) I. their, II. his, III. her, IV. its c) surgeons.
d) I. its, II. your, III. its, IV. their d) men.
e) I. his, II. his, III. their, IV. your e) women.

019 | FEI 1997 024 | UFV 2001


Complete: In the sentence "Please give us the intelligence to save what
is left of our environment", the pronouns us and our relate
Stay with while I drive car. to:

a) I – your a) we.
b) she – you b) they.
c) me – your c) I.
d) me – yours d) she.
e) her – yours e) you.

020 | UECE 1998 25 | UNESP 1995


Em "It was HER sister Josephine who told HER", os vocábulos Assinale a alternativa correta:
em maiúsculo classificam-se respectivamente como:
They saw men and women talking to own
a) pronome adjetivo/pronome objeto hearts.
b) pronome substantivo/pronome sujeito
c) pronome adjetivo/pronome sujeito a) his
d) pronome substantivo/pronome objeto b) her
c) them
d) they
e) their
26 | PUCPR 2001
030 | PUCSP 2006 – ADAPTED
Which option contains the correct use of the pronouns? When the Portuguese arrived in Brazil five centuries ago,
they encountered a fundamental problem: the indigenous
I. Could you tell what has happened in the pub?
peoples they conquered spoke more than 700 languages.
II. His uncle gave the money to set up his new
Rising to the challenge, the Jesuit priests accompanying
business.
them concocted a mixture of Indian, Portuguese and African
III. It was kind of you to let me borrow
words they called "língua geral," or the "general language,"
computer.
and imposed it on their colonial subjects.
IV. She ignored father's warning and jumped
into the swimming pool. Adaptado do site www.nytimes.com
V. Just a minute, I'm going to hang jacket in the
wardrobe. – No texto acima, palavra "THEM" refere-se a:
a) I. me; II. him; III. your; IV. her; V. my
a) povos indígenas.
b) I. them; II. her; III. your; IV. her; V. your
b) padres jesuítas.
c) I. him; II. them; III. his; IV. its; V. mine
c) sujeitos colonizados.
d) I. her; II. us; III. their; IV. our; V. yours
d) índios, africanos e portugueses.
e) I. us; II. his; III. her; IV. his; V. him
e) portugueses.
27 | UFRN 2000 – ADAPTED
031 | JFS 2000
Portugal gave her people, her religion, her language, her
Dadas as sentenças:
building and decorative arts, her culture and habits, to
Brazil, to West and East Africa, to the Red Sea, to India and I. THE TITANIC sank in the beginning of the XX century.
Sri Lanka, to China and Japan, to the East Indies. II. THE BABY OF OUR UPSTAIRS NEIGHBORS is crying aloud.
III. MY PET is the smartest of the neighborhood. Everybody
Highlife, London: British Airways. July 1997. p. 121-122.
enjoys seeing him.
– No texto acima, o vocábulo "her" ocorre cinco vezes e, em
todas essas situações, refere-se a: – A alternativa que possui os pronomes que substituem
corretamente os termos em destaque é:
a) cultura.
b) povo. a) It – She – It
c) China. b) It – She – He
d) Portugal. c) He – She – He
d) She – He – He
28 | UFAL 1999 – ADAPTED e) She – It – He
Between 1950 and 1960, Japanese manufacturing output
grew at an average annual rate of 16.7 per cent and 32 | PUCCAMP 1992
GNP (Gross National Product) at about 10 per Peter L. Berger, one of America's most important
cent. sociologists, exhorts politicians to operate with "the ethic of
responsibility" (borrowing a phrase from Max Weber) and
– Preencha corretamente a lacuna do texto: consider the moral consequences of their actions.

a) theirs (Adapted from Dialogue, 2/1989)


b) our
c) it – In the text, the pronoun "their" refers to:
d) ours
e) its a) sociologists.
b) philosophers.
029 | FAAP 1997 c) politicians.
His niece has meals in town. d) consequences.
e) actions.
a) her
b) their
c) your
d) his
e) yours
33 | UNESP 1983
39 | UFSM 2002 – ADAPTED
That sports car is very expensive. The car dealer told me that
Stars do it. Sports do it. Judges in the highest courts do it.
price is 10,000 dollars. Let's do it: that yoga thing.
a) her – Observe que o "it" se repete. A que se refere?
b) his
c) its a) Stars.
d) their b) Sports.
e) hers c) Judges.
d) India.
34 | UNESP 1983 e) Yoga.
I have met that girl before, but I can't remember
name. 40 | CESGRANRIO 1991
Mark the option that contains the appropriate pronouns to
a) her complete the sentences below:
b) his
c) your Animals' teeth are changing (I) composition.
d) its That animal had (II) teeth in perfect conditions.
e) yours He brushes (III) teeth whenever he eats something.
If the patient dies, we call (IV) relatives.
35 | UNESP 1984 The bacteria found (V) way to the stomach.
Peter brought his dogs and I brought .
a) (I) their, (II) its, (III) his, (IV) his, (V) their
a) my b) (I) its, (II) their, (III) its, (IV) his, (V) its
b) your c) (I) their, (II) its, (III) her, (IV) her, (V) his
c) mine d) (I) his, (II) their, (III) his, (IV) her, (V) their
d) the mine e) (I) their, (II) his, (III) their, (IV) its, (V) her
e) our
041 | UFRS 1996
36 | UNESP 1985 The phrase "a book of mine" could be replaced by:
This dictionary is in fourth edition.
a) mine books.
a) his b) my books.
b) her c) some of my books.
c) its d) a book of my.
d) it's e) one of my books.
e) their
42 | PUCPR 1996
37 | UNESP 1986 Choose the alternative that best completes the dialogue
Which team won the game? below:
team did.
Bob: Do you always get good marks on
a) Theirs examinations?
b) They James: Yes, I do. I guess it's because I do
c) Their homework assignments and study a little every day.
d) Them Bob: How about Maria? Are grades good too?
e) Yours James: Yes. She's very bright and enjoys studying very much.

38 | UNESP 1996 a) yours – my – his


He said he was going to pass exam. b) you – my – hers
c) your – me – your
a) his d) your – mine – yours
b) her e) your – my – her
c) its
d) their
e) our
43 | ITA 1997 – ADAPTED
47 | UNESP 2004
The computer giant IBM has offered $1.1 million (730.000
Nas orações – “Your click on the Fund Free Mammograms
pounds) for a chess rematch between Garry Kasparov and
button helps fund free mammograms” e “The National
ITS super-computer, Deep Blue.
Cancer Institute and U.S. Department of Health and Human
Services recommend that women in their forties and older
– O termo ITS em maiúsculo no texto refere-se:
have mammograms every one to two years”, os adjetivos
possessivos YOUR e THEIR referem-se, respectivamente:
a) ao computador de Garry Kasparov;
b) a Deep Blue;
a) ao clique e a quarenta anos ou mais.
c) à IBM;
b) ao botão e a recomendar.
d) ao computador gigante da IBM;
c) ao leitor e às mulheres.
e) a Garry Kasparov.
d) ao leitor e a quarenta anos ou mais.
e) ao botão e às mulheres.
44 | UDESC 1997
Complete the sentence with the CORRECT alternative:
48 | UFPE 1996 – ADAPTED
Read the following sentence:
- Whose are these shoes?
- They are shoes. They belong to .
Computers and networks isolate us from .
They are .
– The correct choice to fill in the blank space is:
a) their – them – theirs
b) yours – you – your
a) each other’s.
c) his – he – him
b) ourselves.
d) our – us – ourself
c) one another.
e) hers – she – hers
d) themselves.
e) herself.
45 | UNESP 1999
Assinale a alternativa correta:
49 | UNESP 1997
Those two women always help .
In some cities people do not pay for tickets.
a) other each
a) them
b) each other
b) his
c) one other
c) our
d) other one
d) her
e) another each
e) their
50 | PUCPR 2007
046 | ITA 1999
Lucy hates John and John hates Lucy. Lucy and John hate
Leia o recado de Ho Chi Minh aos franceses, em 1946.
.
"You can kill 10 of my men for every one I kill of
a) themselves
, yet even at those odds, you will lose and I will
b) itself
win."
c) each other
d) herself
– A lacuna encontrada na frase acima deve ser preenchida
e) himself
por:
051 | UEL 1994
a) yours.
Here is some money. Go and buy some decent
b) them.
clothes.
c) you.
d) theirs.
a) myself
e) your.
b) herself
c) themselves
d) himself
e) yourself
52 | UNESP 1988
057 | UNESP 2000
Assinale a alternativa correta:
Assinale a alternativa que preenche corretamente a lacuna
da frase apresentada:
People should know about .
Catherine is making a dress.
a) yourself
b) herself a) to him
c) himself b) to her
d) themselves c) himself
e) yourselves d) herself
e) they
53 | UNESP 1996
Assinale a alternativa correta: 058 | UFV 2000
In the sentence "We look for answers within OURSELVES",
You can do that . the capital word has a meaning related to:

a) myself a) them.
b) himself b) him.
c) herself c) you.
d) yourself d) us.
e) ourselves e) her.

054 | FAAP 1997 059 | UERJ 2006


Assinale a alternativa correta: Reflexive pronouns have two distinct uses: basic and
emphatic. The reflexive pronoun used emphatically is found
I took my husband to the airport . in the option:

a) himself a) The oppressed resign themselves to their doom.


b) oneself b) They tacitly adjust themselves to oppression.
c) myself c) The enforcement of the law itself is a form of peaceful
d) herself persuasion.
e) yourself d) Our end is a community at peace with itself.

055 | FAAP 1997 60 | PUCRS 2008 – ADAPTED


Assinale a alternativa correta para preencher o espaço na The pronoun themselves in the sentence “the things
sentença a seguir: themselves had existed from the beginning of the world” is
used:
Mr. Dean's secretary was ill yesterday, so he had to type the
letters . a) as the complement to the verb "had existed".
b) to emphasize the subject of the verb "had existed".
a) yourself c) in relation to people taken in general.
b) themself d) to specify which things are arranged.
c) himself e) as a personal pronoun.
d) herself
e) itself 61 | UNITAU 1995
Assinale a alternativa que corresponde à denominação do
056 | UECE 1996 pronome, em destaque, a seguir:
Choose the incorrect alternative:
ANY day is a good day for walking.
a) The hunter shot itself with his own gun.
b) She wants to buy herself a new coat. a) Adjetivo possessivo.
c) Most girls like to look at themselves in the mirror. b) Adjetivo indefinido.
d) I locked myself out of the house. c) Adjetivo demonstrativo.
d) Adjetivo relativo.
e) Adjetivo definido.
062 | UEL 1994
67 | FUVEST 1997
Assinale a alternativa que preenche corretamente a lacuna
Choose another way of saying "There isn't anything really
da frase a seguir: like that":
- "Why didn't you buy that sweater? It was such a good
a) There is nothing really like that.
offer!"
b) There aren't many things really like that.
- "Because I didn't have money on me." c) There aren't no things really like that.
d) There is anything hardly really like that.
a) a
e) There are a few things really like that.
b) no
c) any 68 | UEL 1997 – ADAPTED
d) some
Assinale a alternativa correta:
e) none
Despite this violent activity, poltergeists in fact never hurt
63 | UNESP 1985
.
Assinale a alternativa correta:
a) something
Those organisms pose danger to human life. b) nothing
c) none
a) any d) nobody
b) none
e) anybody
c) no
d) not 069 | UEL 1997
e) no one
Assinale a alternativa que preenche corretamente a lacuna
da frase a seguir:
64 | UNESP 1986
said she is right. I will longer stand his bad manners.
a) Somebody
a) no
b) Anybody b) very
c) Anyone
c) too
d) Something
d) much
e) Anything
e) many
65 | UNESP 1987 070 | UFRS 1997
Assinale a alternativa correta:
Only about 160,000 red squirrels remain, against an
onslaught of some 2.5 million grays.
Would like to hear music tonight?
– A palavra SOME poderia ser substituída sem alteração do
a) somebody sentido por:
b) someone
c) anything a) fully.
d) anyone
b) partly.
e) something c) approximately.
d) more than.
066 | JFS 2008
e) average.
I do not want to stay home tonight. I want to go
. 071 | FATEC 1999
Escolha a alternativa que mantém o mesmo significado de
a) nowhere NO ONE em "no one passes or fails a TOEFL":
b) somewhere
c) nowhere else a) Anybody
d) everywhere else b) Everybody
e) none c) Nobody
d) Somebody
e) Someone
072 | FEI 1997
076 | UFRS 2002
Complete o diálogo:
A expressão HARDLY ANY em “He was a big, beefy man with
hardly any neck” poderia ser traduzida por:
- "Would you like
apples?"
- "No, thank you, I don't want apple." a) raramente visto.
- "And you?"
b) dificilmente algum.
- "Yes, I'd like ."
c) bom tamanho.
a) some – any – any d) quase nenhum.
b) an – any – no e) especialmente longo.
c) any – no – some
d) some – any – some 077 | PUCRIO 2002
e) an – some – any

073 | ITA 1999 – ADAPTED


Hardly took Louis Frank seriously when he first
proposed, more than 10 years ago, that Earth was being
bombarded by cosmic snowballs at the rate of as many as 30
a minute.

– A lacuna deve ser preenchida por:

a) somebody.
b) anybody.
c) someone.
d) everybody.
e) nobody.

74 | PUCPR 2003
Put in the missing words: "International Herald Tribune", August 30, 2001.

I. I want more tea, please. – Mark the sentence which must be completed with
II. I go fishing. "anywhere":
III. It doesn't rain .
IV. I'm sorry, but I have to give you. a) The manager had to go off else for an
V. knows it's wrong. appointment.
b) The dangerous dog was approaching but there was
a) I. some; II. Every time; III. someday; IV. nothing; V.
to hide.
Somebody
c) Britney says she didn't go yesterday.
b) I. any; II. Sometimes; III. every day; IV. anything; V.
d) This is part of the original castle build around
Everybody
c) I. some; II. Sometimes; III. every day; IV. nothing; V. 1700.
Everybody e) Have you seen my glasses? I've looked for
d) I. any; II. Every day; III. sometimes; IV. nothing; V. them.
Everyone
e) I. some; II. Everywhere; III. every time; IV. anything; V. 078 | UNESP 2004
Somebody Eating disorders class, cultural, or gender
boundaries. Therefore, they can affect .
75 | UFRRJ 1998
The sentence "there aren't any elephants here" in the a) knows no – anyone.
affirmative form is: b) know no – someone.
c) know some – nobody.
a) there are many elephants here. d) can know – nobody.
b) there are plenty of elephants here. e) don't know any – anyone.
c) there are some elephants here.
d) there are a few elephants here.
e) there are a lots of elephants here.
079 | UEL 1998
084 | MACKENZIE 2000
Assinale a letra correspondente à alternativa que preenche
I'm a person technical knowledge of computer
corretamente a lacuna da frase apresentada: will impress .
Here is a riddle for you: Which hand should you stir soup
a) who – everyone
with?
b) which – someone
. You should use a spoon. c) for whom – nobody
d) whom – everybody
a) Both
e) whose – anyone
b) Either
c) All 085 | UFSM 2002
d) Neither
Assinale a alternativa que preenche corretamente a lacuna
e) No one da frase apresentada:
80 | MACKENZIE 1998 There are many evidences that can live alone.
Indicate the alternative that best completes the following Those marriage ends generally suffer from
sentence: depression.
Being considerate means thinking about , not a) any – who
only about yourself. b) nobody – whose
c) no – whose
a) the others d) nobody – who
b) others e) any – which
c) another
d) the other 86 | UFRS 2000 – ADAPTED
e) every other The expression "whatever it likes" in the text below could be
translated as:
81 | MACKENZIE 1999
Vote for candidate you like. The screenplay (…) dares to imagine whatever it likes about
the link between Shakespeare's artistic passions and his mad
a) wherever yearning for a certain aristocratic beauty.
b) whenever
c) whoever a) qualquer que.
d) whomever b) seja lá o que for.
e) whichever c) nem tudo que.
d) todos que.
82 | UNESP 2000 e) nem sempre que.
Assinale a alternativa correta:
87 | UNITAU 1995
finds the money may keep it. Assinale a alternativa que corresponde ao referente do
pronome relativo em destaque a seguir:
a) Who he
b) Whom Both research and commercial perspectives are considered,
c) Whose making the event essential for all researchers, designers and
d) Whomever manufacturers WHO need to keep abreast of developments
e) Whoever in HCI.
083 | UFSM 2003 a) research and commercial perspectives
campaign benefits children b) developments in HCI
education is worth doing. c) interface design, user modelling, tools, hypertext, CSCW,
and programming
a) Some – which d) recent trends and issues
b) Any – whose e) all researchers, designers and manufacturers
c) Any – which
d) None – that
e) Some – what
88 | UNITAU 1995
92 | UNESP 1986
Assinale a alternativa que corresponde ao referente do That is the one I always use.
pronome relativo em destaque a seguir:
a) whose
Since then, microchips, satellites and nuclear power have
b) who
become realities THAT define everyday life.
c) what
d) which
a) every day
e) whom
b) life
c) intellectuals 93 | UNESP 1990
d) realities
The doctor to Mrs. Jones went told her to eat
e) scientists
less.
89 | CESGRANRIO 1994
a) where
WHERE in "They are limited to texts where the possibilities
b) what
of linguistic error are minimal" could be replaced by:
c) whose
d) who
a) that.
e) whom
b) which.
c) whose. 94 | CESGRANRIO 1990
d) in which.
In "Men don't often have the lump-in-the-throat feeling that
e) whereby.
many women experience", the pronoun THAT could be
replaced by:
90 | CESGRANRIO 1995
The pronoun WHO is used in "To help anxious shopaholics, a) who.
who often wind up with major financial and personal b) whom.
difficulties, researchers at several universities in the United c) whose.
States are working on a variety of therapeutic approaches".
d) what.
Mark the option in which WHO and THAT are
e) which.
interchangeable:
95 | CESGRANRIO 1991
a) The drug you ordered last week has arrived.
In the sentence "This same syndrome is reflected in the
b) The hospital your father recommended is
models who are shown in current advertising", the relative
now closed.
pronoun WHO could be replaced by THAT. The item in which
c) We hope the psychiatrist will arrive soon. the relative WHO could NOT be replaced by THAT is:
d) This is the researcher handled the project.
e) I met your doctor, but he didn't know I was. a) Journalists who also write ads earn a lot of money.
b) The girl recognized the man who had committed the
091 | UNIRIO 1995 – ADAPTED crime.
To make spending time outdoors safer, a company called c) The salesgirl told the manager who had stolen the dress.
Frogskin, Inc., located in Scottsdale, Arizona, is marketing a d) Some advertisements show models who are quite exotic-
line of clothing called Frogware THAT, wet or dry, protects looking.
the user from the damaging effects of the sun more e) One of the boys who visited us yesterday is a model.
effectively than sunscreens.
096 | FEI 1996
– The word THAT can be replaced with: Escolha a alternativa correta para completar a frase a seguir:

a) what. It was Eiffel constructed the metal framework.


b) whose.
c) which. a) whom
d) who. b) which
e) whichever. c) whose
d) why
e) who
97 | CESGRANRIO 1993
101 | FEI 1997
In the sentence "it's time to meet people who work from Complete:
their homes", the pronoun WHO can be replaced by THAT.
My neighbor, is very beautiful, was here this
– Mark the option that can only be completed with the
morning.
relative pronoun WHO:

a) That is the consultant I met in São Paulo last a) which


week. b) whose
b) My brother, works as a consultant, makes c) who
much money. d) when
c) Would you like to work with bosses are e) what
understanding?
d) Employees work from home part of the time 102 | CESGRANRIO 1997
are happier. Mark the sentence that can only be completed with WHOSE,
e) The tele-commuter to I was introduced the relative pronoun:
yesterday is Asian.
a) This is Patricia, sister you met last week.
b) One should be loyal to one is married.
98 | PUCPR 1996
c) She's married to a doctor of you have heard.
Fill in correctly with a relative pronoun:
d) AIDS, kills thousands of people, hasn't been
The flower exhibit was held in the Botanical wiped out.
Garden in Curitiba, last September, showed beautiful orchids e) I don't like people lose their tempers easily.
from all over the world.
103 | PUCPR 1998
a) that Choose the right alternative to complete the spaces:
b) whose
c) whom I. George Washington, became president of the
d) where United States, never told a lie.
e) who II. In Norway, is a Baltic country, you can see
the midnight sun.
99 | PUCPR 1997 III. Melanie Griffith, with Antonio Banderas got
Insert the appropriate relative pronoun: married, is very jealous.
IV. Bernard Shaw, books were known in all the
Gossips, to you should pay no attention, is a world, was a very clever writer.
bad thing. V. Le Corbusier, about we are learning now, was
Dr. Smith, car is outside, has come to see a a famous modernist architect.
patient.
My friend Jack, is in hospital, is very ill. a) whose – that – which – who – whose
This is my Uncle John, you have heard so much b) whom – which – that – whose – whom
about. c) who – which – whom – whose – whom
d) which – whom – who – whom – which
a) which, whose, who, whom e) that – who – whose – which – who
b) that, whose, whom, which
c) which, whom, that, who 104 | MACKENZIE 1999
d) whom, whose, that, whom Choose the correct alternative:
e) that, whom, who, which
a) Caetano Veloso's, who latest CD, I bought last week, is
100 | MACKENZIE 1997 wonderful.
Os períodos simples provenientes do composto SHE WAS A b) Caetano Veloso's latest CD, which I bought last week, is
GIRL WHOM IT WAS DIFFICULT TO KNOW WELL são: wonderful.
c) Caetano Veloso's latest CD whom is wonderful I bought
a) She was a girl. Whom it was difficult to know well. last week.
b) She was a girl. It was difficult to know her well. d) Caetano Veloso, which is wonderful, I bought last week
c) She who was a girl. It was difficult to know well. latest CD.
d) She was a difficult girl. It was difficult to know well. e) Caetano Veloso's latest CD, whose I bought last week, is
e) She was a girl. It was difficult to know whom well. wonderful.
105 | JFS 2010
109 | UFV/PASES 2000
In his last book, the author decided to talk about the people Complete the sentence below correctly:
and the places he loved.
Don Pedro, was one of the visitors, was also
a) who
very impressed with Bell's invention.
b) whom
c) which
a) who
d) that
b) whose
e) whose
c) when
d) where
106 | FATEC 2002
e) which
O pronome which em "Many mobile phone operating
companies would rather give out new handsets than see
110 | UFRRJ 2000 – ADAPTED
their clients defect to rival services, which often try to lure
In the sentence "about 20% of lung-cancer patients are
customers by offering the latest mobile phones free for
found to have a tumor WHOSE biological characteristics and
switching services" refere-se a:
small size give them a good chance of being cured if the
malignant growth is surgically removed", the capital word
a) clients.
refers to:
b) handsets.
c) rival services.
a) patients
d) customers.
b) blacks.
e) mobile phone operating companies.
c) tumor.
d) lung cancer.
107 | UNIOESTE 1999 – ADAPTED
e) about 20%.
Assinale a(s) alternativa(s) na(s) qual(is) os pronomes that,
who e which estão utilizados corretamente:
111 | PUCPR 2003
( ) Athletes are basically the consumers who are going to Supply the sentences with the correct alternative:
buy Jui2ce.
( ) Jui2ce is a juice that offers a series of benefits to your I. This is the hardest problem I have ever had to
health. face.
( ) Mandarin Mango is a flavor which provides vitamin A. II. A doctor, patients trust him, has great
( ) Calcium is one of the components who is good for health. responsibility.
( ) Young people that usually drink the Jui2ce say it is III. Vesuvius, is a lofty volcano, overlooks the
delicious. Bay of Naples.
( ) Jui2ce has beta-carotene, who is said to be excellent for IV. My friend Marcello, is in hospital, is very ill.
health. V. There's something I must tell you in
confidence.
a) V F V F F F
b) V F V F V V a) I. that; II. which; III. what; IV. who; V. that
c) V V V F F V b) I. which; II. whose; III. that; IV. whose; V. which
d) V V F F V F c) I. that; II. whose; III. which; IV. who; V. that
e) V V V F V F d) I. what; II. who; III. which; IV. that; V. what
e) I. that; II. whose; III. what; IV. which; V. that
108 | UFRRJ 2000 – ADAPTED
In the passage "I have learned there are large numbers of 112 | UNESP 2003
Americans (maybe not the majority) who are passionate Assinale a alternativa correta:
about, or at least interested in, shaping their lives to be
humane, individual, socially tolerant and contributing, and Children who are exposed to TV can learn ideas
spiritual by some definition", the word WHO could be may be taken away from it.
replaced by:
a) whoever
a) which. b) whom
b) whom. c) who
c) that. d) which
d) whose. e) where
e) the word cannot be replaced.
113 | UFRRJ 2003 – ADAPTED
116 | UECE/2ª FASE 2007
In the sentence “Under a microscope you can see the In the sentences: "Gold's novel was also the start of the
bacteria that lives in your gums. It's called gram-negative 'Jewish-American' novel, WHICH BECAME AN IMPORTANT
bacteria and it produces a toxin or poison that destroys the TYPE OF LITERATURE IN THE FIFTIES AND SIXTIES. Gold
bones around your teeth”, the underlined word can be describes the failure of the 'American Dream' for those WHO
replaced by: HAD LEFT EUROPE LOOKING FOR A NEW AND BETTER LIFE",
the parts in CAPITAL LETTERS are, respectively:
a) who.
b) whose. a) defining adjective clause and non-defining adjective
c) whom. clause.
d) which. b) non-defining adjective clause and defining adjective
e) what. clause.
c) defining adjective clause and defining adjective clause.
114 | ITA 2006 d) non-defining adjective clause and non-defining adjective
clause.

117 | UECE/2ª FASE 2008


The sentence: "The mimetic theory was dominant for
centuries, only falling into disfavor in the late 18th century
with the rise of Romanticism, which took poetry to be
essentially an expression of personal feeling" contains a/an:

a) object noun clause.


b) subject noun clause.
c) non-defining relative clause.
d) defining relative clause.

118 | UECE/2ª FASE 2008


– "A man named", no primeiro quadrinho, é equivalente a: The sentence: "the texts that make up English literature are
a part and a product of the English language and cannot be
a) a man whose name is. separated from it" contains a/an:
b) a man that the name is.
c) a man who the name is. a) conditional clause.
d) a man whom the name is. b) adverbial clause.
e) a man that is name. c) relative clause.
d) noun clause.
115 | PUCRIO 2006 – ADAPTED
In the expression "One-third of the youngest children in the 119 | UECE/2ª FASE 2008
United States – babies through age 6 – live in homes where The writer I am talking about is the one:
the television is on almost all the time", where could be
rephrased CORRECTLY with: a) whom hates giving interviews.
b) which has just written his autobiography.
a) Live in homes in which the television is on almost all the c) who wrote "Travels in Scriptorium".
time. d) whom was persecuted because of his ideas.
b) Live in homes that the television is on almost all the time.
c) Live in homes which the television is on almost all the 120 | FUVEST 1996
time. Choose the question for the statement: 'Plague also cropped
d) Live in homes the television is on almost all the time. up in 1994, in India':
e) Live in homes in that the television is on almost all the
time. a) How long did plague crop up in India?
b) How did plague crop up in 1994?
c) When did plague crop up in India?
d) What did plague crop up in India?
e) Why did plague crop up in India?
121 | UNIFESP 2008
125 | UNESP 1992
No trecho:
Assinale a pergunta correta para a resposta apresentada a
seguir:
"Some soy plantations in central Brazil are being
transformed to sugarcane ethanol operations and
For two weeks.
environmentalists say that could lead soy farmers to move
into the Amazon for their crop, which is also in high demand
a) How long have you had it?
worldwide, particularly from China".
b) How many time do you have it?
c) How long did you had it?
– a palavra which refere-se:
d) How much time you have got it?
e) There is how long you've got it?
a) ao etanol de cana.
b) aos produtores de soja. 126 | UFPE 2007 – ADAPTED
c) à soja. The word THAT, in: "the United Nations reported THAT of
d) à Amazônia. the 41 countries it monitors"; and in: "because THAT might
e) à China. jeopardize their economic growth", and in: "costly mandates
and controls THAT harm the economy":
122 | JFS 2000
O pronome completa corretamente a sentença ( ) functions differently in each phrase.
abaixo e, sintaticamente, é classificado como . ( ) has equivalent meanings in the three examples.
( ) is a conjunction in both, the first and the last examples.
A coward is one thinks with his legs every time
( ) functions as a demonstrative pronoun in the second
he is in danger.
example.
( ) is a relative pronoun in the last example and refers only
a) who – objeto to controls.
b) who – sujeito
c) whom – sujeito a) F F V V F
d) that – objeto b) F F F V F
e) which – sujeito c) V V V V F
d) V F F V F
123 | JFS 2002 e) V F F F V
Those firemen, saved the little girl from the fire,
are local heroes. 127 | UNESP 1993
Assinale a alternativa correta:
a) who
b) that
is your hat?
c) whom
d) which a) When
e) a e b estão corretas b) Who
c) Whose
124 | UERJ 2005 – ADAPTED d) Where
Pronouns may have different functions according to the e) How many
contexts where they occur. The use of the pronoun THAT
establishing reference to the previous content of the
128 | CESGRANRIO 1995
sentence is found in:
"A compulsive shopper told a researcher that she could
never go to a supermarket and buy just one bottle of milk".
a) That is a magical and mutually rewarding form of love
between writer and reader.
Mark the question to which this sentence is an answer:
b) A battle scar, a light limp, hair that is an untamable mass
of curls, or any other minor flaw will make the character
a) Where a compulsive shopper buys her milk?
more relatable, more lovable.
b) What did a compulsive shopper tell a researcher?
c) What could be more fulfilling than a book that caresses
c) Who did a compulsive shopper tell her habits to?
the reader with love, wit, sensuality and a feeling of
d) Why has a compulsive shopper told a researcher about
goodness?
her habits?
d) A romance novel that combines those elements will
e) How has a compulsive shopper told a researcher her
seduce the reader from the first chapter to the last.
routine?
129 | JFS 2000
133 | UNESP 1984
Complete:
Assinale a alternativa correta:
- Henry is a scientist wants to know how comets
a) Who did discovered America?
are formed.
b) Who discovered America?
- The thief stole my wallet must be mad now. It
c) Did who discover America?
was empty.
d) What Columbus discovered?
- These precious moments you are living now
e) What did Columbus discovered?
won’t last forever.
- They are exploring a continent surface is icy.
134 | UNESP 1985
- The woman about we were talking is an expert
Can you tell me ?
on Astronomy.
a) how much does a box of matches cost
a) that – who – * – which – whom
b) how much a box of matches costs
b) who – * – that – which – who
c) how much did a box of matches cost
c) who – that – which – whose – whom
d) how much has a box of matches cost
d) who – that – whose – which – whom
e) how much costs a box of matches
e) who – that – which – of which – who
135 | UNESP 1999
130 | FUVEST 1977 people in Brasília?
Qual a pergunta que segue a afirmação "I know that you are
the winner"? a) How many – are there
b) How much – are there
a) Whom said I am the winner? c) What many – was there
b) Which one told you that I am the winner? d) What many – were there
c) Who told you so? e) How many million – was there
d) Who told that to you?
e) Who said that I am the winner? 136 | UNESP 1999
farm is that large one? It is .
131 | UNESP 1983
a) Which – Peter's
Assinale a alternativa correta:
b) Whose – Peter's
c) Whose – of Peter
- These blue jeans are mine. are those on the
d) Which – for Peter
sofa?
e) What – Peter's
- They're Peter's.
137 | PUCPR 1996
a) Which
Choose the alternative that best completes the dialogue
b) What
below:
c) Where
d) Whom
Mr. Wilson is applying for a job. Right now, he is being
e) Whose interviewed by Mrs. Taylor, head of the personnel
department.
132 | UNESP 1983 Mrs. Taylor: is your full name, please?
Assinale a alternativa correta: Mr. Wilson: Thomas Wilson.
Mrs. Taylor: are you from?
Do you know ? Mr. Wilson: Canada.
Mrs. Taylor: were you born?
a) where your brother bought that car Mr. Wilson: I was born on March 7, 1956.
b) where did your brother buy that car
Mrs. Taylor: did you know about our job offer?
c) where does your brother buy that car
Mr. Wilson: Through the ad you put in the newspaper.
d) where will your brother buy that car
e) where has your brother bought that car a) How – Where – Why – Who
b) What – Where – How – Why
c) Who – How – Where – When
d) What – Where – When – How
e) What – Who – When – How
138 | CESGRANRIO 1994
141 | PUCPR 2007
"The new telephone can deal with 'hello' and other words
We don't know with he was talking on the
well enough." phone.
This sentence contains the answer to all question below
a) that
EXCEPT one. Mark it:
b) whom
c) what
a) What can the new telephone deal with?
d) which
b) What can deal with "hello" and other words well enough?
e) whose
c) How can the new telephone deal with "hello" and other
words? 142 | UFV 2000 – ADAPTED
d) Whose words can the telephone deal with well enough? Match the Question Words with the appropriate sentences.
e) Which words can the new telephone deal with well All question words must be used:
enough?
a) Where
139 | CESGRANRIO 1994 – ADAPTED
b) How
A new telephone system translates words and phrases from c) Which
English into other languages. Mark the item which contains a d) What
suitable English equivalent for the following Portuguese e) Why
phrases that may be part of a telephone conversation: f) Who
I. "Como vai você?"
( ) do you prefer: fish or meat?
II. "Quem está falando?"
( ) didn't they call the police?
III. "Ele não está. Quer deixar recado?" ( ) are we going to help her?
( ) should I spend my Christmas vacation?
a) (I) How do you do? – (II) What's your name? – (III) He's
( ) will win the next Nobel Prize for literature?
out. Can I give him a messages?
( ) was he doing when the lights went off?
b) (I) How have you – (II) Who's speaking? – (III) He's off. Are
there any messages?
a) C – B – E – A – F – D
c) (I) How are you? – (II) Who's this? – (III) He's not in. Do
b) C – E – B – F – A – D
you want to leave a message?
c) C – E – B – A – F – D
d) (I) Are you okay? – (II) Who are you? – (III) He left. Do you d) E – C – B – A – F – D
want to leave him a note? e) E – C – B – A – D – F
e) (I) What's up? – (II) Who's talking? – (III) He's not here
right now. Would you like to leave a message? 143 | FUVEST 2000
Choose the question for the statement: "the Sydney
140 | PUCPR 2001
delegates promised that theirs would be the most
Fill in the blanks in the sentences below choosing the best environmentally-friendly Games ever".
alternative:
a) Whose Games the Sydney delegates promised that would
I. knows how to speak decent French to talk to
be the most environmentally-friendly Games ever?
the tourists?
b) Who did the Sydney delegates promise that would be the
II. The ticket costs $8. are you going to pay? most environmentally-friendly Games ever?
III. can I take the subway to the Guggenhein
c) Who did promise that theirs would be the most
Museum?
environmentally-friendly Games ever?
IV. of those buildings is the hospital?
d) Whose Games did the Sydney delegates promise that
V. will your sister travel to London?
would be the most environmentally-friendly Games ever?
e) Which Games the Sydney delegates promised that would
a) I. Who; II. How; III. Where; IV. Which; V. When
be the most environmentally-friendly Games ever?
b) I. Whose; II. Who; III. How; IV. What; V. Why
c) I. Which; II. Why; III. When; IV. How; V. Whose
d) I. Whom; II. What; III. Which; IV. Where; V. How
e) I. How; II. When; III. What; IV. Why; V. Where
144 | UEL 1996
148 | EFOMM 2009
- How about having a party soon?
In the sentence: “If the oceans die, it could cause great
- . destruction”, the pronoun it refers to:
a) Yes, sure
a) oceans.
b) I bet he doesn't
b) great destruction.
c) It won't last
c) atmosphere.
d) I never do
d) the oceans’ death.
e) We aren't in it at all
e) the cause.
145 | JFS 2000
149 | EFOMM 2009
course do you think is the best one of this
Typhoon “Ida” left a trail of destruction in wake.
university?
It swept the country from coast to coast.
a) What
b) Whose a) mine
c) How b) ours
d) Which c) his
e) Why d) hers
e) its
146 | PUCPR 1999
Fill in the balloons with the right interrogative pronouns. 150 | JFS 2012
Relate the numbers given to the pronouns: What would the world be like without the Web?
almost had an answer, if only for a day. To
protest Congress’s consideration of the onerous Stop Online
Piracy Act (SOPA) – which would require Internet service
providers to block access to Web sites that let people
infringe on copyrighted material – Internet companies
including Google, Facebook, and Twitter were rumored to be
coordinating a blackout, taking offline
temporarily. I must admit I was kind of hoping the Internet
companies would go through with , just so we
could remember how we lived before the Internet. And it
seemed for a while that actually would,
especially after a lawyer who runs a trade group that
represents Internet companies claimed that big-name Web
companies were pondering the AWOL option.

a) 1. Whose, 2. Who, 3. Why, 4. How, 5. Who – Fill in the gaps correctly.


b) 1. Who, 2. Which, 3. What, 4. Why, 5. How
c) 1. What, 2. Whose, 3. What, 4. Why, 5. What a) We – themselves – it – they
d) 1. Which, 2. Who, 3. How, 4. What, 5. How b) They – ourselves – it – we
e) 1. Who, 2. Whose, 3. What, 4. Why, 5. How c) We – ourselves – it – they
d) They – themselves – them – we
147 | UFRS 2006 e) We – themselves – them – they
Em quais das frases a seguir o pronome WHAT é usado
corretamente?
“It is by acts and not by ideas that people live.”
I. What impresses the visitor about Seattle is its wateriness. Anatole France
II. What time does the ferry boat leave for Bainbrigde
Island?
III. Seattle offered plentiful resources, what attracted Arthur
Denny's clan.

a) Apenas em I.
b) Apenas em II.
c) Apenas em III.
d) Apenas em I e II.
Quantifiers and Intensifiers 5 | MACKENZIE 1996
Indicate the alternative that best completes the following
1 | UEL 1996 – ADAPTED sentences:
Assinale a letra correspondente à alternativa que preenche
corretamente as lacunas da frase apresentada: I. Could you give me water? I'm so thirsty.
II. I have very money. I need more.
"Did you like the film?" III. Who has friends than John? Nobody I think.
"No, not very ." IV. Only people came to the party yesterday. It
" was wrong with it?" was boring!
"The actors were good but the story was too sentimental." V. I have time to stay with my family nowadays.

a) much – What a) I. less; II. little; III. lesser; IV. a few; V. few
b) many – Why b) I. some; II. a little; III. few; IV. little; V. lesser
c) few – When c) I. a little; II. little; III. fewer; IV. few; V. less
d) little – How d) I. little; II. a little; III. less; IV. a few; V. least
e) so – Where e) I. few; II. less; III. least; IV. little; V. more

2 | UNESP 1991 006 | UFPR 1992


Assinale a alternativa que preenche corretamente as lacunas of our history and the lives of of
da frase a seguir: our great men and women are recreated by the movies.

Do politicians work and earn – Choose the alternative(s) that can complete the sentence
money? above correctly:

a) little – many 1) much – much


b) very – much 2) much – many
c) much – few 04) many – much
d) little – much 08) a lot – a lot
e) hard – many 16) a lot – much
32) a lot – many
3 | UNESP 1993
Assinale a alternativa correta: a) 02 + 08 + 16 + 32 = 58
b) 01 + 04 + 08 + 32 = 45
It is not easy to learn a foreign language. It requires c) 02 + 04 + 08 + 16 = 30
years of study. d) 01 + 02 + 32 = 35
e) 02 + 08 + 32 = 42
a) many
b) much 007 | UNESP 1990
c) little Assinale a alternativa correta:
d) lot of
e) any How shoes are there in the shop windows?

4 | FUVEST 1979 a) much


Assinale a alternativa que preenche corretamente as b) many
lacunas: c) few
d) a few
Give me tea with sugar. e) a lot of

a) many – much 008 | ITA 1998


b) some – a lot of Assinale a opção cuja frase esteja gramaticalmente correta:
c) short – many
d) bit – a lot of a) There is fewer people at the party than Mary expected.
e) some – many b) There is less people at the party than Mary expected.
c) There are less people at the party than Mary expected.
d) There are fewer people at the party than Mary expected.
e) There was less people at the party than Mary expected.
9 | CESGRANRIO 1990
13 | UFRS 1998 – ADAPTED
Which of the following sentences can be completed with the Na sentença “the Tamagotchi is a Keychain-size plastic egg
word MANY as in "the laser has many applications"?
that houses a small LCD in which ‘lives’ a creature that you
nurture by pushing a variety of buttons”, a expressão a
a) The laser beam is being used by telephone
variety of buttons significa o mesmo que:
companies.
b) The laser beam has caused advance in
a) little buttons.
various areas.
b) several buttons.
c) Science has gained from the latest
c) all the buttons.
applications of the laser.
d) very small buttons.
d) effort has resulted in significant technological
e) too many buttons.
improvement.
e) Scientists have devoted time to research in
14 | UFRS 1998 – ADAPTED
the field of communication.
These pioneers began a revolution that has culminated in a
firmly established belief among most American employers
10 | UNESP 1998
that women can do the job – any job – as well as men. Many
Assinale a alternativa que preenche corretamente a lacuna
thought that day would never come.
da frase adiante:

Must you always make so noise? – Na frase "Many thought that day would never come", a
palavra mais adequada para completar o sentido de many é:
a) much
b) many a) beliefs.
c) most b) most.
d) few c) women.
e) less d) standards.
e) discrimination.
11 | CESGRANRIO 1997
The item that presents, respectively, a synonym for BUT and 15 | UEL 1998 – ADAPTED
the opposite of MUCH in "but there is much to be gained" is: When the first men arrived in Samoa they found blind men
who could see well to describe things in detail
a) nevertheless – anything just by holding their hands over objects. In France, just after
b) even though – a few the First World War, Jules Romain tested hundreds of blind
c) besides – a little people, and found a that could tell the
d) however – few difference between light and dark. He narrowed their
e) yet – little photosensitivity down to the nose or in the fingertips.

12 | CESGRANRIO 1999 – Assinale a alternativa que preenche corretamente as


The following sentences should be completed with FEW or lacunas do texto:
LITTLE:
a) so – some
I. Many of us tried but very succeeded. b) very – any
II. To our surprise, changes in foreign policy were c) enough – few
. d) little – one
III. That school is so expensive that only children e) less – plenty
can attend it.
IV. That crane can lift objects weighing a 016 | UFSM 1999
hundred pounds. A expressão a few numbers em "the new phones can be
V. We had chance of success. programmed to dial only a few numbers" indica:

– The sentence which must be completed which FEW are: a) número controlado.
b) grande quantidade.
a) I and IV, only. c) número insuficiente.
b) II and III, only. d) número ilimitado.
c) I, II and V, only. e) número incompleto.
d) I, II, III and IV, only.
e) II, III, IV and V, only.
17 | UFPE 1998 – ADAPTED
021 | UFSM 2001
Identify the equivalent phrases to the one which is in capital
Em "A LOT OF these goods", a expressão em maiúsculo pode
word: ser substituída, sem alteração do sentido, por:
Not since the O.J. Simpson criminal trial have SO MANY a) a great deal of.
Americans been discussing one single issue. b) a few of
c) more of
(1) such a small number of d) the majority of.
(2) such a large number of e) much of.
(3) such a few
(4) such a considerable number of 22 | PUCPR 2000
(5) such a fair number of Mark the correct alternative to fill the gaps of the dialogue
below:
a) F V F V F
b) F V F V V At the Supermarket...
c) F F F V V
d) V V V F V Wife: Do we need (I) wheat?
e) V V F V F Husband: Yes, we do. We haven't got (II) wheat.
Husband: We need (III) apples, don't we?
18 | MACKENZIE 1998 Wife: No, we don't. We have got (IV) apples. But we have (V)
Indicate the alternative that best completes the following carrots and (VI) cheese. Let's get some...
sentences:
a) I. some; II. much; III. any; IV. few; V. many; VI. little
A: I don't like stroganoff. Would you like (I) pizza instead? b) I. much; II. any; III. many; IV. too much; V. few; VI. few
B: Oh no! Let's buy (II) loaves of bread and make c) I. few; II. some; III. little; IV. many; V. little; VI. little
sandwiches. d) I. any; II. much; III. some; IV. many; V. few; VI. little
A: But we have (III) time to do that. We're late for school. e) I. few; II. many; III. few; IV. no; V. much; VI. many
B: Alright, but we've got (IV) ham and (V) hot dogs.
23 | UNESP 1997
a) I. a little; II. a few; III. very little; IV. only a little; V. few I have a friends.
b) I. only a little; II. many; III. very little; IV. a few; V. any
c) I. few; II. very few; III. more; IV. little; V. some a) ten
d) I. very little; II. only a few; III. many; IV. a few; V. more b) few
e) I. little; II. much; III. few; IV. a few; V. very little c) some
d) many
019 | UFV 2000 e) several
In the sentence "MANY thinkers have tried to give us
answers," the capital word has a meaning close to: 24 | CESGRANRIO 1997
LAUGHTER is an uncountable noun. Mark the sentence
a) a few. below that shows the correct usage of countable and
b) few. uncountable nouns:
c) little.
d) several. a) I'll have just a bread for dinner.
e) much. b) John has fewer money than Paul.
c) Can you give me an advice?
020 | FATEC 2002 d) I need some informations.
Assinale a alternativa que apresenta o uso gramatical e) My mother is in very good health.
correto de "there is":
025 | AFA 2007
a) There is few teachers and parents talking to the children. “Only a relatively small amount of heat is lost through the
b) There is some children who need help. windows”. The boldfaced expression in the sentence can
c) There is a lot of frightened adults after the attack. only be substituted for:
d) There is many events happening at the same time.
e) There is much anger among people in New York. a) a great deal of.
b) plenty of.
c) a small number of.
d) a little.
25 | CESGRANRIO 1995
031 | JFS 2009
LITTLE is used in "There is still very little known about
Read the following excerpt:
compulsive spenders". Check the item in which it must also
be used to complete the sentence meaningfully.
Watched by his wife, Zahra Rahnavard, Mr. Mousavi told an
audience of female supporters in Tehran: "We should reform
a) people claim they don't like to go shopping.
laws that are unfair to women." As Ms. Rahnavard spoke,
b) Many adults are compulsive spenders, but very
many in the crowd shouted protests against the morality
children suffer from this obsession.
police, who regularly arrest women they deem
c) Researchers are willing to spend thousand
inappropriately dressed.
dollars to find out more about compulsive shopping.
d) As the famous psychiatrist was talking about compulsive
– Which of the following is the best option to be used after
shoppers, the audience interrupted very .
the word many in the sentence “many in the crowd shouted
e) Apparently, changes in the treatment recommended to
protests against the morality police” in order to complete its
obsessive shoppers were very .
meaning?
027 | AFA 2007
a) women.
In the sentence “Two little mice and two little men live in a
b) people.
labyrinth searching for some cheese”, the word little can
c) citizens.
only be substituted for and refers to the
d) dwellers.
.
e) wives.
a) small – emphatic opinion given by the author to the
032 | AFA 2004
characters.
“An elderly German decided to commit suicide. Took a lot of
b) few – size of the men and mice.
pills, tied a briefcase full of stones around his neck, rowed
c) brief – height of them
out into the middle of the Rhine and was found sound
d) short – the lack of importance showed by the author
asleep in his boat.”
28 | EFOMM 1994
(Buffalo News)
Tom takes luggage in his trips. He usually takes
suitcase.
In the sentence “Took a lot of pills”, which other expression
of quantity couldn’t be used in it?
a) a few – no
b) little – one
a) a large number of
c) very little – any
b) a couple of
d) very few – one
c) a great deal of
e) a little – no
d) plenty of
29 | EFOMM 2007
033 | EN 2010
We should use time we have available to
Analyse the sentences below. Which alternative is correct?
discuss John’s proposal.
a) A little people passed the exam because it was too
a) the much
difficult.
b) the little
b) Few people passed the exam because it was too difficult.
c) a few
c) Much people passed the exam because it was too difficult.
d) a little
d) Many people passed the exam because it was too
e) the little of
difficult.
e) A lot of people passed the exam because it was too
030 | UEL 1994
difficult.
I don't like to spend my vacation in the country. There isn't
to do.

a) some
b) much
c) many
d) nothing
e) everything
034 | EFOMM 2012
Choose the option which completes the sentences below Verbs
correctly:
001 | FAAP 1997
It is use trying to change her mind. Assinale a alternativa que preenche corretamente a lacuna
Slowly, children began coming to school. da frase a seguir:
Unfortunately, he had friends.
Could you possibly give me help? In my school many foreign students.

a) a little / a few / few / little a) has


b) a little / a few / little / little b) there are
c) a little / few / few / a little c) there is
d) little / few / little / a little d) have
e) little / a few / few / a little e) there have

035 | JFS 2012 2 | UNESP 1994


Taking the following sentences mark the option which fills Assinale a alternativa que preenche corretamente a lacuna
the blanks correctly. da frase a seguir:

i. A great people who voted for her in the last There two main obstacles.
election will not be doing so this time.
ii. Have you heard of Polly recently? a) is
iii. The repairs to our car cost more than we b) was
were expecting. c) be
iv. I know quite people who've had the same d) to be
problem. e) were
v. With training she could do very well.
3 | PUCPR 1997
a) i. much; ii. much; iii. very; iv. few; v. little Complete the following text with the correct form of the
b) i. much; ii. much; iii. very; iv. a few; v. a little verb:
c) i. many; ii. little; iii. much; iv. few; v. little
d) i. many; ii. little; iii. much; iv. a few; v. a little A long time ago London an important city, but it
different from London today. There
e) i. many; ii. much; iii. much; iv. a few; v. a little
not very many big buildings. There
a lot of small boats on the river.
“Wear the old coat and buy the new book.”
a) is, is, are, are
Austin Phelps
b) was, is, are, are
c) was, was, were, were
d) was, is, were, were
e) had been, is, were, are

4 | MACKENZIE 1999
Indicate the alternative that best completes the following
sentence:

She German very well.

a) speaks and both writes


b) both speaks and writes
c) speaks both and writes
d) speaks and write both
e) both speaks and both writes
5 | FATEC 2002
10 | UNITAU 1995
Reescreva corretamente a frase a sentença apresentada a
Assinale a alternativa que mostra a ordenação em um
seguir: diálogo coerente das frases numeradas de 1 a 15 a seguir:
Yes, it does depend on the age of the child.
1) I am pregnant.
2) Yes, what is it?
a) Yes, it really depends on the child's age. 3) Hello, I would like to talk to Mike.
b) Yes, it really depends on an aged child.
4) Is he home?
c) Yes, it really depends on what age have the child.
5) I don't believe it!
d) Yes, it depends on what age the child does has.
6) It is true, honey.
e) Yes, it does depend on what age does the child has. 7) Hi. It is Mike speaking.
8) Who is that?
6 | UNESP 1992 9) Fine, thanks.
Escolha a alternativa que melhor interage com a oração 10) Well Mike, how have you been?
dada:
11) It's Candice.
12) Yes, of course I do remember you.
How do you do?
13) I must tell you something, Mike.
14) You are my ex-wife.
a) I forgot!
15) Don't you remember me?
b) It is easy!
c) How do you do?
a) 15, 14, 13, 12, 11, 10, 9, 8, 7, 5, 6, 4, 1, 2, 3.
d) I don't know.
b) 1, 2, 3, 4, 5, 6, 7, 8, 9, 10, 11, 12, 14, 15, 13.
e) But I don't! c) 7, 3, 2, 1, 5, 6, 8, 9, 10, 11, 12, 14, 13, 15, 4.
d) 2, 8, 6, 5, 7, 9, 11, 13, 15, 10, 12, 4, 1, 3, 14.
7 | UNESP 1993
e) 3, 4, 7, 8, 11, 15, 12, 14, 10, 9, 13, 2, 1, 5, 6.
Which is the suitable answer to this question?
011 | UEL 1994
Do you want some ice-cream?
Assinale a alternativa que é a correta versão da frase
apresentada:
a) Last night.
b) In my pocket.
"Eu quero que eles se sintam em casa."
c) Last Sunday.
d) It is not black.
a) I want that they feel the house.
e) No, thank you.
b) I wish they would touch home.
c) I wish they had homely feelings.
8 | UNESP 2004
d) I want them to feel at home.
People weight loss is a temporary endeavor,
e) I'd like them to make themselves a house.
a lifelong action plan.
012 | UEL 1996
a) who thought – don't design
Nesta questão uma certa situação é sugerida. Assinale a
b) that thought – has never designed
alternativa mais adequada para a situação:
c) who design – think of
d) who think – don't care about designing
- "Do you still have tickets to Londrina?"
e) who think – design
- "Yes, we do."
- "When does the next bus leave?"
9 | FUVEST 1977
-" ."
Qual a forma correta?
a) R$ 35,00
a) This book tells me as long the river was;
b) Seats 21 and 22
b) This book tells how long the river is;
c) Only aisle seats
c) This book tells how long the river is ago;
d) From gate 5
d) These books tells me how long the rivers were;
e) In twenty minutes
e) This book tells me how long the river is.
13 | UNESP 1987
18 | MACKENZIE 1998
Assinale a alternativa correta:
Indicate the alternative that best completes the following
sentence:
Why go home now?
I when that I have to study.
a) aren't we
b) didn't we a) don't like – she says
c) haven't we
b) never like – she will tell me
d) don't we
c) can't like – she says
e) wouldn't we
d) mustn't like – she speaks
e) don't like it – she tells me
14 | UNESP 1996
Assinale a alternativa correta: 019 | JFS 2012
Indicate the alternative that best completes the following
He doesn't anymore. sentence:
a) smoking She for you since 4 p.m.
b) no smoking
c) smokes
a) has been waiting
d) smoked b) had been waiting
e) smoke
c) waits
d) is waiting
15 | UDESC 1996 e) was waiting
Mark the CORRECT alternative to complete the sentence:
20 | FUVEST 1998
His body in the cemetery.
Choose the question for the statement "The spirit lives on”:
a) lies
a) Where does the spirit live?
b) laid
b) What does the spirit live on?
c) lays
c) Which lives on?
d) lain
d) Who lives on the spirit?
e) lied
e) What lives on?
16 | FUVEST 1997
21 | PUCRIO 1998
A forma correta do singular de "Why do bees fuss about so
In the sentence "Everyone lies", the present tense is being
much when they fly?" é:
used to express a fact that will never change in time
(historical present). In which of the alternatives below is the
a) Why does bee fuss about so much when it fly?
present tense being used to express a similar idea?
b) Why do an bee fusses about so much when it flies?
c) Why does a bee fuss about so much when it flies?
a) It is hot and sunny today.
d) Why does the bee fuss about so much when it fly?
b) Water freezes at 0° Celsius.
e) Why does a bee fusses about so much when it flies?
c) My plane leaves at 5pm tomorrow.
d) My cousin studies Computer Science.
17 | MACKENZIE 1997
e) Joe is late for work today.
Indicate the alternative that best completes the following
sentence:
22 | MACKENZIE 1999
Indicate the correct alternative to complete the sentence:
She his proposal, but she a
decision for a while. He the now.

a) considers – doesn't need to make a) could remind – girls' name


b) is considering – doesn't want to make b) has reminded – girl's name
c) has considered – had to take c) is remembering – girl's name
d) has been considering – is taking d) reminds – name of the girl
e) considered – needs to take e) remembers – girl's name
023 | UFPB 1998
27 | FATEC 1999
International advertising can be a risky business. When Assinale a alternativa que apresenta a idéia equivalente a
McDonald's launched Le Big Mac in Paris, it discovered that "We used to fight like cats and dogs, but this has brought us
in local slang this meant "the big pimp". It is not just a closer together":
question of language either; national advertising styles also
vary considerably. The British like humour and irony in their a) We are not close together anymore.
ads, whereas the Germans regard this approach as frivolous. b) We usually fight like cats and dogs.
The French are more sexist than the British and will use c) We are used to fighting like cats and dogs.
seminaked women in almost any context. The Italians d) We are not so close together as we used to be.
generally like to see beautiful people wearing beautiful e) We don't fight like cats and dogs anymore.
clothes driving beautiful cars. These are not just national
stereotypes, but based on hard experience. Different 28 | PUCPR 1999
countries also prefer different products. Choose the alternative with the right sentences:
– The text is PREDOMINANTLY in the: I. Why don't you cut the orange yourself?
II. You must learn how to speak English well.
a) present tense. III. He is just likes my father.
b) past tense. IV. I do not know like to sing this song.
c) future tense. V. They cut each other with a knife.
d) present perfect tense.
e) present progressive tense. a) I, II, IV
b) II, III, V
024 | UFRS 1998 c) II, III, IV
The correct verbal forms of the nouns "seduction", d) I, II, V
"betrayal", and "destruction" are: e) I, IV, V
a) seduce – betray – destroy 29 | MACKENZIE 2000
b) seduct – betray – destroy A: How do you like your coffee?
c) seduce – betrayal – destruct B:
d) seduct – betrayal – destruct
e) seduce – betray – destruct a) Only if you're having one too.
b) Strong, with 2 spoonfuls of sugar, please.
025 | UFSM 2001 c) Please do. I'm very hungry.
Assinale a alternativa que melhor expressa a idéia da d) Come on now. You can't be serious.
pergunta "Sound like science fiction?": e) No, thanks. I'm on a diet.
a) Is the sound like in science fiction? 30 | UNESP 2003
b) Do you sound like science fiction? Children's interest in TV between ages of 5 and
c) Does it sound like science fiction? 10.
d) Do you like the sound of science fiction?
e) Does science fiction have sound? a) keeps on growing
b) kept on grow
26 | PUCRS 2000 c) is keeping in growing
Choose the alternative that best completes the sentence: d) keep
e) keeps to grow
Today's issue of The Inquirer that another
couple wants to have their wedding sponsored. It also 31 | UNESP 2003
a photo of them. When children are frequently exposed to violent scenes on
TV, they about violence anymore because they
a) says – runs anything wrong in it.
b) say – run
c) said – running a) care – don't see
d) saying – ran b) don't care – can't see
e) says – running c) don't care – didn't see
d) didn't care – couldn't see
e) don't care – couldn't see
32 | UFRRJ 2004
037 | AFA 2007 – ADAPTED
In the sentence, "biological agents do not survive well", the
As a survivor of the Holocaust, I lost the life I led more than
use of the Present Tense implies: 60 years.
a) doubt.
– The Present Tense of the underlined verb is:
b) condition.
c) probability. a) led.
d) objectivity.
b) lead.
e) certainty. c) lid.
d) leaden.
33 | UFSCAR 2004
A frase "teens would rather look something up" equivale a:
038 | EEAR 2008
Choose the best alternative to answer the question below:
a) teens prefer to look something up.
b) teens must look something up. What do you do?
c) teens dislike to look something up.
d) teens should look something up. a) We are pilots.
e) teens wish to look something up. b) I’m fine, thanks.
c) I live in New York.
34 | PUCPR 2006 d) We are 20 years old.
When Carlos has a headache, he some tea.
039 | EEAR 2008
a) is drinking An umbrella a very ordinary object. It
b) drank people against the rain and hot sun. You can
c) used to drink fold most umbrellas, so it is easy them.
d) drinks
e) would drink – Choose the best alternative to complete the blanks in the
Paragraph:
035 | UFRS 2006
Gerald Middleton was a man of mildly but persistently a) is – puts – to hide
depressive temperament. Such men are not at their best at b) was – keeps – to help
breakfast, nor is the week before Christmas their happiest c) was – brings – to buy
time. d) is – protects – to carry

– What justifies the use of verbs in the present tense in the 040 | EEAR 2008
second sentence is the fact that that sentence expresses a: The stepmother smiled and said: “Of course you
go, Cinderella. If you your work first and if you
a) generalization. a dress to wear.”
b) systematization.
c) formalization. a) may – do – have
d) simplification. b) could – was – bought
e) formulation. c) might – are doing – lend
d) ought to – would finish – washed
036 | UECE 1998
O infinitivo de "stood" e "felt": 41 | EFOMM 2007
The companies are expanding their business and they
a) stand – feel all the help they can get. So they
b) steal – fall several people.
c) stride – fly
d) stay – fear a) need – are employing
b) are needing – are employing
c) needed – are employing
d) are to need – employed
e) needing – employ
42 | UNESP 1988
47 | FATEC 2004
Assinale a alternativa que preenche corretamente a lacuna Assinale a alternativa que apresenta o uso correto do
da frase apresentada: presente contínuo como em "The British Army is now
installing it in its tanks":
Many countries with nuclear reactors.
a) The British Army is liking the new program.
a) is experimenting b) The British Army is understanding the needs of the
b) experiments population.
c) experimenting c) The British Army is listening to the population.
d) would experiment d) The British Army is preferring the new general.
e) are experimenting e) The British Army is possessing many tanks.
043 | FAAP 1997 48 | UFV 2004 – ADAPTED
Complete: Choose the alternative in which the capital word -ING form
is an example of the present continuous:
The population of the world is .
a) Only 1 child in 100 can be classed as a real screen addict, a
a) going child who spends a WORRYING 7 hours or more watching TV
b) covering or playing computer games.
c) finding b) INCREASING prosperity has also contributed to the rise of
d) growing the bedroom culture.
e) beginning c) Children from the age of 9 are now TURNING to their
bedrooms as a place to socialise.
044 | FAAP 1997 d) 57% of children say they still enjoy READING, and 1 in 5
Assinale a alternativa correta: teenagers can be classed as a book-lover.
e) It is getting harder to control children's VIEWING.
The whole world against drugs now.
49 | MACKENZIE 2006 – ADAPTED
a) is fighting The battle for digital control (I) in the movie business, but (II)
b) fought virtually over in music.
c) had been fighting
d) has fought – The words and verb forms which properly fill in blanks (I)
e) fight and (II) in the sentence are:
45 | UNESP 2000 a) has still raged – it'll have been
Assinale a alternativa que preenche corretamente cada b) will have raged – it's being
lacuna da frase apresentada: c) is still raging – it's
d) was still raged – it had been
I to the radio every day, but I e) would still be raged – it has been
listening to it now.
50 | UNESP 1992
a) listen – am not Escolha a alternativa que responde corretamente à pergunta
b) listened – had apresentada:
c) listening – was not
d) was listening – not What order did he receive?
e) not listen – was
a) He ordered to stop smoking.
46 | MACKENZIE 2000 b) He was ordered stop to smoke.
Em inglês, "Você está esperando alguma carta?" seria: c) They ordered to him stop smoking.
d) They ordered him to stop smoke.
a) Have you been waiting for a chart? e) He was ordered to stop smoking.
b) Are you expecting a letter?
c) Are you attending any lecture?
d) Are you staying for the lecture?
e) Have you been hoping for a lecture?
51 | AFA 2004 – ADAPTED
56 | UNESP 1991
Assinale a alternativa que preenche corretamente a lacuna
Watching every motion in my foolish lover's game da frase a seguir:
On this endless ocean finally lovers know no shame
Turning and returning to some secret place inside
Can you tell me where my bike yesterday?
Watching in slow motion as you turn around and say
a) did you leave
(Trecho de Take my breath away –
b) you left
Giorgio Moroder/Tom Whitlock)
c) you have left
d) you leaved
– How can we classify the first verb in the first verse? It’s: e) did you left
a) a present participle used as part of the present 057 | FEI 1994
progressive. Assinale a forma verbal que está no PAST SIMPLE TENSE:
b) a gerund and it is used as the subject of the sentence.
c) not used as infinitive, just as gerund because it’s a special a) shows
expression.
b) has discovered
d) a gerund used with certain idiomatic expressions, for the c) making
most part, recreational activities.
d) found
e) have prepared
52 | UNESP 1994
He to return to his home.
058 | UEL 1996
When I asked Jim if he liked his job he replied that he:
a) not wanted
b) wanted a) did.
c) did wanted
b) does.
d) does wanted c) do.
e) to want d) doing.
e) has done.
53 | UNESP 1995
The mayor it difficult to refuse. 59 | UNESP 1986
he do the work last night?
a) find
b) finding a) Do
c) founded b) Does
d) found
c) Have
e) to find d) Has
e) Did
54 | FUVEST 1977
Qual a pergunta a anteceder a resposta "yes, I did"?
60 | UNESP 1996
Brazil last year's world soccer championship.
a) Did you buy a car?
b) Will you buy a car?
a) win
c) Didn't you have a nice car? b) won
d) Have you bought it?
c) wins
e) You didn't.
d) to win
e) winning
55 | FUVEST 1978
She did not tell me the truth. She to me. 61 | UNESP 1997
John me some money last week.
a) lie
b) lain a) sends
c) laid b) send
d) lay c) sent
e) lied d) sending
e) to send
62 | UNESP 1998
67 | PUCPR 2000
Assinale a alternativa correta: Choose the option to complete the text correctly:
I the river in a boat and swam the
CLAUDE MONET in Paris on 14 November 1840
stream.
and on 6 December 1926. He a
leading member of the French Impressionist painters,
a) across – cross especially concerned with the effect of outdoor light and
b) cross – crossed shade. He especially noted for his quick,
c) acrossed – crossing spontaneous style, developed to catch the fleeting moment.
d) crossed – across
Although now acclaimed as a great French painter, Monet
e) crossing – across
himself, like most artists, never that he had
achieved the perfect conclusion of the ideas that were in his
63 | MACKENZIE 1997 mind.
Indicate the correct alternative:
(Adapted from the book "The life and the works of Monet",
Kelly her eyebrows in extravagant . by Edmund Swinglehurst
a) risen – shock
a) was born – died – was – is – felt
b) raise – horror
b) borns – dies – is – is – fells
c) rise – panic c) borned – died – was – was – felled
d) raised – surprise
d) had born – had died – had been – had been – had felt
e) to raise – arrogance
e) is born – died – was – was – fell
64 | UNESP 1999 68 | MACKENZIE 2000
When World War II ?
Assinale a alternativa que corretamente preenche as lacunas
I, II e III das frases a seguir:
a) did – started
b) do – started
He (I) me a favor 2 months ago.
c) does – started
They (II) an attempt to escape.
d) do – star I (III) an important decision last night.
e) did – start
a) did – made – made
065 | UECE 1999
b) made – did – made
Marque o "past tense" de SPEND, LEAVE e KNOW: c) did – made – did
d) made – made – made
a) spended – leaved – known
e) made – did – did
b) spent – left – knew
c) spended – left – knew 69 | MACKENZIE 2000
d) spent – leaved – known Which is the best sentence?
66 | UNESP 2000 a) I used to smoke, but I don't anymore.
Assinale a alternativa que preenche corretamente a lacuna
b) I've smoked, but I don't anymore.
da frase apresentada:
c) I smoked, but I didn't anymore.
d) I had smoked, but I haven't anymore.
The kids were hungry and all the pie.
e) I would smoke, but I can't anymore.
a) eats
70 | MACKENZIE 2000
b) ate Thanks the financial aid he , he
c) eating
to attend the university.
d) eaten
e) not ate a) to – receives – had been able
b) for – has received – will be able
c) to – is receiving – wasn't able
d) to – received – was able
e) for – received – would be able
71 | UFRRJ 2000
075 | UFRS 2005
The infinitive and past simple forms of the verbs KNOWN
Complete the gap in the sentence below with the best
and BEEN are: alternative:
a) know and be.
In the past, English cavaliers swords while
b) knew and being. on the left.
c) know – knew and be – was/were.
d) known – knew and be – was/were.
a) drew – riding
e) know – knew and be – were/was. b) draw – would ride
c) were drawing – riding
72 | UFV/PASES 2000 d) drew – have ridden
In the sentence "Thousands of visitors were surprised when
e) had drawn – rode
they SAW and HEARD this invention", the capital verbs are
the past tenses of:
76 | FATEC 2003
Assinale a alternativa que apresenta a forma interrogativa
a) save and hide.
correta da frase "the death rate rose 33 percent":
b) say and hate.
c) sit and have. a) Did the death rate rose 33 percent?
d) sing and hum.
b) Did the death rate raise 33 percent?
e) see and hear. c) Did the death rate rise 33 percent?
d) Does the death rate rise 33 percent?
73 | UFV 2002 – ADAPTED
e) Does the death rate rose 33 percent?
Based on your knowledge of English grammar, correct the
sentence in the cartoon below: 77 | FATEC 2007
Assinale a alternativa que corresponde à forma afirmativa
do segmento "Engineers didn't think":

a) Engineers thought.
b) Engineers though.
c) Engineers through.
d) Engineers thru.
e) Engineers throw.

78 | UNESP 2008
Indique a alternativa que completa corretamente a sentença
a seguir:

The lady was sorry the Application to eight


pages, but she it to get the information she
a) 'he took my chair' – it's 'my chair was took'. .
b) 'he took my chair' – it's 'my chair was taken'.
c) 'he took my chair' – it's 'my chair was taked'. a) runs – needs – wanted
d) 'he taken my chair' – it's 'my chair was taken'. b) runs – need – wanted
e) 'he taken my chair' – it's 'my chair was taked'. c) run – needs – wants
d) ran – needed – wanted
074 | UFRS 2001 e) run – need – want
O verbo "lead" forma o passado e o particípio passado do
mesmo modo que: 79 | EFOMM 2005
Susie was watching TV when her husband .
a) meet.
b) cut. a) arrived
c) tear. b) had arrived
d) see. c) was going to arrive
e) bear. d) has arrived
e) would arrive
80 | EFOMM 2007
85 | UNESP 1986
- ‘David, Mr. Willcox and Mr. Alex Assinale a alternativa correta:
the P&O Shipping Company in 1837?’
- ‘That’s right. That was the first year.’
When John came in a book.
a) did – found a) she was reading
b) has – found
b) Mary is reading
c) do – find
c) will read
d) does – found
d) should read
e) have – founded
e) reads
81 | UNITAU 1995
86 | UECE 1998 – ADAPTED
Assinale a alternativa que corresponde ao verbo que tem She was beginning to recognize this thing that was
duas formas distintas para pessoas diferentes no passado approaching to possess her, and she was striving to beat it
simples: back with her will.
a) To have. – Na sentença, emprega-se o tempo:
b) To do.
c) To go.
a) simple past.
d) To become. b) past perfect.
e) To be.
c) past continuous.
d) present perfect.
082 | FEI 1997
Preencha os espaços em branco com a forma verbal correta: 87 | UDESC 2002
Find the correct answer:
When she I to do my work.
I was watching television.
a) has arrived – had tried
b) arrived – was trying
a) past continuous
c) arrives – was trying
b) conditional
d) has arrived – has tried
c) simple past
e) arrived – try
d) present perfect continuous
083 | UEL 1996
88 | UNESP 1995
Samuel Ryder a friendly game between some
Assinale a alternativa que preenche a lacuna da frase a
British professionals and the American players.
seguir corretamente:
– Assinale a letra correspondente à alternativa que preenche
He will almost everything you ask him.
corretamente a lacuna da frase apresentada:
a) do
a) is watching
b) to do
b) watches
c) doing
c) will watch
d) does
d) was watching
e) did
e) has watched
89 | UNESP 1998
84 | UNESP 1989
Assinale a alternativa que preenche corretamente a lacuna
Assinale a alternativa correta:
da frase adiante:
They about art last night.
I'll soccer this afternoon.
a) talks
a) playing
b) talk
b) played
c) was talking
c) to play
d) talking
d) play
e) were talking
e) plays
90 | UEL 1998 – ADAPTED
94 | PUCRIO 2000
That's exactly what you experience aboard the In the sentence "For the first time in human history, early in
high speed Eurostar passenger train.
the next millennium, there will be more people living in cities
than on the rest of the planet", the future form is used to
– A lacuna do texto é corretamente preenchida pela
express a prediction. In which of the alternatives below is
alternativa:
the future form used to express a similar idea?
a) are
a) Will someone help me with the luggage?
b) had
b) It will snow heavily in two days' time.
c) does
c) If it rains, the match will be cancelled.
d) go
d) Don't worry. I'll watch your dog carefully.
e) will
e) Waiter, I'll have some salad for lunch.
91 | MACKENZIE 2002
95 | UEL 2000 – ADAPTED
Indicate the alternative that best completes the following
Na frase "You'll find some monster savings on books at
sentence:
amazon.co.uk", a forma verbal em YOU'LL indica:

Julia isn't going to London.


you going a) hábito.
?
b) futuro.
c) necessidade.
a) Aren't – either
d) permissão.
b) Aren't – too
e) vontade.
c) Are – neither
d) Are – either
096 | PUCPR 2006
e) Are – too
I'm sorry, but I able to meet you for lunch
tomorrow.
092 | ITA 2002 – ADAPTED
Qual das expressões sublinhadas a seguir NÃO indica a) haven't been
expectativa/ação futura? b) can't be
c) don't be
a) Democrats hope to capitalize on public disenchantment d) won't be
with the Bush energy plan. e) wasn't
b) Who shall run the program – the Medicare system or
states and private insures? 097 | JFS 2000
c) A fast-track bill without provisions to protect the You and I together if we don’t want to fail at the
environment or international labor standards will face admission exam this year.
trouble.
d) They expect a fight if Daschle concludes that the White a) will to study
House is trying to pack the judiciary with conservative b) shall not to study
activists. c) shall study
e) They expect a fight if Daschle concludes that the White d) will not to study
House is trying to pack the judiciary with conservative e) won’t study
activists.
098 | UEL 1994
093 | FEI 2000 Assinale a alternativa que preenche corretamente a lacuna
"I don't think". Coloque na forma positiva e no tempo da frase a seguir:
futuro:
- “What to do when you get to Rio?”
a) I do think. - “I don't know yet.”
b) I am thinking.
c) I think. a) are you going
d) I won't think. b) were you
e) I'll think. c) did you
d) do you
e) you go
099 | ESPCEX 99
104 | FUVEST 1997
Choose the correct alternative: Considere a imagem a seguir:
A: Brrr. Who turned up the air conditioner? It’s really cold in
here. My nose and my fingers are cold.
B: I you a hot cup of tea.
A: Thanks. That sounds good.

a) will bring
b) won’t bring
c) will not break
d) will break
e) won’t break

100 | JFS 2008


Read the following sentence:

They're going to have a baby in the spring.


– Qual seria o correspondente, no passado, de "if I ever
– It expresses something that:
catch" and "I'll wash"?
a) is not probable to take place.
a) If I ever were to catch – I'll wash
b) is not being planned or expected.
b) If I ever caught – I'd wash
c) suddenly happens.
c) If I ever would catch – I washed
d) is certain or expected to happen.
d) If I ever caught – I'd have washed
e) will not happen without planning.
e) If I had ever caught – I would wash
101 | UNESP 1993
105 | UFSM 2003
Assinale a alternativa correta:
If people were honest, they buy fake products.
I did not think she come.
a) would
b) did
a) was
c) won't
b) were
d) wouldn't
c) would
e) don't
d) don't
e) doesn't
106 | UFRS 2005 – ADAPTED
In "If not for a girl named Kitty Wu, I probably would have
102 | FEI 1995
starved to death", the form would have starved indicates a:
Em "If there were no cracks glass would be stronger than
steel", a forma verbal "would be" significa:
a) habit long acquired.
b) condition in the future.
a) será.
c) permission granted.
b) foi.
d) possibility in the past.
c) seria.
e) obligation in the present.
d) teria sido.
e) é.
107 | UEL 1994
Life is so dull! I anything interesting happen to
103 | PUCPR 1996
me in ages!
If I won a lottery I around the world.
a) had
a) travel
b) have not
b) traveled
c) have had
c) will travel
d) don't have
d) would travel
e) haven't had
e) am traveling
108 | FUVEST 1977
113 | FGV 1995 – ADAPTED
Qual destas sentenças está correta?
Brutal competition from the Third World and the Soviet block
has stalled the developed nations.
a) I don't have never taken a course in Japanese.
b) I have never taken a course in Japanese.
– No texto, a melhor tradução para HAS STALLED é:
c) I never didn't take a course in Japanese still.
d) I ever did not take a course in Japanese. a) tem incentivado.
e) I took not a course in Japanese ever.
b) aqueceu.
c) crescer.
109 | FUVEST 1977
d) tem assustado.
Indique a resposta certa para "Has he heard the news?" e) parou.
utilizando "no":
114 | PUCCAMP 1992
a) No, I didn't. Assinale a letra correspondente à alternativa que preenche
b) No, I haven't heard the news. corretamente as lacunas da frase apresentada:
c) No, he hasn't.
d) No, I haven't. Sandy: Hi, Jack.
e) No, you haven't. Jack: Hi, Sandy.
Sandy: Gosh! I you for ages!
110 | FUVEST 1979 Jack: That's true. I from a trip to Japan just
Assinale a alternativa que preenche corretamente a lacuna:
yesterday.
Have you the correct alternative?
a) saw – am returning
b) saw – returned
a) choose
c) have seen – have returned
b) chase
d) haven't seen – returned
c) choosed
e) haven't seen – have returned
d) chose
e) chosen
115 | ITA 2003 – ADAPTED
If all my relatives suddenly died and all my friendships dried
111 | UNESP 1991
up and all of my subscriptions were cancelled and all of my
Assinale a alternativa que preenche corretamente as lacunas
bills were paid, I (I) guaranteed mail – two
da frase a seguir:
pieces a week, by my estimation – for the credit card
companies (II) me.
He learning English five years ago but he
it yet.
– Assinale a opção que poderia preencher respectiva e
corretamente as lacunas I e II do texto apresentado:
a) has started – does not learn
b) started – has not learned
a) would still be – would still want
c) has started – learn
b) will still be – will still want
d) started – have not learned
c) would still have been – would still have wanted
e) have started – did not learn
d) still am – still want
e) may still be – may still want
112 | FGV 1995 – ADAPTED
Black Nigerian students have, on average
116 | UNESP 1984
consistently better academically than their white European
Assinale a alternativa correta:
classmates.
a) I live here since 1970.
– Assinale a letra correspondente à alternativa que preenche
b) I have lived here since 1970.
corretamente a lacuna do texto:
c) I am living here since 1970.
d) I will live here since 1970.
a) did
e) I would live here since 1970.
b) does
c) do
d) done
e) doing
117 | UNESP 1985
122 | UFRS 1997
We're still waiting for Bill. He yet.
Choose the best alternative to complete the sentence below
correctly:
a) hasn't come
b) haven't come
Mexico many difficult crises in history, but now
c) didn't come it its own future.
d) doesn't come
e) hadn't come
a) has faced – is shaping
b) faced – was shaped
118 | CESGRANRIO 1993
c) have faced – shapes
In only a short time, the computer the way in d) have been facing – shaped
which many jobs .
e) faces – has been shaped
a) had changed – do 123 | UFRS 1998
b) changed – have done
Complete a frase a seguir com a forma verbal mais
c) has changed – are done
adequada para cada lacuna:
d) are changing – were done
e) will change – have been doing The kids in love with the Tamagotchi when they
first it, but they with it lately.
119 | UNIRIO 1996
A forma verbal has arrived em "Yes, the future has arrived: a
a) fell – saw – have not played
movie theater at home, thanks to laser" traduz-se por:
b) fall – see – did not play
c) fell – see – did not play
a) está chegando. d) have fallen – seen – do not play
b) tem chegado. e) fall – saw – have not played
c) chegará.
d) chegou. 124 | UFRN 1999 – ADAPTED
e) chega. Since 1935 researchers have known that when laboratory
rats and mice are fed a very-low-calorie diet – 30 to 50
120 | ITA 1998 percent of £their normal intake – they live about 30 percent
A frase "I never came across such a set in all my life" foi longer than their well-fed confreres, as long as they get
extraída de "Three Men in a Boat", escrito por Jerome K.
sufficient nutrition.
Jerome em 1889.
– A locução verbal HAVE KNOWN indica uma noção de
– No seu entender:
temporalidade referente a:
a) A frase não apresenta restrição gramatical.
a) dois momentos no passado.
b) "I have never come across..." teria sido uma melhor opção
b) passado e futuro.
gramatical.
c) passado, exclusivamente.
c) "I have never came across..." teria sido uma melhor opção
d) passado e presente.
gramatical.
d) "I never come across..." teria sido uma melhor opção
125 | ITA 1999
gramatical.
"Since 1985 the Shop a Company limited by
e) "I am never coming across..." teria sido uma melhor o
guarantee with charitable status; its aim is primarily to
opção gramatical.
relieve poverty in developing countries".
121 | UNESP 1998
(Panfleto da loja ONE WORLD SHOP, em Edimburgo, Escócia)
Assinale a alternativa correta:
– A alternativa que melhor preenche a lacuna do texto acima
Have you my Uncle Jack?
é:
a) not meet
a) is
b) meets
b) was
c) meeting
c) had been
d) met
d) have been
e) meet
e) has been
126 | ITA 2000 – ADAPTED
130 | FATEC 2003
But what has been so frustrating about the market reactions Assinale a alternativa que apresenta o uso correto do
in recent months is that despite the surging economy, Presente Perfeito do verbo "become", como em "Internet
inflation has not been rising. It has remained flat, at around cafes have become this generation's equivalent of the
3 percent, and ¢ yet Wall Street, certain that the shadow it telephone booth":
sees is the ghost of higher inflation come to haunt the
trading floors, has been clamoring to the Federal Reserve for a) He has become a political leader in 1984.
higher rates. b) She has become a U.S. citizen before she moved to
Australia.
The New York Times Magazine. May 22, 1994.
c) She has become a widow right after the war.
d) She has become his wife five years ago.
– O que determinou a utilização do Present Perfect Tense no
e) He has become ill.
parágrafo acima foi:
131 | UFRRJ 2003
a) o estilo do autor.
In the sentence, "I HAVE HAD leading positions in political
b) a referência a um tempo passado não explicitado no and public office FOR nearly 30 years", the words in capital
texto. letters describe an action which:
c) a referência a acontecimentos e/ou sentimentos
desencadeados no passado e que continuam no presente. a) was repeated in the past.
d) a atribuição de maior ênfase ao que se pretende dizer. b) continues up to the present.
e) a referência a sentimentos e/ou acontecimentos que c) continues to the future.
ocorrem no presente. d) goes to the past.
e) was completed in the past.
127 | MACKENZIE 2000
Indicate the alternative that best completes the following 132 | UERJ 2004
sentence: In the sentences:
plan that was presented . The large scale entrance of women into the professions
since the 1960s has posed many ideological and aesthetic
a) Not every – was suitable challenges.
b) Not all – had suited
c) Neither all – had suitability Many of the basic principles, associated with exclusively
d) Almost any – h as suited male executive office subcultures, have endured.
e) Every – suitably
– The temporal reference expressed by the verb forms has
128 | UFSM 2001 posed and have endured is best analyzed as:
Se o sujeito da oração "Machines HAVE already
CONVERGED" estivesse no singular e fosse mantido o tempo
a) situations beginning at a prior point continuing into the
do verbo, a forma verbal destacada:
present.
b) actions occurring at a specified prior time with current
a) ficaria inalterada.
relevance.
b) seria trocada por "had converged".
c) actions completed in the past prior to other past points in
c) se transformaria em "is being converged".
time.
d) seria substituída por "has converged".
d) situations developed over a prior time period and now
e) passaria para "is converging". completed.
129 | UFV 2002
133 | MACKENZIE 2005
In the sentence "Fortunately, people HAVE INVENTED The same verb tense used in "The jokes haven't stopped yet"
countless ways of amusing themselves", the capital verb is appropriately used in:
tense is:
a) The books have been read last week.
a) present perfect continuous.
b) They've done that before.
b) present perfect simple.
c) The noise has stopped when I went to bed.
c) simple present.
d) The film has started at 6:00 p.m. before long.
d) simple past.
e) We've seen each other the night before.
e) past perfect.
134 | UFRS 2006
138 | UECE 2008
Consider the verb form in the sentence below: The sentences "Plato acknowledged the power of poetry",
"This is simply the accepted mode of referential writing",
Britain HAS INVESTED very little in Chinese studies.
"Traditionally, literary texts have been easy to identify" and
"In the 20th century, much attention has been given to the
– The same verb form is used correctly in the sentence
language of literature" are respectively in the:
a) I haven't met my Chinese friends since July.
a) simple past, present perfect, present perfect and present
b) The children have read a Chinese story yesterday.
perfect.
c) Have you learned Mandarin when you were in school?
b) simple past, simple present, present perfect and present
d) They have seen many Chinese films last year.
perfect.
e) His parents have lived in China in the 1960's.
c) simple past, simple present, past perfect and present
perfect continuous.
135 | PUCPR 2008
d) past perfect, simple present, present perfect and present
Find the correct use of the Present Perfect Tense:
perfect continuous.
1) I've answered all the questions.
139 | EEAR 2008
2) He has stayed in that position for half an hour.
Some men no jobs lately.
3) Jane's written a book.
4) The writer has written a new book last year.
a) haven’t found
5) Lice has been a problem to mankind for years.
b) have found
6) Some thieves have robbed the bank a week ago.
c) doesn’t find
7) My men has slept for five hours.
d) has found
– Choose the right alternative:
140 | EFOMM 2006
The paint is wet because he the picture.
a) 1 – 2 – 5 – 7
b) 1 – 2 – 4 – 5
a) is just ending
c) 1 – 4 – 5 – 6 – 7
b) just ended
d) 2 – 3 – 7
c) have just ended
e) 1 – 2
d) has just ending
e) has just ended
136 | FATEC 2008
Assinale a alternativa que contém o uso correto do tempo
141 | EFOMM 2007
verbal Present Perfect", como no exemplo a seguir:
These machines have idle since the factory
closed.
Evidence has begun to show that animals have personalities
after all.
a) laid
b) lied
a) Brazil has won the world cup in 2002.
c) lay
b) When America was discovered, Indians have lived in the
d) to lay
land for a long time.
e) lain
c) Her grandfather has won the lottery.
d) They have finished their assignment before the end of
142 | ITA 1995
class.
A alternativa que melhor preenche a lacuna na sentença
e) The president has arrived from Europe the previous night.
abaixo é:
137 | ITA 2008
You look as if you a monster!!! Are you all right?
Assinale a opção em que a contração dos verbos ('s) ou ('d)
está representada corretamente:
a) just saw
b) have just seen
a) He's more useless than the gunman. – has
c) have just been seeing
b) The car's only got a few minutes left on it. – is
d) just see
c) I wish this bloke'd hurry up. – had
e) are just seeing
d) The car's not even worth it. – has
e) He's just turned twenty. – has
143 | CESGRANRIO 1990
148 | JFS 2008
Mark the sentence which can be completed with the verb
Jeffrey was proud of his son who a prize at
between parentheses in the same verb tense as in school.
“Scientists have found that the laser beam can transmit
human voices”: a) win
b) won
a) A century ago, scientists not able to predict c) have won
the applications of the laser beam. (BE)
d) has won
b) Doctors started to use the laser beam only after e) had won
communication experts it. (USE)
c) Today laser researchers the laser beam is 149 | UNITAU 1995
here to stay. (BELIEVE)
Assinale a alternativa que corresponde à denominação do
d) Scientists the importance of the laser since it tempo verbal da frase a seguir:
was discovered. (REALIZE)
e) B
I have been looking for a tree.
y the year 2000, scientists in various areas
the use of the laser beam. (EXTEND) a) Gerund.
b) Simple Present.
144 | EFOMM 1997
c) Present Perfect.
Which is the correct form?
d) Present Continuous.
e) Present Perfect Continuous
a) Have ever you been in England?
b) Has you ever been in England?
150 | UNITAU 1995
c) Have you ever been to England? Assinale a alternativa na qual se incluem a forma do futuro
d) Have you been ever in England? simples e a forma do presente perfeito contínuo da sentença
e) Have you ever be to England? a seguir:
145 | UEL 1995 Two teams of 11 players attempt to guide an inflated ball
He returned home after he the office. into goal cages.
a) leaves a) Two teams of 11 players will attempt to guide an inflated
b) does leave ball into goal cages./ Two teams of 11 players have been
c) had left attempting to guide an inflated ball into goal cages.
d) will leave b) Two teams of 11 players would attempt to guide an
e) didn't leave inflated ball into goal cages./ Two teams of 11 players has
been attempting to guide an inflated ball into goal cages.
146 | UFRS 2001 c) Two teams of 11 players attempted to guide an inflated
When earth to be, the angels' war in heaven ball into goal cages./ Two teams of 11 players are
. attempting to guide an inflated ball into goal cages.
d) Two teams of 11 players may attempt to guide an inflated
a) came – had ended ball into goal cages./ Two teams of 11 players will be
b) comes – has ended attempting to guide an inflated ball into goal cages.
c) had come – ended e) Two teams of 11 players do attempt guiding an inflated
d) came – had been ending ball into goal cages./ Two teams of 11 players will has
e) comes – was ending attempted to guide an inflated ball into goal cages.

147 | UNESP 2008 151 | FUVEST 1978


The lady that she the PIN number Assinale a alternativa que preenche corretamente a lacuna:
on the number of button presses required to access her
account balance. I the book for a couple of hours now.

a) wrote – has modeled a) had read


b) writes – would be modeling b) am read
c) was writing – modeled c) had been read
d) wrote – has been modeling d) having read
e) wrote – had modeled e) have been reading
152 | FUVEST 1996 – ADAPTED
157 | UFPEL 2006
As military spending has fallen, Brazil's arms makers
For the past three years, she says, she has been submitting
.
essays bought and copied from the internet and passing
them off as her own. She is currently working on her final-
a) have been struggling
year project and most of the materials in the dissertation are
b) would be struggling
coming off the net. Anna (not her real name) says she cheats
c) had been struggling
because it is easy to get away with it.
d) has struggled
e) were struggling Pode-se observar, no texto acima, a ocorrência de três
tempos verbais distintos na língua inglesa. As afirmativas a
153 | UDESC 1996 seguir contêm idéias relativas a cada um desses tempos:
Mark the sentence that is written in CORRECT English:
I. Algo que Anna faz com regularidade.
a) I teach English since five years ago. II. Algo que Anna tem feito há algum tempo.
b) I'm teaching English for many years. III. Algo que Anna está fazendo no momento.
c) I've taught English since some years.
d) I had taught English since some years. – Com base nas asserções, assinale a alternativa que
e) I have been teaching English for five years. apresenta a idéia contida em cada um desses tempos
verbais, segundo a ordem em que aparecem nos referidos
154 | UDESC 1996 parágrafos:
Find the CORRECT alternative to complete the following
sentence: a) II, I e III.
b) III, I e II.
- "You look tired." c) III, II e I.
- "Yes, I non-stop all day." d) I, II e III.
e) II, III e I.
a) am working
b) have been working 158 | JFS 2008
c) work I hard for 10 years before I that
d) will work promotion.
e) will have worked
a) have been working – gotten
155 | UDESC 1997 b) have been working – get
Find the CORRECT answer: c) had been working – got
d) had been working – gotten
He letters since lunch. e) had been working – get

a) is writing 159 | FUVEST 1977


b) have been writing Qual dessas sentenças está correta?
c) have written
d) writes a) News have to be sent by telegraph.
e) has been writing b) News has to be sent by telegraph.
c) News are sent through telegraph.
156 | UFPE 1995 – ADAPTED d) News had to be sent with telegraph.
As Brazil economic and social upheaval, many e) News is to be sent with telegraph.
Brazilians dream of moving to foreign parts to try out a new
160 | FATEC 1998
life.
Em "THAT'S the buzz about creatine, a muscle-building
supplement THAT'S become as common as sweaty towels in
– Select the correct choice to fill in the blank space of the
gyms across the country", a contração THAT'S corresponde
sentence above:
respectivamente a:
a) had gone through
a) that is – that is
b) has been going through
b) that has – that is
c) will go through
c) that is – that has
d) is going through
d) that has – that has
e) could go through
e) that has – that was
161 | UEL 2000 – ADAPTED
165 | UFG 2006
Addicted to portable electronics but hate adding to the 60
billion or so alkaline batteries that get thrown away every
year?

– A sentença acima é uma pergunta redigida de forma


coloquial. A forma gramaticalmente correta do verbo seria:

a) Were you addicted...


b) Are you addicted...
c) Do you addict...
d) Did you addict...
e) Are you addicting...

162 | UEL 2000 – ADAPTED


When the stuff finally get discarded, it breaks
down into harmless rust. Adapted from www.unitedmedia.com
– É exemplo de agramaticalidade característica da linguagem
– A lacuna do texto deve ser preenchida com uma forma que oral:
dê ênfase ao verbo. Essa forma pode ser:
a) "You know what I wonder?"
a) sure b) "Sometimes I wonder..."
b) too c) " ... pleased with me."
c) is d) "Do you ever wonder...?"
d) do e) "He just has to be!"
e) does
166 | EFOMM 2006
163 | PUCPR 2004 Lucy: “- Has Betty finished writing that letter?”
If the sentence "Ain't that sweet!" were to be changed from Tom: “- No, and she three hours ago”.
colloquial speech into more formal language, it would be:
a) should had finished
a) Aren't those sweets! b) should have finished
b) Wasn't that sweet! c) should to finished
c) Isn't that sweet! d) should finished
d) Is that as sweet! e) should finish
e) Was that sweet!
167 | JFS 2008
164 | PUCPR 1998 I the job, but I too tired.
Choose the CORRECT option that completes the phrases
below: a) should have finished – were
b) could have finished – was not
I. I'd rather you in the car. c) cannot have finished – am
II. This wouldn't have happened it you been d) would have finished – was
more careful.
III. When he was a boy he be thin. 168 | UNESP 1992
IV. No sooner arrived it started to rain. Escolha a alternativa que responde corretamente às
V. He cannot leave the hospital until his cut has . perguntas apresentadas:

a) wouldn't smoke – had – would – they had – cured What happened to the bridge?
b) not to smoke – should have – was used to – did they –
cure a) Oh! We should have blew up it!
c) no smoking – have – used to – they had – been cured b) Oh! We should blown it up!
d) didn't smoke – had – used to – had they – cured c) Oh! We should to have blown up it!
e) don't smoke – would have – got used to – they were – d) Oh! We should have blown it up!
cures e) Oh! We should had blown up it!
169 | CESGRANRIO 1994
172 | AFA 1999
Choose the alternative which completes the following Complete the text:
sentences with the adequate verb forms:
City residents tired of noisy car alarms that go off at all hours
I. T of the night this: an auto-security system that
he first translation program for computers uses smoke, not noise. Called the Dragon Vehicle Defense
invented in the late forties. (BE) Machine, it robbery by filling the car with a
II. Scientists research on machine translation cloud of smoke so dense that the thief can’t see. It
since the 50's. (DEVELOP) at car stores in June. Cost 35 dollars.
III. M
ost contemporary translators nowadays a) love – will prevent – is
computers to perform their task. (USE) b) will love – prevents – will be
c) won’t love – is preventing – won’t be
a) (I) had been – (II) have developed – (III) will use d) loving – is going to prevent – is being
b) (I) was – (II) have been developing – (III) use
c) (I) to be – (II) are developing – (III) are using 173 | AFA 2008
d) (I) has been – (II) develop – (III) will be using
e) (I) would have been – (II) developed – (III) have been using Lovers' Moon

170 | PUCPR 1996 The fabled Lovers' Moon


Lucy: How is your hotel? illuminates the night.
Sally: Great! It's the best hotel I in. Shining upon a couple with its magic light.
Lucy: Is Ben the holiday? They treasure just one thought
Sally: Ben is really . You know, the last time we two hearts so crystal clear.
to London was 10 years ago and the city To hold in their arms one that is so dear.
a lot since then. One that makes life worth living just by near.

They promise that forever


– Choose the best option to complete the conversation:
together they as they
bathe in the magic that others do not
a) stayed – enjoying – pleasing – have come – have changed
see.
b) stay – enjoyed – pleased – come – changed
So if you feel that you lonely
c) ever stayed – enjoying – pleased – have come – have
and hope to find love soon.
changed
Look toward the heavens and make
d) have ever stay – enjoying – pleased – came – changed
a wish on the Lovers' Moon.
e) have ever stayed – enjoying – pleased – came – has
changed
From the Internet, Quacmoto 1/4/00
171 | JFS 2008 – Complete the brackets with the right tense of the verb to
Fill in the gaps correctly: be and mark the correct alternative:
Coldplay a rock band formed in London,
England in 1997. Coldplay 33.9 million albums, a) being – will be – are
and also known for their hit singles, such as b) to be – to be – ‘re going to be
"Yellow", "The Scientist", "Speed of Sound", "Fix You", "Viva c) been – are – will be
la Vida" and the Grammy Award-winning "Clocks". Coldplay d) been – are going to be – are
worldwide fame with the release of their single
"Yellow", followed by their debut album, Parachutes (2000), 174 | UFF 2000
which nominated for the Mercury Prize. "Even had we known from the beginning he was suffering
from yellow fever it would not have changed the
From Wikipedia treatment".

a) are – have sold – are – achieved – was – The verb phrases in bold indicate that the change in
a) are – has sold – is – achieved – was treatment:
a) are – have sold – is – achieves – was
a) is – has sold – are – achieved – were a) will happen in future.
a) is – have sold – are – achieves – were b) may happen in future.
c) could have happened but didn't.
d) can happen but will not any way.
e) might still happen.
175 | ITA 2005
177 | JFS 2007
Fill in the text below with the following instructions:

During the same 24 hours that BenQ (I)


it quits on the digital camera market, Kodak
reportedly (II) similar thoughts when eying its
low-end camera lineup. According to CNET, Kodak President
Antonio Perez shared that the firm would be "abandoning
the low-end of the digital camera business" at the JPMorgan
Technology Conference in Boston. He also added that while
the company " (III) much money" in that
segment, it (IV) its own five-megapixel CMOS
sensor to be used in a (presumably mid-range) Kodak-
branded digicam. More interesting, however, was the
addition that this very sensor also
(V) its way into "several Motorola cell phones by the end of
the year." (…)

www.engadget.com

I. The Simple Past Tense of To Call


II. The Present Perfect Tense of To Have
(Time, May 12, 2003) III. The Past Progressive Tense of To Make
IV. The Past Progressive Tense of To Develop
Considere as seguintes asserções: V. The Simple Conditional Tense of To Make

* Em "We're" e "they'd", "'re" e "'d" são, respectivamente,


– The correct sequence is:
contrações de flexões verbais dos verbos I e II.
* Uma outra forma de expressar a oração "If we told you a) called – has ... had – weren't making – were developing –
everything, they'd have to kill us." é III. would ... make
b) called – has ... had – wasn't making – was developing –
– A opção que melhor preenche as lacunas I, II e III é: would ... make
c) called – has ... have – wasn't made – was developing –
a) I. are; II. would; III. They'd kill us, unless we told you would ... make
everything. d) had called – has ... had – wasn't making – had been
b) I. are; II. had; III. They had to kill us, unless we told you developing – would ... make
everything. e) had called – has ... had – wasn't making – was developing
c) I. were; II. would; III. Unless we told you everything, they – would ... made
would have to kill us.
d) I. were; II. could; III. Unless we told you everything, they 178 | CESGRANRIO 1992
could kill us. Check the alternative which contains the correct verb forms
e) I. are; II. would; III. They wouldn't have to kill us, unless to complete the sentences below:
we told you everything.
1. My neighbor sleeping pills since he lost his
176 | IME/CG 2011 – ADAPTED job. (TAKE)
By 1996, significant components of the aircraft’s defensive 2. Someone who from insomnia finds it difficult
managing system, just one small part of its electronics, to sleep. (SUFFER)
obsolete. 3. John to see a specialist in sleeplessness three
weeks ago. (GO)
a) will be 4. As I entered the drugstore, I saw someone that I
b) is 6 years earlier. (MEET)
c) are
d) were a) has been taking – suffers – went – met
e) was b) took – has been suffering – has gone – did meet
c) has taken – has suffered – went – have met
d) had taken – had suffered – had gone – met
e) has taken – suffers – went – had met
179 | EFOMM 2012
Choose the correct option to complete the sentences: Modal Auxiliaries
1. Where have you been? I for you for two 1 | UNESP 1992
weeks! Assinale a pergunta correta para a resposta apresentada:
2. Their bus at 3:00 p.m.
3. Tim to the beach, when he heard the Take the second on the left and then ask again.
weather forecast and changed his mind.
4. Research that excessive use of cell phones a) Can you give me an information?
may cause headaches. b) Excuse me. Where the Town Hall is?
5. When the police arrived, the thieves . c) Excuse me. Can you tell me where the Town Hall is?
d) Could you tell me where does the Town Hall is?
a) have been looking / has arrived / drove / showed / had e) Do you know when is the Town Hall?
run away
b) am looking / has arrived / was driving / has shown / has 2 | CESGRANRIO 1995 – ADAPTED
run away Shopaholics could be sexually frustrated, might suffer from
c) have been looking / has arrived / drove / has shown / ran lack of self-esteem, or they may just have a neurotic reaction
away to television commercials and glossy advertisements.
d) have been looking / arrives / was driving / has shown /
had run away – The modals COULD, MIGHT and MAY appear in the text to
e) am looking / arrives / was driving / showed / ran away express the idea of:

180 | PUCPR 2000 a) permission.


Mark the correct option: b) possibility.
c) intention.
Language is the most important development in human d) prohibition.
history. The arts, sciences, laws, economic systems, and e) ability.
religions of the world not exist without
language. Humans biologically for some 40 3 | UNIRIO 1995 – ADAPTED
thousand years. However, our ability to communicate Research shows that sunscreens may not be as effective as
us from the cave all the way to the moon. hoped at preventing sunburn. Users may be spending long
Little about the birth of language. Written hours in the sun with a false sense of security.
records that are more than 4 thousand years old
, but anthropologists agree that humans – The word MAY expresses the idea of:
thousands of years before that.
a) permission.
a) could – have not changed – has led – has known – are b) possibility.
being found – have been speaking. c) prohibition.
b) could – have not changed – will lead – known – is being d) obligation.
found – have spoken. e) expectation.
c) could – have not changed – has led – is known – have
been found – were speaking. 4 | FUVEST 1977
d) can – will – will lead – is known – have found – were Qual destas expressões corresponde a "ele não deveria ter
speaking. feito isso"?
e) should – have not changed – have led – is known – were
found – have been speaking a) He mustn't have made it;
b) He shouldn't have done that;
c) He could not have made it;
“I believe that if one always looked at the sky, d) He might not have done that;
one would end up with wings.” e) He cannot have done that.
Gustave Flaubert
5 | FUVEST 1977
010 | UEL 1994
Qual a forma correta?
Assinale a alternativa que preenche corretamente a lacuna
da frase a seguir:
a) the mail must go on whether there are a hundred storms;
b) the mail can go on whether there are a hundred storms; - "Excuse me, sir. you tell me the time?"
c) the mail should go on when there are a hundred storms; - "Sure, it's 5:20."
d) the mail must go on if there are a hundred storms;
e) the mail is going on if there are a hundred storms. a) May
b) Do
6 | FUVEST 1979 c) Can
He avoid mistakes. d) Have
e) Shall
a) ought – making
b) must – make 011 | UEL 1996
c) shall – make
Assinale a tradução correta da frase entre aspas apresentada
d) needs – make no diálogo a seguir:
e) should – making
- "Can you tell me how to get there?"
7 | FGV 1995 – ADAPTED - Of course I can.
we conclude, in line with the opinions of some
scholars, that black Nigerians are genetically more intelligent a) Você pode me dizer como se consegue isso lá?
than Europeans? b) Quem pode me contar como se faz isso?
c) Você pode me ensinar o caminho?
– Assinale a letra correspondente à alternativa que preenche d) Como se pode ir de lá para cá?
corretamente a lacuna do texto: e) Você consegue atravessar para o outro lado?
a) Have 012 | UEL 1996
b) Are
Assinale a versão correta da frase entre aspas:
c) Can
d) Is "Não posso comprar um carro novo."
e) Had
a) I shouldn't be thinking of a new car.
8 | PUCCAMP 1992 b) I can't afford a new car.
Janet: Look, our boat is sinking! c) I can't buy anything new.
Peter: Oh, dear! Can you swim? d) If it is new, I don't want it.
Janet: Yes, but we won't have to, there's a life boat on e) Who needs a new car anyway?
board.
13 | UEL 1996 – ADAPTED
– In the above dialogue, the verbs CAN and HAVE TO express Assinale a letra correspondente à alternativa que preenche
respectively and . corretamente a lacuna da frase apresentada:
a) ability – obligation Bob, you do me a real favor?
b) permission – prohibition
c) possibility – prohibition a) may
d) permission – possibility b) could
e) ability – necessity c) must
d) ought
009 | UEL 1994 e) should
Assinale a alternativa correta:
14 | UNESP 1984
We hurry. The bus leaves in 10 minutes. This place is not good. we go elsewhere?
a) can a) Will
b) must b) Will not
c) do c) Ought
d) did d) Shall
e) would e) Let's
15 | UNESP 1989
020 | FAAP 1997
Assinale a alternativa correta: Assinale a alternativa correta:
Doctors and dentists should always with their
I'm sorry the train was late and I arrive earlier.
training.
a) couldn't
a) continued
b) ought not
b) continue
c) don't
c) continuing
d) mustn't
d) will continued
e) wouldn't
e) have continue
21 | CESGRANRIO 1990
16 | UNESP 1996 The phrase BE ABLE TO in "the laser light signals will also be
Assinale a alternativa correta: able to transmit video telephone conversations in the
future" expresses the idea of:
Could I earlier tomorrow?
a) permission.
a) to leave
b) assumption.
b) leave
c) obligation.
c) leaves d) ability.
d) left
e) necessity.
e) leaving
22 | CESGRANRIO 1991
17 | UNIRIO 1996 In "servilities that must be avoided" the modal auxiliary
The word CAN in "Loneliness itself is hard to define. People MUST expresses an idea of obligation. The form which would
aren't always lonely when they're alone, but they CAN feel NOT express the same idea in this same context is:
lonely when surrounded by other people" expresses:
a) had better.
a) obligation. b) have to.
b) necessity. c) need to.
c) permission. d) ought to.
d) possibility. e) have got to.
e) intention.
023 | FEI 1996
18 | UECE 1997 – ADAPTED Complete:
I should get my hair cut.
I'd prefer to stay here. That's what I'd .
– 'Should' means:
a) do
a) would. b) do best
b) had to. c) have done
c) ought to. d) rather do
d) might. e) doing
019 | FAAP 1997 24 | MACKENZIE 1997
Assinale a alternativa correta: Indicate the alternative that best completes the following
sentence:
- "Do I have to do it again?"
- "Yes, you ." "He might have done it." So, .
a) had a) he was permitted to do it.
b) would b) we don't know whether he did it or not.
c) must c) he didn't do it.
d) did d) he did it.
e) were e) he wasn't able to do it.
25 | UNESP 1997
31 | UNIRIO 1999 – ADAPTED
The rain can our shoes.
How dreadful for them to sense, as they must, the lack of
enthusiasm of the audience.
a) spoil
b) spoils
– "As they must" sugere:
c) spoiled
d) to spoil
a) obrigação.
e) spoiling
b) advertência.
c) recomendação.
26 | UECE 1996 – ADAPTED
d) dedução.
He must have seen the old priest's corpse lying there.
e) permissão.
– Taking "must" as an example, choose the alternative
where the modal is used correctly: 32 | UFRN 1999 – ADAPTED
I would like to find an agency or agencies that might be
a) This card should be sent at once. interested in using the images for textbooks, calendars or
b) She should be sent us a post card. postcards.
c) They ought not go to the beach on Friday.
d) He must not to come to the party tonight. – A forma verbal MIGHT BE exprime:

027 | UERJ 1998 a) necessidade.


The word CAN in "consumers can start questioning b) certeza.
advertising" expresses: c) probabilidade.
d) condição.
a) possibility.
b) probability. 033 | UERJ 1999
c) permission. In “the computer cannot know that it is the year 2000, and
d) intention. must effectively work on the presumption that it is 1900”,
the word must expresses:
28 | UFRS 1998 – ADAPTED
In “he should have reached the door of 10 Downing Street”, a) intention.
the form "should have reached" indicates the same as: b) certainty.
c) necessity.
a) must have reached. d) prohibition.
b) was expected to reach.
c) will have reached. 034 | PUCRIO 1998
d) can have reached. The modal auxiliary ought to in "Liars ought to have good
e) was going to reach. memories" indicates:

29 | MACKENZIE 1998 a) possibility.


Look how wet the ground is. It last night. b) probability.
c) obligation.
a) might be sunny d) ability.
b) must have rained e) permission.
c) should have been warm
d) may be snowing 035 | UFSM 2000
e) ought to dry Research suggests the cause may lie internally, in terms of
abnormal biological functioning.
30 | UNESP 2001
Universities abroad may a more recent result – A melhor tradução para o segmento "may lie" é:
when the IELTS was taken a long time ago.
a) pode estar.
a) requires b) consegue revelar.
b) require c) parece encobrir.
c) to require d) deixa passar.
d) requiring e) permite enganar.
e) required
36 | MACKENZIE 1998 – ADAPTED
041 | UEL 2000
Your government check may not arrive, your insurance Andrew Parker, a researcher at the Australian Museum,
policies may have expired. discovered that a bevy of tropical beauties are capable of
harnessing solar power for murderous ends.
In the sentence above, "may have expired" has the meaning
of: – A expressão ARE CAPABLE OF significa o mesmo que:
a) permission. a) can.
b) real deduction. b) must.
c) unreal future. c) might.
d) uncertainty. d) should.
e) logical conclusion. e) will.
37 | UNESP 2002 042 | UEL 2000
Yoshinori Haga said that a great potential and The mummies may also provide anthropologists with new
that, among other reasons, toys used for knowledge about capac cocha, the lncas' ritual sacrifice of
entertainment. children.

a) there was – can be – O uso de MAY, na frase acima, indica que o autor:
b) there is – were
c) there were – couldn't be a) tem certeza do que afirma.
d) there was – could be b) evita fazer afirmações categóricas.
e) there wasn't – can't be c) pede permissão do leitor para fazer afirmações.
d) quer a confirmação para suas afirmações.
38 | FATEC 2002 – ADAPTED e) procura convencer o leitor sobre suas afirmações.
Assinale a alternativa que apresenta idéia equivalente a "it
may lead to more confusion": 43 | PUCSP 2001
Nas frases:
a) It may be due to more confusion.
b) It may be caused by more confusion. * Knowledge about the effects of DNA variations among
c) It may be the result of more confusion. individuals can lead to revolutionary new ways to diagnose,
d) It may result in more confusion. treat, and someday prevent the thousands of disorders that
e) It may result from more confusion. affect us.

39 | PUCMG 2001 * DNA sequences can lead to an understanding of their


The sentence "You can't avoid colds" means that: natural capabilities.

a) you don't have to do any exam. – A palavra CAN indica a idéia de:
b) you mustn't stay inside your home.
c) you can't keep away from colds. a) conhecimento.
d) you shouldn't be exposed to viruses. b) permissão.
c) habilidade.
040 | UFRS 2000 d) confirmação.
The verb which can be classified both as a FULL VERB and a e) probabilidade.
MODAL is:
44 | UFRRJ 1998
a) gets. In the sentence "MAY I ask why you do it?", the word that
b) turned. best replaces the capital word is:
c) dares.
d) envisions. a) should.
e) enjoy. b) ought to.
c) need.
d) can.
e) must.
45 | FATEC 1999
50 | PUCRIO 2003
Assinale a alternativa que apresenta o significado
correspondente mais próximo a "Sales may get a boost":

a) Sales are going to get a boost.


b) Sales are able to get a boost.
c) Sales will get a boost.
d) Sales must get a boost.
e) Sales might get a boost.

46 | MACKENZIE 2000
In which of the statements does the modal MUST express
necessity?

a) There's somebody in the other office. It must be my boss!


b) You mustn't smoke here!
c) She must be a very good student. She always gets A's.
d) I must go right now!
e) Who must he be? There are lots of people around him! – In "you should start each day with a song... in your soul",
"should" expresses an idea of:
47 | MACKENZIE 2000
In the sentence, "You may be wrong, but you may be right", a) certainty.
MAY means: b) obligation.
c) impossibility.
a) possibility. d) probability.
b) permission. e) advice.
c) ability.
d) deduction. 51 | PUCPR 2003
e) obligation. Match the columns below. Then mark the option which
provides the correct order of the second column:
48 | UFPEL 2000
A expressão "business may sound complicated and Coluna I
expensive" traz consigo a idéia de: (1) Shall I switch off
(2) Could I carry
a) permissão. (3) Can I take
b) possibilidade. (4) I'll help
c) certeza. (5) Would you like
d) habilidade.
e) obrigação. Coluna II
( ) you do the washing up.
49 | PUCMG 2004 ( ) your coat?
Recent advances in human embryology and genetic ( ) those bags for you?
engineering have raised the issue of how this knowledge ( ) something to drink? (
ought to be used, and it is now a matter of considerable ) the lights?
public concern and debate.
– The option that shows the correct order of the second
– The words ought to suggest: column is:
a) advice. a) 4 – 3 – 2 – 5 – 1
b) ability. b) 5 – 3 – 2 – 1 – 4
c) possibility. c) 4 – 2 – 3 – 1 – 5
d) prohibition. d) 4 – 3 – 2 – 1 – 5
e) 5 – 2 – 3 – 4 – 1
52 | MACKENZIE 2004
056 | UNESP 2005
A different and possible ending to the sentence "If I put my Indique a sentença que expressa um conselho:
shoes on the right feet" is:
a) When depressed, teens always ask for adult guidance.
a) ...I can have been consider a normal person.
b) Teens see more of what life has to offer and then they
b) ...I ought to be considered a normal person. become depressed.
c) ...I would have had considered like normal.
c) Adolescents who never make new friends become
d) ...I should had considered to be normal.
depressed.
e) ...I must have be considered a normal person.
d) Adolescents don't try to make new friends when they feel
depressed.
53 | UFRRJ 2004 e) When teens become depressed, they should try to ask an
In the sentence, "the casualties were fewer than might have adult for help.
occurred", the underlined word implies:
057 | UEL 2005
a) possibility.
b) permission.
c) necessity.
d) assumption.
e) capacity.

054 | UFRS 2004


Consider the following sentences:

I. The magic should come from another place.


II. It would be necessary for the magic to come from
elsewhere.
III. The magic had to come from any other place.

– Which of them means the same as the sentence THE


MAGIC WOULD HAVE TO COME FROM SOMEWHERE ELSE?

a) Only I.
b) Only II. Disponível em:
c) Only III. http://www.loc.gov/rr/print/list/listguid.html
d) Only I and III. Acesso em: 14 out. 2004.
e) Only II and III.
– Com base no texto, "ought to" é usado para indicar o que é
055 | ITA 2005 considerado:
Assinale a opção que contém as respectivas melhores
traduções para os verbos destacados nos trechos a seguir: a) Errado.
b) Correto.
– "But the extinct languages of which we have some c) Necessário.
historical record in this part of the world MUST be only a d) Improvável.
fraction of those for which we have nothing." e) Obrigatório.
– "It is easy to see that no sensible estimate CAN be
obtained about the rate at which languages have died in the 58 | PUCRS 2005
past." O verbo da oração principal na sentença "People faced with
– "We CAN of course make guesses at the size of the mental health concerns can find it difficult to get facts about
population in previous eras, and the likely size of symptoms and treatments, the services that are available to
communities, and work out possible numbers of languages." them, or even to find someone who is prepared to listen" é:

a) devem; pode; pode. a) faced.


b) devem; pode; podem. b) concerns.
c) devem; pode; podemos. c) can find.
d) deve; podem; pode. d) get.
e) deve; podem; podemos. e) are.
59 | PUCRIO 2006
62 | PUCRIO 2007
In the sentence "One reason TV can become instructive,
In "This study appears to show us that the use of energy
when parents explain why children should not copy what drinks might predispose people to abuse alcohol", might can
they saw", the word can expresses the idea of:
be correctly substituted by:
a) ability. a) must.
b) request.
b) shall.
c) inability. c) had to.
d) permission.
d) could.
e) possibility.
e) ought to.
60 | MACKENZIE 2005 63 | PUCRIO 2007
In "It could be a plane crashing into the World Trade
Center", could can be correctly substituted by:

a) might.
b) must.
c) had to.
d) shall.
e) ought to.

64 | PUCRS 2008
The clause "we are now able to fly" can be substituted,
without a change in meaning, by "we fly now".

a) are going to
– "Don't have to" and "must not" indicate: b) ought to
c) should
a) absence and lack of obligation. d) could
b) required permission and necessity. e) can
c) lack of necessity and prohibition.
d) no choice and permission. 65 | UNIRIO 1998
e) requirements and power. When your emotions are riding high, "the ever-changing
clouds and colors of the sky are a reassuring reminder that
61 | PUCRIO 2006 your own state of mind is temporary. It's a relief to
The only option in which the detached expression transmits remember that, This too shall pass.
an idea of obligation/necessity is:
– The word SHALL in "This too shall pass" conveys the
a) An email distribution list on Star Trek MAY HAVE close to meaning of:
one hundred members.
b) The communication which takes place there COULD BE a) certainty.
either one-way or merely informational. b) likelihood.
c) Users MUST BE invited to join the community by someone c) possibility.
already there. d) suggestion.
d) Similar to Friendster, Orkut goes a step further BY e) expectation.
PERMITTING 'communities of users'.
e) InCircle WAS INTENDED for use by former university 66 | EFOMM 2008
students. The invention of the elevator by Elinsha Gray transformed
architecture. If Elinsha hadn’t invented the elevator,
skyscrapers have been built.

a) mustn’t
b) might
c) should
d) can
e) wouldn’t
67 | EFOMM 2008
71 | UFPE 1998 – ADAPTED
- “How come Bob argued with the waiter?”
"Must" in the sentence "Why the monarchy must stay" is
- “The food awful.” equivalent to:
a) had better be
(1) could
b) should have been (2) ought to
c) can be (3) is obliged to
d) must have been
(4) should
e) ought to be (5) has to
68 | PUCRIO 2008 – The correct sequence is:
Mark the only alternative that contains a correct
correspondence between the verb form in capital letters and
a) F V V V V
its meaning:
b) F V F V V
c) F V V V F
a) Today the world faces what MIGHT be called a 'clash of d) V F V V V
emotions' as well. – Obligation
e) V V V F V
b) There are some areas that SEEM to display all of them
simultaneously. – Necessity
72 | UFPE 1998 – ADAPTED
c) The first priority for the West SHOULD be to recognize the "MAY" in "Blacks with different tribal tongues MAY have
nature of the threat. – Advice
been forced to create this common black vernacular,"
d) It is a war nonetheless and one that the West CAN lose. – indicates:
Permission
e) But it MUST find a solution to the Palestinian problem (1) possibility.
first. – Ability
(2) probability.
(3) certainty.
069 | JFS 2007
(4) obligation.
In the sentences below: (5) permission.
* I might come and visit you in America next year, if I can
– The correct sequence is:
save enough money.
* We oughtn't to have agreed without knowing what it a) V V F F V
would cost. b) V V F V F
* When you got lost in the forest you must have been very c) V F F V F
frightened. d) V V F F F
e) F V V F F
– The Modals MIGHT, OUGHT and MUST express,
respectively:
73 | EFOMM 2010
a) possibility, advice and obligation
"People must be aware of the consequences of their actions.
b) possibility, prohibition and probability
One can do whatever he pleases as long as he doesn't do
c) possibility, advice and probability. harm to others. This may not be followed by many people,
d) capacity, advice and probability but it certainly should. If a person has many friends, he must
e) capacity, prohibition and advice know this already."
070 | JFS 2007 – The underlined modal verbs express:
Fill in the following sentence correctly:
a) advice / permission / permission / advice / obligation
In my opinion, Marla study harder. She b) advice / ability / permission / ability / obligation
be approved, but she improve. c) obligation / permission / possibility / advice / deduction
d) obligation / ability / possibility / obligation / deduction
a) can – could – ought e) deduction / ability / permission / obligation / obligation
b) should – can – must
c) ought to – should – can
d) should – must – cannot
e) ought – can – must
074 | AFA 2005
Active and Passive Voice
1 | UNITAU 1995
Assinale a alternativa que corresponde à forma ativa da
frase a seguir:

A detailed description (...) is given.

a) The authors give a detailed description.


b) The authors will give a detailed description.
c) The authors have given a detailed description.
d) The authors gave a detailed description.
e) The authors will have given a detailed description.

2 | UNITAU 1995
“I don’t care what planet you’re from, you can’t run around Assinale a alternativa que corresponde à voz ativa da
Earth stark naked!” sentença a seguir:

– Which sentence has the same idea as the modal can’t The 1994 cup was hosted by the United States.
in the sentence above?
a) The United States hosted the 1994 Cup.
a) You don’t have to run around Earth… b) The United States will host the 1994 Cup.
b) You needn’t run around Earth… c) The United States have hosted the 1994 Cup.
c) You mustn’t run around Earth… d) The United States had hosted the 1994 Cup.
d) You aren’t able to run around Earth… e) The United States will have hosted the 1994 Cup.

075 | JFS 2012 3 | UNITAU 1995


Assinale a alternativa que corresponde à voz passiva da frase
Match the following sentences with the best meaning a seguir:
expressed by the modal verb in each of them.
Future generations may regard the scientific indictment of
I. She can speak four languages. smoking as a major contribution to preventive medicine and
II. I wonder if I might have a quick look at your newspaper. the health of the western world.
III. You should not talk to your brother this way.
a) Future generations will be regarded by the scientific
IV. Luggage must not be left unattended.
indictment of smoking as | ...
b) The scientific indictment of smoking has been regarded by
a) ability / possibility / advice / prohibition
future generations as | ...
b) capacity / possibility / suggestion / obligation
c) The scientific indictment of smoking may be regarded by
c) ability / permission / advice / probability
future generations as | ...
d) capacity / permission / suggestion / necessity
d) The scientific future generations may be regard by
e) ability / permission / advice / prohibition
smoking as | ...
e) Future generations of smoking will be regarded by the
indictment as | ...
“So high as a tree aspires to grow,
so high will it find an atmosphere suited to it.”
4 | UNITAU 1995
Henry Thoreau
Assinale a alternativa que corresponde à forma passiva da
sentença a seguir:

We encourage the kids to go swimming.

a) The kids were encouraged to go swimming.


b) The kids have been encouraged to go swimming.
c) The kids will be encouraged to go swimming.
d) The kids may be encouraged to go swimming.
e) The kids are encouraged to go swimming.
5 | CESGRANRIO 1994
010 | FAAP 1996
Mark the sentence below which is NOT in the passive voice: The passive form of the sentence "The International Court of
Hague rejected an attempt by New Zealand to stop further
a) A revolutionary telephone system was unveiled. French nuclear tests in the South Pacific" is:
b) A sophisticated computer was programmed.
c) It was instructed to translate "out of sight, out of mind". a) An attempt by New Zealand to stop further French
d) The Russian translation was then fed into the computer. nuclear tests in the South Pacific have been rejected by The
e) A computer will invariably have difficulty in making sense International Court of Justice in Hague
of it. b) An attempt by New Zealand to stop further French
nuclear tests in the South Pacific would be rejected by The
6 | UNIRIO 1995 International Court of Justice in Hague
The PASSIVE construction equivalent to "in addition, c) An attempt by New Zealand to stop further French nuclear
Frogwear absorbs very little water" is: tests in the South Pacific was rejected by The International
Court of Justice in Hague
a) In addition, very little water is absorbed by Frogwear. d) The International Court of Justice has rejected an attempt
b) In addition, very little water can absorb by Frogwear. by New Zealand to stop further French nuclear tests in the
c) In addition, very little water would be absorbed by South Pacific.
Frogwear. e) An attempt by New Zealand to stop further French
d) In addition, very little water has been absorbed by nuclear tests in the South Pacific is being rejected by The
Frogwear. International Court of Justice in Hague
e) In addition, very little water was absorbed by Frogwear.
11 | MACKENZIE 1996
007 | FEI 1995 Change the following sentence to the Passive Voice:
Leia as sentenças a seguir e marque a que está na VOZ
PASSIVA: Somebody left the lights on all night.

a) We were experimenting with the use of sound waves. a) All night somebody left the lights.
b) It was a very low-tech start. b) The lights are left on all night.
c) Madonna has now been replaced by high energy waves. c) The lights didn't leave on all night.
d) All glass is weak because it cracks. d) The lights were left on all night.
e) Glass products have microscopic cracks in them. e) The lights was left on all night.

8 | UNESP 1991 12 | MACKENZIE 1996


Assinale a alternativa que preenche corretamente a lacuna Change the following sentence to the Passive Voice:
da frase a seguir:
You don't need to wind this wonderful watch.
Those people are happy because they love in
their childhood. a) This wonderful watch isn't needed to be wind.
b) This wonderful watch doesn't need to winded.
a) was given c) This wonderful watch doesn't need to be wound.
b) has given d) This wonderful watch don't need to be wounded.
c) were given e) You don't need to be wounded by this wonderful watch.
d) have being given
e) be given 13 | MACKENZIE 1996
A voz passiva de "Somebody must send me the new books"
9 | MACKENZIE 1996 é:
Change the following sentence to the Passive Voice:
a) I must send the new books.
They feed the seals twice a week. b) The new books must be sent to me.
c) I will be sent the new books.
a) The seals are fed twice a week. d) The new books would be sent to me.
b) The seals are feeded twice a week. e) The new books must be send by somebody.
c) The seals are found twice a week.
d) Twice a week they are feeding the seals.
e) The seals are being fed twice a week.
14 | MACKENZIE 1997
19 | CESGRANRIO 1991
The Passive Voice of "It is alleged that he stole a car" is: The sentence Young people today think that "Thank you",
"You're welcome" and "Excuse me" are servilities that must
a) He is alleged to have stolen a car. be avoided, is in the passive voice.
b) The car is alleged to have stolen by him.
c) It was alleged that the car is stolen by him. – Mark the option which does NOT have a verb in the
d) It is alleged that he has stolen a car. passive voice:
e) He alleged that the car was stolen by him.
a) The most famous model in the world was born in Brazil.
15 | MACKENZIE 1997 b) The same syndrome is reflected in some advertisements.
Change the following sentence to the Passive Voice: c) Children who haven't been raised in boarding schools are
happier.
Thousands of people ride the underground every morning. d) This piece of advertising was chosen among many others.
e) If she hadn't been taught how to model, she wouldn't be
a) The underground is ridded by thousands of people every famous now.
morning.
b) The underground was rode by thousands of people every 020 | UEL 1994
morning. Assinale a alternativa que preenche corretamente a lacuna
c) The underground is rode by thousands of people every da frase a seguir:
morning.
d) Thousands of people is ride by the underground every In many parts of the world, the future productivity of the soil
morning. by man's ill use of it.
e) The underground is ridden by thousands of people every
morning. a) endangers
b) endangered
16 | MACKENZIE 1997 c) will endanger
The Passive Voice of "Man has made the world much more d) are endangered
complex" is: e) is endangered
a) Much more complex has made the world. 21 | PUCPR 1997
b) The world was been made much more complex by man. Choose the correct form in the passive voice for Mr. Brown
c) Much more complex was the world made. gave us a lesson:
d) Complex has been made the world much more by man.
e) The world has been made much more complex. a) We had given a lesson.
b) A lesson will be given us by Mr. Brown.
17 | MACKENZIE 1997 c) A lesson is being given us by Mr. Brown.
Change the following sentence to the Passive Voice: d) We were giving a lesson.
e) We were given a lesson by Mr. Brown.
Did the army surround the city?
22 | UFRS 1997 – ADAPTED
a) Is the city surrounded by the army?
A correct active version of the underlined expression in
b) Was the city surround by the army?
Hammett came to the foot of the stairs and in a whisper
c) The city was surround by the army.
said, "Come down. Be very quiet. When you get to the last
d) Was the city surrounded by the army? few steps, crouch very low so that you can't be seen through
e) Was the army surrounded by the city? the window" is:
18 | MACKENZIE 1997
a) so that anything can't see you.
Change the following sentence to the Passive Voice:
b) so that nothing can't see you.
c) so that anybody can see you.
In 1945 the allied powers defeated Germany.
d) so that no one can see you.
e) so that none can't see you.
a) In 1945 Germany was defeated by the allied powers.
b) In 1945 Germany did defeated by the allied powers.
c) In 1945 Germany are defeated by the allied powers.
d) The allied powers were defeated by the Germany in 1945.
e) In 1945 Germany was defeat by the allied powers.
23 | CESGRANRIO 1999
28 | MACKENZIE 1999
All the following sentences have a verb in the passive voice, Our plan by the members of the committee.
EXCEPT one. Mark it:
a) will consider
a) More powerful tobacco health-warning labels may be b) has being considered
required. c) has considered
b) Where are the warnings that alcohol may lead to violence, d) have been considered
may cause death? e) is being considered
c) Government and public opinion seem to be content to
allow alcohol to be portrayed as a fun, benign substance. 29 | MACKENZIE 1999
d) With tobacco, the product has been demonized. Change the following sentence to the Passive Voice:
e) With alcohol, it is the user who has been held responsible.
Did the idea interest them?
024 | UFPB 1998
This sentence is in the PASSIVE VOICE: a) Were they interested in the idea?
b) Was the idea interest to them?
Women are warned by doctors. c) The idea was interested to them?
d) Were they interest in the idea?
– The ACTIVE VOICE is: e) Are they interested in the idea?

a) Doctors warn women. 30 | MACKENZIE 1999


b) Women warn doctors. Indicate the alternative that best completes the following
c) Doctors are warned by women. sentence:
d) Women are being warned.
e) Doctors warned women. Not about the accident since that
time.
25 | FUVEST 1999
Choose the correct active voice form for "They have been a) much – has said
widely praised": b) much – has been said
c) a lot – is said
a) The press had widely praised them. d) many – has said
b) People praised them widely. e) very much – has being said
c) One has widely praised them.
d) The press has widely praised them. 31 | UFRS 1998 – ADAPTED
e) People has widely praised them. Tess of the D'Urbervilles is the story of the seduction,
betrayal, and destruction of an innocent girl, Tess
26 | MACKENZIE 1998 Durbeyfield, who is led by her foolish parents into thinking
Indicate the alternative that best completes the following she comes from an ancient noble family, the D'Urbervilles.
sentence:
– The active form "is led by her foolish parents" is HER
When the manager arrived, the problem . FOOLISH PARENTS:

a) was been solved already. a) are lead.


b) should to be solved yet. b) have led.
c) had already been solved. c) lead.
d) has still been solved. d) leads.
e) had already solved. e) led.

27 | MACKENZIE 1999 32 | UNESP 2001


He responsible for the accident. As the IELTS tests all four skills, it worldwide to
assess proficiency in English.
a) was holding
b) has been holing a) is
c) held b) has used
d) was held c) had been used
e) would hold d) has been using
e) has been used
33 | FUVEST 2001 – ADAPTED
37 | FATEC 2000
At the moment, so-called genetically modified (GM) crops
Assinale a alternativa em que a voz do verbo (voz passiva) é
are in disgrace. Consumers, particularly in Europe, are wary a mesma empregada em THE PESTICIDE HAS BEEN BANNED:
of buying food that may contain them. Environmental
activists are ripping up fields where they are being tested a) The chemicals are still a serious threat everywhere.
experimentally. And companies that design them are selling b) It's so effective in controlling mosquitoes that carry the
off their GM subsidiaries, or even themselves, to anyone malaria parasite.
willing to take on the risk. c) Small amount of pesticide can disrupt the working of
human hormones.
The Economist, July 1st 2000
d) Some of the highest concentrations of DDT are found in
– Choose the correct ACTIVE VOICE FORM for "fields where polar bears, penguins.
they are being tested experimentally": e) They condense and fall to the ground in cold weather.

a) fields where scientists have been testing them 038 | FEI 2000
experimentally "He was elected". Indique a forma ativa:
b) fields where environmentalists are testing them
experimentally a) He elects.
c) fields where genetic engineers had been testing them b) He elected.
experimentally c) He had elected.
d) fields where genetic engineers are testing them d) He has elected.
experimentally e) He is electing.
e) fields where one has been testing them experimentally
39 | UNESP 2002
34 | PUCRS 2001 Indique a alternativa que expressa o mesmo significado de:
The correct active voice for the sentence "20 billion were
spent on diet products and services" is "People Japanese toymakers now see senior citizens as their most
20 billion on diet products and services". dynamic market.

a) spent a) Senior citizens are now seen as their most dynamic


b) have spent market by Japanese toymakers.
c) will spend b) Senior citizens were seen as the Japanese toymakers'
d) had spent most dynamic market.
e) spend c) Senior citizens' most dynamic market is seen as the
Japanese toymakers.
35 | PUCRS 2002 d) Senior citizens and Japanese toymakers are seen as the
The correct active voice of "The Athena guidance is now most dynamic market.
being negotiated with other groups" is: e) Senior citizens are seen as Japanese toymakers by their
most dynamic market.
a) They are now negotiating the Athena guidance with other
groups. 40 | UNESP 2003
b) The Athena guidance with other groups is now being Indique a alternativa que expressa o mesmo significado de:
negotiated.
c) Other groups are now negotiating the Athena guidance. When children watch TV, they encounter a wide range of
d) The Athena guidance is now negotiating with other places, people, and information.
groups.
e) They have been negotiating the Athena guidance. a) When children watch TV, a wide range of places, people,
and information will be encounter.
36 | UFRRJ 1998 b) When children watch TV, a wide range of places, people,
The sentence "it keeps the elephants away", becomes in the and information are encountered.
Passive Voice: c) When children watch TV, a wide range of places, people,
and information is encountered.
a) "in keeping with the elephants away". d) When TV was watched, a wide range of places, people,
b) "the elephants are keeping the way". and information are encountered by children.
c) "the elephants are kept away". e) A wide range of places, people, and information will be
d) "the elephants is kept away". encountered when children watched TV.
e) "the elephants keep the way".
41 | PUCRS 1999
46 | FATEC 2003
The correct passive form of "This town of 1,500 has acquired
Assinale a alternativa que apresenta a voz ativa correta da
a unique status" is "A unique status: frase "Few of these businesses are run by corporations":
a) was acquired by this town of 1,500."
a) Corporations ran few of these businesses.
b) has been acquired by this town of 1,500."
b) Corporations run few of these businesses.
c) have been acquired by this town of 1,500."
c) Corporations are ran by few of these businesses.
d) was being acquired by this town of 1,500."
d) Corporations were run by few of these businesses.
e) had been acquired by this town of 1,500."
e) Corporations have run few of these businesses.
42 | PUCRS 2000
047 | UFRS 2001
The correct Active Voice for "Most of the product examples
The sentence "Lucifer is ordered to obey the Son of God"
were found in word problems in books" is "People:
means the same as:
a) will have found most of the product examples in word
problems in books." a) The Son of God orders Lucifer to obey Him.
b) found most of the product examples in word problems in b) Lucifer orders the Son of God to obey him.
books." c) God orders His Son to obey Lucifer.
c) will be finding most of the product examples in word d) Lucifer obeys the orders of the Son of God.
problems in books." e) Someone orders Lucifer to obey the Son of God.
d) had been found most of the product examples in word
problems in books." 48 | MACKENZIE 1998
e) had found most of the product examples in word Indicate the alternative that best completes the following
problems in books." sentence:

43 | UFRRJ 2000 A small number of visitors to come to the


Another way of saying "I was shocked by their intensity" is: meeting.

a) I shocked the with the intensity. a) are expecting


b) Their intensity shocked me. b) are expected
c) Shocking them was intense. c) will expect
d) Their intensity was shocked by me. d) have expected
e) Their intensity was shocking me. e) is expected

44 | FATEC 2002 49 | MACKENZIE 1998


Assinale a alternativa que corresponde à voz ativa da frase cheats on the test sooner or later.
the young Baartman was lured away em "In 1810, the
young Baartman was lured away from her Khoisan kin in a) Whomever – will caught
South Africa": b) Whatever – will catch
c) Whichever – will have caught
a) they lured the young Baartman away. d) Wherever – will be catching
b) they lure away the young Baartman. e) Whoever – will be caught
c) they have lured away the young Baartman.
d) they had lured the young Baartman away. 50 | MACKENZIE 2003
e) they were lured the young Baartman away. The sentence "He was told to take memantine with his
regular pills" in the active voice will be:
45 | FATEC 2003
Assinale a alternativa que apresenta a voz ativa correta da a) He said his regular pills were taken with memantine.
frase destacada em "EPHEDRA HAS BEEN LINKED TO A b) If he takes memantine with his regular pills, said the
NUMBER OF STROKES, heart attacks and seizures and more doctor, he will be cured.
than 100 deaths": c) Someone said the pills that he took were taken with
memantine.
a) They linked ephedra to a number of strokes. d) He must take memantine with his regular pills, have said
b) A number of strokes have been linked to ephedra. the doctor.
c) They have linked ephedra to a number of strokes. e) The doctor said that he should take memantine with his
d) A number of strokes has been linked to ephedra. regular pills.
e) They had been linked ephedra to a number of strokes.
51 | MACKENZIE 2003
55 | PUCRS 2004
The sentence "Nair has produced a readable work that
A voz ativa correspondente a "The pulsations in a red giant's
questions some modern assumptions" in the passive voice luminosity are caused by dramatic fluctuations in the star's
would be:
temperature" é:
a) A readable work that questions some modern
a) Dramatic fluctuations in the star's temperature caused the
assumptions has been produced by Nair.
pulsations in a red giant's luminosity.
b) A readable work has been produced by questions about
b) The star's temperature cause the dramatic fluctuations in
modern assumptions by Nair. a red giant's luminosity.
c) Some modern assumptions are questioned by a readable c) The red giant's luminosity and the star's temperature
work produced by Nair. caused the dramatic fluctuations on the stars.
d) Questions that have been produced by Nair have been d) Dramatic fluctuations in the star's temperature cause the
worked by readable modern assumptions. pulsations in a red giant's luminosity.
e) Modern assumptions that have been worked by Nair are
e) The star's temperature have caused dramatic fluctuations
producing readable modern questions.
in a red giant's luminosity.
52 | MACKENZIE 2004 56 | UNESP 2004
The sentence "Their longstanding authority over the City
Indique a alternativa que expressa o mesmo significado de:
development had never been seriously challenged" in the
active voice will be: More than 5 million Americans are affected by serious and
often life-threatening eating disorders.
a) Their longstanding authority had never challenged
seriously the City development. a) Serious and often life-threatening eating disorders affect
b) Nobody had ever seriously challenged their longstanding more than 5 million Americans.
authority over the City development. b) Serious and often life-threatening eating disorders
c) The City development had never been challenged affected more than 5 million Americans.
seriously by their longstanding authority. c) More than 5 million Americans affect serious and often
d) The development over the City's longstanding authority life-threatening eating disorders.
had ever been challenged. d) More than 5 millions serious and life-threatening eating
e) Seriously had the City development ever been challenged disorders often affect Americans.
by their longstanding authority. e) Serious and life-threatening eating disorders have been
often affected by more than 5 million Americans.
53 | FATEC 2004
Many sorts of work by different groups of 057 | UFRS 2004
economists. Select the correct alternative to complete the sentence
below:
a) are been accomplished
b) was accomplished The active version of the sentence The ghost has been seen
c) have been accomplished by several living members of my family is the sentence
d) were accomplish SEVERAL LIVING MEMBERS OF MY FAMILY .
e) has been accomplished
a) saw the ghost
54 | FATEC 2004 b) had seen the ghost
Considere a frase "Still, it offers no protection from biological c) were seeing the ghost
or chemical weapons". d) have seen the ghost
e) were to see the ghost
– Assinale a alternativa em que a transposição dessa frase
para a voz passiva está correta, completando a frase abaixo: 058 | UFSM 2004
Em "The proposal established by France
Still, no protection from biological or chemical weapons May 1950", assinale a alternativa que completa
. as lacunas:

a) are offered a) will – in


b) is offered b) was – in
c) was offered c) would – at
d) were offered d) is – at
e) have been offered e) were – on
59 | UFV 2004 – ADAPTED
62 | PUCCAMP 2005 – ADAPTED
Choose the alternative which presents BOTH examples in the O significado da sentença "Não se pergunta a um atleta da
passive voice: Irlanda se é católico ou protestante" corresponde, em inglês,
a:
a) The distinction between individualistic media use and
social activities such as chatting with friends is less extreme a) It is not asked an Irish athlete if he is Catholic or
than "is commonly assumed"./ Only 1 child in 100 "can be Protestant.
classed" as a real screen addict. b) An Irish athlete is not asked if he is Catholic or Protestant.
b) Increasing prosperity "has also contributed" to the rise of c) Do not ask an Irish athlete if he is Catholic or Protestant.
the bedroom culture./ British teenagers "have always d) One should not ask an Irish athlete if he is Catholic or
retreated" to their bedrooms. Protestant.
c) The distinction between individualistic media use and e) No asking an Irish athlete if he is Catholic or Protestant.
social activities such as chatting with friends is less extreme
than "is commonly assumed"./ Increasing prosperity "has 63 | UNESP 2005
also contributed" to the rise of the bedroom culture. Indique a alternativa que expressa o mesmo significado de:
d) Only 1 child in 100 "can be classed" as a real screen
addict./ British teenagers "have always retreated" to their Depression is defined by doctors as an illness that affects the
bedrooms. ability to function.
e) The distinction between individualistic media use and
social activities such as chatting with friends is less extreme a) Doctors had defined depression as an illness that affects
than "is commonly assumed"./ British teenagers "have the ability to function.
always retreated" to their bedrooms. b) Doctors define depression as an illness that affects the
ability to function.
60 | UFV 2005 – ADAPTED c) Doctors would define depression as an illness that affects
Choose the alternative in which BOTH verbal forms are the ability to function.
examples of the passive voice: d) Doctors are defining depression as an illness that affects
the ability to function.
a) Discovered in the early 1950s, the Yanomami "were left e) Doctors are used to defining depression as an illness that
alone" for much of the next three decades./ At least two affects the ability to function.
thousand Yanomami have been massacred or "have died" of
epidemics of measles, tuberculosis, and hepatitis. 64 | UNESP 2005
b) Discovered in the early 1950s, the Yanomami "were left Indique a alternativa que expressa o mesmo significado de:
alone" for much of the next three decades./ At least two
thousand Yanomami "have been massacred" or have died of Adults expect teens to act moody.
epidemics of measles, tuberculosis, and hepatitis.
a) Teens expected adults to act moody.
c) I "could scarcely have found" a friendlier people./ At least
b) Adults are expected by teens to act moody.
two thousand Yanomami "have been massacred" or have
c) Adults and teens are expected to act moody.
died of epidemics of measles, tuberculosis, and hepatitis.
d) Teens are expected to act moody.
d) I "could scarcely have found" a friendlier people./ At least
e) Teens always act moody, although it is never expected by
two thousand Yanomami have been massacred or "have
adults.
died" of epidemics of measles, tuberculosis, and hepatitis.
e) I "could scarcely have found" a friendlier people./
65 | FATEC 2005
Discovered in the early 1950s, the Yanomami "were left
Assinale a alternativa que apresente a forma correta da voz
alone" for much of the next three decades.
passiva da seguinte frase:
61 | UFRRJ 2005 The decline of Education threatens our future as a nation
The sentence "People are scared by this proposition" is and as a people.
equivalent to:
a) Our future is threatened by the nation and the
a) This proposition scared people. Education's decline.
b) This proposition scares people. b) Our future was threatened by the decline of Education as
c) This proposition has scared people. a nation and as a people.
d) This proposition had scared people. c) Our future is to be threatened by the people as a nation.
e) This proposition is scaring people. d) Our future as a nation and as a people is threatened by
the decline of Education.
e) A nation and a people are threatening the Education's
decline.
66 | FUVEST 2006
070 | ITA 2006
Choose the correct passive voice form for:

No one has made any attempt to tackle the issue.

a) No attempt has been made to tackle the issue.


b) No attempt is made by anybody to tackle the issue.
c) It could not be made any attempt to tackle the issue.
d) It is not made any attempt to tackle the issue.
e) No attempt was made by anybody to tackle the issue.

67 | MACKENZIE 2004
The sentence "E.M.D.R. helps victims of trauma reprocess
disturbing thoughts and memories" in the passive voice will
be:

a) Victims of trauma are helped to reprocess disturbing – "I was dragged", no início do quinto quadrinho, significa:
thoughts and memories by E.M.D.R.
b) Disturbing thoughts and memories are reprocessed by a) Fui surpreendido.
victims of trauma which are helped by E.M.D.R. b) Fui arrastado.
c) Victims of trauma are reprocessed disturbing thoughts c) Fui capturado.
and memories by the help of E.M.D.R. d) Fui exposto.
d) Disturbing thoughts and memories are helped to e) Fui atirado.
reprocess victims of trauma by E.M.D.R.
e) E.M.D.R. is helped to reprocess disturbing thoughts and 71 | UFPE 2006 – ADAPTED
memories by victims of trauma. The phrase "The carbon in biofuels emissions has simply
been captured from the atmosphere by crops" has, as its
68 | MACKENZIE 2005 active counterpart:
The sentence "She counsels them to give 'urgent priority' to
finding a marriage partner fast" in the passive voice will be: a) Crops are simply capturing the carbon in biofuels
emissions from the atmosphere.
a) Finding a marriage partner fast and give 'urgent priority' b) Crops simply capture the carbon in biofuels emissions
to them is counselled by her. from the atmosphere.
b) 'Urgent priority' to finding a marriage partner fast is c) Crops simply captured the carbon in biofuels emissions
counselled by them. from the atmosphere.
c) To give 'urgent priority' to finding a marriage partner fast d) Crops have simply captured the carbon in biofuels
was counselled by her to them.
emissions from the atmosphere.
d) She counselled them to be given 'urgent priority' to e) Crops had simply captured the carbon in biofuels
finding a marriage partner fast. emissions from the atmosphere.
e) They are counselled to give 'urgent priority' to finding a
marriage partner fast. 72 | PUCRIO 2006
The passive voice is used in "Orkut was quietly launched on
69 | MACKENZIE 2005 January 22, 2004".
The sentence "Daniel L. Schacter explores the memory
miscues that occur in everyday life" in the passive voice will – Now, find the sentence that is also in the passive voice:
be:
a) Communities have never rejected new members.
a) The memory miscues that occurs in everyday life is
b) Good ideas took shape at the end of the session.
explored by Daniel L. Schacter.
c) Some communities have been able to control their
b) The memory miscues that occur are explored by everyday
growth.
life.
d) Several social groups could be connected by the Internet.
c) In everyday life the memory miscues have explored by
e) Young students are never tired of chatting with friends on
Daniel L. Schacter.
email.
d) The memory miscues that occur in everyday life are
explored by Daniel L. Schacter.
e) Life that occurred everyday by memory miscues explores
by Daniel L. Schacter.
73 | PUCPR 2007
077 | UECE 2008
"The seeds will be eaten by the birds" is the passive voice The sentences: "critics have attached importance to the
for: ethical purpose of literature" and "a textbook is written in
continuous prose" are respectively in the:
a) The birds will eat the seeds.
b) The birds ate the seeds. a) passive voice and active voice
c) The birds will be eaten by the seeds. b) active voice and passive voice
d) The seeds will eat the birds. c) passive voice and passive voice
e) The birds are going to eat the seeds. d) active voice and active voice
74 | MACKENZIE 2007 78 | UNESP 2003
The sentence "In English medium schools in particular a low The text a study in which 100 preschool
level of English may impede students' acquisition of children both before and after watching TV.
knowledge" in the passive voice would be:
a) reported – is observed
a) A low level of English in English medium schools in b) reports – observed
particular may have impeded students' acquisition of c) reported – had been observed
knowledge.
d) had reported – were observed
b) Students' acquisition of knowledge may impeded in
e) reports – had observed
English medium schools in particular by a low level of
English.
79 | MACKENZIE 1998
c) Students' acquisition of knowledge might impeded in
Mark the option that best completes the following sentence:
English medium schools in particular by a low level of
English.
A prize to whoever solves this equation.
d) In English medium schools in particular students'
acquisition of knowledge may be impeded by a low level of a) has given
English. b) should give
e) In English medium schools students' acquisition of c) is giving
knowledge might have been impeded by a low level of d) will be given
English in particular. e) must have given
75 | MACKENZIE 2008 80 | MACKENZIE 2005
Choose the correct alternative: Reading about Peter Jackson is sheer fun. But why (I) the
movies he (II) before "Lord of the Rings" never (III)?
a) This book chronicles the events. – Immediate Future
b) The book was released. – Passive Voice – The alternative that contains the verbs which complete
c) Deathly Hallows broke sales records. – Past Progressive blanks I, II and III in their appropriate tense is:
d) The previous record had been held. – Present Perfect
e) Deathly Hallows is published. – Present Progressive a) are – made – mentioned
b) have – has made – to be mentioned
076 | UECE 2007 c) are – being made – been mentioned
In "It is a world that had been created without thought of d) have been – has made – mentioned
him", "Steinbeck's description of this social injustice shocked e) are – is to be made – to be mentioned
the nation", and "In time, laws were passed to help people
like the Joads", the sentences are respectively in the: 081 | AFA 2007
Mark the option that means "Heating bills can be reduced
a) passive voice, active voice, active voice. when double glazing is installed":
b) passive voice, active voice, passive voice.
c) active voice, active voice, passive voice. a) When double glazing is installed heating bills can reduce
d) active voice, passive voice, passive voice. it.
b) Double glazing can reduce heating bills when it is
installed.
c) Double glazing is installed to reduce heating bills.
d) When you install double glazing you reduce heating bills.
082 | AFA 2007
86 | EFOMM 2006
Change the sentence below into passive voice:
In: "The invention of the automobile has changed American
life in several ways", the passive voice is:
Chlorates and nitrates usually provide oxygen for the
reaction.
a) “American life is being changed in several ways”.
b) “American life was being changed in several ways”.
a) Oxygen is provided for the reaction usually by chlorates c) “American life is changed in several ways”.
and nitrates.
d) “American life has been changed in several ways”.
b) The ones that usually provide oxygen for the reaction are e) “American life would be changed in several ways”.
chlorates and nitrates.
c) Oxygen for the reaction is usually provided by chlorates 87 | EFOMM 2007
and nitrates.
In: "This expedition will use a special Russian-owned ship",
d) Chlorates and nitrates are usually provided by oxygen for the passive voice is:
the reaction.
a) A special Russian-owned ship will use by this expedition.
083 | PUCPR 1998
b) A special Russian-owned ship would use by this
Choose the RIGHT alternative to complete the passage: expedition.
c) A special Russian-owned ship will be used by this
Everything ready for the party. The room expedition.
, the furniture . There d) A special Russian-owned ship would be used by this
bottles of wine and food on the table. A jazz record
expedition.
and the atmosphere was just right.
e) A special Russian-owned ship would used by this
expedition.
a) is – is clean – is moved – are – is playing
b) was – had been cleaned – had been moved – were – was 88 | EFOMM 2008
playing The problem discussed by the board of
c) had been – had been cleaned – had been moved – were – directors when it was proposed again by the supervisors.
had been played
d) was – had cleaned – had moved – was – had played a) had already
e) were – was cleaned – was moved – were – was playing b) is already
c) had already been
084 | EEAR 2007 d) has already
What’s the active voice for "The first roller skates were e) has already been
made in 1760 by Joseph Merlim"?
089 | UFRRJ 2003
Joseph Merlim the first skates in 1760. The sentence "Scientists think they have found what causes
people to sneeze" is equivalent to:
a) made
b) makes a) what causes people to sneeze is founded by scientists.
c) has made b) what causes people to sneeze were found by scientists.
d) was making c) what causes people to sneeze has been found by
scientists.
085 | EFOMM 2005 d) what causes people to sneeze was found by scientists.
So far the President . e) what are the causes of sneezing by people.

a) has not been elected 090 | FEI 1996


b) will never be elected I don't think the windows need cleaning. They don't need:
c) would be elected soon
d) had been elected a) to clean.
e) would have been elected b) to be clean.
c) to be cleaning.
d) to be cleaned.
e) to cleaning.
091 | UNESP 2006
96 | FUVEST 1979
Indique a alternativa que expressa o mesmo significado de: Assinale a alternativa que corresponde à frase:
It seems that some theories can't explain the origins of Preciso mandar fazer um terno para o casamento.
terrorism.
a) I must have a suit made for the wedding.
a) It seems that the origins of terrorism weren't explained by b) I have to have a suit done for the wedding.
all theories. c) I have to tell to do a suit for the marriage.
b) It seems that the origins of terrorism shouldn't be d) I need to order to make a suit for the wedding.
explained by theories. e) I must send to do a suit for the marriage.
c) It seems that all theories might be explained by the origins
of terrorism. 97 | MACKENZIE 1996
d) It seems that the origins of terrorism are explained by all Choose the correct alternative to complete the following
theories. sentence:
e) It seems that the origins of terrorism can't be explained by
some theories. Since I haven't got , I will .
092 | FASM 2000 a) enough time – have the cake made
Critics call the data misleading in the Passive Voice is: b) time enough – get someone to make the cake
c) enough time – bake the cake myself
a) Misleading is called data by critics. d) any time – make the cake
b) Data is called misleading by critics. e) time enough – ask somebody to bake the cake
c) Data misleading is called by critics.
d) Data are called misleading by critics. 98 | PUCCAMP 1994
e) Critics are called misleading by data. Assinale a letra correspondente à alternativa que preenche
corretamente as lacunas da frase apresentada:
093 | JFS 1999
Mark the correct Passive Voice of the following sentence: "Hi, Jane!"
"Hi, Susan. How have you been?"
The helicopter is dropping the food supplies. "Oh, just fine. What have you done? You look so different!"
"Me? Different? I don't know."
a) The food supplies is being dropped by the helicopter.
"Maybe it's your hair."
b) The food supplies are been dropped by the helicopter. "Oh, that maybe ."
c) The food supplies are being dropped by the helicopter. "It looks quite nice."
d) The food supplies are being droped by the helicopter.
"Thank you. Well, I've got to go. See you around."
e) The food supplies have been being dropped by the "See you. Bye."
helicopter.
a) I'm cutting my hair
94 | EFOMM 1997
b) I just cut your hair
Somebody opened the door. c) I'm going to have a haircut
d) You had a haircut
a) The door was opened.
e) I just had my hair cut
b) The door opens.
c) The door is open.
099 | JFS 2008
d) The door open.
Read the following dialogue:
e) The open door.
"This floor is very dirty, Ferdinand. It needs sweeping."
95 | EFOMM 2000 "Yes, sir. Don’t worry. I will tell someone to do it at once."
"She had been told about the meeting", the active voice is:
– The underlined sentence is equivalent in meaning to:
a) Nobody told her about the meeting.
b) Somebody had told her about the meeting.
a) I will clean it by myself.
c) Everybody would tell her about the meeting.
b) someone should have cleaned.
d) She had told somebody about the meeting.
c) I will have it done.
e) The meeting was told about her.
d) you should clean it.
e) I will get him to do it.
100 | JFS 2008
Fill in the blanks correctly: Direct and Indirect Speech
I. Paulson and Patrick hurt during the game 1 | FUVEST 1977
yesterday. Qual destas formas está correta se usada por uma pessoa
II. Chris a terrific necklace during her birthday que narra um acontecimento?
party last weekend.
III. Do you think I look prettier? I have had my nose a) He thought she has not known what she is going through;
. b) He thought she will never know what she will have to go
IV. It said that the price of oil will go again. through now;
V. Will the meeting at noon or after the coffee c) He thought she would never know what she had gone
break? through then;
d) He thinks she did not know what she was going through;
– Now, mark the correct sequence: e) He will think she did not know what she went through
then.
a) were got – was given – straightened – has been – be
realized 2 | FUVEST 1977
b) got – was given – straighten – had been – have been Qual é a forma indireta, correspondente à forma direta "The
realized teacher said, Are you sure you have all understood me"?
c) have got – were given – straightened – is – can be realized
d) got – was given – straightened – is – be realized a) The teacher said if you are sure you had understood him;
e) gotten – were given – straighten – was – be realized b) The teacher asked whether we were sure we did
understand him;
c) The teacher said if we all are sure we have understood
“To accomplish great things, we must not only act, him;
but also dream; not only plan, but also believe.” d) The teacher asked if they were sure they had all
Anatole France understood him;
e) The teacher asked them to be sure to understand him.

3 | FUVEST 1977 – ADAPTED


Complete o diálogo a seguir:

John: I congratulate you on your success.


Peter: Pardon me.
John: I said .

a) that I had congratulated him on his success.


b) that you congratulated me on your success.
c) that I congratulated him on your success.
d) that I congratulated you on your success.
e) that I had congratulated you on my success.

4 | FUVEST 1978
Assinale a alternativa que equivale ao seguinte:

Suddenly Peter said to me, "Are you hungry?"

a) Suddenly Peter said that I was hungry.


b) Suddenly Peter told me that I was hungry.
c) Suddenly Peter asked if he were hungry.
d) Suddenly Peter asked me whether I was hungry.
e) Peter informed me that he was hungry.
Change the sentences from 05 to 11 into the Reported
10 | MACKENZIE 1997
(Indirect) Speech:
Warren said to me, "I can't find my glasses in this room."
5 | MACKENZIE 1996
a) Warren told me that he couldn't found his glasses in that
Sally said to me, "Do you know what time it is?"
room.
b) Warren told me he couldn't find her glasses in these
a) Sally asked me if I knew what time it was. room.
b) Sally told me whether she knew what time it was.
c) Warren told me that he couldn't find his glasses in that
c) Sally asked me whether she know what time it is.
room.
d) Sally asked me if I know what time it is.
d) Warren told me that he can't find his glasses in those
e) Sally told me if I knew what time was it.
room.
e) Warren said to me that he could not found his glasses in
6 | MACKENZIE 1996
this room.
Jeff said to Meg, "You don't understand me."
11 | MACKENZIE 1997
a) Jeff told Meg she didn't understand him.
Helen said to Paul, "Is this a free country?"
b) Jeff asked Meg that her didn't understand herself.
c) Jeff told Meg that she didn't understood him.
a) Helen told Paul if this was a free country?
d) Jeff told Meg that he didn't understand her.
b) Helen asked Paul if that was a free country.
e) Jeff told Meg she did understand him. c) Helen asked Paul whether that is a free country.
d) Helen told Paul this is a free country.
7 | MACKENZIE 1996
e) Helen told Paul if that is a free country.
"Are there any messages for me?", said Helen.
12 | PUCPR 1996
a) Helen asked if there is any messages for her.
Choose the correct indirect form for:
b) Helen asked whether there were any messages for she.
c) Helen asked whether were there any messages for herself. Oliver said to her: "What will you do tomorrow?"
d) Helen asked if there were some messages for her.
e) Helen asked if there were any messages for herself.
a) He asked her what she would do the following day.
b) He told her what she would do the following day.
8 | MACKENZIE 1996 c) She wondered what he will do the next day.
The director said to the boys, "Behave yourselves." d) He wanted to know what he would do the following day.
e) She asked what she would do the next day.
a) The director asked the boys to behave yourselves.
b) The director told the boys to behave himself
013 | UFPB 1998
c) The director asked the boys to behave themselves.
Read this sentence:
d) The director told the boys to behave ourselves.
e) The director told the boys, "Behave themselves."
The doctor says: "I'm happy we found this trend toward
reduced risk."
9 | MACKENZIE 1996
The patient said to me, "How long have the doctors been – The INDIRECT SPEECH is:
operating her?"
He said that he this trend toward reduced risk.
a) The patient asked me how long the doctors had been
operating her. a) is happy they found.
b) The patient told me how long her had been operating by b) has been happy we have found.
the doctors. c) was happy they had found.
c) The patient asked me how long had the doctors been d) will be happy we will find.
operating her. e) would be happy they would find.
d) The patient told me whether have the doctors been
operating her.
e) The patient asked me how long had been the doctors
operating her.
014 | UFRS 1998
18 | MACKENZIE 2000
Considere a frase: If you don't feed your Tamagotchi, it will
The sentence They said, "Do parents know their kids?" in the
die. reported speech would be:
– Escolha a melhor opção para reescrevê-la, começando com
a) They said that did parents know their kids?
She told me that:
b) They asked that parents know their kids.
c) They said that parents knew the kids.
a) if I won't feed my Tamagotchi, it would die.
d) They argued that do parents know their kids.
b) if you didn't feed your Tamagotchi, it had died.
e) They asked if parents knew their kids.
c) if I didn't feed my Tamagotchi, it would have died.
d) if I didn't feed my Tamagotchi, it would die. 19 | MACKENZIE 2000
e) if you haven't fed your Tamagotchi, it will have died.
The sentence He said, "Can Asians think?" in the reported
speech would be:
15 | MACKENZIE 1998
A forma indireta de "Would you like to go out tonight?" é:
a) He asked can Asians think?
b) He said that Asians can think.
a) He asked her if she would have liked to go out tonight.
c) He asked that Asians could think.
b) He asked if would she like to go out that night. d) He asked if Asians could think.
c) He asked whether she'd like to go out that night.
e) He said that could Asians think.
d) He asked whether she had liked to go out that night.
e) He asked if she'd liked to go out tonight.
20 | PUCRIO 2000
The girl said to her parents: "Mom and Dad, the police were
16 | UNIRIO 2000 here while you were gone". If we turned this statement into
"How would you describe yourself?" is a direct question.
reported speech, we would have The girl said to her parents
Complete the sentence below with the appropriate indirect
that the police:
question form:
a) had been there while they had been gone.
Daniel Hart asked an African-American teenager:
b) had been here while they had gone.
c) have been there while they were gone.
a) how to describe it.
d) have been here while they would be gone.
b) how to describe yourself.
e) would have been there while they have been gone.
c) how he would describe yourself.
d) to describe himself.
21 | UNESP 2006
e) to describe yourself.
John Arquilla declared that the greatest advantage of the
internet stealth, and that terrorists
17 | PUCRS 2000
in an ocean of bits and bytes.
The correct INDIRECT STATEMENT for the sentence "I don't
think our children should be subjected to needless
a) was – swim
advertising" said Ms. Mazzoni is "Ms. Mazzoni said she:
b) is – swam
c) is – has swum
a) doesn't think their children should be subjected to
d) was – swam
needless advertising".
e) was – swimming
b) has not thought their children should be subjected to
needless advertising".
22 | FATEC 2002
c) would not think their children should be subjected to
Considere a frase "It's a perfect setup for heart disease and
needless advertising".
diabetes, says Stampfer". Assinale a alternativa em que a
d) will think their children should be subjected to needless
transposição dessa frase para o discurso indireto está
advertising".
correta, completando a frase a seguir:
e) thinks their children should be subjected to needless
advertising".
– Stampfer says:

a) it was a perfect setup for heart disease and diabetes.


b) it is a perfect setup for heart disease and diabetes.
c) it has been a perfect setup for heart disease and diabetes.
d) it had been a perfect setup for heart disease and diabetes.
e) it will be a perfect setup for heart disease and diabetes.
s

d
e

d
e

d

e
;

d

28 | UFPE 2006 – ADAPTED
31 | MACKENZIE 2006
The two-year old girl said: "Daddy, draw me a spider". The The sentence "Why has evolution burdened humans with
reported speech for this sentence is: such seemingly irrational passions?" in the reported speech
will be:
She requested him:
a) Fisher asked evolution why it had burdened humans with
a) that he draws her a spider. such seemingly irrational passions.
b) that he had drawn her a spider. b) Fisher asked why evolution had burdened humans with
c) if he had drawn her a spider. such seemingly irrational passions.
d) to draw her a spider. c) Fisher asked why had evolution been burdened humans
e) whether he should draw her a spider. with such seemingly irrational passions?
d) Fisher said that why had evolution burdened humans with
29 | UFMG 1995 – ADAPTED such seemingly irrational passions?
A wife is telling us what happened this morning. Complete e) Fisher asked that evolution has burdened humans with
the following text according to the comic strip below: such seemingly irrational passions.

32 | MACKENZIE 2007
The sentence Mr. Redstone said, "We don't think someone
who effectuates creative suicide and costs the company
revenue should be on the lot" in the reported speech would
be:

a) Mr. Redstone believed that they didn't think someone


who would effectuate creative suicide and cost the company
revenue should have been on the lot.
b) Mr. Redstone stated that we didn't think someone who
had effectuated creative suicide and costed the company
revenue should have been on the lot.
This morning, my husband stepped on our scale and shouted c) Mr. Redstone implied that they didn't think someone who
that (1) that much. He pointed at the machine and said it (2) effectuated creative suicide and costed the company
a dirty liar. revenue should have been on the lot.
d) Mr. Redstone affirmed that they hadn't thought someone
a) he didn’t weigh – was who had effectuated creative suicide and cost the company
b) he don’t weigh – was revenue should be on the lot.
c) he doesn’t weigh – were e) Mr. Redstone believed that they didn't think someone
d) he did weigh – were who effectuated creative suicide and cost the company
e) he hadn’t weigh – was revenue should be on the lot.

030 | JFS 2008 033 | AFA 2008


Complete the following sentences meaningfully: Mark the option which contains an indirect form to
complete the prophet’s idea in the following gap:
1. I told him:
2. I didn't know: The prophet in silence the secrets of the days
and the nights.
a) 1. what the homework was./2. what he meant.
b) 1. what was the homework./2. what he mean. a) said to the man whether his heart has known
c) 1. what was to be the homework./2. what did he mean. b) said to people’s hearts know
d) 1. what is the homework./2. what did he meant. c) told him: your heart knows
d) told them that their hearts knew
034 | UEL 1998
Transformando-se a fala de Sigrid Koch-Baumgarten em Conditionals
discurso indireto tem-se:
001 | UNESP 1991
As social scientists we are interested in trying to understand Assinale a alternativa que preenche corretamente a lacuna
the Diana phenomenon. da frase a seguir:

a) Sigrid Koch-Baumgarten said that as social scientists we If he put it this way, everybody with him.
are interested in trying to understand the Diana
phenomenon. a) would agree
b) Sigrid Koch-Baumgarten says that as social scientists we b) should have agreed
are interested in trying to understand the Diana c) will agree
phenomenon. d) would has agreed
c) Sigrid Koch-Baumgarten has said that as social scientists e) agreeded
they would be interested in trying to understand the Diana
phenomenon. 2 | ITA 1996 – ADAPTED
d) Sigrid Koch-Baumgarten said that as social scientists they just call our 24-Hour Card Replacement, and
were interested in trying to understand the Diana we'll have a new one in your hands usually by the end of the
phenomenon. next business day.
e) Sigrid Koch-Baumgarten says as social scientists they were
being interested in trying to understand the Diana – No texto acima omitiu-se uma oração. Preencha-a com a
phenomenon. opção que representa a melhor redação:

035 | EFOMM 2005 a) If you need a lost or stolen card replaced


Mark the correct option. Helen said: “Somebody must send b) If you need replace a lost or stolen card
me the new books!” She said that: c) If a lost or stolen card needs to replace
d) If it is needed to replace a lost or stolen card
a) somebody had to send her the new books. e) If a lost or stolen card needs to be replaced by us
b) somebody had sent her the new books.
c) the new books were sent to her. 3 | PUCCAMP 1994
d) she was going to receive the new books. Assinale a letra correspondente à alternativa que preenche
e) she must have sent the new books. corretamente as lacunas da frase apresentada:

- "Frederick, what's the matter with you? This is the third


“People die of fright and live of confidence.” assignment you haven't turned in!"
Henry Thoreau - "I know, Mr. Dwarf. I would have turned them in
but I've been extremely busy."
- "But that's no excuse. You must understand that I'll have to
fail you if you don't complete your requirements."
- "Yes, I know. I'll try to catch up."

a) when I have time


b) if I had time
c) if I had had time
d) if I will have time
e) when I would have time

004 | UEL 1995


If you don't go, very angry.

a) I feel
b) I am
c) I was
d) I'll be
e) I have been
005 | UFMG 1995
009 | ITA 1998
Leia a seguir o comentário publicado pela revista
Love Among the Laundry NEWSWEEK:

When Sally found a man's striped sock curled among her


clothes at the launderette she returned it to the tall dark "He had lots of German in him. Some Irish. But no Jew. I
young man with a shy smile. They met there every week for think that if he a little Jew he it
several months, then were seen no more. One of their out".
wedding presents had been a washing machine.
Singer Courtney Love,
(Molly Burnett) on the suicide of her rock-star husband, Kurt Cobain.

– If they had not got married, they would probably have: – As lacunas (I) e (II) do comentário anterior devem ser
preenchidas, respectivamente, por:
a) changed their dirty clothes.
b) lost their socks forever. a) I. had had, II. would have stuck
c) rented a washing machine. b) I. has had, II. would stick
d) returned to the launderette. c) I. have had, II. had had stuck
e) sold their striped socks. d) I. had had, II. had stuck
e) I. had, II. would stuck
6 | MACKENZIE 1996
Indicate the alternative that best completes the following 010 | UFRS 1996
sentence: The alternative that does not finish the sentence "If it rains
we..." correctly is:
If you had taken my advice, you .
a) couldn't go out.
a) would learned the lesson b) won't go out.
b) would have learnt the lesson c) mustn't go out.
c) should learned the lesson d) shouldn't go out.
d) would learn the lesson e) can't go out.
e) should understand the lesson
11 | UFRS 1997 – ADAPTED
7 | CESGRANRIO 1991 A frase sublinhada em "if you are otherwise healthy, just call
Mark the item that shows the correct ending to the your doctor" seria melhor traduzida como:
following sentence:
a) se você não é saudável de outras maneiras.
If the process happens each time we eat sugar, we: b) se você tiver pouca saúde.
c) se você for saudável e esperto.
a) will have dental problems. d) se você não tem outras doenças.
b) would have dental problems. e) se você tem um plano de saúde.
c) would have had dental problems.
d) could have dental problems. 12 | UNESP 2001
e) may have had dental problems. Candidates who get a poor result always regret:

008 | ITA 1997 If I had studied more before sitting for the test, I
Lady Astor MP: "If you (I) my husband I (II) poison your it.
coffee".
Churchill: "If you (III) my wife I (IV) drink it." a) would pass
b) passed
– Os termos que melhor preenchem as lacunas I, II, III e IV c) have passed
são: d) would have passed
e) had passed
a) were (I), would (II), were (III), had (IV).
b) was (I), would (II), was (III), would (IV).
c) were (I), had (II), were (III), had (IV).
d) was (I), could (II), was (III), would (IV).
e) were (I), would (II), were (III), would (IV).
013 | UFC 2001
016 | MACKENZIE 2002
If I my raincoat, I a cold.
Amelia's Letter
a) had worn – wouldn't have gotten
Dear Members of the UNICEF, b) wear – would have get
c) didn't wear – wouldn't have get
I am an eleven-year-old girl and come from a small village. I
d) am wearing – would have gotten
left home to come to the city and to work to send money to
e) hadn't wear – couldn't have get
my family. Now I make T-shirts in a factory. I work twelve
hours a day for very little money. The factory is very dirty
017 | FUVEST 2003 – ADAPTED
and hot. The boss is very mean and often beats us. He makes
Diana had been hoping to get away by five, so she could be
us work very hard without breaks. My friends and I want to
at the farm in time for dinner. She tried not to show her true
leave but we know that working in the factory is better than
feelings when at 4.37 her deputy, Phil Haskins, presented
begging in the streets. The boss tells us this every single
her with a complex twelve-page document that required the
day...
signature of a director before it could be sent out the client.
Could you please tell us what to do?
Haskins didn't hesitate to remind her that they had lost two
Adapted from: "Voices of Youth": similar contracts that week.
http://www.unicef.org/voy/meeting/rig/casestud.html
– Choose the item which best completes the sentence,
– Choose the option that correctly completes the sentence: according to the passage:

If Amelia , she . Diana wouldn't be at the farm in time for dinner unless she
by five.
a) had left her family behind – could have gotten a job
b) hadn't come from a small village – wouldn't be eleven a) would get away
years old b) gets away
c) worked less than 12 hours a day – wouldn't be beaten by c) got away
her boss d) had got away
d) hadn't gotten a job in a factory – might have worked with e) can get away
her mean boss
018 | ITA 1997
e) weren't afraid of ending up begging in the streets – would
have already left the factory New technologies allow total strangers to know almost
everything about a person. Author Peter F. Eder writes
14 | PUCRS 2002 about the ongoing invasion of personal privacy which will
The sentence "Had he made Patch Adams, Salles would have get much worse unless better safeguards are quickly
focused on the medical profession" could be rewritten, established.
without change in meaning, as:
– De que forma o trecho "unless better safeguards are
a) If Salles focused on the medical profession, he would have quickly established" poderia ser reescrito, sem alteração do
made "Patch Adams". significado do texto?
b) Salles would focus on the medical profession if he would
have made "Patch Adams". a) ... if better safeguards are established quickly.
c) If Salles had focused on the medical profession, he would b) ... if better safeguards quickly established.
have made "Patch Adams". c) ... if better safeguards are not quickly established.
d) If Salles made "Patch Adams", he would focus on the d) ... when better safeguards are quickly established.
medical profession. e) ... when better safeguards are not established.
e) Salles would have focused on the medical profession if
he'd made "Patch Adams". 19 | UNESP 2002
If senior citizens more pessimistic toward
15 | MACKENZIE 2002 technology, Web developers and marketers to
If she had gone to the movies, . emphasize two things: ease of use and value.

a) she might be sick now a) became – have


b) Jane would be busy b) become – have
c) she would like to call Jane c) became – would have
d) she would have met Jane d) became – will have
e) Jane will be happy e) became – had
20 | ITA 2004 – ADAPTED
24 | MACKENZIE 2005
Had they been born just a few years earlier, they would have The sentence "If Orwell's book were to be rewritten, you
been part of that powerful and long-lasting generation that would have a nightmare vision of the world" in the THIRD
entered the military during World War II and filled the CONDITIONAL will be:
universities immediately after the war.
a) If Orwell's book had to been rewritten, you would have
– Assinale a opção que contém uma expressão equivalente been a nightmare vision of the world.
a: "Had they been born", em "Had they been born just a few b) If Orwell's book had been rewritten, you would have had
years earlier..." e que, portanto, poderia vir a substituí-la no a nightmare vision of the world.
texto: c) If Orwell's book had rewritten, you would have a
nightmare vision of the world.
a) They had been born... d) If Orwell's book had been rewriting, you would had have a
b) When they had been born... nightmare vision of the world.
c) As they had been born... e) If Orwell's book had rewritten, you would have been a
d) Whether they had been born... nightmare vision of the world.
e) If they had been born...
25 | UFC 2006 – ADAPTED
21 | PUCPR 2005 – ADAPTED What's the relation between the two clauses in the
Although it is very big (a female adult measures 9 cm) and underlined sentence below?
poisonous, the Italian tarantula does not represent a serious
problem for people. If this dental dream becomes a reality, stem cells will be
taken from the patient, cultured in a lab and then
– According to the text, one alternative is correct: reimplanted under the gum in the patient's jaw where the
tooth is missing.
If the Italian tarantula had been so poisonous, a lot of
people: a) Consequence.
b) Conclusion.
a) will be killed. c) Result.
b) would have to kill. d) Contrast.
c) would have killed. e) Condition.
d) would be killed.
e) would have been killed. 023 | MACKENZIE 2005
The sentence "If you run into someone on the street in
22 | MACKENZIE 2005 Taiwan, he's likely to greet you by asking 'Have you eaten?'"
If you (I) a friend or relative for his or her in the THIRD CONDITIONAL will be:
favorite awards-show moment, you (II) about
the kiss between Madonna and Britney Spears at the 2003 a) If you had run into someone on the street in Taiwan, he
MTV Video Music Awards. might have probably greeted you by asking 'Have you
eaten?'
– Mark the correct alternative to fill in blanks I and II: b) If you ran into someone on the street in Taiwan, he would
have been likely to greet you by asking 'Have you eaten?'
a) were asked about – can tell c) If you ran into someone on the street in Taiwan, he would
b) asked – have been told likely greet you by asking 'Have you eaten?'
c) could have asked – should tell d) If you had run into someone on the street in Taiwan, he
d) were to ask – might be told would have been likely to greet you by asking 'Had you been
e) must ask – have to be told eaten?'
e) If you could run into someone on the street in Taiwan, he
026 | UNESP 2006 would have likely to greet you by asking 'Have you eaten?'
If some anti-terrorist commentators the causes
of terrorism, they justifications for it. 27 | PUCPR 2007
If he loses election, he from public life.
a) accept – also accepted
b) don't refuse – wouldn't refuse a) retired
c) accepted – would also accept b) retire
d) wouldn't refuse – didn't refuse c) going to retire
e) accepted – won't refuse d) will retire
e) has retired
28 | MACKENZIE 2006
031 | UECE 2007
I'd have gone to that party if they me.

a) have invited
b) had invited
c) will invite
d) are inviting

032 | UECE 2007


They felt as if they on thin ice.

a) are walking
b) have walked
c) were walking
d) will walk
– The sentence that contains "if-clauses" correctly used is: 033 | UECE 2008
In the sentence "IF THE CRITERIA OF QUALITY BECOME
a) If you don't vote, you wouldn't have a say in the future of EXACTING, a canon may emerge ..." the part in capital letters
your country. is a/an:
b) Would you get married if you had been in love?
c) George might have become an architect if he went to a) relative clause.
school. b) conditional clause.
d) If she was traveling far, she always flies. c) noun clause.
e) Had I had money, I would have moved. d) restrictive clause.
29 | PUCRIO 2007
34 | UNESP 2008
"Had the buildings been full, about 14,000 people would If the pension deposit earlier, the lady
probably have died" means the same as: in trouble.

a) The buildings had been full of 14,000 dead people. a) arrived – would haven't been
b) Fourteen thousand people died because the buildings b) had arrived – wouldn't be
were full. c) had arrived – wouldn't have been
c) Though the buildings were full, about 14,000 people didn't d) arrives – wouldn't be
die. e) has arrived – would be not
d) Had the buildings been filled with 14,000 people, no one
would have died. 35 | PUCPR 2001
e) About fourteen thousand people could have been killed if Select the best alternative to complete the sentences below:
the buildings had been full.
I. If they money, they will build a big house.
030 | UNESP 2007 II. If you out more, you would meet a few
Indique a alternativa que preenche corretamente a people.
sentença: III. If he his restaurant, he would have got more
customers.
If the snow cover longer, it the IV. If she so fast, she wouldn't have crashed her
plants and the water cycle. car.
V. If we a car, we wouldn't have to spend all our
a) lasted – would damage – will disturb time waiting for buses.
b) would last – would damage – disturb
c) lasts – will damage – disturb a) have; went; had cleaned; hadn't been driving; had
d) will last – damages – disturbs b) have; go; cleaned; wasn't driving; have
e) lasts – will damage – disturbed c) had; went; was cleaned; hadn't driven; has
d) have had; have gone; had cleaned; hadn't been driven;
have had
e) have; have gone; has cleaned; hadn't driven; have
036 | ITA 1996
041 | JFS 2011
If the U. S. Justice Department had not derailed Gate's bid to Complete the sentence below meaningfully:
acquire Intuit, the deal realize Microsoft's
ambition to make money from almost every commercial they harder, they .
transaction in cyberspace.
a) Unless – had studied – wouldn’t have failed
a) is going to help
b) Unless – hadn’t studied – would have failed
b) would have helped
c) If – had studied – would have failed
c) would help
d) Unless – had studied – would have failed
d) helps
e) If – hadn’t studied – wouldn’t have failed
e) is likely to help
042 | JFS 2010
37 | EFOMM 2005 "Earthquakes don't kill — they don't create damage —
If my boss had not phoned the police on the spot, the thief
," said Eric Calais, a Purdue University
.
geophysicist studying the Haiti quake.
a) would get away with it – A lacuna presente no excerto acima deve ser preenchida
b) got away with it por:
c) should got away with it
d) would have gotten away with it a) unless there's nothing to be damaged.
e) has gotten away with it
b) unless there's nothing to damage.
c) if there's little to damage.
38 | EFOMM 2007 d) if there's nothing to damage.
The loan back if they hadn’t been held up last e) if there's almost nothing to be damaged.
week.
43 | EFOMM 2012
a) would have paid Choose the correct alternative to complete the following
b) would pay sentence:
c) would have been paid
d) will have paid "If she hadn’t been so bad-tempered, I her."
e) will be paid
a) shall have married
039 | AFA 1999
b) ought to have married
"Come back to me and you will really know what happiness c) might have married
can be" means:
d) will have married
e) must have married
a) If you come back to me, you will know what happiness can
be. 44 | EFOMM 2010
b) If coming back makes you happy, it could bring happiness The conditions expressed in the sentences "If they had
to me. known the outcomes, they would have taken different
c) If you come back to me, you would know what happiness measures" and "you should know people better, if you really
could be. want to make more friends" are, respectively:
d) Come back to me or else you won’t know what could be
happiness. a) unlikely / unlikely
b) impossible / unlikely
040 | JFS 2000
c) likely / unlikely
Complete meaningfully the following sentence:
d) impossible / likely
e) likely / impossible
Had they studied hard, they .

a) would pass
“Chance is perhaps the pseudonym of God
b) wouldn’t have passed
when He did not want to sign.”
c) would have passed
Anatole France
d) would have pass
e) would haven’t passed
Question Tags 006 | FEI 1997
He'll be back soon, ?
1 | UNITAU 1995
Assinale a alternativa que corresponde à sequência de a) will he
Question Tags adequados para completar as frases a seguir: b) doesn't he
c) shall he
1. He isn't at home, ? d) won't he
2. That will happen, ? e) couldn't he
3. She hasn't a cue, ?
4. It rains a lot, ? 7 | FUVEST 1998
Escolha a question tag correta para "I knew I would be a
a) isn't he; won't it; has he; doesn't it scientist":
b) is it; will it; does she; has it
c) isn't he; will it; has she; hasn't it a) didn't I?
d) is he; won't it; has she; doesn't it b) wasn't I?
e) isn't he; won't he; has she; does it c) won't I?
d) don't I?
2 | FUVEST 1978 e) would I?
Assinale a alternativa que preenche corretamente a lacuna:
8 | UDESC 1997
He doesn't study here, he? Complete with the CORRECT alternative:

a) doesn't The sun shone the whole day, it?


b) do
a) is
c) did
b) did
d) does
c) doesn't
e) don't
d) didn't
e) isn't
3 | UNESP 1987
Assinale a alternativa correta:
009 | FMTM 1998
You can sing well, ?
Your name is Mary, ?
a) didn’t you
a) isn't you
b) can’t you
b) isn't it
c) don’t you
c) aren't it
d) doesn’t you
d) aren't you
e) couldn’t you
e) isn't he
010 | JFS 1999
4 | UNESP 1988
Roy read the newspaper this morning before his father,
Politics is a science, ?
?
a) weren't they a) didn’t he
b) isn't it b) doesn’t he
c) wouldn't he c) is he
d) wasn't it d) did he
e) won't they e) does he
005 | UEL 1995 011 | JFS 2002
He hasn't seen you lately, ? Em qual das alternativas abaixo a 'question tag' está errada?

a) has he a) Let’s go to a disco, shall we?


b) is it b) Do the exercises, will you?
c) have you c) Mariah put the book on the armchair, doesn’t she?
d) have we d) Let me go with you, shall I?
e) haven't you e) The teacher came to help us, didn’t she?
012 | JFS 2002
018 | AFA 1997
Complete corretamente: We can’t do without him, ?
The Titanic sank in 1912, ?
a) can us
b) can we
a) didn’t she c) can ours
b) didn’t it
d) can’t he
c) hadn’t it
d) hadn’t she
019 | AFA 2000
e) doesn’t it
Choose the correct question for the context: You are
speaking to your daughter. You want to make sure that she
013 | ITA 1984
turned off the stove. You ask her:
Michiko and Yamashiro are not Japanese, ?
a) You turned off the stove, did you?
a) are
b) You do turn off the stove, didn’t you?
b) aren't they
c) You didn’t turn off the stove, did you?
c) aren't them
d) You did turn off the stove, didn’t you?
d) are they
e) are they not
020 | EFOMM 2012
Choose the option with the correct tag questions for the
014 | ITA 1985
sentences below.
You know you have to study more, ?
1. You weren’t listening, ?
a) do you 2. She doesn't know him, ?
b) don't you 3. I'm a bit overweight, ?
c) do you not
4. Don't open your eyes, ?
d) not know
e) not you know
a) weren't you / does she / aren't I / do you
b) were you / doesn't she / aren't I / do you
015 | ITA 1991 c) were you / does she / aren't I / will you
I am not as good at football as he is, ? d) weren't you / does she / am I not / will you
e) were you / doesn't she / am I not / do you
a) aren’t I
b) is he
c) no
“It's better to be a pirate than to join the Navy.”
d) am I
Steve Jobs
e) am

016 | ITA 1992


A alternativa que corretamente preenche o claro de One
never knows what to expect, ? é:

a) isn’t it
b) does one
c) one knows
d) knows one
e) do we

017 | EN 1989
The cheque of the customer had not been returned:

a) hadn't it?
b) does it?
c) hasn't it?
d) did it?
e) had it?
005 | EFOMM 2006
Rejoinders Philip: "- She's got a place at college!"
Kate: "- !"
001 | FUVEST 1978
Selecione a alternativa que preenche corretamente a lacuna: a) So have I
b) So I have
My mother doesn't drink tea and do I.
c) And I
d) I am too
a) or
e) So am I
b) also
c) too
006 | JFS 2008
d) either
Mr. Anderson can’t speak Chinese.
e) neither
a) Neither I can.
2 | MACKENZIE 1997
b) Neither can I.
Indicate the alternative that best completes the following
c) So can I.
sentence:
d) So I can.
Anne disliked our new roommate, and .
“Be more prompt to go to a friend in adversity
a) I didn't too
than in prosperity.”
b) I didn't either
Chilo
c) neither did I
d) never did he
e) so did I

3 | MACKENZIE 1999
Indicate the alternative that best completes the following
sentence:

I haven't finished the homework and .

a) my brother hasn't either


b) neither my brother
c) so did my brother
d) either hasn't my brother
e) neither does my brother

4 | MACKENZIE 2002
Indicate the alternative that best completes the following
sentence:

Paulo knows how to drive a truck and .

a) Mark does neither


b) either does Mark
c) so does Mark
d) nor does Mark
e) Mark does either
005 | PUCPR 1996
Articles Fill in the blanks of the following sentence with the definite
article:
001 | UNITAU 1995
Assinale a alternativa que corresponde à sequência que Brazil is most industrial country in
completa as lacunas a seguir: South America, while United States
holds same position in North America.
Indian the ecologist saw, started
horrible fire because of ordinary yellow bird
a) the; the; the; the; the; the
flew over his head. b) *; the; the; *; the; *
c) *; the; *; the; the; *
a) a; the; a; who d) *; the; the; the; the; *
b) an; the; a; whose e) the; *; *; *; the; the
c) the; a; an; which
d) the; an; a; whom
006 | PUCCAMP 1992
e) a; an; a; which
Fred: I've been thinking of buying .
Sam: Really? Which make are you considering?
002 | FUVEST 1977 – ADAPTED Fred: That doesn't matter as long as is
In beginning, religion played
important part in economical.
Brazil. history of
a) a car – some car
a) a / the / a / the / * b) a car – the car
b) * / the / an / the / * c) some car – car
c) the / the / a / the / * d) the car – a car
d) the / * / an / the / * e) car – a car
e) the / the / an / the / *
007 | UFPB 1998
3 | FUVEST 1978 – ADAPTED Read this sentence:
money is very important, but you can't buy
good idea is good idea, whether
bit of luck with all money you it's done in one, three or 33
have. countries.

a) The / a / the – It is completed by the following sequence:


b) * / a / the
c) A / a / the a) A / no article / the / the
d) Some / a / the b) no article / no article / a / a
e) The / a / * c) The / the / no article / no article
d) A / a / no article / no article
4 | UNESP 1991 e) The / no article / a / no article
Assinale a alternativa que preenche corretamente as lacunas
da frase a seguir: 8 | FATEC 1998
Indique a alternativa em que a tradução de a ou an difere do
It was honor for us to see Queen of seu significado habitual de artigo indefinido (um, uma) em
England. língua portuguesa:
a) a – the – the a) However, the abduction of Gutiérrez would be a new
b) * – a – * twist.
c) an – a – the b) After his family paid an undisclosed ransom.
d) the – * – an c) An American businessman help for eight days was
e) an – the – * released just last week.
d) Kidnapping has become rampant in Mexico with hundreds
of cases a year.
e) Editorials are already referring to him as a "political
kidnapee".
9 | PUCPR 2001
013 | EEAR 2007
In which of the sentences we MUSTN'T use the article THE to
Choose the alternative in which the definite article is used
complete the blanks? correctly:
a) Statue of Liberty is visited by thousands of
a) The Brazilians are very friendly.
tourists every year.
b) The old man is arriving right now.
b) Lots of people enjoy amount of time they c) The spring is the season of flowers.
spend outdoors. d) The New York is a very beautiful city.
c) Richard Claydermann will play piano for
hundred people in the theatre tomorrow. 014 | JFS 2008
d) Economically, London is considered one of Mark the correct option:
the most important cities in Europe.
e) Who is next to be interviewed? Mr. Smith called you when you were out.

10 | EFOMM 1994 a) A
I want can of peaches, b) An
sugar, and pound of raspberry jam. c) The
d) No
a) the / the / a / the / *
b) a / * / * / a / * 015 | ITA 1994
c) the / * / a / * / * Complete corretamente o texto a seguir:
d) a / the / the / * / the
e) a / the / a / the / a The pianist I told you about lives in (I) one-story
building on Main Street. Although she isn’t (II)
11 | EFOMM 2007 professional musician, she plays (III) piano
lemon originated in China and extremely well.
spread south to Malaysian islands and west to
India. a) an – a – the
b) * – * – a
a) A / the / the / * c) a – a – the
b) * / * / * / * d) an – * – *
c) The / the / the / the e) the – * – *
d) A / the / * / *
e) The / * / the / * 016 | AFA 2001
milk and meat are good for
12 | PUCPR 2000 our health.
Which is the correct alternative about the use of the article
"the" in the phrases below? a) * / * / *
b) * / the / *
I. You mustn't smoke in class. c) The / the / *
II. Marcos has all the right qualifications for job. d) The / the / the
III. Sometimes there are shows in Central Park.
IV. Mercury is the smallest planet in 017 | UNITAU 1995
Solar System. Assinale a alternativa que corresponde à sequência na qual
V. liberty and democracy are se inclui um uso inadequado do artigo em inglês:
idealized since French Revolution.
a) a watch; a pity; an orange.
a) Only in sentences I and II it's necessary to use the article b) an umbrella; a real effort; a year.
THE. c) a small plane; an idea; a whale.
b) It's correct to use THE in all blank spaces. d) a one-man show; an university; a private investigator.
c) In alternatives I, III, IV and V it's correct to complete the e) an egg; an uncle; a book.
spaces with THE.
d) About alternative V, it only needs the article in the third
space.
e) We have to use THE only in the second space of phrase
number IV.
018 | JFS 2000
022 | JFS 2009
Complete corretamente as sentenças abaixo:
Read the following text and fill in the gaps with the correct
sequence of articles:
- life you want is really fascinating.
- I felt love in her touch.
The Strokes are American rock band formed in
- Carlson, teacher, has become rich 1998 that rose to fame in early 2000s as
man. leading group in garage rock
- Everybody intends to enter in university. revival. Upon release of their acclaimed debut
- Johnson bought ewe. album Is This It in 2001, many critics hyped
group as the "saviors of rock" for their stripped
a) The – the – a/ a – an – a
down sound, heavily influenced by bands such as The Velvet
b) The – * – the/ a – a – a
Underground. Since then, band has maintained
c) The – the – the/ a – a – an large fan base and has enjoyed much
d) * – * – the/ a – an – a success, particularly in United
e) The – * – the/ a – an – an Kingdom.

019 | UFF 1996 Adapted from http://en.wikipedia.org/


In the expression such a reaction, the word such is followed
by the indefinite article a. Mark the sentence in which the a) an – an – a – the – the – the – a – the – a – no article – the
indefinite article must also be used after such. b) an – the – an – a – the – the – no article – the – a – a – the
c) an – the – a – the – the – the – no article – the – a – no
a) Germans will not listen to such article – the
businessmen. d) an – the – a – the – the – the – no article – the – a – no
b) A British businessman would not believe such article – no article
nonsense. e) an – the – a – no article – a – the – no article – the – a – a
c) A French counselour would not give him such – the
silly advice.
d) An American would not sign such contract. 023 | JFS 2009
e) Those students have never heard such I was watching MTV show last night. What
beautiful music. wonderful clips they exhibited!

020 | ITA 1990 a) the – a


Marque a alternativa que corretamente preenche as lacunas b) a – a
I e II da sentença a seguir: c) an – a
d) an – *
(I) Dr. Brown refused to talk to (II) e) the – *
doctor who wanted to help him é:
024 | JFS 2009
a) the – the Read the following text and fill in the gaps with the correct
b) * – the sequence of articles:
c) * – an
d) the – a Despite universal derision of
e) the – an literary establishment, which could never comprehend
its inherently noble spirit, Tolkien's The Lord of
021 | EOMM 2012 the Rings was recently voted greatest work of
The indefinite article can be appropriately used in: fiction of 20th Century by thousands of
Waterstones' customers.
a) information * = No article
b) wool
c) furniture Adapted from http://library.flawlesslogic.com/
d) eggs
e) hypothesis a) * – the – * – the – *
b) * – the – * – a – the
c) the – the – * – the – the
d) the – the – * – a – the
e) the – * – * – the – a
Plural of the Nouns 006 | FEI 2000
ALL HE NEEDED. Escolha a sentença correta, colocada no
1 | FUVEST 1979 plural, tempo futuro simples:
Assinale a alternativa que preenche corretamente as
lacunas: a) All they will need.
b) They all will need.
Boys have big and girls have small . c) They would need all.
d) All they would need.
a) foots – ones e) All they didn't need.
b) feet – ones
c) feet – one 7 | PUCPR 1996
d) feets – ones Match the columns below so that the words in the second
e) foot – one column fit the sentences provided in the first one:

2 | CESGRANRIO 1990 1.Tom and Mary love their .


KNIVES is the plural of KNIFE. Which of the words below 2. Put the oranges inside those .
does not form its plural in the same way? 3. I can't walk. My are aching a lot.
4. The are flying south.
a) Wife. 5. The cat is hunting the .
b) Life. 6. When Jane fell over, she broke two of her .
c) Leaf.
d) Chief. ( ) feet
e) Half. ( ) teeth
( ) children
3 | CESGRANRIO 1991 ( ) geese
The word that DOESN'T have an irregular plural form like ( ) mice
tooth – teeth is: ( ) boxes

a) ox. – Choose the correct alternative:


b) foot.
c) cloth. a) 6 – 3 – 1 – 4 – 5 – 2
d) goose. b) 3 – 6 – 1 – 4 – 5 – 2
e) mouse. c) 3 – 6 – 2 – 4 – 5 – 1
d) 5 – 3 – 4 – 2 – 6 – 1
4 | FATEC 1998 e) 3 – 6 – 4 – 1 – 2 – 5
Observe a frase: "Did hundreds of Japanese KIDS suffer a TV-
induced epileptic attack last week". Indique a alternativa em 8 | PUCRS 2007
que todas as palavras têm a mesma forma de plural de KIDS: Nouns in English can be divided into countable or
uncountable (e.g.: apple X water). In order to indicate some
a) attack – mouse kind of "measurement" in the case of uncountable nouns,
b) episode – Japanese another noun is required (e.g.: "glasses" or "liters" of water).
c) cartoon – trigger Accordingly, the expression below that is equivalent to the
d) show – child structure "blades of grass" is:
e) explosion – Japanese
a) structures of steel.
5 | PUCRIO 1999 b) classes of Chinese.
Check the only word that could be used in the plural form: c) cups of coffee.
d) floors of wood.
a) Information. e) letters of complaint.
b) Land.
c) Research.
d) Energy.
e) Employment.
9 | MACKENZIE 2003
012 | ITA 1988
Dadas as afirmações de que o plural de:

1. BASIS é BASIS
2. DATUM é DATAS
3. BUSINESS é BUSINESSES

– Constatamos que está(estão) correta(s):

a) Apenas a afirmação no 1.
b) Apenas a afirmação no 2.
c) Apenas a afirmação no 3.
d) Apenas as afirmações nos 1 e 3.
e) Todas as afirmações.

013 | ITA 1989


O plural dos substantivos abaixo:
– Which alternative shows the correct plural form of the
words given? I. knife
II. tooth
a) mouse – mice/ goose – geese/ phenomenon – III. woman
phenomena/ deer – deer
b) mouse – mices/ chick – chicken/ person – persons/ child – – é, na ordem:
children
c) mouse – mouses/ goose – geeses/ deer – deers/ news – a) knifes – teeth – womans
news b) knifes – teeths – women
d) mouse – mouses/ new – newses/ bus – buses /person – c) knives – tooths – women
people d) knives – teeth – women
e) mouse – mises/ child – children/ police – polices/ news – e) knive – teeth – women
news
014 | JFS 2000
010 | ITA 1987 Considering that the plural form of the following words:
Dadas as afirmações de que o plural de:
1. Man is Men
1. OX é OXEN 2. Woman is Women
2. CHIEF é CHIEFS 3. Roman is Romen
3. ROOF é ROOVES
– The incorrect datum(a) is(are) the:
– Constatamos que está(estão) correta(s):
a) number 1.
a) Apenas a afirmação nº 1. b) number 2.
b) Apenas a afirmação nº 2. c) number 3.
c) Apenas a afirmação nº 3. d) numbers 1 and 2.
d) Apenas as afirmações nºs 1 e 2. e) numbers 1 and 3.
e) Todas as afirmações.
015 | ESPCEX 1999
011 | EFOMM 2000 The plural of wife, goose, mouse and hero are:
The plural forms of the underlined words in the sentence
"Jane is afraid of mouse and louse" are respectively: a) wifes, gooses, mouses, heroes.
b) wifes, geese, mice, heroes.
a) mice – lice c) wives, geese, mice, heros.
b) mices – lices d) wives, geese, mice, heroes.
c) mouses – louses e) wifes, gooses, mouses, heros.
d) mice – louses
e) mouses – lice
016 | JFS 2000
019 | ITA 1990
Complete as sentenças a seguir utilizando as formas corretas Dadas as afirmações de que o plural de:
de cada vocábulo apresentado:
1. Chief é Chieves
– He bought a car.
2. Radius é Radii
– She is a rock star. 3. Leaf é Leaves
– I have good to give you.
– The police in front of the building.
– Constatamos que está (estão) correta(s):
a) sport – famous – informations – is a) Apenas a afirmação nº 1.
b) sport – famous – informations – are
b) Apenas a afirmação nº 2.
c) sports – famous – informations – are
c) Apenas a afirmação nº 3.
d) sport – famouses – information – is d) As afirmações nos 2 e 3.
e) sports – famous – information – are
e) Todas as afirmações.
017 | ITA 1987
020 | JFS 2000
Algumas vezes, o significado da forma plural de um Marque a alternativa que possui as sentenças abaixo
substantivo (em inglês), é diferente do seu significado na reescritas, corretamente, no plural:
forma singular. Dadas as palavras (já na sua forma plural):
1. She writes a letter to her sister every day.
1. GOODS
2. He and his friend are going to buy a new house.
2. SCALES
3. The news is good, I think you will like it.
3. SPIRITS 4. This old photo brings me a good recollection.
5. The phenomenon happened yesterday night.
– Constatamos que confere(m) com a afirmação acima:
a) 1. They writes letters to their sisters every day./ 2. They
a) Apenas a palavra nº 1. and their friends are going to buy new houses./ 3. The news
b) Apenas a palavra nº 2. is good, we think you will like them./ 4. These old photos
c) Apenas a palavra nº 3. bring us good recollections./ 5. The phenomena happened
d) Apenas as palavras nºs 1 e 2. yesterday night.
e) Todas as palavras.
b) 1. They write letters to their sisters every day./ 2. They
and their friends are going to buy new houses./ 3. The news
018 | JFS 2008
are good, we think you will like them./ 4. These old photos
Give the correct plural form of the words below:
bring us good recollections./ 5. The phenomena happened
yesterday night.
– Buffalo;
c) 1. They write letters to their sisters every day./ 2. They
– Eskimo;
and their friends are going to buy news houses./ 3. The news
– Concerto;
is good, we think you will like them./ 4. Those old photos
– Person;
bring us good recollections./ 5. The phenomena happened
– Manservant;
yesterday night.
– Thesis;
d) 1. They write letters to their sisters every day./ 2. They
– Sister-In-Law;
and their friends are going to buy new houses./ 3. The news
– Die.
is good, we think you will like them./ 4. These old photos
bring us good recollections./ 5. The phenomena happened
a) Buffalos/ Eskimos/ Concertos/ Personas/ Menservants/
yesterday night.
Theses/ Sisters-in-law/ Dice
e) 1. They write letters to their sisters every day./ 2. They
b) Buffaloes/ Eskimos/ Concertos/ Persons/ Menservants/
and their friends are going to buy new houses./ 3. The news
Theses/ Sisters-in-law/ Dice
is good, we think you will like they./ 4. These old photos
c) Buffaloes/ Eskimos/ Concertoes/ Persons/ Manservants/
bring us good recollections./ 5. The phenomenons happened
Theses/ Sisters-in-law/ Dice
yesterday night.
d) Buffaloes/ Eskimos/ Concertos/ Persons/ Menservants/
Theses/ Sister-in-laws/ Deaths
e) Buffalos/ Eskimoes/ Concertoes/ Personas/ Manservants/ “Existence would be intolerable
Thesis/ Sister-in-laws/ Deaths if we were never to dream.”
Anatole France
Genitive Case 5 | UDESC 1997
father is in Europe.
1 | UNESP 1994
Assinale a alternativa que preenche corretamente a lacuna: a) The Mary's and George's
b) Mary's and George
The uncle was dead. c) Mary and George's
d) Mary's and Georges's
a) writer e) The Mary and George's
b) writers
c) writer of 6 | UNESP 1999
d) writer's farm is that large one? It is .
e) writers of the
a) Which – Peter's
2 | UNITAU 1995 b) Whose – Peter's
Assinale a alternativa que corresponde à tradução mais c) Whose – of Peter
adequada da frase a seguir: d) Which – for Peter
e) What – Peter's
My mother's maid has just bought the dog's meat.
007 | UFRS 2001
a) Minha mãe e a empregada acabam de comprar a carne do O possessivo, usado como em "Woody Allen's Sweet and
cachorro. Lowdown", está correto em todas as alternativas abaixo,
b) A empregada de minha mãe acaba de comprar a carne do EXCETO em:
cachorro.
c) Minha mãe acabou de fazer a carne do cachorro. a) There was a two hours' delay at the airport in London.
d) Minha mãe fará compras com a empregada e o cachorro. b) Anthony Burgess's A Clockwork Orange is a milestone in
e) Minha mãe é empregada e comprou carne de cachorro. modern literature.
c) In our last holidays we had to cope with our young
3 | FUVEST 1979 – ADAPTED relatives' weird ideas.
Reescreva empregando o caso genitivo: d) Elizabeth I's interest on sea voyages brought development
to England.
John and Mary are cousins. Have you met the parents of e) Maggie and Millie's eyebrows are so thin you can hardly
John and of Mary? see them.

a) John and Mary are cousins. Have you met John and Mary's 8 | FATEC 2003
parents. Assinale a alternativa que apresenta o uso correto do caso
b) John and Mary are cousins. Have you met John's and possessivo, como no substantivo "media" em "the media's
Mary's parents. collective attention":
c) John and Mary are cousins. Have you met John's and Mary
parents. a) mens' garment.
d) John and Mary are cousins. Have you met John's and b) womens' wear.
Mary's parents’. c) mental's disturbance.
e) John and Mary are cousins. Have you met John's and d) children's clothes.
Mary's parent’s. e) disappointment's feeling.

004 | UFRS 1996 9 | UNESP 2005


The phrases "Americans' encounter" the nation's energies" Indique a alternativa que expressa o mesmo significado da
and "America's physical geography" are examples of: expressão em destaque na sentença:

a) passive voice. It is important to remember that THE BEHAVIOR OF


b) the infinitive. DEPRESSED CHILDREN may change.
c) the gerund,
d) the genitive. a) the depressed children's behavior
e) indirect speech. b) the behavior's depressed children
c) the behavior of the depressed children's
d) the children's depressed behavior
e) the depressed behavior's children
010 | UFRS 2007
015 | AFA 1999
While the danger does not seem to dampen anyone's
The honor to a woman is to:
partying spirit, violence is much feared and the threat is
much discussed among the locals.
a) refer to the daughter of her mother’s.
b) have the same of her daughter’s name.
– The use of 'S is the same in ANYONE'S PARTYING SPIRIT
c) be referred to as her daughter’s mother.
and in:
d) be called by the name of her daughter’s.
a) Everyone's invited for Carnival in Rio.
016 | JFS 2000
b) The American's luggage was checked carefully.
Complete:
c) My friend Jeremy's arrived.
d) Nobody's pleased with the situation.
wives arrived together.
e) The Mexican tourist's coming tomorrow.
a) Alan’s and Victor’s
011 | ITA 1990
b) Alan’s and Victor
Assinalar a alternativa onde o uso do caso genitivo esteja
c) Alan and Victor’s
CORRETO:
d) Alan’ and Victor’
e) Alan’ and Victor’s
a) For goodness’ sake, this is my brother-in-law’s dog.
b) For goodness’ sake, this is my brother's-in-law dog.
017 | JFS 2010
c) For goodness sake's, this is my brother-in-law's dog.
Leia o fragmento a seguir:
d) For goodness sake's, this is my brother's-in-law dog.
e) For goodness sake's, this is my brother-in-law dog's.
"When you look at the architecture in Chile you see buildings
that have damage, but not the complete pancaking that
12 | ESPCEX 1999
you've got in Haiti," said Cameron Sinclair, executive director
The correct sentence is:
of Architecture for Humanity, a 10-year-old nonprofit that
has helped people in 36 countries rebuild after disasters.
a) My father’s friend called me yesterday.
received 400 requests for help the day
b) The table’s leg is broken.
after the Haiti quake but he said it had yet to receive a single
c) I have an appointment at the office’s doctor.
request for help for Chile.
d) My brother neighbour’s sister is a nurse.
e) The girls school is far from St Bartholomew’s.
– O espaço em branco deve ser preenchido por qual das
seguintes opções?
13 | EFOMM 1994
Betty, Jane and I were invited to a party at
a) Sinclair San Francisco’s based organization
home.
b) Sinclair's San Francisco-based organization
c) Sinclair's San Francisco’s based organization
a) your friend’s Carol
d) San Francisco-based organization by Sinclair
b) our friend Carol
e) San Francisco’s based organization by Sinclair
c) our friend Carol’s
d) your friend’s Carol’s
018 | UDESC 1999
e) her friends’ Carol
Choose the correct answer to complete the sentence:
14 | EFOMM 2000
The offices are very modern.
His sickness is worrying him very much.
a) businessmen’
a) mother’s-in-law
b) businessmens’
b) mother-in-law
c) businessmans’
c) mother’s-in-law’s
d) businessmen’s
d) mother-in-law’s
e) mothers-in-law’s
“When desire dies, fear is born.”
Baltasar Gracián y Morales
005 | AFA 2001
Numbers What’s the right answer for the numerical expressions
below?
001 | FEI 2000
Indique o ordinal referente a "four":
1 3 16
5 ; 2 5 ; 15
a) forty
b) fourteen
a) One fives / two thirty-five / sixteen fifteen
c) fourteenth
b) One five / two and third fifth / sixteen fifteens
d) fourth
c) One fifth / two and three fifths / sixteen fifteenths
e) fortieth
d) First fifths / second thirty-five / sixteenth fifteenths
002 | FUVEST 1979 – ADAPTED
006 | JFS 2008
Reescreva a frase colocando por extenso os numerais, na
Some stats about the Olympic Games in Beijing:
sua forma ordinal:
* 28 Olympic programs, 302 sub-categories
Her (21) birthday will be on the
* 302 gold medals
(11).
* 10,500 athletes are expected to participate
* 21,880 torchbearers will run 137,000 km over 130 days
a) Her twenty-first birthday will be on the eleventieth.
* The National Stadium (Bird’s Nest) covers an area of
b) Her twenty-one birthday will be on the eleven.
258,000 sq. meters
c) Her twenty-first birthday will be on the eleven.
* The Bird’s Nest has 91,000 seats
d) Her twenty-one birthday will be on the eleventh.
* The surface of the National Aquatics Center is covered by
e) Her twenty-first birthday will be on the eleventh.
1,437 pieces of transparant material
* The highest price for the opening ceremony tickets is 5000
003 | JFS 2000
Renminbi, the lowest is 200 Renminbi
Marque a alternativa que possui os resultados corretos das
* Beijing expects 550,000 international visitors and 2.4
operações abaixo:
million domestic spectators
* Over 800 star-class hotels and 4,000 hostels will provide
2×9=?
about 420,000 overprized rooms
14 – 11 = ?
? + 4 = 16
– Give the marked numbers in full:
a) eighteenth – three – twelve
a) three hundreds and two; ten thousands and five
b) eighty – thirty – two
hundreds; one hundred and thirty-seven thousands; two
c) eighteen – third – twelve
hundreds and fifty-eight thousands; one thousand and four
d) eight – thirteen – twenty
hundreds and thirty-seven; two millions and four hundreds
e) eighteen – three – twelve
thousands; four hundreds and twenty thousands
b) three hundred and two; ten thousand and five hundred;
004 | JFS 2000
one hundred and thirty-seven thousand; two hundred and
Solve the problems below:
fifty-eight thousand; one thousand and four hundred and
thirty-seven; two million and four hundred thousand; four
1. Two into ten goes times.
hundred and twenty thousand
2. A quarter plus three-quarters makes .
c) three hundred and two; ten thousand and five hundred;
3. Twenty-eight from fifty leaves .
one hundred and thirty-seven thousand; two hundred and
fifty-eight thousand; one thousand and four hundred and
a) five – one – twenty-one
thirty-seventh; two million and four hundred; four hundred
b) four – four – twenty-one
and twenty thousand
c) five – four – twenty-two
d) three hundred and two; ten thousand and five hundred;
d) five – one – twenty-two
one hundred and thirty-seven thousand; two hundred and
e) four – four – twenty-two
fifty-eight thousand; one thousand and four hundred and
thirty-seven; two million and four thousand; fourth hundred
and twenty thousand
007 | JFS 2008
Read the following sentence and fill in the blanks Prepositions
meaningfully:
001 | PUCRIO 2004
More than twenty people were inside the Mark the only sentence that CANNOT be correctly
stadium, but more were outside because they completed with the preposition FROM:
didn’t get to buy the tickets in time.
a) Commercial sales of drugs derived this one
a) thousand – hundreds plant are about US$160 million a year.
b) thousands – hundreds b) Madagascar's rosy periwinkle, a plant Africa,
c) thousand – thousand provides two important anti-tumor agents.
d) hundred – thousand c) Quinine, an aid in the cure of malaria, is an alkaloid
e) hundreds – hundreds extracted the bark of the cinchona tree found in
Latin America and Africa.
008 | ITA 1990 d) 1960, only 19 percent of Hodgkin's disease
A alternativa que corretamente preenche as lacunas I, II e III sufferers had a chance for survival.
de: e) Until recently, wild yams Mexico and
Guatemala provided the world with its entire supply of
1. Five from six leaves I. diosgenin, an active ingredient in birth control pills.
2. Two into eight goes II times.
3. The third power of two is III. 002 | UFRS 2006
In the phrase "Exports to China are expected to quadruple
– é: by 2010", BY is being used with the same meaning as in:

a) 30; 8; 10 a) I'll finish reading the book by midnight.


b) 11; 10; 10 b) We could go to Gramado by car.
c) 30; 12; 8 c) He paid the hotel expenses by cheque.
d) 1; 4; 8 d) Forty divided by eight is five.
e) 1; 8; 1 e) Do you know "The Da Vinci Code" by Dan Brown?

3 | PUCRS 2006
“There is no duty more obligatory The alternative which contains the prepositions that best
than the repayment of kindness.” complete the sentences below is:
Cicero
Prepaid meters have been launched with the aim
improving water service; however, they might
be a problem those who cannot afford paying
water services.

a) to – for – of
b) on – to – for
c) of – to – on
d) for – on – of
e) of – for – for

4 | UNESP 1994
Something is cooking the oven.

a) up
b) to
c) in
d) into
e) for
5 | ITA 1995 – ADAPTED
10 | FUVEST 1977 – ADAPTED
It's clear that Gossard and the rest of Pearl Jam no longer
Complete com as palavras necessárias:
want to "rely" anger and craziness to drive the
band. I was born 2 o'clock the morning,
a Sunday April the
– A preposição que deve acompanhar o verbo "rely", year 1958, a farm a small village
relacionado no texto, é: called Sta. Cruz, the state of Goiás, Brazil.

a) at a) on / in / on / in / in / in / in / in
b) on b) on / in / on / in / of / in / in / in
c) in c) at / in / on / in / of / on / in / on
d) for d) at / in / in / in / of / in / in / in
e) with e) at / in / on / in / of / in / in / in

6 | ITA 1995 – ADAPTED 11 | CESGRANRIO 1995


Scientists have been talking about producing better foods The program Dr. Black is working his colleagues
genetic engineering ever since the technology the department psychiatry will
first became available more than 20 years ago. build on a pioneering study done 1989.

– A preposição que preenche a lacuna corretamente é: – Mark the item which contains the prepositions that
complete the passage above:
a) by.
b) for. a) with, of, about, in
c) over. b) with, on, from, in
d) through. c) with, in, of, in
e) with. d) without, at, by, on
e) without, from, after, on
7 | UNESP 1993
He walked the room. 012 | FAAP 1996
An executive presiding over a lunchtime meeting
a) at a busy San Francisco restaurant was having no
b) on luck getting the waiter's attention. So, using his cellular
c) between phone, he called the restaurant and asked have
d) into some menus sent over his table. It worked.
e) among
Reader's Digest – Sep./95
8 | UNESP 1995
I read a chapter politics. – Quais preposições completam corretamente o texto
anterior?
a) on
b) at a) in, about, to
c) above b) at, to, in
d) before c) in, for, on
e) after d) for, for, to
e) in, to, to
9 | FUVEST 1979
the circumstances you must go 13 | UNESP 1996
foot. Assinale a alternativa correta para completar o espaço em
branco na sentença a seguir:
a) Under – with
b) Under – by She is very proud her children.
c) On – on
a) at
d) Under – on
b) in
e) On – under
c) on
d) with
e) of
14 | MACKENZIE 1996
18 | CESGRANRIO 1990
Complete with the appropriate prepositions:
The sentence in which FOR is used in the same way as in
"These knives are now used for some general surgery" is:
I. How kind you to invite us your
party! a) The laser has been used for years.
II. I'm sorry him but even so I'm not sorry b) The technician is leaving for Tokyo.
what I did. c) This telephone records word for word.
III. Is the director confident his abilities? d) The laser has become popular for its wide applicability.
e) Doctors use laser for several types of operations.
a) I. for/for; II. about/for; III. about
b) I. of/to; II. for/about; III. of 19 | CESGRANRIO 1990
c) I. in/for; II. for/about; III. of In "The sweet-and-lovely look is OUT; the aggressive punk
d) I. on/to; II. for/in; III. in pose is IN", the capital words stand for:
e) I. for/on; II. of/for; III. with
a) out of work / in vogue
15 | MACKENZIE 1996 b) out of sight / in mind
Complete with the appropriate prepositions: c) out of order / in memory
d) out of date / in fashion
I. My wife was very glad taking a trip abroad. e) out of mind / in sight
II. Are you hopeful receiving a nice gift?
III. Whatever is good you will be acceptable 20 | FEI 1996 – ADAPTED
me. One of the most famous monuments in the world, the
IV. Criminals belong jail. Statue of Liberty, was presented to the United States of
America in the nineteenth century by the people of France.
a) I. in; II. about; III. for/for; IV. to
b) I. on; II. of; III. about/for; IV. to – Preencha a lacuna de acordo com o texto:
c) I. with; II. for; III. for/to; IV. on
d) I. about; II. of; III. for/to; IV. in Who was the statue presented ?
e) I. of; II. in; III. to/to; IV. at
a) from
16 | UDESC 1996 b) for
Choose the CORRECT alternative to complete the sentence: c) by
d) at
That girl the corner told everybody she is going e) in
leave New York seven
tomorrow night, a huge airplane. 21 | UEL 1997 – ADAPTED
Olajuwon should have no trouble promoting his product. "All
a) by – for – on – in – at I drink is water", says he. OVER a gallon a day.
b) under – below – by – at – for
c) through – into – onto – on – on – A palavra OVER, no texto, significa:
d) on – to – for – at – by
e) on – to – for – in – by a) sobre.
b) em cima de.
17 | MACKENZIE 1996 c) super.
Indicate the alternative that best completes the following d) mais do que.
sentence: e) abaixo de.

22 | MACKENZIE 1997
you know, Jack is a soldier, but he walks the end, he gave discussing
a general. his father and said he would go
medicine.
a) How; how
b) As; like a) In – up – with – in for
c) As; as b) At – up – with – in
d) Like; as c) In – out – about –into
e) How; like d) At – in – with – out in
e) In – at – on – up at
23 | UNESP 1986
28 | UNIRIO 1997
Assinale a alternativa correta: The word LIKE in "Premier researchers use the Net to test
projects like real-time, 3D models of colliding galaxies or
Fried potatoes are called "French Fries" the rampaging tornadoes" introduces elements of:
United States.
a) exemplification.
a) on b) generalization.
b) about c) reformulation.
c) of d) comparison.
d) from e) addition.
e) in
29 | ITA 1997 – ADAPTED
24 | UNESP 1987 Specialty Minerals do Brasil, an international, research-
Assinale a alternativa correta: based company, (I) several openings for Operator
Technicians at its plant in Jacareí. The successful candidate
Very little is known nuclear energy. will be responsible (II) operating a computer-controlled
process, performing quality control lab tests, unloading of
a) of bulk product, loading tanker trucks with finished product,
b) over and other duties as assigned. Availability (III) travel abroad is
c) in required.
d) into
e) about – As lacunas I, II a III devem ser preenchidas
respectivamente por:
25 | UNESP 1988
Assinale a alternativa correta: a) I. has, II. for, III. for
b) I. have, II. by, III. of
Aspirin is the best drug to fight headache. c) I. have, II. for, III. of
d) I. has, II. for, III. to
a) on e) I. has, II. by, III. to
b) against
c) with 030 | ITA 1997
d) to
e) for Mensagem Capadócia

26 | UNESP 1989 Um adesivo "made in Paraguai" anda circulando nos vidros


Assinale a alternativa correta: dos carros paulistanos: "Good girls go to heaven, bad girls go
to everywhere". É de doer, posto que o correto seria:
That experiment was performed important " ".
scientists.
Bárbara Gancia, Folha do São Paulo de 7/06/96.
a) by
b) to – A perspicácia da colunista se faz notar de diversas formas
c) from no texto anterior. Uma delas é através da correção de uma
d) against impropriedade normativa. Qual seria a frase final do texto,
e) for aqui omitida propositalmente?
27 | UNESP 1997 a) Good girls go heaven, bad girls everywhere.
Assinale a alternativa correta: b) Good girls go heaven, bad girls go everywhere.
c) Good girls go to heaven, bad girls to go to everywhere.
We stayed in Rome two months. d) Good girls go to heaven, bad girls go everywhere.
e) Good girls go heaven, bad girls go to everywhere.
a) since
b) at
c) in
d) on
e) for
31 | ITA 1997 – ADAPTED
034 | UFRS 1997
Probably the only thing that Brazil's two pay TV
Fill in the blank below with the best alternative:
heavyweights, Globo and TVA, agree is that the
country's multichannel business is on the verge of a boom.
Political corruption and civil unrest are Mexico's
modern problems.
– Qual a preposição que melhor preenche a lacuna?
a) because
a) for
b) between
b) on
c) throughout
c) at
d) among
d) by
e) although
e) in
035 | FEI 1997
32 | CESGRANRIO 1998
Complete:
The following sentences must be completed with "between"
or "among": Pablo said that Spain, everybody sleeps
1 and 4 PM.
I. There were no radicals her close friends.
II. The students talked quietly themselves
before the test started. a) with – among
III. The father and the mother sat in the sofa, with the baby b) among – between
them. c) between – among
IV. The Queen of England is not very popular now d) among – at
the British people. e) in – at
V. There is much difference the American and
the Brazilian education systems. 036 | MACKENZIE 1999
Indicate the alternative that best completes the following
– "Between" must be used in sentences: sentence:

a) III and V only.


She's used running the park
b) I, II and III only.
6 p.m.
c) II, III and IV only.
d) II, IV and V only.
a) for – at – at
e) I, II, III and V only.
b) for – in – at
c) at – in – before
033 | CESGRANRIO 1998
d) into – at – about
Fill in the blanks of the text below with the correct
e) to – in – after
prepositions:
37 | UNESP 1999
No higher education reforms are likely to be adopted Jim wanted to buy a ticket the nine o'clock bus.
time to affect the choice that a student or his
family makes about where the student should go to college a) for
today. For a student, not having to worry about cost would b) into
be a wonderful option. But almost every c) out
student to be able to go to school these days, working out d) of
matters of cost is an essential part choosing the e) over
best college.
38 | MACKENZIE 1998
a) on, to and in She sent a beautiful birthday card.
b) on, for and at
c) in, for and of a) for her teacher
d) in, before and in b) to her teacher
e) about, to and of c) into her teacher
d) her teacher
e) up to her teacher
039 | UFRS 1998
043 | CESGRANRIO 1999
Escolha a melhor alternativa para preencher as lacunas da Advertising is capitalism's soft sell. Girls growing up and
frase a seguir: housewives worried achieving their roles are
the foremost consumers. Advertising sets out to make
1948 an American woman was employed people identify characters
the first time a jet pilot advertisements, to make them jealous the
an American airline. person they would become if they bought the product.

a) In – at – as – for – Check the item that contains the missing prepositions that
b) During – by – like – in complete the text above:
c) From – on – with – by
d) On – for – like – by a) with – to – in – of
e) In – for – as – by b) with – with – of – of
c) with – through – of – at
040 | UEL 1998 d) about – with – in – of
The not-for-credit series of 13 interdisciplinary lectures e) about – with – on – at
focuses on the creation of myths and explores parallels to
Eva Perón and the Virgin Mary, others. 044 | MACKENZIE 1998
Christmas people usually sing .
– Assinale a alternativa que preenche corretamente a lacuna
do texto: a) On – pop music
b) On – musics
a) between c) In – lyrics
b) among d) About – lullabies
c) above e) At – carols
d) under
e) across 045 | UECE 1999
"They may find a house to live in for the winter". A partícula
041 | PUCPR 1998 IN, usada na frase, emprega-se de modo INCORRETO, no
Choose the RIGHT alternative to complete the spaces: seguinte exemplo:

I. I stayed in New York two months. a) There are three girls in the group.
II. The film didn't begin nine o'clock. b) They walked home in the rain.
III. I go there an hour. c) In her mind, he is guilty.
IV. They've been mending the road last d) He began his new job in Monday.
Monday.
V. I'll be working in a bank three years. 46 | UNESP 2000
The boys and girls ran the street.
a) by – in – since – for – until
b) for – until – in – since – for a) above
c) by – until – in – before – for b) with
d) since – by – before – until – by c) at
e) until – since – by – for – since d) down
e) back
042 | MACKENZIE 1998
47 | UFSM 2000 – ADAPTED
Indicate the alternative that best completes the following
A conflict that goes back to the 1300s cannot be solved by
sentence:
bombing the warring parties. The solution can come only
from within the Balkans and its people.
"Apartments rent are difficult
nowadays", said the tenant. – A melhor tradução para a expressão from within é:
a) on – for find a) dos.
b) for – for finding b) até o fim dos.
c) to – for finding c) de fora dos.
d) on – to be found d) à moda dos.
e) for – to find e) desde o início dos.
48 | UNESP 2001 – ADAPTED
052 | UFSM 2001
In cyberspace, we can talk, exchange ideas, and assume
A palavra em maiúsculo na expressão "BY 1900 Britain had
personae of our own creation. We have the opportunity to become a major world power" tem o mesmo sentido de:
build new kinds of communities, virtual communities, in
which we participate with people from all over the world, a) beyond.
people with whom we converse daily, people with whom we b) at
may have fairly intimate relationships but whom we may c) among.
never physically meet. d) through.
e) around.
– After
I read the text above, I could realize that my friend 53 | PUCMG 1999 – ADAPTED
Christine has a terrible problem: She lives 1204
Rudolph, skilled at surviving in the wilderness, vanished in
Reality Boulevard but her husband lives
the mountainous woods of North Carolina.
cyberspace!
– It can be inferred that Rudolph was surviving
a) in – in
in the wilderness.
b) in – on
c) on – at a) good at
d) at –on
b) worried about
e) at – in
c) interested in
d) fond of
49 | PUCRS 2001
e) anxious about
The prepositions in and on are correctly used in all
alternatives BUT:
54 | PUCPR 1999
Choose the correct alternative to fill in the blanks:
a) On September I'll be in vacation.
b) He'll go on a leave in the summer.
I. What are you thinking ?
c) She's always in a bad mood on Mondays. II. He died the injuries caused by a terrible
d) In two months you can be on the road. accident.
e) In the evenings I see her on TV.
III. They succeeded breaking the door open.
IV. Everybody laughed him when he said that.
50 | UNESP 2002 V. Why don't you concentrate your studies?
Assinale a alternativa que preenche corretamente a lacuna
da frase apresentada: a) in, of, from, on, at
b) in, of, from, at, on
There are some barriers that prevent senior citizens
c) of, from, in, at, on
taking part in an online market.
d) of, at, in, from, on
e) of, from, in, on, at
a) of
b) to 55 | UFAL 1999 – ADAPTED
c) for
Ericsson is a global leader mobile telephony,
d) from supplying state-of-the-art technology and quality equipment
e) on to customers worldwide.
051 | UFRS 2000 – Preencha corretamente a lacuna do texto:
The preposition into is used INCORRECTLY in:
a) at
a) He stood into the room, hands in his pockets.
b) in
b) The wicked witch turned the prince into a frog. c) off
c) His texts have been translated into many languages.
d) on
d) He then went into the details of his dream.
e) over
e) They ran into each other at the corner of the street.
56 | PUCRIO 2000
060 | PUCPR 2003
Mark the sentence which must be completed with on and in,
Fill in the blanks of the following sentences with the
respectively: appropriate option:
a) I was talking the phone when I heard a knock
I. They deliver the mail ten o'clock.
the door. II. it was raining, we went for a walk.
b) The boy got a bike his birthday, and is now III. Don't eat so much you go bathing.
keeping it his parents' garage.
IV. He ran away he saw the policeman.
c) The Smith family lives the countryside,
V. You won't win you try hard.
a very cozy farm house.
d) John was invited to speak the conference a) I. after; II. While; III. unless; IV. before; V. until
behalf of the company's president.
b) I. until; II. Before; III. after; IV. while; V. although
e) Several workers decided to go strike c) I. unless; II. When; III. until; IV. after; V. before
the same day their boss announced his d) I. at; II. Although; III. before; IV. when; V. unless
bankruptcy. e) I. before; II. Until; III. although; IV. unless; V. when

057 | UFV/PASES 2000 061 | UFRS 2004


Choose the best option to complete the sentence: Complete the sentence below with the best alternative:

Bell used electricity to send the human voice Tolkien wrote much his trilogy
one place another. World War II, but denied that his stories were analogous
that great battle.
a) on – in
b) from – to a) of – during – to
c) in – to b) of – at – with
d) at – to c) about – during – into
e) above – below d) for – in – to
e) of – in – into
058 | UFRS 2001
The word into is used correctly in all alternatives below 062 | PUCMG 2005 – ADAPTED
EXCEPT: According to Stuff magazine, the iPod is the "coolest thing to
come out of California since the Beach Boys".
a) He ran into some old friends at the airport.
b) He remained into that room where they had always met. – The word "since" conveys an idea of:
c) She walked into his life as a breath of fresh air.
d) It came into view when the clouds cleared the sky. a) manner.
e) She went into the house carrying a bunch of flowers. b) place.
c) time.
059 | UFRS 2002 d) result.
Complete the following sentence with the correct
alternative: 063 | UFSM 2005
The first games athletes a disability
In New England, we drove hours along country held 1948.
roads and stayed an old sea captain's home
the sea. – Selecione a alternativa que completa corretamente as
lacunas:
a) for – in – off
b) during – into – by a) for – without – was – on
c) up – near – from b) to – with – has – at
d) during – at – out c) from – with – is – on
e) for – in – by d) to – without – are – at
e) for – with – were – in
064 | UFRS 2005
068 | JFS 2000
Fill in the gaps in the following sentence correctly:
The cat jumped the table in order
get the food that was it.
Man walked the moon the first
time 1969. a) up – to – on
b) about – for – up
a) on – for – in c) over – for – about
b) across – at – in d) on – to – on
c) across – by – on e) onto – to – on
d) in – on – at
e) on – at – after 069 | ITA 1991
A alternativa abaixo que preenche a lacuna de:
065 | JFS 2012
Fill in the following sentences correctly: Buses here never arrive time.

I. The climbers stopped 300m the top of the – Dando idéia de pontualidade:
mountain.
II. The whole village is water. a) on
III. A king shouldn’t do anything his dignity. b) at
IV. The tunnel goes right the city. c) in
d) by
a) I. below – II. below – III. beneath – IV. underneath e) up
b) I. below – II. under – III. beneath – IV. underneath
c) I. below – II. under – III. beneath – IV. under 070 | ITA 1991
d) I. under – II. under – III. below – IV. beneath what he says, she was born March
e) I. under – II. below – III. below – IV. beneath 25, 1970.

066 | UNIFESP 2007 – ADAPTED a) According to – in


Em "Since levels of lung function were in the normal range b) According with – on
at the start of the study, the researchers say, the possibility c) Accordance to – in
that poor lung function led to hostility rather than the other d) According to – on
way around is unlikely", a expressão rather than significa, e) Accordance with – in
em português:
071 | ITA 1992
a) pelo contrário. A alternativa que corretamente preenche os claros (I), (II),
b) ao invés de. (III) de:
c) a menos que.
* The Declaration of Independence was signed
d) mais que.
(I) July 4, 1796.
e) devido a.
* (II) first, I thought you were a thief.
* (III) present, I am living in Brazil.
067 | ITA 1995
'Without Fear of Be Happy' (Sem medo de ser feliz) é o título, – é:
em inglês que um periódico paulista atribuiu ao livro do
jornalista americano Ken Silverstein sobre a campanha de a) at – at – at
Lula à Presidência da República em 1989. Examinando o b) on – at – at
título, você diria que: c) on – by – in
d) in – in – in
a) Está estruturalmente correto. e) in – by – on
b) Deveria ser: "Without Fear of Been Happy"
c) Deveria ser: "Without Fear of to Be Happy" 072 | AFA 1999
d) Deveria ser: "Without Fear of Being Happy" 14% the force female, we cannot
e) Deveria ser: "Without Fear to Be Happy" run a military today women.

a) At / in / on
b) With / from / over
c) With / of / without
d) Both / in / without
073 | FUVEST 1977
78 | EFOMM 2006
Qual a sentença correta? I made seven different plans for my vacation, however
the end I went to the Bahamas again.
a) We were deprived from playing games for a week.
b) We were deprived of playing games for a week.
a) to
c) We were deprived of playing games through a week. b) by
d) We were deprived at playing games for a week.
c) with
e) We were deprived from playing games by a week.
d) in
e) at
074 | EEAR 2007
Choose the best alternative to complete the blanks:
79 | EFOMM 2006
There is nothing illegal about my business dealings.
Julie was born July 3, night Everything is strictly board.
New York.
a) on
a) in / at / at b) above
b) on / at / in c) by the
c) in / at / in d) over
d) on / in / at e) onto

075 | EFOMM 2005 080 | EFOMM 2006


Choose the only option with the correct preposition: The word since can be used to appropriately fill in the blank
of which sentence below?
What earth are you doing here so early in the
morning? a) I am studying here 2004.
b) She has slept two hours.
a) under c) They had been staying at the hotel many days
b) on before they decided to move to an apartment.
c) in d) Bob has worked at that big company he
d) at moved to Washington.
e) behind e) Carol lives in Boston she was born.

76 | EFOMM 2006 081 | EFOMM 2007


Choose the only option with the correct preposition: Crowded a lifeboat, Ruth Becker stared
disbelief as the luxury liner Titanic slipped
The man jumped the horse and went away. the icy Atlantic Ocean.

a) of a) in / in / in
b) under b) in / in / into
c) out of c) on / on / onto
d) into d) on / on / onto
e) onto e) on / in / into

77 | EFOMM 2006 082 | UFPE 1998


We’ve had such a busy day! At least twelve people called. "On this planet, more people get their news from TIME than
Oh, the way, there’s a message here for you any other single source – over 30 million people,
from your cousin. worldwide."

a) over TIME, August 12, 1996, Vol. 148, N° 7, page 2.


b) in
c) by – OVER in "OVER 30 million people" signifies:
d) on
e) into a) less than.
b) by.
c) not as much as.
d) not so many as.
e) more than.
083 | JFS 2008
a) in / under / in / on / on
Ryan drove me without stopping and drove off
b) at / under / at / on / off
the downtown.
c) in / in / on / in / in
d) at / in / in / at / off
a) from / into
e) at / on / at / in / on
b) towards / over
c) along / up
087 | JFS 2011
d) past / towards
Fill in the gaps correctly:
e) in / next to
He aimed the bird and shot it!
84 | EFOMM 2010
Choose the option in which the prepositions complete the a) at – in
verb phrases with accuracy, respectively: b) on – at
c) on – on
I. The ship is bound Africa. d) at – on
II. We set sail a tour of the Caribbean. e) at – at
III. The captain was responsible the incident.
IV. Attention has to be given the weather 088 | UFV 2003
conditions. The expression regardless of in the sentence "You can profit
V. Crew members are expected to comply from it regardless of your level of formal education", can be
safety regulations. replaced by:

a) for / to / for / for / with a) unless.


b) in / for / for / to / to b) because.
c) to / for / for / to / with c) although.
d) to / for / to / to / to d) however.
e) for / to / to / for / with e) in spite of.

85 | EFOMM 2010 089 | JFS 2011


Choose the correct option to complete the sentences: Which prepositions must be used to fill in the gaps in the
sentences below?
I. I am familiar that song.
II. Were you aware the regulations against - The operation, unprecedented in the city's history, began
smoking in this area? around 8am.
III. What John said is contrary common sense. - According to police the favela had been "conquered"
IV. This winter, Paul will be eligible a three- around 9.30am.
week vacation.
a) at – by
V. How do you account this discrepancy?
b) at – in
a) with / of / with / to / for c) at – on
b) with / of / to / for / for d) on – by
c) to / about / to / to / to e) on – at
d) with / about / with / to / of
090 | UNIRIO 2002 – ADAPTED
e) to / of / to / for / to
In the 1960s, only about 200 golden lion tamarins remained
86 | EFOMM 2010 in the wild, DUE TO the destruction of their habitat, Brazil's
Choose the option in which the prepositions complete the Atlantic coastal rain forest – 'Mata Atlântica'.
sentences with accuracy, respectively:
– The capital word discourse marker due to in the text above
I. Stress can make us quite forgetful illustrates:
times.
II. The New York Port Authority operates daily a went off. They were able to act promptly though.
lot of pressure.
III. After a storm, the fishing boat was lost sea.
IV. The remains of the wreckage were found the
shore.
V. They were call when the emergency alarm
a) result.
b) consequence.
c) purpose.
d) reason.
e) contrast.
Conjunctions 5 | UNIRIO 1995 – ADAPTED
Research shows that sunscreens may not be as effective as
001 | ITA 1995 hoped at preventing sunburn. Users may be spending long
'It's "weird" he muses, 'we're getting more comfortable hours in the sun with a false sense of security, and though
playing live, and we're playing more consistently every night. lotions may protect against sunburning UVB rays, it does
Yet sometimes it's more difficult to get in that state of mind little to block out the potentially more dangerous UVA rays.
where you can just lose yourself to the music.'
– The word THOUGH (ref.:2) can be replaced with:
Stone Gossard, Pearl Jam
a) but.
– A palavra "Yet", relacionada no texto, poderia ser b) however.
substituída por: c) therefore.
d) besides.
a) Furthermore. e) yet.
b) And.
c) Nevertheless. 6 | FGV 1995 – ADAPTED
d) Already. The idea that executives need to fly on business class
e) Rather. they can work is bogus.

2 | UNITAU 1995 – Assinale a alternativa que preenche corretamente a lacuna


Assinale a alternativa que corresponde a um sinônimo para a do texto:
palavra, em destaque, a seguir:
a) though
DESPITE the growing use of computers in classrooms b) so
American universities are still graduating millions of c) for
technological illiterates. d) until
e) since
a) spite
b) spite the 007 | FGV 1995
c) in spite of The new economic order was supposed to bring rapid
d) spite of growth for the industrialized nations, as emerging capitalist
e) spite of the countries joined a global free trade system
brutal competition from the Third World and the Soviet
3 | CESGRANRIO 1995 block has stalled the developed nations.
The sentence "Though overshoppers later experience
considerable remorse, they find shopping exciting" contains – A palavra que preenche melhor a lacuna do texto é:
an idea of:
a) Because.
a) addition. b) Instead.
b) alternative. c) Moreover.
c) cause. d) Therefore.
d) condition. e) While.
e) contrast.
008 | ITA 1996
4 | UNIRIO1995 We don't believe your needs should have to wait just
A palavra que poderia substituir a expressão AS A RESULT because it's 2 o'clock in the morning.
em "As a result, Struve claims, he can halve the time
required to grow a 1.5-inch diameter red oak" sem alteração – A palavra because, em destaque no texto, poderia ser
do significado é: substituída por:

a) Therefore. a) while.
b) However. b) how.
c) Moreover. c) like.
d) Besides. d) since.
e) Anyhow. e) for.
9 | MACKENZIE 1996
013 | UEL 1995
Yes, I know Mario quite well; ,I to A lacuna é corretamente preenchida pela alternativa:
see him at the club last weekend.
he is lazy, he makes a lot of money.
a) furthermore – had
b) thus – wanted a) But
c) in fact – happened
b) Thus
d) whereas – waited
c) Due to
e) despite – liked d) Unless
e) Although
10 | MACKENZIE 1996 – ADAPTED
Corporations can no longer afford lifetime employment and
14 | CESGRANRIO 1993
the seniority system, whereas young workers do not
Freedom, however, poses its own problems. The biggest
consider company life the most important.
advantage is that you are in charge of everything. And the
biggest disadvantage is that you are in charge of everything.
– The meaning of whereas in the text is: Typical problems include a sense of isolation, lack of
motivation, and, conversely, the feeling that you can never
a) sufficiently.
get away from your work.
b) theoretically.
c) at present. – The word CONVERSELY means:
d) intensely.
e) while.
a) on the other hand.
b) additionally.
11 | MACKENZIE 1996
c) firstly.
Indicate the alternative that best completes the following d) especially.
sentence: e) even though.
He had a headache; , he the
15 | MACKENZIE 1996
invitation.
Indicate the alternative that best completes the following
sentences:
a) hence – declined
b) then – dismissed
I. Study harder; you'll fail.
c) otherwise – refused
II. I know nothing about it; , I can't help you.
d) so – quit III. The editors continue to publish, irregularly,
e) however – failed two journals.
IV. being a good actor, he is also an excellent
12 | UFPR 1994 – ADAPTED soccer player.
IN ADDITION, much of the water is polluted and salty.
a) I. otherwise; II. thus; III. albeit; IV. Besides
– In the sentence above, the expression in capital letter can
b) I. although; II. therefore; III. for; IV. And
be replaced by:
c) I. so; II. so that; III. but; IV. Moreover
d) I. or else; II. consequently; III. besides; IV. Also
1) Consequently
e) I. also; II. as a consequence; III. then; IV. In addition
2) Besides
04) Also
16 | UNIRIO 1996
08) In contrast
The word SO in "So despite the extent to which loneliness
16) As soon as
affects nearly everyone at various times, it presents a
32) However
challenge to researchers" expresses:
64) Moreover
a) comparison.
a) 01 + 02 + 04 + 64 = 71 b) consequence.
b) 01 + 02 + 08 + 16 = 27 c) purpose.
c) 02 + 04 + 08 + 32 = 46 d) contrast.
d) 02 + 04 + 64 = 70 e) cause.
e) 02 + 04 + 32 + 64 = 102
017 | FAAP 1997
022 | CESGRANRIO 1991 – ADAPTED
I don't know she is as optimist or a pessimist In the sentences:
a) each
1. Now, if this process happens each time we eat sugar, we
b) how many
can see that eating excessive amounts of sugar causes more
c) weather and more tooth decay.
d) like that
2. However, sweets are often eaten as snacks between
e) whether
meals and during the day, times when people generally do
not brush after eating.
018 | CESGRANRIO 1990
3. Therefore, the dangerous process of tooth decay is
Mark the word that can appropriately be used to fill the
allowed to continue.
blank and expand the sentence "Men not only cry less
frequently, they also do it somewhat – The words NOW, HOWEVER and THEREFORE could be
differently. substituted by:

a) yet a) Than – But – Thus


b) but b) These days – Perhaps – So
c) and c) Because – Although – Meanwhile
d) although d) Recently – Even though – Besides
e) however e) Well – Nevertheless – Consequently

19 | CESGRANRIO 1990 023 | UEL 1997


A less significant but perhaps more curious use of the laser in A lacuna é corretamente preenchida pela alternativa:
medicine is to remove tattoos. Whereas before tattoos were
virtually impossible to remove without considerable I'd like to talk to him he arrives.
difficulty and pain, now they can be removed relatively
painlessly. a) while
b) rather than
– The word WHEREAS means: c) since
d) as soon as
a) as. e) because
b) when.
c) while. 024 | UFF 1997
d) where. In the sentence "It was as if he forgot who I was", AS IF
e) because. means:

20 | CESGRANRIO 1991 a) even though


The clause "As one eats…" as in" As one eats, particles of the b) as though
sugary food get stuck between the teeth and around the c) although
gums" can be paraphrased as: d) as for
e) if possibly
a) While we eat...
b) When they eat... 25 | PUCSP 1998
c) Though we eat... No período "The struggle to have a piece of land to work on
d) Because you eat... for a decent living has produced rifes and conflicts between
e) As one thing is eaten... the landless peasants, ON ONE HAND, and the powerful
landowners and the government, ON THE OTHER", as
21 | UEL 1997 – ADAPTED expressões on one hand e on the other indicam uma relação
you want to stay young, sit down and have a de:
good think.
a) alternância.
a) Though b) adição.
b) But c) oposição.
c) So d) consequência.
d) Then e) causalidade.
e) If
26 | CESGRANRIO 1992 – Uma outra forma de se escrever o trecho "despite
'A good story,' he thought. So he went there – it took 36 concerns about monthly book/bill ratios", em maiúsculo, no
hours by train and canoe. texto é:
– The relationship between these two sentences is NOT one a) ...in case of the concerns about monthly book/bill ratios.
of: b) ...because of the concerns about monthly book/bill ratios.
c) ...due to the concerns about monthly book/bill ratios.
a) consequence. d) ...rather than getting concerned about monthly book/bill
b) conclusion. ratios.
c) result. e) ...although they are concerned about monthly book/bill
d) cause. ratios.
e) time.
030 | UFRS 1996
027 | ITA 1998 subdue the new territory, Americans had to
In an early article, McKay suggested that the occurrence of face severe conditions.
PAHs (polyaromatic hydrocarbons) and textural and
mineralogical features in the Martian meteorite ALH84001 a) So as
were consistent with the presence of past life on Mars. A b) In order that
series of technical comments and responses address c) As for
whether abiotic processes could have instead produced d) In order to
these features. e) By order that
SCIENCE – December 20, 1996. 031 | UERJ 1997
The word BUT in the sentence "But she's the only one"
– Assinale a conjunção que poderia ligar as idéias contidas indicates:
nos dois períodos do texto anterior:
a) cause.
a) Whereas b) contrast.
b) Moreover c) addition.
c) Thus d) conclusion.
d) However
e) Hence 032 | UFPB 1998
In the following verses:
28 | ITA 1997 – ADAPTED
At present, neither offers full access to the Internet – they And can understand nothing
are linked by a "gateway" through which e-mail can be sent But the unusual laughter
and received, but which denies access to many of the
delights the Net has to offer. , both companies – “But” means:
are widening the gateways in the near future.
a) however.
– A alternativa que melhor preenche a lacuna é: b) also.
c) although.
a) Besides. d) because.
b) In addition. e) except.
c) However.
d) Furthermore. 33 | UERJ 1998 – ADAPTED
e) Finally. Consumers can start questioning advertising in the category
generally even the intent is to provide worth-while
29 | ITA 1997 – ADAPTED information.
Many manufacturers believe that the only MEANS to greater
production capacity is augmenting the old with the new. – The underlined word indicates:
Virtually all of the world's LEADING semiconductor
companies are building new fabs to satisfy projected a) comparison.
demand, DESPITE CONCERNS ABOUT MONTHLY BOOK/BILL b) purpose.
RATIOS. c) addition.
d) contrast.
34 | UERJ 1998 – ADAPTED
038 | UFRS 1998
Photojournalists are often people who feel a powerful social
all the conquests already achieved by women, a
responsibility to document the atrocities of humanity IN
lot still remains to be done.
ORDER TO provide evidence to the world.
a) In spite of
– The capital world expression is a marker of:
b) Because of
c) Although
a) result.
d) In order to
b) reason.
e) Supposing
c) purpose.
d) consequence.
39 | UECE 1998 – ADAPTED
Assinale a frase em que THAT não é pronome relativo:
35 | UNIRIO 1998
The word AS in "And as stress begins to ebb you may find
a) Knowing THAT Mrs. Mallard was afflicted with a heart
that it's replaced by a renewed sense of possibility and
trouble, great care was taken to break to her as gently as
optimism" expresses the idea of:
possible the news of her husband's death.
b) There stood, facing the open window, a comfortable,
a) consequence.
roomy armchair. Into this she sank, pressed down by a
b) comparison.
physical exhaustion THAT haunted her body and seemed to
c) purpose.
reach into her soul.
d) contrast.
c) There was something coming to her and she was waiting
e) time.
for it, fearfully. What was it? She did not know; it was too
subtle and elusive to name. But she felt it, creeping out of
036 | UFRS 1998
the sky, reaching toward her through the sounds, the scents,
In case you are childless or an alien from outer space and
the color THAT filled the air.
managed to avoid this craze, the Tamagotchi is a Keychain-
d) She was beginning to recognize this thing THAT was
size plastic egg that houses a small LCD (Liquid Crystal
approaching to possess her.
Display) in which "lives" a creature that you nurture by
pushing a variety of buttons.
40 | FUVEST 1999
Choose the item which best completes the sentence,
– A expressão "in case" pode ser substituída, sem alteração
according to the passage:
de sentido, por:
some Scottish fans bare their bottoms, they are
a) So.
readily forgiven.
b) As.
c) When.
a) No matter
d) Indeed.
b) Always if
e) If.
c) All the time
d) Even when
037 | UFRS 1998 – ADAPTED
e) In spite of
Na frase "Moreover, PCs and telecommunications
technologies have enabled more women and men to work at
41 | MACKENZIE 1998
home, increasing employment options and bringing the
Indicate the alternative that best completes the following
perennial kids-and-career battle to an end", a palavra
sentence:
MOREOVER pode ser substituída por:
She cleaned the house she ironed the clothes.
a) However.
b) Such as.
a) whatsoever
c) In addition to that.
b) and after that
d) Provided that.
c) nevertheless
e) Notwithstanding.
d) so
e) and after
42 | MACKENZIE 1998
046 | ITA 1999
Indicate the alternative that best completes the following Considerando a "charge" a seguir bem como a sua tradução,
sentence:
assinale a opção cuja conjunção corresponda à palavra que
foi excluída (entre parênteses) da fala do assessor de
He is very mature .
Clinton:
a) spite of his age
b) despite his age
c) instead of his age
d) despite of his aging
e) in spite his age

43 | MACKENZIE 1998
Indicate the alternative that best completes the following
sentence:

Philip never borrowed money:

a) not either from his parents or from the bank


b) neither from his parents or from the bank "A opinião pública ainda está a seu favor...
c) both from his parents and from the bank a não ser que ela esteja mentindo para você".
d) either from his parents or from the bank
e) neither from his parents and from the bank a) although
b) whereas
44 | PUCRIO 1999 – ADAPTED c) in spite of that
Compare that job market to the prospective immigrant labor d) unless
force. Of recent arrivals, only 63% have finished high school. e) for
No surprise that, while immigrants make up only 12% of
today's workforce, they clean half the restaurant tables in 047 | ITA 1999
the U.S. Yet immigrants also are 50% more likely than Leia o recado de Ho Chi Minh aos franceses, em 1946:
Americans to have a graduate degree.
"You can kill 10 of my men for every one I kill of yours, yet
– The sentence "Yet immigrants also are 50% more likely even at those odds, you will lose and I will win".
than Americans to have a graduate degree" introduces:
– Assinale a opção cuja conjunção tenha significado
a) a conclusion. semelhante ao de "yet":
b) a definition.
c) a contrast. a) therefore
d) a result. b) despite
e) an example. c) thus
d) moreover
45 | PUCRIO 1999 – ADAPTED e) however
As you are reading these words, you are taking part in one of
the wonders of the natural world. For you and I belong to a 048 | UERJ 1999
species with a remarkable ability: we can shape events in Of course the programmers who created this system could
each other's brains with exquisite precision. foresee this would cause a problem, but AS WELL AS a
degree of short-termism there was a widespread disbelief
– In "For you and I belong to a species... precision", the that anyone would be using the same computers and
conjunction FOR can be understood as: programs nearly thirty years later.

a) however. – The expression "as well as" indicates:


b) already.
c) and. a) conclusion.
d) because. b) condition.
e) therefore. c) contrast.
d) addition.
49 | UFRRJ 1999 – ADAPTED
54 | ITA 2000 – ADAPTED
The explosive growth in these regions is DUE not only TO
But what has been so frustrating about the market reactions
high birth rates but to the young age at which mothers have
in recent months is that despite the surging economy,
their children, telescoping the time between generations.
inflation has not been rising. It has remained flat, at around
3 percent, and yet Wall Street, certain that the shadow it
– The expression DUE TO could be replaced by:
sees is the ghost of higher inflation come to haunt the
trading floors, has been clamoring to the Federal Reserve for
a) since.
higher rates. (...)
b) towards.
c) from.
The New York Times Magazine. May 22, 1994.
d) because of.
e) during.
– "Yet" quer dizer:
50 | UFRRJ 1999
a) apesar disso.
The construction "such a... that" as in "Eating is such a
b) ainda.
passion here that it even shapes the cityscape" expresses:
c) já.
d) consequentemente.
a) condition.
e) até o momento.
b) purpose.
c) result.
55 | UNIRIO 2000
d) contrast.
The teenager believes that Camden has a bad
e) manner.
reputation, it can change for the better.
051 | UFSM 1999
– The option which completes correctly and meaningfully
The new phones can be programmed to dial only a few
the sentence above is:
numbers, LIKE home, or a parent's office.
a) in spite.
– A palavra em destaque indica uma relação de:
b) even though.
c) because.
a) oposição.
d) unless.
b) comparação.
e) however.
c) exemplificação.
d) adição.
56 | UNIRIO 2000
e) causa – efeito.
Speech is natural, , we learn to speak before we
learn to read and write.
52 | PUCRIO 1998
In the sentence "If you wrote about the international
– The item which completes the sentence above in a
banking systems for bankers, your language and information
meaningful way is:
would be more technical", the author intends to transmit an
idea of:
a) but.
b) although.
a) conclusion.
c) however.
b) addition.
d) nevertheless.
c) condition.
e) therefore.
d) comparison.
e) contrast.
057 | UERJ 2000
SIMILARLY, there has been a mad dash to book cabins on
53 | UNESP 2000
cruise ships.
Mr. Foley was reading the newspaper, Mrs.
Foley was watching television.
– The capital word in expression indicates:
a) While
a) comparison.
b) As long
b) explanation.
c) In the meantime
c) conclusion.
d) Because of
d) analogy.
e) How
058 | UERJ 2000
63 | ITA 2001 – ADAPTED
Crews may need to watch radar screens FOR oncoming
traffic instead of popping corks. SINGAPORE

– The capital word has the idea of: In a 1975 survey, only 27 per cent of people over age 40
claimed to understand English, whereas among 15-20-year-
a) cause. olds, the proportion was over 87 per cent. There is also
b) purpose. evidence of quite widespread use in family settings. In such
c) direction. an environment, therefore, it is not surprising that a local
d) explanation. variety ('Singaporean English') should have begun to emerge.

059 | UERJ 2001 MALAYSIA


A single conjunction may express multiple meanings. In
"Short of trying to collect every book in existence, as the Malay-medium education was introduced, with English as an
ancient library did", the word AS conveys the idea of: obligatory subject but increasingly being seen as a value for
international rather than intranational purposes – more a
a) cause. foreign language than a second language.
b) comparison.
c) simultaneity. The Cambridge Encyclopedia of the English Language
d) contradiction. David Crystal – CUP, 1995

060 | UFPE 2001 – ADAPTED – Sinônimos para THEREFORE (texto sobre Singapura) e para
The giant panda is China's national symbol. But it is an RATHER THAN (texto sobre Malásia) são, respectivamente:
endangered species, with just 1,000 animals believed to
exist in the wild. a) however – more than
b) altogether – before
– In "BUT it is an endangered species" the connective BUT c) thus – despite
gives the idea of: d) as a consequence – and
e) consequently – instead of
a) consequence.
b) conclusion. 64 | PUCRS 2001 – ADAPTED
c) contrast. In an era of feminist and politically correct values, not to
d) addition. mention the belief that all men and women are created
e) emphasis. equal, the fact that all men and women are not – and that
some are more beautiful than others – disturbs, confuses,
061 | UFF 2001 even angers.
In the sentence, "In the meantime, Judge Robert Kaye will
issue his final judgment on the jury's verdict", IN THE – The word "even" is used in the same meaning as in:
MEANTIME could be replaced by:
a) We will not be even until you pay me.
a) During. b) Even if he attends, he may not participate.
b) Nevertheless. c) He is willing, even eager, to do it.
c) Actually. d) She numbered all the even pages of the book.
d) Meanwhile. e) After washing, the colors may not be even.
e) However.
65 | PUCRS 2002 – ADAPTED
062 | UEL 2001 Once they are agreed, the new guidelines will go through a
Na sentença "Carr, HOWEVER, thought of a way to spot series of trials, at first primarily at Cambridge University,
them", a palavra HOWEVER poderia ser substituída por: before being issued nationally.

a) nevertheless. – O termo "Once" pode ser substituído por:


b) also.
c) since. a) Before.
d) never. b) As soon as.
e) but. c) If.
d) Unless.
e) At any time.
66 | PUCRS 2002 – ADAPTED
69 | PUCRIO 2002 – ADAPTED
The first time I saw Walter Salles's brilliant Brazilian film The sentence "While French at one time was the foreign
Central Station (Central do Brasil) was on the same day I had language most favored by educated Brazilians, nowadays
to see Robin Williams's "Patch Adams", which is more or less many younger citizens are required in school to learn English
its American counterpart. (…) Both are about optimism in as a second language and are seen ¡chatting in English at the
the face of adversity, with Salles dealing with the mall, exercising their bilingual skills" expresses an idea of:
reformation of a cynical woman, while Williams strives to
regenerate the entire medical profession. a) addition.
b) result.
– A palavra while é usada com o mesmo sentido no texto e
c) definition.
na alternativa:
d) conclusion.
e) contrast.
a) We must have been burgled while we were asleep.
b) You can go swimming while I am working on this paper.
70 | MACKENZIE 2002
c) While I am willing to help, I do not have too much time
In which of the sentences can "since" be replaced by
available.
"because"?
d) While you are good at science, your friend is absolutely
hopeless. a) I have enjoyed science since I was a child.
e) They chatted for a while before they walked into the b) I am interested in this article since I like science.
theater. c) I've been studying science since 1988.
d) Lots of strange things have happened here since last
67 | UNIFESP 2002 – ADAPTED
week.
Na frase "The rise of molecular biology since the late 1950s e) Since his trip to the States, he's been very ill.
has had the gradual and quite unforeseen effect of turning
the eyes of medical scientists increasingly toward the basic 71 | PUCSP 2002
mechanisms of life, rather than disease and death", a Among married women, labor-force participation rates rose
expressão rather than pode ser substituída, sem mudar o 10 percentage points per decade for each 10-year period
sentido, por: from 1940 to 1990. So today, the rates are over 70 percent
for all women age 25 to 64, and a little over 80 percent for
a) even if. women who have bachelor's degrees.
b) in order to.
c) moreover.
– Na frase "So today, the rates are over 70 percent for all
d) furthermore
women age 25 to 64", a palavra so pode ser substituída, sem
e) instead of. alterar o sentido, por:
68 | UNIFESP 2002 – ADAPTED
a) Therefore.
Until the mid-1960s, medical research was primarily driven b) If.
by the desire to solve the problems of sick people. Although c) However.
Aristotle was what might be termed today a pure laboratory d) Nevertheless.
investigator, with no thought of the clinical usefulness of his e) Even if.
findings, the vast majority of those physicians later
influenced by his contributions to biology were trying to 72 | FATEC 2000
solve the mysteries of human anatomy and physiology for Assinale a alternativa que contém uma palavra equivalente a
the distinct purpose of combating sickness. thus, empregada no seguinte texto: "A child who spends too
much time on video games may not disengage from a
– A palavra "although" indica uma idéia de:
simulated world and THUS may be confused in the real one":
a) alternância. a) although
b) exemplificação.
b) instead
c) oposição.
c) therefore
d) condição. d) however
e) enumeração
e) whereas
73 | UFRS 2000 – ADAPTED
76 | PUCSP 2001 – ADAPTED
Swiss cheese? Roquefort? Is it Gouda? Well... we all know Na frase "To help achieve these goals, researchers also are
the moon isn't really made of cheese. Cheese is just one of studying the genetic makeup of several nonhuman
the many different images seen in the charcoal-gray, black, organisms", a palavra TO pode ser substituída por:
and white markings created by various lunar craters and
basins. (…) The Chinese see a rabbit in the dark areas and a a) such.
toad in the white. b) in order to.
c) because.
(Fonte: Astronomy, Sept. 1999) d) from.
e) then.
– De acordo com o texto, complete a sentença abaixo com a
alternativa correta: 77 | MACKENZIE 2001
Indicate the alternative that best completes the sentence:
The Chinese can see a rabbit a toad
on the lunar markings. The road was in bad condition:

a) neither – nor a) despite, I was very late.


b) and – or b) however, we didn't stop.
c) either – and c) nonetheless, the road was long.
d) both – or d) in fact, it was 7 o'clock.
e) both – and e) if I had more time, I'd call you.

74 | UFSM 2001 – ADAPTED 78 | MACKENZIE 2001


A Florida panther rests quietly in the Everglades of it rained hard, the plane took .
southwestern Florida. ALTHOUGH protected by the
Endangered Species Act, only 30 Florida panthers are a) In spite of – on
believed to survive in the Everglades victims of disease and b) Unless – out of
shrinking habitat as well as illegal hunting and automobiles. c) If – out
d) Although – off
– A palavra destacada no excerto acima estabelece uma e) But – over
relação de:
079 | UERJ 2002
a) finalidade. What they have in common is the sight of Brazil as it sheds
b) adição. its image as eternally easygoing.
c) tempo.
d) concessão. – The two instances of the word AS in the sentence above
e) condição. establish the following semantic relations:

75 | PUCSP 2001 – ADAPTED a) causality and addition


Besides providing clues to understanding human biology, b) alternation and purpose
learning about nonhuman organisms' DNA sequences can c) concession and contrast
lead to an understanding of their natural capabilities that d) temporality and comparison
can be applied toward solving challenges in health care,
energy sources, agriculture, and environmental cleanup. 80 | UFSCAR 2000
That's not to say humanity can't become extinct. A 50-mile-
– A palavra BESIDES, sublinhada no fragmento acima, indica wide asteroid crashing down from space would do it. So
uma relação de: could a sudden and thorough collapse of earth's ecosystem
through pollution, deforestation and the like – unless we
a) adição. establish some colonies in space beforehand.
b) oposição.
c) consequência. – A palavra unless indica uma relação de:
d) exemplificação.
a) adição.
e) finalidade.
b) exemplificação.
c) ressalva.
d) oposição.
e) consequência.
81 | PUCRS 2000 – ADAPTED
085 | FURG 1999
The word while as in "The groom got the idea of corporate GOOD NEWS Some of the most notorious chemicals in the
sponsorships while working in a small struggling animation 1960s and 1970s such as the pesticide DDT, and PCBs – used
studio that often had to barter for services" is used with the in a variety of goods, from electrical equipment to paint –
same meaning as in: have been banned or heavily restricted. Shipments of toxic
waste are carefully controlled by an international treaty.
a) I watched the show a while ago.
However, we have little idea of the long-term effects of all
b) Sabrina is blonde while Tom is redheaded.
but a few of the 70,000 and more chemicals that are in
c) While she knows he's not rich, she still wants to marry
regular use.
him.
d) I met her while I was studying at PUC. – O termo HOWEVER é empregado para:
e) We haven't seen them for a while.
a) expressar idéia de tempo.
82 | UFRN 2000 – ADAPTED b) dar idéia de consequência.
Through Lisbon flowed not only the gold of Brazil and West c) fazer referência a uma idéia já mencionada.
Africa and the spices of the East Indies, but new treasures of d) apresentar idéias semelhantes.
knowledge, new maps, accounts of different peoples and e) introduzir uma idéia que se opõe à anterior.
societies, new animal and botanical species.
086 | PUCRS 1999 – ADAPTED
– A expressão correlativa not only... but indica: But neither the park nor the mountains are enough to
explain why Hay attracts so many visitors.
a) adversidade.
b) exclusividade. The expression "neither... nor" excludes both "the park" and
c) negação. "the mountains." If they were to be included, the correct
d) adição. expression(s) would be:

83 | FEI 2000 – ADAPTED I. not only... but also


He'd drifted into an alternative school for troubled youths, II. either... or
where, despite special classes, Frank's principal still III. as well as
considered him "the worst kid of the whole lot."
– The correct alternative is:
– De acordo com o texto, como você traduz DESPITE?
a) I.
a) No lugar de. b) I and II.
b) Apesar de. c) I and III.
c) A respeito de. d) I, II and III.
d) Enquanto que. e) II and III.
e) Em vez de.
087 | UNIT 1999
The sentence "Although no one can predict the full effect to
84 | FATEC 1999 – ADAPTED
the current information revolution, we can see changes in
Because of its vision-disturbing side effects, the drug has
our daily lives" expresses an idea of:
also been suspected of contributing to at least one plane
crash. Indeed, a Federal Aviation Administration pamphlet a) addition.
recommends a prudent "six hours from Viagra to throttle". b) cause.
Still, the news about Viagra is mostly happy. It works for c) contrast.
many men, and Pfizer, the manufacturer, estimates that 5 d) time.
million prescriptions have been written. e) consequence.
– Assinale a alternativa que apresenta a palavra que 88 | MACKENZIE 2000
expressa a mesma idéia de still em "Still, the news about capable of walking upright, medieval men did
Viagra is mostly happy": so for short periods of time.
a) Thus. a) As if
b) Therefore. b) Since
c) Hence. c) Until
d) Nevertheless. d) Because
e) Finally. e) Though
89 | MACKENZIE 2000
94 | UNIRIO 2002
Choose the alternative in which WHILE is being used to
Whether it spreads joy or alarm, the body-shaping trend is a
express a contrast: boom that is still growing.
a) While the machines are working, let's have some coffee.
– The capital word discourse marker WHETHER introduces a
b) I usually take a shower while my mom is preparing (an):
breakfast.
c) What were you doing while Peter was studying?
a) result;
d) While she was in bed, he read the paper.
b) example;
e) My dad is a dreamer, while my mom is too realistic
c) condition;
d) comparison;
90 | UFRRJ 2000
e) choice of alternatives.
"I was giving a talk in a large auditorium in New England
WHEN A WOMAN SITTING IN THE BALCONY STOOD UP". 095 | JFS 2012
Mark the synonym to the words in bold in the sentence
– The selected passage expresses an idea of: below:
a) purpose. Inasmuch as you are the older employer, you are
b) cause. responsible for the performance of these men.
c) place.
d) time.
a) Unless
e) condition.
b) Still
c) In addition
091 | UFSM 2002
d) Since
There is great evidence of the benefits of Yoga, , e) Hence
some people still question this practice.
96 | ITA 2003 – ADAPTED
a) consequently If all my relatives suddenly died and all my friendships dried
b) thus up and all of my subscriptions were cancelled and all of my
c) however bills were paid, I would still be guaranteed mail – two pieces
d) despite a week, by my estimation – for the credit card companies
e) while
would still want me.
92 | UFSM 2002 – ADAPTED
– A palavra FOR, em destaque na linha 4, poderia ser
O oposto do termo destacado em "But according to clinical substituída por:
psychologist Simon Gelsthorpe, at Bradford Community
Health Trust, loneliness and depression are not always about
a) yet.
being alone" é:
b) why.
c) still.
a) in relation to.
d) but.
b) as a result of. e) because.
c) concerning to.
d) in disagreement with.
97 | FATEC 2003 – ADAPTED
e) in reference to. A palavra though em "Anyone can contract lupus, though it's
most common among those between 15 and 44" indica uma
93 | FATEC 2002 relação de:
A palavra although em "Although many became ill the next
day, some guests didn't show symptoms for several days" a) tempo.
indica uma relação de sentido de: b) concessão.
c) consequência.
a) tempo. d) explicação.
b) explicação. e) causa.
c) consequência.
d) concessão.
e) adversidade.
098 | JFS 2012
102 | UERJ 2003
We were unable to get sponsoring and had to
And, since the "who" and "where" of our lives are always
abandon the project. changing, so is our understanding of truth.
a) provided
– SINCE and SO in the sentence above may be notionally
b) regardless replaced by:
c) notwithstanding
d) despite
a) if and thus.
e) ergo b) therefore and but.
c) hence and instead.
099 | UNIFESP 2003 – ADAPTED d) because and likewise.
Moreover, they do not allow patients to act meaningfully.
103 | PUCSP 2003 – ADAPTED
– Na frase acima, a palavra MOREOVER pode ser substituída, In 1995, the Brazilian daily "Folha de São Paulo" bore this
sem mudar o sentido, por: headline: "World Bank Report Indicates Brazil Is the Country
with the Greatest Social and Economic Disparity in the
a) Furthermore.
World". The article reports that 51.3 percent of Brazilian
b) However.
income is concentrated in 10 percent of the population. The
c) Thus.
wealthiest 20 percent own 67.5 percent of Brazil, while the
d) Nevertheless. 20 percent who are poorest have only 2.1 percent. It was
e) Rather.
that way when I was a boy, and it is still that way. As we
reached adolescence, my generation dreamed of inverting
100 | UFRS 2001 – ADAPTED this brutal legacy.
Woody Allen's 'Sweet and Lowdown' has received great
critical acclaim, not least in the perceptive review of it by – Na frase "As we reached adolescence, my generation
Jonathan Romney. But not even he has discussed the aspect dreamed...", a palavra As pode ser substituída, sem mudar o
of the film I found the most intriguing. sentido, por:
– A palavra "But" poderia ser substituída sem alteração de
a) like.
sentido por:
b) whatever.
c) when.
a) although.
d) as well as.
b) nevertheless.
e) then.
c) therefore.
d) whatever. 104 | PUCSP 2003
e) despite.
Na frase "Students were either leftist or they would keep
their mouths shut". A expressão either... or indica uma idéia
101 | PUCPR 2003
de:
Mark the alternative that offers the best way to complete
the sentences below: a) exclusão.
b) inclusão.
I. Francis is engaged to be married, she is still in
c) gradação.
doubt about her getting married.
d) predominância.
II. Tom does not have a new car, does he have e) enumeração.
an apartment of his own.
III. After dinner, Lee either plays cards watches 105 | UNIRIO 2003
TV. In "You hate air conditioning, YET your houses deal with
IV. Joanne loves dancing singing. changes in the weather," the discourse marker yet denotes:
V. M
ichael and Peter should read more books, a) addition.
they will take a literature test soon. b) cause.
c) contrast.
a) but – nor – and – and – for d) comparison.
b) but – or – or – and – so e) condition.
c) yet – nor – or – in addition – so
d) but – nor – and – or – so
e) yet – nor – or – and – for
106 | PUCRS 2003
110 | PUCPR 2005
In the sentence "But Soderlholm and his wife, Gunvor, are
happy to pay", the word "but" could be replaced by:

a) Except for.
b) Therefore.
c) Yet.
d) Just.
e) While.

107 | UFSM 2003 – ADAPTED


Finding a public toilet in most of China is not difficult: you
can smell it long before you see it. But that's all beginning to
change. Recently, Beijing authorities promised a "toilet
revolution" (…). To prove they're serious, officials have
unveiled a star-rating system for bathrooms located at – In "... BECAUSE there weren't any malls, SO everyone was
popular tourist sites-ranking each facility with zero to five depressed", the ideas expressed by the capital words are
stars. Within three years Beijing hopes to have 471 four-star respectively:
toilets at places like the Forbidden City and Summer Palace.
a) comparison – result
– A palavra like indica: b) reason – addition
c) contrast – result
a) comparação. d) consequence – condition
b) exemplificação. e) reason – consequence
c) concordância.
d) contraste. 111 | JFS 2012
e) consequência. No texto abaixo, os vocábulos em destaque expressam,
respectivamente:
108 | PUCSP 2005
Com base na seguinte frase, responda à questão: Newsweek circa 1965 and the iPad may seem worlds apart,
but what they have in common outweighs their differences.
"Yet, just as those made of wood, bark, reeds, feathers, and Both eschew style over substance while wholeheartedly
animal skins are fragile, even ephemeral, so is this culture". embracing substance with style. Marrying the 1965 version
of Newsweek to the 2012 iPad's technology, we created a
– A expressão "... just as ... so is ..." indica uma relação de: design that respectfully tips its hat to the past without ever
getting stuck there. Call it retro with mojo. Won't you join us
a) diferença. on the elevator.
b) comparação.
c) consequência.
a) contraste e concessão.
d) reiteração.
b) contraste e adição.
e) ponto de vista. c) contraste e simultaneidade.
d) concessão e contraste.
109 | UNESP 1997 e) concessão e simultaneidade.
Marque a alternativa que completa as lacunas na sentença a
seguir corretamente: 112 | UECE 2000
O vocábulo BUT, na frase But Bobby Crabtree and his wife
Charles ... Mary are brother ... sister. thought Matilda was staying with friends that night,
classifica-se gramaticalmente como:
a) or / and
b) and / or a) verbo
c) and / but b) pronome
d) and / and c) conjunção
e) but / and d) advérbio
113 | UFC 2005 – ADAPTED
116 | FAAP 1975
The twelve days of the event led to a series of infrastructure Assinale a alternativa correta:
changes that will very much improve the quality of life for
citizens in Greece, especially groups of people with he was waiting for the news, he looked very
restricted mobility. , the greatest legacy of the angry.
Athens 2004 Paralympic Games will be a new social
perception of the disabled. a) If
b) While
– The appropriate expression to complete the blank in line c) Why
04 is:
d) Since
e) How
a) Besides
b) However
117 | UERJ 2006
c) In general
Connectors establish a set of semantic roles while linking
d) Once more
clauses. Observe the kind of link employed in the sentence
e) For instance
below.
114 | UFC 2000
"It isn't simply that he accepts the factual existence of power
The sentence “Yet their ambitions and their dreams are a
or legitimacy; RATHER, it's that he accepts that an authority
mirror image of our own – and our response to them says a
figure is justified in making a decision without also
great deal about us as individuals, as a country” is equivalent
explaining the reason for that."
in meaning to:
– The information that follows the spotted connector
a) In other words, their ambitions and their dreams are a
functions as:
mirror image of our own – and our response to them says a
great deal about us as individuals, as a country.
a) enumeration.
b) As a result, their ambitions and their dreams are a mirror
b) replacement.
image of our own – and our response to them says a great
c) reinforcement.
deal about us as individuals, as a country.
d) exemplification.
c) In addition, their ambitions and their dreams are a mirror
image of our own – and our response to them says a great
118 | FATEC 2007
deal about us as individuals, as a country.
O advérbio rather than em "Rather than buying more guns
d) Similarly, their ambitions and their dreams are a mirror
or patrol cars, Bogotá's cops went for something bigger:
image of our own - and our response to them says a great
science" poderia ser substituído, sem prejuízo de significado,
deal about us as individuals, as a country.
por:
e) Still, their ambitions and their dreams are a mirror image
of our own – and our response to them says a great deal
a) instead of.
about us as individuals, as a country.
b) in addition to.
c) as long as.
115 | ITA 2006
d) now that.
Leia atentamente todo o período transcrito abaixo, verifique
e) as far as.
as idéias contidas nas orações introduzidas por unless e
hence e assinale a opção que, respectivamente, expressa
119 | UNIFESP 2007
tais idéias.
Em "The United States might buy up credits instead of
reducing their own emissions", a expressão instead of
Unless the administration changes its approach, it will
indica:
continue to erode America’s good name, and hence its
ability to effectively influence world affairs.
a) reiteração.
b) substituição.
a) uma causa e uma concessão.
c) alternância.
b) uma explicação e uma adição.
d) sugestão.
c) uma condição e uma explicação.
e) causalidade.
d) uma explicação e uma conclusão.
e) uma condição e uma conclusão.
120 | JFS 2011
124 | JFS 2012
In the sentence "Even though some investors thought the New York City closed several blocks of Broadway in 2009 to
upgrade had been long overdue, few expected it to create a pedestrian plaza around Times Square – a much-
materialise before the end of the year due to current global publicized experiment that in February became permanent
uncertainty", even though and due to express, respectively, policy, even though it did not improve traffic flow as much
ideas of: as hoped. The Big Apple has also dabbled in shorter-term but
larger-scale street closures, barring cars on a stretch of
a) cause and effect.
streets leading from the Brooklyn Bridge to Central Park on a
b) contrast and addition.
series of summer Saturdays in 2008 and 2009. And on June
c) contrast and resulting.
7, New York City Mayor Michael Bloomberg announced a
d) complementarity and consequence.
somewhat less sexy but nonetheless significant change in
e) resulting and effect.
the city's infrastructure, instituting dedicated bus-only lanes
on Manhattan's East Side to speed transit up and down the
121 | JFS 2012
island.
Which of the following sentences can be filled in correctly
with the word ‘instead’?
– As expressões em NEGRITO no excerto acima podem ser
substituídas, respectivamente, por:
a) I think I'd like to stay at home this evening go
out.
a) although e yet.
b) You can go me, if you want. b) besides e nevertheless.
c) There's no coffee – would you like a cup of tea
c) since e even though.
?
d) however e therefore.
d) Small cities aren't very noise they can be fun. e) though e moreover.
e) all my work, I still failed the exam.
125 | EFOMM 2010
122 | IME/CG 2012
She has tried to reach them four times on the phone without
They seem to have the same number of stars as fully grown success. Hence she needs to write them as her last option.
galaxies in the present-day Universe. , their sizes
are four to five times smaller and their densities are a
– The underlined connective expresses the idea of:
hundred times larger than their present-day counterparts.
a) addition
a) Whenever
b) conclusion
b) Even though
c) enumeration
c) Though d) contrast
d) However e) concession
e) Although

123 | IME 2012 – ADAPTED


“Because your own strength is unequal to the task,
Leia o texto seguinte e complete as lacunas com as palavras do not assume that it is beyond the powers of man;
correta. but if anything is within the powers and province of man,
believe that it is within your own compass also.”
Atmospheric Physics is the study of physical processes in the Marcus Aurelius
atmosphere; understanding the climate and the weather,
and the physical reasons it is as it is and how it
might change in the future. My own particular research
background is in radiative transfer and by that I mean the
transport of electromagnetic radiation through the
atmosphere; the Sun’s radiation coming in,
getting absorbed and scattered in the air, and the transfer of
heat radiation emitted by the Earth’s surface.

a) as – neither
b) because – if
c) for – but
d) why – both
e) that – not only
Subjunctive, Imperative, 005 | UEL 1994
Assinale a alternativa correta:
Infinitive and Gerund
He stopped only after the doctor said he was
001 | ITA 1995 going to die.
Without fear of be happy (Sem medo de ser feliz) é o título,
em inglês, que um periódico paulista atribuiu ao livro do a) drank
jornalista americano Ken Silverstein sobre a campanha de b) drink
Lula à Presidência da República em 1989. Examinando o c) drinks
título, você diria que: d) drinking
e) to drink
a) Está estruturalmente correto.
b) Deveria ser: "Without Fear of Been Happy". 6 | UNESP 1990
c) Deveria ser: "Without Fear of to Be Happy". Both Mary and Roger enjoy tennis.
d) Deveria ser: "Without Fear of Being Happy".
e) Deveria ser: "Without Fear to Be Happy". a) plays
b) play
002 | ITA 1995 c) to playing
A lacuna da sentença abaixo deve ser preenchida por: d) playing
e) played
Astronomers are used all kinds of wild things in
outer space. 7 | MACKENZIE 1996
Indicate the correct alternative:
a) find
b) to find I had hoped my letter.
c) finding
d) to finding a) her answer
e) found b) her answering
c) that she answer
003 | UNESP 1985 d) that she would answer
Assinale a alternativa que preenche corretamente a e) to her answer
sentença a seguir:
008 | ITA 1995
Most people cannot learn verbs without them. A melhor forma de concluir a sentença a seguir é:

a) to study Although personal appearance is of great importance when


b) studying going to an interview for a job, the candidate should be
c) study careful .
d) studied
a) to not overdress
e) studies
b) to do not overdress
c) not to overdress
004 | JFS 1999
d) do not overdress
Assinale a alternativa correta:
e) not overdress
Nothing can be done except the results of the
009 | ITA 1996
exam.
Leia a frase e preencha a lacuna:
a) to waiting
By signing below, I ask that an account opened
b) waits
for me and Card(s) issued as I request, and that you renew
c) to wait
and replace them until I cancel.
d) waiting
e) wait a) to be
b) are
c) is
d) will be
e) be
010 | FEI 1996
015 | ITA 1996
I regretted answering like that. I was sorry so.
A alternativa que deve preencher a lacuna no quadro a
seguir é:
a) to have done
b) in doing
c) to do
d) to be doing
e) for do

011 | FEI 1996


Excuse you.

a) me to interrupt
b) me for interrupt
c) me for interrupting
d) me in interrupting
e) me to interrupting

012 | FEI 1996


Assinale a alternativa correta:
a) there is
b) you do
Would you mind coming with me? I'd like .
c) there be
d) have
a) you to come
e) may be
b) to come
c) coming
016 | JFS 2000
d) you coming
Relacione as colunas A e B, encontrando o Imperativo
e) you come
correto.
013 | UEL 1997
Coluna A
The best time an international call is between
11 p.m. and 8 a.m.
1. I'm hungry.
2. I'm very tired.
a) you for to make
3. I'm sad today.
b) for you make
4. I'm pissed off.
c) that you to make
5. I'm thirsty.
d) that to make you
e) for you to make
Coluna B
014 | MACKENZIE 1997
( ) See a comedy movie.
Indicate the alternative that best completes the following
( ) Take vacation.
sentence:
( ) Eat something.
( ) Drink some water.
The teacher mentioned some changes that we can expect
( ) Chill out and take it easy.
by the year 2000.
a) 1 – 2 – 3 – 4 – 5
a) have been taking place
b) 5 – 4 – 3 – 2 – 1
b) to take place
c) 3 – 2 – 1 – 4 – 5
c) to have taken place
d) 3 – 2 – 1 – 5 – 4
d) taking place
e) 2 – 3 – 4 – 1 – 5
e) have had taken place
017 | UFSC 1997
Select the proposition(s) which is(are) in the Imperative Phrasal Verbs
Form:
1 | MACKENZIE 1996
(1) Please, place all valuables in the hotel safe.
Choose the correct alternative which indicates the meaning
(2) You are advised not to take money on the beach. of blow over in the following sentence:
(04) We ask you to return your keys before your departure.
(08) You must carry your suitcases. They're hoping that this crisis will BLOW OVER and be
(16) Don't make noise at night. forgotten.
(32) Be careful with your tickets.
a) happen suddenly and make you able to accept a pleasant
– Soma = ( )
experience.
a) 49 (01 + 16 + 32) b) explode.
b) 48 (16 + 32) c) succeed in spite of difficulties.
c) 50 (02 + 16 + 32) d) become stronger and cause trouble for other people.
d) 56 (08 + 16 + 32) e) pass without doing harm.
e) 17 (16 + 01)
2 | UDESC 1996
018 | JFS 2007 Find the correct alternative:
Which of the following sentences is not in the Imperative
Mood? a) to put on – to remove
b) to put out – to increase
a) Get plenty of calcium. c) to put off – to dress
b) Don’t mess too much with your hair. d) to put on weight – to grow fat
c) But, be patient with those who supply it. e) to put together – to keep
d) Painting over the ugly parts.
e) But trust me on the sunscreen. 3 | FUVEST 1979
Assinale a alternativa que melhor traduz a seguinte frase:
019 | ITA 2010
Assinale a opção em que o termo em negrito apresenta I have run out of vinegar.
função gramatical diferente das demais.
a) Corri para buscar vinagre.
a) ... depending on whether they were playing at home or b) Derramei o vinagre.
away. c) Preciso sair para pegar vinagre.
b) Knowing that appearance affects people’s mood and d) Meu vinagre acabou.
outlook,... e) Joguei fora o vinagre.
c) Using data from the 1970s and 1980s, they found that...
d) But while the 1988 finding has become a classic in 4 | MACKENZIE 1996
psychology, ... Choose the correct alternative. Indicate the meaning of
e) …when they started wondering about the effect of being catch on to in the following sentence:
able to alter…
We didn't quite CATCH ON TO what the teacher said.
020 | IME 2012
Na sentença a seguir, encontram-se em destaque cinco a) reach the same standard or level
termos. Assinale a alternativa correspondente ao termo cujo b) hear
emprego está INCORRETO. c) confirm
d) believe
I am currently studying for a PhD at the British Antarctic e) understand
Survey working on the structural interpretation of gravity
and magnetic going from east Antarctica, including both 5 | UNESP 1990
numerical and mechanical modelling of data. He took his hat.

a) studying a) off
b) working b) out
c) going c) in
d) including d) into
e) modelling e) on
006 | UECE 1997
10 | PUCRIO 1999
SPLIT UP in “After getting stores and various other
The only option in which MAKE UP has the same meaning as
commodities together in Mamfe, John and I decided to split
immigrants make up only 12% of today's workforce is:
up” means that:
a) The committee is made up of representatives from all the
a) they decided to separate.
university.
b) they decided to end their relationship.
b) You must make up what you owe before the end of the
c) they decided to stay together.
month.
d) they decided to talk over their plans.
c) She never goes without making herself up first.
e) they decided to sleep together.
d) I can make up a bed for you on the floor.
e) He made up an incredible story about his holiday.
7 | CESGRANRIO 1998
The verb to go may be followed by many adverbial particles
11 | EFOMM 2000
and prepositions that add different meanings to it. Match
“Is that my new TV in that package?’
the two columns and then mark the correct alternative:
“Yes, just .”
I – go up
a) open it up, take it out, plug it in and turn it on.
II – go on
b) open it up, take it off, plug it on and turn it on.
III – go away
c) take it out, open it on, plug it off and turn it up.
IV – go by
d) take it in, plug it out, turn it down and open it up.
e) open it out, take it up, turn it on and plug it on.
R. continue
S. leave
12 | FUVEST 1978
T. rise
Assinale a alternativa que traduz melhor a seguinte
U. move
sentença:
V. pass
I can't tell one from the other.
– Mark the correct match.
a) Não distingo uma da outra.
a) I – T; II – U; III – S; IV – R
b) Não conto com nenhuma outra.
b) I – T; II – R; III – S; IV – V
c) Não falo com nenhuma outra.
c) I – T; II – V; III – U; IV – R
d) Não posso falar de uma para outra.
d) I – U; II – R; III – S; IV – V
e) Não posso dizer para a outra.
e) I – U; II – V; III – S; IV – R
013 | ITA 2007
8 | UDESC 1997
A expressão BREAK OUT em “During a poker match, an
Choose the CORRECT answer:
argument breaks out between Napoleon and Mr. Pilkington
when they both play an Ace of Spades, and the animals
The woman is TAKING CARE of her baby.
realize how they cannot tell the difference between the pigs
and the humans” significa:
– She is:
a) to make a sudden, quick advance.
a) looking back on her baby.
b) to bring or come to an end.
b) looking for her baby.
c) to develop suddenly.
c) looking after her baby.
d) to force or make a way through.
d) looking down on her baby.
e) to cause to separate into pieces suddenly or violently.
e) looking over her baby.
014 | EFOMM 2012
009 | AFA 1986
The initials V.I.P. Very Important Person.
Qual alternativa possui os três verbos na ordem natural de
seu uso?
a) means
b) stand for
a) wake up/sleep/get up
c) are about
b) sleep/get up/wake up
d) describe
c) get up/wake up/sleep
e) accounts for
d) sleep/wake up/get up
015 | JFS 2010
Fill in the gaps in the sentences below meaningfully: False Cognate Words
1. the computer and let's go. 001 | UFV 1996
2. Her new boyfriend her on their The word particular in the sentence Much to the surprise of
second date. everyone, the remarkable conclusion of the research
3. She the flu after being sick for a week. showed that people of a certain group actually were born
under a PARTICULAR planet, means the same as:
a) 1. Shut … down – 2. stood … up – 3. got over
b) 1. Shut … down – 2. stood … down – 3. got down a) private
c) 1. Shut … off – 2. stood … over – 3. got over b) specific
d) 1. Shut … off – 2. stood … down – 3. got down c) ordinary
e) 1. Shut … off – 2. stood … up – 3. got on d) personal
e) general
016 | JFS 2012
You’d better give . This issue has no solution. 2 | UNITAU 1995
Assinale a alternativa que corresponde à tradução mais
a) out adequada da frase nominal, entre aspas, a seguir:
b) up
c) over Future generations may regard the scientific indictment of
d) away smoking as 'a major contribution' to preventive medicine
e) off and the health of the western world.

17 | EFOMM 2010 a) uma contribuição importante


In: b) uma contribuição do major
c) a maior contribuição
"Did you buy a new bike?" d) uma contribuição para o major
"Yes, I bought one two weeks ago." e) a importante contribuição

– Which phrasal verb can substitute the underlined word? 3 | MACKENZIE 1996
The meaning of EXQUISITELY in the sentence “Their children
a) ran into were exquisitely dressed” is:
b) let down
c) picked out a) strangely or unexpected.
d) turned out b) extremely beautiful in appearance.
e) picked up c) oddly with an unusual appearance.
d) peculiarly with a special or unique quality.
18 | EFOMM 2010 e) very unattractive and unpleasant to look at.
Choose the correct alternative to complete the statements.
4 | MACKENZIE 1997
I. They the search after three hours. It was The meaning of CONSISTENT in the sentence Mr. Brown's
hopeless. behavior is not consistent with his ideas is:
II. Somebody’s got to those high principles.
III. We shouldn’t their arguments so easily. a) coherent.
IV. I’m trying to my own work. b) contradictory.
V. W c) right.
hat a preposterous idea! Jane will never d) persuasive.
with it! e) strong.

a) gave up / stand up for / give in / catch up on / get away 5 | UNIRIO 1997


b) carried out / stand up to / give over / catch up on / get A opção que contém um falso cognato é:
ahead
c) gave up / stand up to / give over / catch up with / get a) enthusiasts
across b) processes
d) gave up / stand up for / give over / catch up with / get c) outdoors
along d) spades
e) carried out / stand up to / given in / catch up on / get e) parts
through
006 | JFS 2000
010 | EN 1990
Dadas as sentenças: In “They pretended to be dead”, TO PRETEND means:
1. The boys the game yesterday night.
a) entender
2. He gave a of diamond to his mother.
b) pretender
3. I didn’t like the film, it was .
c) fingir
4. They finished the test.
d) tender
– Marque a alternativa que completa corretamente os e) fazer
espaços em branco:
011 | EN 1990
a) assisted / collar / terrific / eventually In “They suffered from the injuries sustained in the crash”,
b) assisted / necklace / terrific / eventually INJURIES means:
c) watched / necklace / terrible / finally
d) watched / collar / terrific / finally a) ofensas
e) watched / necklace / terrific / eventually b) ferimentos
c) injúrias
007 | ITA 1987 d) calúnias
Dadas as afirmações de que: e) fraturas

1. INGENIOUS significa HÁBIL, ENGENHOSO. 012 | CESGRANRIO 1991


2. TO RESUME significa RETOMAR. A synonym for the word ACTUALLY as in “This is actually the
3. SYLLABUS pode significar SÍLABA. decaying of the tooth” is:

– Constatamos que está(estão) correta(s): a) now


b) today
a) Apenas a afirmação nº 1. c) really
b) Apenas a afirmação nº 2. d) normally
c) Apenas a afirmação nº 3. e) theoretically
d) Apenas as afirmações nos 1 e 2.
e) Todas as afirmações. 013 | JFS 2007
What does the word ESTATE mean?
008 | ITA 1988
Dadas as afirmações de que em português: a) estado
b) estatal
1. SCHOLAR significa LETRADO. c) propriedade
2. SENSIBLE significa SENSATO. d) fortuna
3. FABRIC significa TECIDO. e) riqueza

– Constatamos que está(estão) correta(s) : 014 | JFS 2008


Which of these words is not a False Cognate?
a) Apenas a afirmação no 1.
b) Apenas a afirmação no 2. a) apologize
c) Apenas a afirmação no 3. b) response
d) Apenas as afirmações nos 1 e 2. c) support
e) Todas as afirmações. d) largest
e) resume
009 | ITA 1988
A alternativa que corretamente traduz (mantendo a ordem 015 | JFS 2012
das palavras: Ele puxou ou empurrou a porta quando chegou I will the with his new security
lá?, é: .
a) Did he push or pull the door when he got there? a) assist – mayor – policy
b) Did he pushed or pulled the door when he arrived there? b) attend – major – police
c) Did he pull or push the door when he got there? c) support – official – politics
d) Pushed or pulled he the door when he gots there? d) assist – mayor – politics
e) Pushed or pulled him the door when he arrived there? e) attend – major – policy
016 | PUCRIO 1995
The verb TO ATTEND is used in “The first semester I Mixed Topics
attended college, I became friends with one of the American
students in my class”. 001 | ITA 2001
Assinale a opção correta em relação às frases abaixo,
– Check the ONLY alternative in which the same verb can be extraídas de um calendário americano:
used.
I. "A really great talent finds its happiness in execution." –
a) The receptionist's job was to the phone. Goethe.
b) The doctor carefully the patient. II. "There are many things in life that will catch your eye, but
c) The professor a convention to present his only a few will catch your heart… pursue these." – unknown.
work. III. "By cultivating the beautiful we scatter the seeds of
d) He the radio to listen to the news. heavenly flowers, as by doing good we cultivate those that
e) The interviewer questions from the public. belong to humanity." – V. Howard.

017 | JFS 2012 a) Na sentença (I), "its" refere-se a "happiness".


Em inglês, há algumas palavras que, ao serem pluralizadas, b) Na sentença (II), "these" refere-se a "things in life that will
podem mudar o sentido, tornando-se falsas cognatas. Como catch your heart."
exemplo, o vocábulo spirit: “espírito”, no singular, e c) Na sentença (II), "those" refere-se a "flowers".
“disposição”, no plural. Qual das opções abaixo segue o d) Os termos "but" na sentença (II) e "as" na sentença (III)
mesmo padrão? são conjunções que podem ser consideradas sinônimas no
contexto em que se encontram.
a) Minute. e) Os termos "beautiful" e "good", na sentença (III), exercem
b) Human. a função de adjetivos.
c) State.
d) Fruit. 002 | UNITAU 1995
e) Diplomat. Assinale a alternativa que relaciona as construções
adequadas e as construções inadequadas em inglês dentre
018 | JFS 2012 as listadas a seguir:
Complete as sentenças a seguir coerentemente:
1. The football is in the our hearts.
2. Romário was not born in the United States.
I. Many were killed during the battle.
3. Romário does not come from the United States.
II. It's a typical country with a large house for
4. I see he and she.
the owner, farm buildings and workers' houses.
5. Pelé is more popular than Nixon.
III. The secret to public speaking is to get the on
6. A cat is more big than a mouse.
your side.
7. Parreira have a little sister.
IV. Over two hundred people the funeral.
8. An elephant is smarter than a feminist.
V. Some of his colleagues envy the enormous wealth that he
9. The Brazilian and the Italian teams have no problems
has .
here.
10. I am seeing the cup.
a) privates – state – listeners – answered – spread
b) privates – estate – audience – attended – amassed a) Adequadas: 2, 4, 6, 8. Inadequadas: 1, 3, 5, 7, 9, 10.
c) privates – estate – audience – attended – spread b) Adequadas: 1, 2, 3, 4, 5; Inadequadas: 6, 7, 8, 9, 10.
d) corporals – estate – listeners – answered – amassed c) Adequadas: 6, 7, 8, 9, 10. Inadequadas: 1, 2, 4, 6, 7.
e) corporals – state – audience – attended – amassed d) Adequadas: 2, 3, 5, 8, 9. Inadequadas: 1, 4, 6,7, 10.
e) Adequadas: 3, 2. Inadequadas: todas as demais.

“It is possible to store the mind with a million facts 003 | UFSM 1999
and still be entirely uneducated.” Over last century, women been
Alec Bourne the opportunities play a greater
role in government and business.

a) the – had – give – by


b) that – has – given – for
c) the – have – given – to
d) the – has – given – by
e) that – have – give – to
004 | UFSM 2001
Assinale a alternativa que completa corretamente os 007 | EEAR 2007
espaços no trecho a seguir: Computer viruses can travel one place to
another fast a phone call.
Britain been an industrialised nation for two
a) from – as – as
centuries. It a variety of industries
b) over – as – than
can be divided three main categories.
c) to – more – than
d) between – * – than
a) has – has – which – into
b) have – has – who – in 008 | EN 2001
c) has – have – which – in Choose the alternative that best completes the paragraph
d) have – has – who – into below:
e) has – has – who – in
a good memory? Maybe you are good
005 | UFSM 2001 remembering events or footballers’ names? But
Assinale a alternativa que completa corretamente os could you memorize order of a pack of cards?
espaços no trecho a seguir: British mega-brain Dominic O’Brien can.
(Amazing body science – Young Telegraph, p. 7)
Invented the late nineteenth century
an eastern European ophthalmologist, a) Did you get/ in/ historic/ a
Esperanto humanity's successful b) Do you get/ on/ history/ an
attempt to create an artificial universal language. c) Have you got/ at/ historical/ the
d) Had you got/ for/ historian/ X (= no article)
a) on – for – remains – most
b) in – by – remain – more 009 | EN 2001
c) in – to – remains – most Choose the alternative that correctly completes the text
d) on – by – remain – more “The nose knows”:
e) in – by – remains – most
The Nose Knows
006 | UFPE 2003
One of the most effective devices monitoring
Manaus pollution may be as plain as the nose your face.
So say scientists from the University of Utretch, in the
Throughout its 303-year history, the tiny Manaós Indian Netherlands. Two psychologists several
village around Fort São José do Rio Negro very hundred residents of the industrial city to pop their heads
distinctive names. At the height of the rubber era, out of their windows every Tuesday evening and take a
its channels became spacious avenues traveled whiff. If they smell anything, they’re to classify in from
by streetcars, it was called the "Brazilian Paris" and the innocuous to extremely annoying.
"Tropical Metropolis". , with the arrival of
industry, it a "Free Zone". Nestled in the middle a) to/ in/ would recruit
of the largest tropical rainforest the planet, it is b) for/ on/ have recruited
now becoming the main "gateway" to the Amazon for c) from/ onto/ had recruited
ecotourists. d) in/ into/ recruited

(From "A pleasant day in Manaus", 10 | MACKENZIE 1997


ÍCARO BRASIL, April 2002, page 33.) Indicate the alternative that best completes the following
sentence:
– Fill in the blanks with the correct sequence of words:
The most material for consumer
a) is given – where – Late – becomes – in goods cardboard.
b) had given – what – Lately – become – of
c) has given – how – The latter – to become – at a) used – package – are
d) was given – when – Later – became – on b) used – packing – are
e) have given – which – The latest – becoming – around c) widely used – packaging – is
d) widest used – package – has been
e) used – packaging – could be
11 | UNITAU 1995
a) ficariam inalterados.
Assinale a alternativa que relaciona as construções
b) seriam trocados para "winded" e "their".
adequadas e as construções inadequadas em inglês dentre
c) se transformariam em "winded" e "it".
as listadas a seguir:
d) seriam substituídos por "wind" e "their".
e) passariam para "wind" e "theirs".
1. Mary is likeing John.
2. Mary needs to help sheself.
014 | JFS 2002
3. I can see him.
Read carefully the sentences below and try to find the
4. John was expected to win the world cup.
correct one:
5. Americans films are goods.
6. They has a big farm in downtown.
a) Daniel and Marla are going to buy a new and expensive
sport car to his son.
a) Adequadas: 3, 4. Inadequadas: 1, 2, 5, 6.
b) The collected data is important to discover the average
b) Adequadas: 1, 2, 5. Inadequadas: 3, 4, 6.
age of our students.
c) Adequadas: 3, 4, 5. Inadequadas: 1, 2, 6.
c) The newcomer neighbours have a two-year-old baby and
d) Adequadas: 1, 3, 6. Inadequadas: 2, 4, 5.
are planning to have another child before the next X-mas.
e) Adequadas: 2, 3, 5, 6. Inadequadas: 1, 4.
d) They are looking up the lost documents inside the office
but I think that they won’t find it before the meeting.
012 | UFMA 2000
e) Parker is used to read the Bible before sleep. He is a very
Catholic man.

015 | MACKENZIE 1997


Indicate the alternative that best completes the following
sentence:

Researches have shown that the twenty-first-century family


will certainly be more than today's family
people's longer and
more than once.

a) complicating – due to – live – marriage


b) complicated – because – living – marriage
c) complicated – as a result of – living – marrying
d) complicated – because of – life – marring
e) complicating – as a consequence of – life – marriage

016 | EN 2001
Choose the alternative that correctly completes the text “No
Copyright  2000 United Feature Syndicate, Inc. celebrations”:
Redistribution in whole or in part prohibited.
No Celebrations
– In the context of the cartoon above, the correct alternative
No big celebration held when I arrived. Huang
is:
Taitai didn’t have red banners greeting me in the fancy room
on the first floor. Tuan-yu was not to greet me.
a) “another” implies no other additional idea.
Instead, Juang Taitai hurried me upstairs to the second floor
b) “can” indicates ability.
into the kitchen, was a place family children
c) “doesn’t have to” indicates permission.
didn’t usually go. This was a place for cooks and servants. So
d) “did” is used to make questions with ordinary verbs in
I my standing.
the Present Tense.
e) “can” could be replaced with the same meaning by “may”.
a) was/ there/ which/ knew
b) would/ here/ where/ have known
013 | UFSM 2002
c) has/ somewhere/ when/ had known
Se o sujeito da oração "that WINDS back 5,000 years in ITS
d) will/ anywhere/ that/ know
native India" se referisse a uma expressão no plural e fosse
mantido o tempo verbal, os termos destacados:
017 | AFA 2005
020 | JFS 2012
Read the ad and answer the question: The magic had audience
participation, with people shouting things the
Lockheed Martin and Sikorsby. Working together for performers and going up stage.
three decades, this team (I) and integrated
(II) any other team in the world. That includes a) performance – lots of – at – over
the SH-60B, (III) the new MH-60R and MH60S b) performance – lots of – to – over
programs. This (IV) delivers the proven Naval c) show – a lot of – to – on
Hawk airframe with advanced multi-mission avionics d) show – lot of – at – on
capabilities to the U.S. Navy and navies worldwide – e) show – a lot of – at – on
together we deliver (V) naval capability to
(VI) customers. As questões de 19 a 23 devem ser respondidas a partir da
imagem abaixo.
SiKorsKy | A United Technology Company

– Fill in the blanks correctly with grammatical subjects that


have been asked in the questions below:

I. Use the Present Perfect Tense of the verb to build.


II. Use a Comparative form of Superiority.
III. Use a Comparative form of Equality.
IV. Use the right Possessive case of nouns.
V. Use the Superlative form of an adjective.
VI. Use the right Pronoun.

a) has built/ more naval helicopters than/ as well as/ team’s


synergy/ the best/ our.
b) have build/ many naval helicopters as/ so good as/ teams’
synergy/ the better/ ours.
c) had build/ much more helicopters than/ less well as/ team 021 | UEL 1999
synergy’s/ more and more/ us. Preenche corretamente a lacuna (I) a alternativa:
d) has builded/ less helicopters than/ not so bad as/ team
synegys’s/ the most/ we. a) hold on
b) hold up
018 | MACKENZIE 1999 c) expect
Indicate the alternative that best completes the following d) tell
sentence: e) say

This guide useful about the city. 022 | UEL 1999


Preenche corretamente a lacuna (II) a alternativa:
a) gives – information
b) have given – information a) I'm
c) is giving – piece of informations b) I'd
d) gave – pieces of informations c) I've
e) will give – pieces of informations d) You'll
e) You're
019 | JFS 2012
After a I enjoy beer 023 | UEL 1999
two. Preenche corretamente a lacuna (III) a alternativa:

a) hard day's work – one – or else a) coming


b) hard work day – either – or else b) comes
c) hard day's work – either – or c) will come
d) hard work day – either – or d) to come
e) hard day's work – a – or e) is coming
024 | UEL 1999
Preenche corretamente a lacuna (IV) a alternativa: Idioms and Vocabulary
a) us 001 | UEL 1997
b) me Assinale a tradução correta dos termos em maiúsculo:
c) my
d) hers I DON'T FEEL LIKE being indoors all day. Let's go for a long
e) mine walk.

025 | UEL 1999 a) Não estou com vontade


Preenche corretamente a lacuna (V) a alternativa: b) Não acho ruim
c) Não creio que é uma boa idéia
a) There d) Não concordo
b) There's e) Não estou pensando em
c) Have
d) It's 002 | CESGRANRIO 1993
e) It Mark the item which contains the adequate translation for
the saying “Out of sight, out of mind”.

“Do not protect yourself by a fence, a) Quem não deve, não teme.
but rather by your friends.” b) Quem quer faz, quem não quer manda.
Czech Proverb c) Deus ajuda a quem cedo madruga.
d) Longe dos olhos, longe do coração.
e) Em terra de cego, quem tem um olho é rei.

003 | ITA 1996


Na frase “He is very, very rich and so powerful that even his
enemies are eager to cut deals with him”, o significado de to
cut é:

a) competir
b) concordar
c) compartilhar
d) tornar–se
e) cortar (qualquer tipo de relação)

004 | UEL 1997


Assinale a versão correta da frase a seguir:

Prefiro ir de carro.

a) I like cars.
b) I dislike cars.
c) I always go by car.
d) I'd rather go by car.
e) I don't have a car.

005 | CESGRANRIO 1991


In “The sweet-and-lovely look is OUT; the aggressive punk
pose is IN” the capital words stand for:

a) out of work / in vogue


b) out of sight / in mind
c) out of order / in memory
d) out of date / in fashion
e) out of mind / in sight
006 | UFRS 1997
010 | UEL 1996
The expressions immediately off and were off, in the Assinale a tradução correta da frase entre aspas:
sentence below, mean respectively:
- "Can you tell me how to get there?"
There were four last stragglers who stopped to examine a
- Of course I can.
small dogwood immediately off the terrace, but one of our
dogs barked in the distant kennels and the deer were off
a) Você pode me dizer como se consegue isso lá?
into the woods.
b) Quem pode me contar como se faz isso?
c) Você pode me ensinar o caminho?
a) close to – went away
d) Como se pode ir de lá para cá?
b) approximately – far off
e) Você consegue atravessar para o outro lado?
c) in – gathered
d) nearby – far away 011 | UEL 1996
e) together – jumped
Assinale a versão correta da frase a seguir:
7 | CESGRANRIO 1995
Não posso comprar um carro novo.
Most names of jobs are used for both men and women.
Mark the one that refers only to males: a) I shouldn't be thinking of a new car.
b) I can't afford a new car.
a) psychiatrist c) I can't buy anything new.
b) social scientist
d) If it is new, I don't want it.
c) waiter
e) Who needs a new car anyway?
d) doctor
e) researcher 012 | FEI 1996
I'd prefer to stay here. That's what I'd .
8 | MACKENZIE 1996
The group was discussing Freud's view of the a) do
and .
b) do best
c) have done
a) consciousness – unconsciousness d) rather do
b) conscious – unconscious
e) doing
c) consciousness – unconscious
d) consciously – unconscious
13 | FUVEST 1977
e) unconsciously – conscious Qual destas sentenças está correta?
009 | UFPE 1996
a) Don't translate word of word.
Identifique a(s) palavra(s) ou expressão(ões) que melhor
b) Don't translate word with word.
complete(m) o sentido da sentença abaixo utilizando V ou F
c) Don't translate word at word.
e, em seguida, marque a sequência correta:
d) Don't translate word to word.
e) Don't translate word for word.
... began the moment the Burma Star contingent started to
march, led unexpectedly by one of its members, the Duke of
14 | FUVEST 1978
Edinburgh.
Assinale a alternativa que corresponde a seguinte sentença
em inglês:
( ) Crying
( ) Cheering
Não deixe de me avisar quando você pretende fazer uma
( ) Rejoicing
viagem.
( ) Applauding with shouts
( ) Encouraging by shouts
a) Don't fail to advise me if you pretend to travel.
b) Don't leave me to know when you intend to go for a trip.
a) F – V – V – V – V
c) Don't let me warn you when you pretend to go for a trip.
b) F – V – V – V – F
d) Don't leave my advice when you intend to travel.
c) V – V – V – V – F
e) Don't fail to let me know when you intend to take a trip.
d) F – V – V – F – V
e) V – F – F – F – F
15 | PUCCAMP 1994
020 | JFS 2010
Assinale a letra correspondente à alternativa que preenche - How many students are there in the classroom?
corretamente as lacunas da frase apresentada.
- A BAKER’S DOZEN.
"Hi, Jane!"
– The capital expression means:
"Hi, Susan. How have you been?"
"Oh, just fine. What have you done? You look so different!"
a) Eleven.
"Me? Different? I don't know."
b) Twelve.
"Maybe it's your hair."
c) Thirteen.
"Oh, that’s maybe ." d) Fourteen.
"It looks quite nice." e) Fifteen.
"Thank you. Well, I've got to go. See you around."
"See you. Bye." 21 | EFOMM 2012
Choose the word that completes the idea of the following
a) I'm cutting my hair
statement correctly:
b) I just cut your hair
c) I'm going to have a haircut Michael is a smoker. He smokes three packs of
d) You had a haircut cigarettes a day.
e) I just had my hair cut
a) violent
16 | ESPCEX 1999
b) fierce
How long John to get to work?
c) big
d) chain
a) does it take
e) strong
b) it takes
c) takes
22 | EFOMM 2012
d) does he take
In the sentences below, seem and appear are used correctly,
e) he takes except in:
017 | UFPE 1999
a) It seemed like a good idea at first.
“Quem dera eu fosse um astro do rock-and-roll” in English is: b) I can’t appear to make her notice the events.
c) It seems a pity, but I can't see you this weekend.
a) I wish I to be a rock-and-roll star.
d) He seems younger than he is.
b) I wish I were a rock-and-roll star.
e) The baby appears hungry.
c) Who wishes to be a rock-and-roll star.
d) I want be a rock-and-roll star.
23 | EFOMM 2010
e) I was wishing I was a rock-and-roll star.
Mark the alternative in which DO and MAKE are all used
correctly:
018 | JFS 2007
a) Do a favor/ make repairs/ make amends/ make a bargain
If you stay up very late or , you
b) Make a bid/ make a demand/ do ends meet/ make an
burn the midnight oil.
entry
c) Make an excuse/ make a face/ make as much as you can/
a) dancing – partying
make business
b) drinking – smoking
d) Do a proposal/ make a scene/ do silly things/ make a
c) working – studying
review
d) sleeping – snoring
e) Do an errand/ make fun of/ make known/ do friends
019 | JFS 2008
024 | JFS 2012
A normal person has:
If someone is bullets, they're very worried or
frightened.
a) twenty fingers and two hands.
b) two shoulders and four knees.
a) shooting
c) three legs and one heart.
b) sweating
d) two lungs and four livers.
c) dodging
e) ten fingers and ten toes.
d) biting
e) killing
025 | JFS 2012
029 | JFS 2012
Fill in the gaps meaningfully:

I. Could you speak a little , please?


II. I must be weight, some clothes of mine don’t
fit me anymore.
III. I can’t the washing machine work.
IV. your brother, I must warn you to be careful.

a) loudly / loosing / make / As


b) louder / losing / make / As
c) aloud / losing / do / As
d) louder / loosing / make / Like
e) louder / loosing / do / Like

26 | EFOMM 2008
In: “We were a bit worried about the new manager because
we heard that she liked to run a tight ship.”, the expression
in bold means: ‘to be well ...’

a) organized
b) disposed Adapted from http://3.bp.blogspot.com/
c) received – No cartun acima, a expressão hit the sack poderia ser
d) placed substituída por:
e) educated
a) turn in.
27 | AFA 2004 – ADAPTED b) sleep over.
Among eighteen recorded CD’s by the London Starlight c) stretch my legs.
Orchestra, eleven are dedicated to some good movies. They d) pull back.
are considered veracious musical photographs as part of the e) fork out.
story of cinema. “Take My Breath Away”, from Moroder and
Witlock, is the love theme of the remarkable movie “Top 030 | JFS 2010
Gun” and also from the album that contains more than “Although the risk of a double-dip recession is still
seventeen of the same type. significant, it is not the most likely scenario,” said Diane
Swonk, chief economist at Mesirow Financial. “Moreover,
According to the title of the song mentioned in the text there are no silver bullets when it comes to fueling
above, what´s the best definition for the lover´s feeling? employment.

a) you keep air my lungs and not let it out. – The expressions in bold can be defined respectively as:
b) you make me feel shocked by surprise.
c) you make me feet brand new. a) The worst-case scenario the economy of a country may
d) you hold my breath deeply face – A doubtful solution
b) When the economy moves back into a deeper and longer
028 | JFS 2012 recession – An impossible solution
Parker's fun at parties, but his brother's a wet blanket. c) A recession twice stronger than the previous one – A
solution that seems magical
– A partir da sentença acima, podemos concluir que o irmão d) A recession followed by a short-lived recovery, followed
de Parker é, em inglês: by another recession – A complete solution to a large
problem
a) a faint-hearted Guy. e) A recession that will be twice longer than the previous
b) a timid person. one – A definitive solution to a huge problem
c) a gatecrasher.
d) a killjoy.
e) a miserable man. “A positive attitude may not solve all your problems,
but it will annoy enough people to make it worth the effort.”
Herm Albright
Synonyms and Antonyms 4 | EFOMM 2004
Regarding the impact the lady’s appearance caused, the
1 | EFOMM 2003 word breathless in verse 16 means:
In “I’m enjoying a peasant life now. I’ve moved out of
London”, the underlined word means: a) simple.
b) light.
a) easy c) unimportant.
b) rich d) remarkable.
c) fancy e) awful.
d) country
e) weird 005 | JFS 2009
In the sentences “The police, firefighters and other
2 | EFOMM 2003 emergency personnel joined about 150 volunteers” and
Which word could replace the underlined word without a “Most of the injured passengers were able to walk out of the
change of meaning? station, but about 20 were carried out on red stretchers”,
the word about cannot be replaced by:
The man deduced what had happened in the playground.
a) some.
b) within.
a) surmised
c) approximately.
b) provoked
d) almost.
c) appealed
e) roughly.
d) condoned
e) discouraged

Just The Way You Look Tonight (Rod Stewart)

1. Someday when I’m awfully low


2. When the world is cold
3. I will feel a glow
4. Just thinking of you
5. And the way you look tonight
6. You’re so lovely
7. With your smile so warm
8. And your cheeks so soft
9. There’s nothing for me but to love you
10. And the way you look tonight
11. With each word, your tenderness grows
12. Tearing my fears apart 006 | JFS 2009
13. And that laugh that wrinkles your nose The verb TO BROIL means the same as:
14. Touches my foolish heart
15. Yes your lovely, never ever change a) to fry.
16. Keep that breathless charm b) to grill.
17. Won’t you please arrange it c) to burn.
18. Cause I love you d) to boil.
19. Just the way you look tonight e) to cook.
20. Just the way you look tonight ...
007 | JFS 2009
3 | EFOMM 2004 In “A pact made by a group of teens to get pregnant and
In verse 1 of the lyrics we find the adverb awfully, which raise their babies together is at least partly behind a sudden
means: spike in pregnancies at Gloucester High School, school
officials said”, the verb to raise is synonymous with:
a) absolutely.
a) to deal.
b) terribly.
b) to praise.
c) fortunately.
c) to increase.
d) wishfully.
d) to rise.
e) certainly.
e) to bring up.
8 | EFOMM 2005
013 | JFS 2010
In “On TV and radio, in print and over the Internet, news is
He may have competed in one of the most passionate
everywhere. But are we better informed or just
derbies on the planet as a player, led a team at the FIFA
overwhelmed?”, the underlined word means:
World Cup™ and be the most successful manager in the
history of British football, but even Sir Alex Ferguson has
a) interested.
admitted that he was shaking the first time he met Nelson
b) agape.
Mandela. "We also met him two years ago and were invited
c) worn out.
to his house and it was amazing how relaxed he was. He
d) familiar.
really seemed to be enjoying himself, which was great to
e) tight.
see. He is a truly amazing man".
9 | EFOMM 2006
– In the fragment above, the word amazing cannot be
Psychologists say kids are being robbed of time they need
replaced by:
for connecting with their parents and siblings.
a) gruesome.
a) colleagues
b) fascinating.
b) partners
c) incredible.
c) relatives
d) stunning.
d) friends
e) wonderful.
e) brother or sister
014 | JFS 2010
10 | EFOMM 2008
In the sentence “Researchers conducting tests in the harsh
Communication is embedded within culture and therefore
environment of Mono Lake in California have discovered the
culturally orientated.
first known microorganism on Earth able to thrive and
reproduce using the toxic chemical arsenic”, the words in
– The word embedded means:
bold can be replaced, respectively, by:
a) dislodged
a) realizing – smooth – get on
b) refused
b) carrying out – strict – develop
c) rooted
c) attaining – sharp – rise
d) remained
d) leading – rough – wear out
e) disturbed
e) handling – mild – grow
11 | EFOMM 2008
015 | JFS 2010
Choose the appropriate synonymous sentence to: “She
In the sentence “the researchers successfully grew microbes
seldom goes to conferences now”.
from the lake on a diet that was very lean on phosphorus”,
the word lean can be replaced by:
a) She almost goes to conferences now.
b) She often goes to conferences now.
a) rich.
c) She hardly ever goes to conferences now.
b) fat.
d) She regularly goes to conferences now.
c) plump.
e) She frequently goes to conferences now.
d) bare.
e) flat.
012 | JFS 2009
Correspondents consider Mr. Mousavi the main reformist
016 | JFS 2008
challenger to President Mahmoud Ahmadinejad, who is
In the sentence “The prize drops to $15 million after that
seeking another term.
date and goes away altogether after 2014”, the verbs to
drop and to go away, and the adverb altogether can be
– In the fragment above, the word term means the same as:
replaced by:
a) post.
a) to increase – to vanish – absolutely
b) job.
b) to lift up – to fade away – actually
c) turn.
c) to fall – to disappear – completely
d) position.
d) to decrease – to leave – utterly
e) task.
e) to curtail – to flee – wholly
17 | EFOMM 2010 – ADAPTED
020 | IME 2010
Read the following sentences: The word underlined in the sentence “That incident, which
took place on 7 November 2007, exhibits many of the
1. NATO warships and helicopters pursued Somali pirates for
hallmarks of the missions in Iraq and Afghanistan ...” can be
seven hours and the highspeed chase only ended when
replaced by which of the following expressions, still keeping
warning shots were fired at the pirates’ skiff.
the same meaning?
2. Seven pirates attempted to attack the Norwegian-flagged
a) landmarks
MV Front Ardenne late Saturday but fled after crew took
b) trademarks
evasive maneuvers and alerted warships in the area
c) stamps
d) signs
3. How the attack was thwarted is unclear, it appears to
e) brands
have been the actions of the tanker.
021 | IME 2012
4. The pirates' release underscores the difficulties navies
On the mountain slopes south of Pompeii, a group of Roman
have in fighting rampant piracy off the coast of lawless
citizens flee the doomed city, leaving their footprints in a
Somalia.
layer of volcanic ash.
– Which option contains words that replace with accuracy
– A synonym for the word ‘flee’ is
the following words that are underlined in the text
respectively: SKIFF – EVASIVE – THWARTED – RAMPANT:
a) arrive
b) investigate
a) vessel – elusive – avoided – increasing
c) run away
b) small boat – dangerous – facilitated – violent
d) meet
c) yacht – ambiguous – planned – violent
e) face
d) small boat – elusive – prevented – uncontrolled
e) vessel – dangerous – spoilt – increasing
022 | JFS 2010
Among the hundreds of thousands of leaked State
18 | EFOMM 2010
Department documents were candid and often unflattering
In the fragment “… naval officers hailed the pirates over
assessments of foreign leaders. The classified diplomatic
loudspeakers and finally fired warning shots to stop them”,
cables were released Sunday by online whistle-blower
the word highlighted could be accurately replaced with:
WikiLeaks. The massive release of material intended for
diplomatic eyes only is sure to ruffle feathers in foreign
a) alerted
capitals since the cables contain many tart comments.
b) threatened
c) called
In the sentence “The cables contain many tart comments”,
d) informed
the words in bold mean respectively the same as:
e) advised
a) wires and sharp.
019 | IME 2010
b) information and luscious.
Modern buildings incorporate exciting forms with glittering
c) links and caustic.
façades and compelling interior spaces. Surveying for these
d) messages and sour.
projects requires sophisticated computation, aggressive
e) chains and bitter.
quality control and close interaction with construction
teams.
023 | JFS 2010
The revolutionary war hero and outstanding silversmith
– Tick the alternative that corresponds to “glittering”, still
made the Sons of Liberty silver bowl in the case.
keeping the same meaning of the sentence.
– The word outstanding cannot be defined as:
a) glossing
b) gliding
a) impressive.
c) gladding
b) remarkable.
d) gluing
c) stunning.
e) gloating
d) feeble.
e) striking.
024 | JFS 2010
a) fairly
Give the correct sequence of synonyms for the highlighted b) doubtfully
words in the excerpt below:
c) faithfully
d) steadily
Egypt’s tough 82-year-old president, in charge for the past
e) eventually
three decades, now says he will go – but only at the end of
his term in September, with dignity and with a subtle threat
028 | JFS 2011
that if he does not get his way, things could turn uglier still.
The conditions for the agreement were very austere.
a) sturdy – turn – slight – become – Which of the following options cannot be used to replace
b) strong – shift – tenuous – change into the underlined word?
c) healthy – cycle – faint – shift
d) stiff – spell – fine – grow into a) severe
e) frail – span – quiet – develop
b) harsh
c) strict
025 | ITA 2007
d) feeble
Even after adding in the 16 ovens and coffee machines in the e) relentless
galley, plus the electronic navigation, communications and
avionics systems, we will still be well short of the electrical 29 | ITA 2009 – ADAPTED
system on board the advanced jets that will go into service At the Eucentre, a research site cofounded by the Italian Civil
between 2006 and 2010.
Protection Department in Pavia, Italy, a young engineer dons
a firefighter’s uniform that has been in testing for six
– A expressão to be well short of pode ser substituída por: months. (...) Though the technology was pioneered in the
U.S., the Europeans have taken the reins in a bid to revitalize
a) to be far from.
their traditional-textile industry, which has been hammered
b) to be adequately supplied with.
by Asian competition. “We want to develop state-of-the-art
c) to be below the limit. know-how that can’t be found in Asia,” says Andreas
d) to be close. Lymberis, a scientific officer with the European Commission
e) to be left off. who has championed smart textiles.
026 | ITA 2008
– Assinale a alternativa em que o termo em negrito no
Assinale a opção que contém os significados mais adequados fragmento acima NÃO pode ser substituído pela opção
para as palavras sublinhadas nos trechos abaixo: apresentada.
I. The Illinois Smoke-free Restaurant Recognition Program a) dons (parágrafo 1) – wears
recognizes those restaurants that have gone entirely smoke- b) the reins (parágrafo 2) – control
free… c) a bid (parágrafo 2) – an attempt
II. Second-hand smoke is a combination of the smoke from a d) hammered (parágrafo 2) – stopped
burning cigarette… e) championed (parágrafo 2) – supported
III. More importantly, it represents a dangerous health
hazard. 30 | ITA 2012 – ADAPTED
IV. EPA estimates that approximately 3.000 American They would keep on developing until they were far more
nonsmokers die… intelligent than we are.
a) I. constantly II. provided by III. complication IV. likely Na sentença acima, o vocábulo grifado poderia ser
b) I. completely II. derived from III. risk IV. nearly substituído por:
c) I. generally II. made by III. infection IV. almost
d) I. slightly II. exhaled by III. loss IV. around a) far away.
e) I. totally II. produced by III. problem IV. exactly b) incredible.
c) much.
027 | JFS 2012 d) distant.
Give a synonym for the underlined word in the sentence e) many.
below:

I am reliably informed that you have been talking about


resigning from the company.
031 | ITA 2012
033 | AFA 2012
This Janus-like quality is true of all intelligence services, I Observe the fragment taken from the blues Blow Wind Blow
suppose, but I have never seen an organization quite like the (Muddy Waters, Paul Butterfield):
ISI. It is at once very secretive and very open, yet ISI officials
get especially peeved at the charge of duplicity: “I cannot go When the sun rose this morning,
on defending myself forever, even when I am not doing what I didn’t have my baby by my
I am blamed for,” wrote one of my ISI contacts, after I had side. When the sun rose this
written a column noting the organization’s “double game” morning, I didn’t have my baby
with the U.S. “I shall do what I think is good for PAKISTAN, by my side. I don’t know where
my country. I am sure you will do the same for US.” she was,
I know she’s out with some another guy.
– A opção cujo significado mais se aproxima do vocábulo
peeved (linha 4) é: – In the context of the song, the word when can be
substituted for:
a) mistrustful.
b) challenged.
a) while.
c) irritated.
b) even though.
d) worried.
c) considering that.
e) encouraged.
d) by the time.
032 | ITA 2011
034 | EEAR 2008
Leia a seguinte imagem:
The underlined word in the sentence below means:

– The appropriate response to “How are you?” that anyone


expects to hear is an automatic “I’m fine”.

a) wish
b) answer
c) excuse
d) greeting

035 | EEAR 2008


In the sentence:

Television has contributed to change the values and


perceptions of Americans.

– The underlined word is closest in meaning to:

a) cultures.
b) behaviour.
c) information.
Adapted from http://www.gocomics.com/tomtoles d) conceptions.

– A palavra breakthrough, na charge, tem o mesmo sentido


de: “When you relinquish the desire to control your future,
you can have more happiness.”
a) customary. Nicole Kidman
b) inept.
c) conventional.
d) innovative.
e) ordinary.
2ND Part | Reading Skills 003 | IME 2011
The text states that human beings instinctively:
and General Review
a) provoke opportunities and threats to other human beings.
Is ‘Facebook’, the social networking website, b) move towards that which threatens them.
making us narcissist? A new book argues we’re much more c) react to sudden changes they experience.
self-absorbed nowadays, stating that technology is to blame. d) interrupt work and family life.
I tweet, therefore I am. Or is it, I tweet, therefore I am e) undermine bursts of information.
insufferable? As if adult celebrities that pop out on the red
carpets weren’t clue enough, we now have statistical 004 | AFA 1998
evidence that we are a lot more in love with ourselves than “Can you tell a green field from a cold steel rail?”
we used to be. This social phenomenon has raised fields of (Excerpt from Pink Floyd’s Wish you were here lyrics)
research to academic studies nowadays. In the book ‘The
Narcissism Epidemic: Living in the Age of Entitlement’, Jean – The questions asks:
M. Twenge, a professor of psychology at San Diego State
University, and W. Keith Campbell, a social psychologist at a) if you can tell a story about the field and the cold steel
the University of Georgia, look to the Narcissistic Personality rail.
Inventory, which measures self-regard, materialism, and lack b) if you are able to say how to go from the green field to
of empathy. They found that the number of college students the rail road.
scoring high on the test has risen by 30 percent since the c) if you know one difference between the green and the
early 1980’s. cold steel rail.
d) if you are able to see how different is a green field from a
001 | IME 2011 cold steel rail.
What kind of human behavior is central to the study
mentioned in the text? Want a Reservation at One Hot D.C. Restaurant?
Prepare to Sign a Contract
a) the alienation of the celebrities from the others around
them. The restaurant may have Rogue in its name, but the
b) addiction to technology joint certainly looks to keep its patrons in line.
c) the high scoring of college students in academic tests Many elite eateries have strict policies on
d) the hard work of social psychologists cancellations and photography, but Rogue 24 has taken
e) excessive positive feelings and admiration of oneself what's usually an implicit verbal contract to a binding level.
That's right, get your pen out.
002 | IME 2011 Rogue 24, headed by chef R.J. Cooper, fuses the
What has NOT been encouraging people to act the way eating experience with an intense visual adventure – after
described in the text? all, it's housed in a grungy D.C. alley surrounded by
dilapidated buildings. But the 2-page contract diners must
a) technology sign along with their reservation calls up air of pretension.
b) narcissism Eater DC provides a look at the binding document
c) materialism which includes a ban on photography and cell phones during
d) psychology dinner. They cite the attempt to create an environment
e) entitlement “free of distraction” – to focus on the food, right? It's worth
noting that the kitchen is in the middle of the restaurant.
Scientists say juggling e-mail, phone calls and other That should provide clamor enough to steal the attention of
incoming information can change how people think and even the most tuned-out diners.
behave. They say our ability to focus is being undermined by Their stated cancellation policy is a demand even
bursts of information. These play to a primitive impulse to more intense than keeping your phone and camera stashed
respond to immediate opportunities and threats. The during dinner. You can cancel your reservation 72 hours
stimulation provokes excitement — a dopamine squirt — before with no penalties. Fair. But when breaking it within
that researchers say can be addictive. In its absence, people that three-day window, prepare to face the penalty. You can
feel bored. The resulting distractions can have deadly cancel up to 3 p.m. on the day of your reservation with only
consequences, as when cell phone-wielding drivers and train a 50% penalty. After 3 p.m. or a no-show to dinner? Rogue
engineers cause wrecks. And for millions of people these 24 doesn't care – they'll put you on the hook for the full
urges can inflict nicks and cuts on creativity and deep price. That's up to $175 per person (if you choose the 24
thought, interrupting work and family life. courses with wine pairing). But how? Simple. Rogue 24 asks
for your credit card number on the contract, so they've got
you on the hook.
Okay, we realize it could be for Rogue 24's own 008 | JFS 2011
protection. , their carefully-protected “Journey” As receitas do restaurante Rogue 24 devem ser
menu runs 24 courses and takes three hours to serve (and extremamente saborosas.
eat). And surely hours of preparation go into each meal,
providing apt backing for the intense cancellation – Qual o equivalente, em inglês, para o vocábulo em negrito
restrictions. Imagine if photos from such a storied menu na sentença acima.
appeared on Twitter? The horror!
But really, no matter how delectable the menu a) Receipts.
might be, since when is a simple meal run like a business b) Recipes.
deal? You might want to call your lawyer before planning c) Prescriptions.
your night out at Rogue 24. It's only fair. d) Guidelines.
Adapted from http://newsfeed.time.com/ e) Instructions.

005 | JFS 2011 Persuading Leonardo


Infere-se a partir da leitura do texto que:
Although both Ben Shneiderman's Leonardo's
a) a multa por um eventual cancelamento de reserva no Laptop: Human Needs and the New Computing Technologies
restaurante Rogue 24 pode chegar a 175 dólares, por and B.J. Fogg's Persuasive Technology: Using Computers to
pessoa, caso o aviso não seja feito com no mínimo 72h antes Change What We Think and Do are written by academics,
do dia marcado. the books transcend academia to provide a different view of
b) não é permitido o uso de equipamentos eletrônicos the Internet's potential. Shneiderman prepares the
dentro do restaurante em virtude do receio que haja groundwork for what he calls the "new computing," while
vazamento de certas técnicas utilizadas pelos chefs do Fogg describes how to make that computing persuasive.
estabelecimento durante a confecção dos pratos. The idea behind Leonardo's Laptop is a
c) a assinatura de contratos no ato de se fazer uma reserva é consideration of what Leonardo da Vinci would demand
uma prática que se torna cada vez mais comum nos from a laptop computer and what he would do with it. To
melhores restaurantes norte-americanos. Shneiderman, who is founding director of the Human-
d) o restaurante, localizado em Washington, solicita o Computer Interaction Lab at the University of Maryland, the
número do cartão de crédito do cliente para garantir que o new computing puts users first. Shneiderman begins with a
pagamento da reserva seja feito antes do dia marcado. brief history of computing and computer applications,
e) o autor não concorda com a política adotada pelo declaring that, "These founders of the old computing
restaurante Rogue 24 em fazer seus clientes assinarem overcame technological limitations to build impressive
contratos no ato de se fazer uma reserva. projects and then turned to producing tools for themselves,
giving little thought to the needs of other users." Although
006 | JFS 2011
not a founder, I admit to being of the old computing
Assinale a opção correta:
generation. I programmed in dead languages such as IBM's
a) o vocábulo patron (linha 2) equivale semanticamente a 1401 Autocoder and 360 Assembler before progressing to
owner. Cobol and RPG. I have now learned Visual Basic and C++, and
b) o vocábulo strict (linha 3) pode ser substituído por I can report that there is nothing intrinsic to any of these
austere. languages that center a programmer's focus on those who
c) o vocábulo binding (linhas 5 e 12) pode ser traduzido use their applications. The new computing is not about
“insensato”. languages but, as Shneiderman suggests, about
d) a expressão up to (linha 24) pode ser substituída por as understanding human activities and human relationships.
far as. With Leonardo as both creator and user, his laptop
e) a expressão on the hook (linha 26) significa “na lista will enable greater creativity and grander goals. This book
negra”. goads you with ideas for applications in e-learning, e-
business, e-healthcare, and e-government. Each area is built
007 | JFS 2011 around a framework for technology innovation that
A lacuna presente no 6º parágrafo deve ser corretamente Shneiderman calls the "four circles of relationships" and the
preenchida por: "four stages of activities." (…)
Although the mental picture of Leonardo with a
a) Inasmuch as notebook computer excites the imagination, as a literary
b) Afterwards device, it does not wear well as the book progresses.
c) After all Nonetheless, Shneiderman achieves the objective of
d) Instead Leonardo's Laptop — creating a foundation for the new
e) Actually computing.
With a new computing application in hand, B.J.
010 | ITA 2009
Fogg's Persuasive Technology: Using Computers to Change
Considere as seguintes afirmações.
What We Think and Do gives you advice on its
implementation. To Fogg, who launched Stanford's I. As duas obras discutidas no texto têm como assunto
Persuasive Technology Lab and who holds seven patents in principal o uso do computador e suas aplicações atuais e
the area of UI* design, a web site must first be credible to be potenciais.
persuasive. Fogg has coined the term "captology" to II. Shneiderman e Fogg, autores do texto, mostram a
describe this branch of the study of computers. From the potencial aplicação da internet nos dias atuais.
book's "Introduction:" III. De acordo com Shneiderman, o computador eficaz deve
Captology focuses on the design, research, and ser, concomitantemente, uma ferramenta capaz de
analysis of interactive computing products created for the persuadir e um agente interativo.
purpose of changing people's attitudes or behaviors.
It is the computer's ability to provide interactivity – Está(ão) correta(s)
that gives its applications an advantage over other forms of
media. a) apenas a I.
Persuasive Technology describes three basic roles b) apenas a II.
that computers play: the computer as a tool, as media, and c) apenas a III.
as a social actor. Further, there are seven types of d) apenas I e II.
persuasive tools described by Fogg. Such tools persuade by e) apenas I e III.
simplifying, tunneling (guiding), customizing, being there at
the right time, removing tedium, rewarding after 011 | ITA 2009
observation, and reinforcing proper behavior. As media, Com relação a Leonardo’s Laptop: Human Needs and the
computers can modify behavior by simulating new New Computing Technologies, NÃO se pode dizer que a
endeavors. As a social actor, computers persuade through obra:
praise. However, no matter the role, to persuade, the
application must be credible. a) tem como foco o usuário de computadores, seja ele um
Perhaps the most interesting parts of Fogg's book iniciante ou especialista no assunto.
are the two chapters that discuss the ways in which b) destaca a importância de programas como Autocoder e
computer applications destroy their own credibility and what Assembler, assim como COBOL, RPG, Visual Basic e C++.
an application or web site must do to be considered, by its c) discute o tipo de uso que Leonardo da Vinci faria, caso
users, trustworthy. According to Fogg, a computing device or tivesse um computador portátil.
application is perceived to be credible only if it is first d) mostra a importância das relações humanas no uso do
perceived as believable — trustworthiness based on computador.
expertise. In brief, an application is trustworthy if it is e) apresenta ao usuário possibilidades de diferentes usos do
thought to be fair and unbiased. It is trustworthy if its author computador, dentre eles, para negócios eletrônicos.
or origin is thought to be skilled and knowledgeable. The
crux of the issue is that credibility matters. 012 | ITA 2009
Both books are thoroughly documented and both Com relação a Persuasive Technology: Using Computers to
are excellent points of departure for a more detailed inquiry Change What We Think and Do, analise as afirmações a
into the available material. If both books are taken to heart, seguir:
using computers and their applications will become
enjoyable and satisfying. I. O trabalho foi idealizado no Laboratório de Tecnologia
Persuasiva da Universidade de Stanford e consiste na sétima
* U.I. - User Inteface criação intelectual do autor.
II. Ao propor um novo conceito na área computacional, o
D. Wohlbruck, Dr Dobb’s Journal, January, 2004. autor destaca mudanças de atitude ou de comportamento
dos usuários.
009 | ITA 2009 III. A obra argumenta que uma página da web deve ser
Indique o gênero, em inglês, ao qual o texto acima pertence. confiável para seduzir o usuário.

a) summary – Está(ão) correta(s)


b) review
c) essay a) apenas a I.
d) abstract b) apenas a II.
e) report c) apenas a III.
d) apenas I e II.
e) apenas II e III.
a) contraste.
b) evolução.
c) crítica.
d) ironia.
e) inexorabilidade.

Looking for love? Formula isn't online, report says

If you’re bemoaning the lack of a Valentine, chances


are you’ve turned to what seem like a gazillion dating
websites for help.
Buyer beware, though, caution a team of
psychologists who’ve just published a lengthy report about
online dating, now a billion-dollar industry.
“There are sites that will tell you, ‘based on decades
of scientific research and basic math, we can find your
compatible mate for you,’” says lead author Eli Finkel, an
associate professor of social psychology at Northwestern
University. “That’s a pretty tantalizing offer.”
The problem, Finkel says, is that these websites
have no scientific evidence to back up their claims that they
can find your soulmate.
Well of course they don’t. Science and romance go
together like Demi and Ashton, right?
Actually, Finkel says, scientists have been studying
Adapted from http://brand.nmsu.edu/ relationships for 80 years or so. And one thing is clear: It’s
impossible to determine that two people have what it takes
013 | JFS 2011 to maintain a long-term relationship before they’ve even
Julgue os itens a seguir: met.
Research has shown that three types of information
I. Os cursos da NMSU abrangem as áreas de magistério, are needed to predict whether a couple will fall in love and
engenharia, hotelaria e artes cênicas. stay in love, Finkel says.
II. A NMSU, por ser a melhor instituição de ensino superior One is demographics. It helps if a potential mate is
do estado, afirma que o sucesso profissional dos graduados age – and geographically appropriate.
é garantido. A second, says Finkel: “What are the actual dynamics
III. A partir do slogan da NMSU, infere-se que o aluno será between two people who have met?”
capaz de aprender não apenas o conteúdo pragmático das And last, “What are the life circumstances that
disciplinas, mas também a harmonizá-los com as vicissitudes affect the couple?” Finkel says. “There’s no way they could
da vida cotidiana. possibly know that a hurricane or a cancer diagnosis or a
IV. As áreas de estudo mencionadas no anúncio podem ser sexy coworker is around the corner.”
encontradas em todos os cinco campi da NMSU. Probably the best-known matchmaking website is
eHarmony.com, which charges $59.95 for a month’s
– Agora, marque a opção correta: subscription. eHarmony asks clients approximately 250
questions about 29 “dimensions of compatibility,” ranging
a) Há apenas uma asserção correta. from conflict resolution to kindness to ambition. eHarmony’s
b) Há somente duas asserções incorretas. “matching algorithm” is proprietary, so the company did not
c) As quatro asserções estão corretas. share it with Finkel and his coauthors.
d) As quatro asserções estão incorretas. In a statement, spokeswoman Becky Teraoka said
e) Apenas as asserções ímpares estão corretas. the proof of eHarmony’s success is in the numbers. On
average, she said, 542 people marry in the U.S. each day as a
014 | JFS 2012 result of being matched on eHarmony, according to a 2009
Leia a seguinte frase: study conducted for the website by Harris Interactive.
“eHarmony’s matching system is based on years of
What we are seeing increasingly is a society of private empirical and clinical research on married couples,” Teraoka
affluence and public squalor. said. “As part of this work, we have studied what aspects of
personality, values and interest, and how pairs match on
– A principal idéia contida na sentença acima expressa:
them, are most predictive of relationship satisfaction.”
Finkel isn’t convinced. Speed-dating, which he’s also
Extract 1
studied, can tell prospective mates more about
than profiles from a website, he says. “The human mind was Japan’s ship yards remain intact after quake
built to size people up pretty quickly. The human mind was
not built to browse a profile and figure out whether Japan’s major ship yards escaped the full impact of
somebody is compatible.” the March 11 earthquake and tsunami that struck the
If you’re looking for love online, Finkel says, your northeastern coast of the country with full force. An official
best bet is to save your money and stick with the less- at the Japan Ship Exporters’ Association said the devastating
expensive websites in which you browse profiles, as natural disaster “will have no impact on future export ship
opposed to those that try to make matches for you. orders at all”. Although several small ship yards in the quake-
But, warns Finkel, who met his wife the old- hit areas were affected, major Japanese ship yards that build
fashioned way through a fix-up arranged by their large vessels for exports are concentrated in western Japan
grandmothers, “get offline fairly quickly, because you’re and remain intact, the official said. Mitsui Engineering & Ship
never going to be able to figure out from a profile and some building sustained “slight material damages” in the
emails whether you’re compatible with somebody.” company’s Kasai Center and Chiba Works but did not
consider such slight damages would cause serious impact on
Adapted from http://todayhealth.today.msnbc.msn.com/ operations. “The rotational schedule of interruption of
power supply due to the earthquake may affect our
015 | JFS 2012 operation at our works and subsidiaries. However, the
Os termos bemoaning (1º parágrafo), tantalizing (3º degree of the impact is not known now,” the company said
parágrafo), ranging (11º parágrafo) e fairly (16º parágrafo) in a statement. Japanese export ship orders rose for the 15 th
podem ser substituídos, respectivamente, por: consecutive month in February on a year-on-year basis.
Japanese ship builders received orders for 277 export vessels
a) lamenting, fascinating, fluctuating e scantily. – 259 bulk carriers, 10 oil tankers and eight general cargo
b) regretting, teasing, varying e pretty. vessels – in the April-February period.
c) complaining, charming, wandering e willingly.
d) weeping for, harassing, stretching e barely. Adapted from www.australianmerchantnavy.com,
e) gloating, taunting, reaching e quite. March 2011

016 | JFS 2012 Extract 2


Levando em consideração o contexto, preencha correta e
coerentemente a lacuna presente no 14º parágrafo: Tsunami Debris Expected on U.S. Shores in 3 Years

The powerful tsunami triggered by the 9.0 Japanese


a) each other
earthquake destroyed coastal towns near Sendai, washing
b) one another
such things as houses and cars into the ocean. Projections of
c) themselves
where this debris might head have been made at the
d) them
International Pacific Research Center, University of Hawaii at
e) everyone
Manoa. What their model predicts about the tsunami debris
017 | JFS 2012 is that they first spread out eastward from the Japan Coast
De acordo com o texto, marque o item correto. in the North Pacific Subtropical Gyre. In a year, the
Papahanaumokuakea Marine National Monument will see
a) Os sites de relacionamento utilizam eficientes métodos pieces washing up on its shores; in two years, the remaining
matemáticos para relacionarem os usuários com os Hawaiian islands will see some effects; in three years, the
parceiros mais possivelmente equivalentes. plume will reach the US West Coast, dumping debris on
b) Ciência e romance são completamente incompatíveis, daí Californian beaches and the beaches of British Columbia,
a ineficiência dos sites de relacionamento. Alaska, and Baja California. The debris will then drift into the
c) Os psicólogos que realizaram o estudo feito sobre os sites famous floating junkyard, the North Pacific Garbage Patch,
de relacionamento concluíram que é impossível determinar where it will wander around and break into smaller and
o grau de compatibilidade entre duas pessoas antes delas se smaller pieces. In five years, Hawaii shore scan expect to see
conhecerem pessoalmente. another barrage of debris that is stronger and longer-lasting
d) Os fatores determinantes, utilizados pelos sites de than the first one. Much of the debris leaving the North
relacionamento, para que duas pessoas se apaixonem e Pacific Garbage Patch ends upon Hawaii’s reef sand beaches.
tenham uma união duradoura são idade, proximidade These model projections will help to guide clean-up and
geográfica, dinamismo conjugal e questões circunstanciais. tracking operations.
e) Eli Finkel e sua equipe não tiveram acesso ao algoritmo
Adapted from www.geog.ucsb.edu,
utilizado pelo site de relacionamentos eHarmony.
April2011
18 | EFOMM 2012
22 | EFOMM 2012
According to the first extract, it is correct to say that: Extract2 explains that the North Pacific Garbage Patch:
a) Japanese ship yards that build large vessels were virtually a) was formed after the Japanese tsunami.
unharmed by the impact of the tsunami. b) will reach the shores of the US West Coast.
b) all Japanese ship yards are not located on the path of the c) is formed by the garbage dumped on US shores.
tsunami. d) is formed by the accumulation of garbage in the ocean.
c) Japanese ship yards located on the path of the tsunami e) releases debris that will eventually end up in Baja
were slightly affected by the devastating waves. California.
d) shipyards in northwestern Japan were the most affected
by the earthquake and tsunami. Wall Street's wild swings last week helped skew
e) the earthquake and tsunami will have a big impact on the both retirement portfolios and mathematical models of the
Japanese ship yard industry over the next decade. financial markets. After all, a standard Gaussian function —
a bell curve — would predict that such extreme dips and
19 | EFOMM 2012 rises would be exceedingly rare and not prone to following
Read the following piece from extract 1 “The rotational one after the other on succeeding days.
schedule of interruption of power supply due to the Gaussian functions might be able to describe the
earthquake may affect our operation at our works and distribution of grades in a big college class, with most
subsidiaries.” It may be inferred that: students getting, say, B–/C+, and enable you to predict how
many students will get A's or fail. But evidently, they do a
a) Operations will be greatly affected by the constant poor job at explaining steep fluctuations in stock prices,
interruption of power supply. (I) some economists and modelers think they
b) The earthquake caused the interruption of power supply are the best tool available to describe financial markets.
without any consequences to the operations. So can any math accurately describe market
c) Some operations are expected to be affected by the behavior and enable you to beat it? To find out, Scientific
interruption of power supply. American spoke with statistical physicist H. Eugene Stanley
d) Subsidiaries and works will benefit from the power supply of Boston University, a proponent of applying the
due to the earthquake. approaches and concepts of physics to economics.
e) Power supply was not affected by the earthquake and
operations will continue. Can mathematical models beat markets?
They haven't yet. Science is about empirical fact.
20 | EFOMM 2012 There is no question that optimistic people think they can
03. According to the following segment from the first beat the market, but they don't do it consistently with
extract: mathematical models. No model can consistently predict the
future. It can't possibly be.
“Japanese export ship orders rose for the 15 th consecutive
month in February on a year-on-year basis.” So what can math predict?
What you can do is predict the risk of a given event.
a) The information above is collected every 15 months. The risk just means the chance that something bad will
b) The passage investigates the decrease in the number of happen, for example. That you can do with increasing
ship orders. accuracy because we have more and more data. It's like
c) The figures above were released before the tsunami. insurance companies: they cannot tell you when you are
d) One can tell for sure that Japanese ship orders were going to die, but they can predict the risk that you will die
decreasing just before the tsunami. given the right information. You can do the same thing with
e) Japanese ship orders have increased for over a year. stocks. If you lose less, you get ahead of those
(II) lose more.
21 | EFOMM 2012
According to the second extract, the model developed by Why do economists and "quants" — those who use
the International Pacific Research Center intends to: quantitative analysis to make financial trades — have such
faith in their mathematical models then?
a) predict the movement of tsunamis in the ocean. If they're just to reduce risk, then they're very
b) study the movement of litter in the ocean. valuable. If you're worried, for example, about the segment
c) indicate the amount of garbage dumped in the ocean. of the Chinese economy that deals with steel, you make a
d) collect the Japanese tsunami debris to clean the ocean. model of what that whole market is all about and then you
e) spread the debris in the ocean after the tsunami. see if we did this what would likely happen. They're right
some of the time. It's better than nothing.
But when they have excessive faith in these models,
a) I. although – II. which – III. one another
it's not justified. Math starts with assumptions; the real
b) I. however – II. who – III. each other
world does not work that way. Economics, which calls itself a c) I. although – II. who – III. one another
science, too often doesn't start with looking at empirical d) I. however – II. whom – III. each other
facts in any great detail. Fifteen years ago even the idea of e) I. although – II. whom – III. one another
looking at huge amounts of data did not exist. With a limited
amount of data, the chance of a rare event is very low,
025 | JFS 2011
which gave some economists a false sense of security that
De acordo com o texto:
long-tail events did not exist.
Why do you argue that financial markets are ruled not by a) as oscilações do mercado financeiro podem ser
Gaussian functions but by power laws — relations in which comparadas aos altos e baixos que a maioria dos estudantes
the frequency of one event varies as a power of some de ensino médio enfrenta no que diz respeitos às notas dos
attribute of that event and are generally more L-shape than exames avaliativos.
bell shape? b) a mais recente crise em Wall Street poderia ter sido
prevista se a função gaussiana tivesse sido aplicada
For anything that is random and fluctuating, like a corretamente.
financial market, a Gaussian function is a wonderful way to c) uma curva de sino, considerada padrão quando a função
make a histogram of the outcome. If the things that gaussiana é aplicada, indica que oscilações no mercado de
fluctuate are not correlated at all with (III), then ações são raras, embora suscetíveis a serem constantes.
it's demonstrable that a Gaussian function is the correct d) há outros métodos, além da função gaussiana, que
histogram. The catch is: in a financial market, everything is
permitem aos economistas fazer análises dos mercados
correlated. The proof of that is that if the stock market were financeiros.
Gaussian, then you'd never have a flash crash *. (…) e) apenas fórmulas matemáticas e conceitos físicos
conseguem descrever o comportamento dos mercados
Adapted from http://www.scientificamerican.com/ financeiros de maneira precisa.
* The Flash Crash was a United States stock market crash on 026 | JFS 2011
May 6, 2010 in which the Dow Jones Industrial Average Marque o título que melhor contempla o tema do texto.
plunged about 900 points — or about nine percent — only
to recover those losses within minutes. (From Wikipedia) a) How to Cope with the Financial Markets’ Crisis?
b) The Stock Markets’ Flash Crash
023 | JFS 2011 c) Economy & Mathematics: How to Join Forces and Solve
H. Eugene Stanley: Crises?
d) The Financial Markets’ Mathematical Revolution
a) acredita que a aplicabilidade da função gaussiana para se e) Can Math Beat Financial Markets?
fazer previsões sobre o comportamento dos mercados
financeiros é válida, porém falha. When football professional in South
b) acredita que o método utilizado pelas companhias de Africa in 1959, 12 clubs broke from the amateur ranks.
seguro para fazer previsões pode ser aplicado ao mercado However, in the strict days of Apartheid, these pioneers
financeiro de maneira mais eficiente do que os métodos whites-only organizations and today, all
vigentes. but a few, defunct. One of the survivors is Arcadia from
c) acredita que a fé depositada pelos economistas, que Tshwane/ Pretoria, an outfit that today competes in the
utilizam a análise quantitativa para fazer transações amateur ranks and concentrates on junior football.
comerciais, em seus modelos não é justificada.
d) acredita que a falsa sensação de segurança, sentida pelos Adapted from http://www.fifa.com/worldcup
economistas, é justificada pela grande quantidade de dados
estatísticos que eles costumam coletar sobre os mercados 027 | AFA 2011
financeiros. Mark the alternative which completes the gaps from the text
e) acredita que a aplicação da função gaussiana na análise correctly.
dos mercados financeiros faria com que o cenário de ocorrer
um flash crash fosse menos provável. a) had gone – have been – were
b) went – were – are
024 | JFS 2011 c) have been – have been – would be
As lacunas I, II e III, presentes no texto, devem ser d) was – had been – will be
preenchidas, respectivamente, por:
028 | AFA 2011
030 | JFS 2011
The text affirms that: O autor do texto acredita que:
a) days of Apartheid were extinguished as well as the a) a religião e o futebol são as duas principais paixões do
prejudice against black football players. povo brasileiro.
b) the pioneers of profession football in South Africa were b) os brasileiros foram os principais responsáveis pela
basically white. profissionalização do futebol.
c) there are no more organizations (professional or amateur) c) pessoas de outras nacionalidades também torcem pela
like the ones from the past. seleção brasileira de futebol.
d) in early 50’s in South Africa there weren’t amateur d) os momentos mais marcantes da história do futebol
football clubs anymore. mundial foram concebidos pela seleção brasileira.
e) várias outras seleções do mundo tentam, em vão, imitar o
estilo de jogo da seleção brasileira.

Adapted from www.google.com

029 | AFA 2011


According to the cartoon, FIFA:

a) pretends to improve the world.


b) has more ambitious purposes than the others.
c) wants to have control over the Roman, Genghis Kan’s, and
British Empires.
d) mustn’t be considered an empire. Adapted from http://www.rampagesoccer.com/

With the 2014 World Cup being awarded to Brazil, it 031 | JFS 2011
could be easily argued that football is going home. Ok, so The expression run a couple of errands (1st balloon) means
football originated in Britain and we invented the game but the same as:
the Brazilians have perfected it. They’ve taken the game to
their hearts and elevated it to the extent that it is a religion a) do tides of chores.
to the people. The blue and yellow clad players of Brazil b) pay two bills.
have given us some memorable moments and play the game c) do some shopping.
with a style and rhythm that no other nation on earth seems d) do a few tasks.
capable of. The game is played to a samba beat that runs e) correct some mistakes.
through the heart and soul of the entire country. When they
are on song, they are unstoppable and people the world 032 | JFS 2011
over clamour to see them play. Chances are, if you asked In the 2nd balloon, take-out is a kind of:
people to name the team they want to see win aside from
their own it would be the magicians from Brazil. a) container.
b) package.
Adapted from http://www.footballworldcupbrazil2014.com/ c) meal.
d) food.
e) duty.
033 | JFS 2011
Too much TV has been associated with violence,
The couple in the cartoon: obesity and social isolation. But TV is having a positive
impact on the lives of billions worldwide, and as the spread
a) has two kids.
of mobile TV, video cameras and YouTube democratize both
b) has just had a baby.
access and content, it will become an even greater force for
c) cheers for different soccer teams.
humbling tyrannical governments and tyrannical husbands
d) enjoys soccer.
alike.
e) has a very busy routine.
Kenny, a development economist, is the
author of a forthcoming book on
innovation, ideas and the global standard
of living

Adapted from http://www.time.com/

034 | ITA 2011


De acordo com o texto, o argumento que melhor justifica o
título TV Will Save the World é:

a) a TV se tornará um meio ainda mais importante para


enfraquecer governos e maridos tirânicos.
b) a TV possibilitará melhoras na educação dos adultos,
TV Will Save the World principalmente no desenvolvimento das habilidades de
In a lot of places, it’s the next big thing leitura.
c) a TV continuará exercendo um impacto positivo nos países
By Charles Kenny em desenvolvimento.
d) a TV propiciará a diminuição da obesidade, da violência e
Forget Twitter and Facebook, Google and the do isolamento social.
Kindle. Forget the latest sleek iGadget. Television is still the e) a TV trará melhoras para a vida de mulheres afegãs.
most influential medium around. Indeed, for many of the
poorest regions of the world, it remains the next big thing — 035 | ITA 2011
poised, finally, to attain truly global ubiquity. And that is a Sobre a presença da TV no mundo, o texto informa que:
good thing, because the TV revolution is changing lives for
the better. a) em países em desenvolvimento, haverá mais aparelhos de
Across the developing world, around 45% of TV do que pessoas até 2013.
households had a TV in 1995; by 2005 the number had b) até 2013, mais de 2/3 das famílias, em todo o mundo,
climbed above 60%. That's some way behind the U.S., where terão aparelhos de TV.
there are more TVs than people, but it dwarfs worldwide c) depois da queda do Talibã, a TV foi declarada ilegal e
Internet access. Five million more households in sub- poucos afegãos possuem um aparelho.
Saharan Africa will get a TV over the next five years. In 2005, d) em 2005, nos países em desenvolvimento, o número de
after the fall of the Taliban, which had outlawed TV, 1 in 5 televisores diminuiu drasticamente.
Afghans had one. The global total is another 150 million by e) nos países que possuem o maior número de televisores, o
2013 — pushing the numbers to well beyond two-thirds of acesso à Internet também é proporcionalmente maior.
households.
Television's most transformative impact will be on 036 | ITA 2011
the lives of women. In India, researchers Robert Jensen and Segundo o texto, um dos impactos que a TV a cabo trouxe
Emily Oster found that when cable TV reached villages, para a vida das mulheres indianas foi que elas:
women were more likely to go to the market without their
husbands' permission and less likely to want a boy rather a) passaram a gostar de ir ao mercado sem a permissão de
than a girl. They were more likely to make decisions over seus maridos.
child health care and less likely to think that men had the b) ficaram menos propensas a preferir ter um filho a uma
right to beat their wives. TV is also a powerful medium for filha.
adult education. In the Indian state of Gujarat, Chitrageet is a c) se mostraram mais dispostas a fazer compras sozinhas.
hugely popular show that plays Bollywood song and dance d) ainda acham que os maridos têm o direito de agredir suas
clips. The routines are subtitled in Gujarati. Within six esposas, apesar de já criticarem esta prática.
months, viewers had made a small but significant e) não gostam mais de tomar decisões sobre os cuidados
improvement in their reading skills. com a saúde das crianças.
Why Urban, Educated Parents Are Turning to DIY Education
When Laurie Block Spigel, a homeschooling
consultant, pulled her kids out of school in New York in the
They raise chickens. They grow vegetables. They knit. Now a
mid-1990s, “I had some of my closest friends and relatives
new generation of urban parents is even teaching their own
telling me I was ruining my children’s lives.” Now, she says,
kids.
“the parents that I meet aren’t afraid to talk about it. They’re
doing this proudly.”
Many of these parents feel that city schools – or any
schools – don’t provide the kind of education they want for
their kids. Just as much, though, their choice to homeschool
is a more extreme example of a larger modern parenting
ethos: that children are individuals, each deserving a
uniquely curated (IV). That peer influence can
be noxious. (Bullying is no longer seen as a harmless rite of
passage.) That DIY – be it gardening, knitting, or raising
chickens – is something educated urbanites should embrace.
That we might create a sense of security in our kids by
practicing “attachment parenting,” an increasingly popular
approach that involves round-the-clock physical contact with
In the beginning, your kids need you – a lot. They’re children and immediate responses to all their cues.
attached to your hip, all the time. It might be a month. It Even many attachment adherents, though, may
might be five years. Then suddenly you are (I) to have trouble envisioning spending almost all their time with
send them off to school for seven hours a day, where they’ll their kids – for 18 years! For Tera Schreiber, it was a natural
have to cope with life in ways they never had to before. You transition. When you have kept your kids so close, literally –
no longer control what they learn, or how, or with whom. she (V) her youngest till Violet was 4 – it can be
Unless you decide, like an emerging population of a shock to send them away.
parents in cities across the country, to forgo that age-old rite Tera’s kids didn’t particularly enjoy day care or
of passage entirely. preschool. The Schreibers wanted a “gentler system” for
When Tera and Eric Schreiber’s oldest child was Daisy; she was a perfectionist who they thought might worry
about to start kindergarten, the couple toured the too much about measuring up. They knew homeschooling
(II) public elementary school a block away from families in their neighborhood and envied their easygoing
their home in an affluent Seattle neighborhood near the pace and flexibility – late bedtimes, vacations when
University of Washington. It was “a great neighborhood everyone else is at school or work. Above all, they wanted to
school,” Tera says. They also applied to a private school, and preserve, for as long as possible, a certain approach to
Daisy was accepted. But in the end they chose a third path: family.
no school at all. Several homeschooling moms would first tell me, “I
Eric, 38, is a manager at Microsoft. Tera, 39, had know this sounds selfish,” and then say they feared that if
already traded a career as a lawyer for one as a nonprofit their kids were in school, they’d just get the “exhausted
executive, which allowed her more time with her kids. But leftovers” at the end of the day. Says Rebecca Wald, a
“more” turned into “all” when she decided that instead of Baltimore homeschooler, “Once we had a child and I realized
working, she would homeschool her daughters: Daisy, now how fun it was to see her discover stuff about the world, I
9; Ginger, 7; and Violet, 4. thought, why would I want to let a teacher have all that
We think of homeschoolers as evangelicals or off- fun?” (…)
the-gridders who spend a lot of time at kitchen tables in the For many of the homeschoolers I met, family is
countryside. And it’s true that most (III) parents more: the very focus of their lives. And they wouldn’t want it
do so for moral or religious reasons. But education observers any other way. One comfort Tera and Eric Schreiber held on
believe that is changing. You only have to go to a downtown to when they started homeschooling was that if it wasn’t
Starbucks or art museum in the middle of a weekday to see working out, they could enroll the girls in school, literally the
that a once-unconventional choice “has become newly next day. That developed into an annual reassessment. By
fashionable,” says Mitchell Stevens, a Stanford professor now their rhythms are deeply their own; they are embedded
who wrote Kingdom of Children, a history of homeschooling. in a community they love. And at the college up the road
There are an estimated 300,000 homeschooled children in there are plenty of calculus tutors, should they need them
America’s cities, many of them children of secular, highly one day.
educated professionals who always figured they’d send their
kids to school – until they came to think, Hey, maybe we Adapted from http://www.thedailybeast.com/newsweek/
could do better.
037 | JFS 2012 041 | JFS 2012
Preencha as lacunas I, II, III, IV e V correta e coerentemente: No 3º parágrafo, o vocábulo affluent pode ser definido
como:
a) I. expecting – II. high-achieving – III. homeschooling –
IV. upbringing – V. breast-fed a) wealthy.
b) I. expecting – II. highly-achieved – III. homeschooling – b) cozy.
IV. upbringing – V. breast-fed c) developing.
c) I. expected – II. highly-achieving – III. homeschooled – d) peaceful.
IV. upbrought – V. breast-feeding e) bustling.
d) I. expected – II. high-achieving – III. homeschooling –
IV. upbringing – V. breast-fed
e) I. expected – II. high-achieving – III. homeschooled – IV.
upbringing – V. breast-feeding HE MAKES
038 | JFS 2012 FORMULA
Segundo o texto:
ONE HAPPEN
a) os pais perdem o controle sobre os filhos quando estes
começam a frequentar a escola. WITH ENERGY,
b) o ato de se enviar os filhos para a escola é considerado
um ritual completamente ultrapassado. DRIVE AND
c) as pessoas que normalmente optam pela educação fora
da escola são evangélicas ou que vivem em áreas isoladas VISION. SO DO
e, portanto, desprovidas de escolas.
d) é bastante comum encontrar pais que optaram pela WE.
educação fora da escola ensinando seus filhos em
locadoras de filmes e museus.
e) o bullying está entre as principais causas que têm Bernie Ecclestone runs the world’s most prestigious sport.
levado muitos norte-americanos a tirarem seus filhos da One reason, five continents, 12 teams and over half a
escola e educarem-nos em casa. billion fans worldwide. It means compromise is not an
option, and it means that speed, teamwork and precision
039 | JFS 2012 are essential. That`s why the man at the top demands the
Marque o item correto. best. And that’s why he chose us to be the official
partner of
F1, ensuring that the entire sport is delivered across the
globe.
a) A educação domiciliar permite que os pais ensinem www.dhl-brandworld.com/F1
atividades do dia-a-dia aos filhos.
b) O fato de se passar o dia todo com os filhos pode ter Excellence. Simply delivered.
um lado negativo: a possibilidade de se criar um indivíduo
inseguro e despreparado para o convívio em sociedade.
c) A flexibilidade da educação domiciliar permite às
famílias que optam por tal sistema que tenham mais
períodos de férias do que as que adotam o método
convencional.
d) Os pais que escolheram a educação domiciliar como
Adapted from TIME, May 24,
forma de ensinarem seus filhos são comumente taxados
2010
de egoístas.
e) A criança educada a partir da educação domiciliar
042 | ITA 2011
precisa ter seu desenvolvimento avaliado anualmente
Assinale a opção que preenche corretamente a lacuna
pelos pais.
presente no texto e que indica o tipo de serviço ofertado
pelo anunciante.
040 | JFS 2012
A opção que descreve a palavra secular (5º parágrafo) é:
a) sports
a) very old.
b) environmental
b) not having any connection with religion.
c) vastly experienced. c) logistics
d) upper middle class. d) finance
e) skilled. e) economy
043 | ITA 2011
047 | JFS 2012
Assinale uma característica, associada à Fórmula 1, que NÃO A mensagem transmitida pela charge denota, em especial:
foi considerada como essencial pelo anunciante.
a) Incoerência.
a) velocidade
b) Contradição.
b) compromisso
c) Incerteza.
c) precisão
d) Equívoco.
d) exigência
e) Afirmação.
e) trabalho em equipe

044 | ITA 2011 048 | JFS 2012


Escolha o termo cuja função gramatical e significado se O vocábulo rather, no 3º quadrinho, equivale a:
aproximam do vocábulo drive, na chamada do anúncio.
a) wholly.
a) comandar b) fairly.
b) percurso c) slightly.
c) dirigir d) enough.
d) dirigente e) somewhat.
e) determinação
The history of technology is full of breakthroughs in
045 | ITA 2011 one field that wound up working wonders in a related one.
O texto informa que Bernie Ecclestone: The 300B vacuum tube, introduced by Western Electric in
1937 to amplify telephone signals, found a far more
a) administra a Fórmula 1. enduring use as a high-fidelity audio amplifier. The atomic
b) é um esportista famoso. clocks first used in the 1960’s by the U.S. military to track
c) é um piloto prestigiado na Fórmula 1. Sputnik and later to validate Albert Einstein’s relativity
d) tem prestígio em todo o mundo. theories are now the basis of Global Positioning System. And
e) é um dos diretores da empresa anunciante. of course, the magnetron, invented in the 1920’s at General
Electric and used in radars during World War II, later found
itself repurposed as the basis for the microwave oven.

049 | IME 2011


According to the text, what is correct to say?

a) The 300B vacuum tube is used in car engines to amplify


the audio communications systems.
b) The atomic clocks are now applied to a different device
than that of its original idealization.
c) The history of technology has proved that it may wound
humans due to its versatility.
d) The atomic clocks were used to play song tracks in high
fidelity quality.
e) The Global Positioning System was validated by Albert
Einstein’s relativity theories.
Adapted from http://www.readingreview.com/
050 | IME 2011
046 | JFS 2012 According to the text, complete the sentence: “The
Consoante o autor do livro que Charlie Brown está lendo, as microwave oven…”
crianças são muito observadoras. Qual das opções abaixo
não contém relação semântica, em inglês, ao vocábulo em a) relies on the proper function of radars.
negrito? b) and the magnetron were repurposed after their
inventions.
a) Watchful. c) is one more example of the technological inventions
b) Perceptive. which have benefited from the innovations generated
c) Overlooking. during the space race.
d) Discerning. d) and radars used during the World War II both count on
e) Insightful. the magnetron as one of its components.
e) was first idealized at General Electric.
Soon enough, say some engineers, miniature
a) on edge
wireless sensors will be located in spots where it would be
b) in the neighborhood of
inconvenient, to say the least, to change their batteries – c) on the brink of
inside your body, within the steel and concrete of buildings, d) in the vicinity of
in the dangerous innards of chemical plants. But today, even e) surroundings
the most robust nodes can be counted on to last only a few
years. Ideally, engineers need wireless sensors that can last
Twenty years ago, when Paul McCartney turned
forever without external power sources or battery changes.
50, he remembers his then-manager pushing the idea of
According to research presented in December at the
retirement. "It's only right," he was told. "You really don't
International Electron Devices Meeting, in Baltimore, that
want to go beyond 50, it's going to get embarrassing." In
dream is within reach.
June, McCartney will be 70 ("I'm never going to believe I'm
70, I don't care what you say," he says. "There's a little cell in
051 | IME 2011
my brain that's never going to believe that"), and he still has
What inconvenience is mentioned in the text?
no plans to stop touring or recording. "You get the argument
'Make way for the young kids,'" he says. "And you think,
a) The fact that batteries are not lifetime loaded.
'Forget that, let them make way for themselves. If they're
b) The spots where wireless sensors are placed nowadays.
better than me, they'll beat me.' Foo Fighters don't have a
c) The micro size of wireless sensors.
problem, they're good. They'll do their thing.
d) To use wireless sensors inside the body.
"If you're enjoying it, why do something else? And
e) That buildings are made of steel and concrete.
what would you do? Well, a good answer is 'Take more
holidays,' which is definitely on the cards, but I don't seem
052 | IME 2011
to do that. I love what I do so much that I don't really want
What does the sentence “According to research presented in
to stop. I'm just kind of casually keeping an eye on how I
December at the International Electron Devices Meeting, in
feel, and onstage, it feels like it's always felt. So for the time
Baltimore, that dream is within reach.” imply about the text?
being, the band's hot, I'm really enjoying myself, still singing
like I sang, not experiencing, touch wood, any sort of
a) In December engineers will come out with a solution for
problems to speak of. If it ain't broke, don't fix it."
the problem.
It doesn't hurt that his touring schedule has been
b) At the International Electron Devices Meeting dreams are
reduced to shorter, intense bursts in recent years, largely
reached.
because of his shared-custody arrangement for his eight-
c) The International Electron Devices Meeting is the ideal
year-old, Beatrice. "We don't do the big sloggo tour, we
meeting to discuss new versions of wireless sensors.
don't do the big U2-Stones go-out-forever thing, and get a
d) Engineers at the International Electron Devices Meeting
bit fed up with it," says McCartney, who's planning some
dream about new inventions in the field of wireless sensors.
dates for later this year. "What we do now is events and
e) It will be possible to produce wireless sensors whose
selective dates. Because of my custody situation, I can only
batteries won’t need to be recharged.
do that. At first, we thought, 'Oh, God, is this going to be a
problem?' and it's actually turned out to be some kind of a
053 | IME 2011
blessing."
The expression ‘to say the least’ in the text suggests that:
He can see himself rocking well into his eighties. "I
a) the situation mentioned may be more than just
can imagine it," he says. "As to whether my imagination will
inconvenient.
come true, I don't know. The last couple of years, I've gotten
b) there is a list of technological researches that could solve
into guitar – so there's all sorts of little things that crop up
the case presented in the text.
that entice you forward, and you go, 'Hmm, I'd like that.'"
c) sensors used inside human body are inconvenient.
I broach the idea of actually dying onstage –
d) wireless sensors used within the steel and concrete
would he be into it? He recoils slightly, then smiles. "What
buildings are a reachable dream.
kind of question is that? I must say, that's not in my
e) engineers need a sensor that can last forever without
imagination. Rocking on until a grand old age... the only thing
external power sources or battery changes.
would be when it's not pleasant anymore, then it would be
'That's a good time to stop.' But it's way too pleasant at the
054 | JFS 2010
moment. And it pays. Good gig, man. But I know exactly
"Many OECD economies are in, or are on the verge of, a
where you're coming from, though. How long can this go
protracted recession of a magnitude not experienced since
on...?"
the early 1980s,'' OECD Chief Economist Klaus Schmidt-
Hebbel warned.
Adapted from http://www.rollingstone.com/
– Slightly altered
– The highlighted expression means the same as:
055 | JFS 2012
a) trabalha em um banco.
No 1º parágrafo do texto, o vocábulo then, utilizado na
b) é funcionário público.
construção then-manager, possui relação semântica com
c) é um trabalhador de “colarinho branco”.
qual das seguintes opções?
d) atua na área administrativa.
e) tem um trabalho enfadonho.
a) latter
b) former
Our Imaginary, Hotter Selves
c) so
Avatars might serve therapeutic purposes,
d) late
helping those with social phobia become more
e) still
confident.
056 | JFS 2012
By Sharon Begley
A expressão on the cards (2º parágrafo) é utilizada pelo ex-
Beatle Paul McCartney para indicar que ele:
Anyone who has ever had a bad hair day, when
looking like a latter-day Medusa makes you feel cranky and
a) não pensa em tirar férias.
antisocial and plodding, can sympathize with the Oakland
b) deseja tirar férias mais longas.
Raiders – and not because the players get helmet hair. The
c) considera a idéia de tirar mais férias.
Raiders alternated between mostly black and mostly white
d) sabe que precisa descansar mais.
uniforms, depending on whether they were playing at home
e) pensa em aproveitar mais feriados.
or away. Knowing that appearance affects people's mood
and outlook, psychologists wondered whether uniform color
057 | JFS 2012
influenced the Raiders' aggressiveness. Using data from the
De acordo com o texto:
1970s and 1980s, they found that the team racked up way
more penalty yards – a measure of aggression – when they
a) Paul McCartney não gosta do fato de estar prestes a
wore black than when they wore white, for infractions both
completar 70 anos.
minor (encroachment) and major (roughing the kicker). The
b) Faz 20 anos que Paul pensou pela última vez na hipótese
pattern held even when the scientists took into account
de se aposentar.
different conditions and styles of play at home and away.
c) A idéia de se aposentar e dedicar-se aos filhos não agrada
But while the 1988 finding has become a classic in
a McCartney.
psychology, the explanation remains controversial. Do
d) O ex-Beatle não teme ser superado pelos artistas mais
referees, because of black's cultural baggage, see black-clad
novos.
players as meaner and badder than those in, say, baby blue?
e) Apesar de não gostar do Foo Fighters, McCartney respeita
Or does wearing black make players see themselves as
o trabalho da banda.
tougher and meaner – and therefore cause them to play that
way?
Jeremy Bailenson and Nick Yee of Stanford
058 | JFS 2012 University had this and other classic studies in mind when
Consoante o texto, Paul McCartney: they started wondering about the effect of being able to
alter one's appearance. They weren't going to study
a) costuma analisar seu trabalho no palco como forma de se wardrobe choices, however. Their quarry is avatars, digital
autocriticar. representations of players in such games as Second Life.
b) ao comentar sobre não ter problemas com sua voz, "Your physical appearance changes how people treat you,"
demonstra-se uma pessoa supersticiosa. says Bailenson. "But independent of that, when you perceive
c) apesar de não demonstrar abertamente, lamenta o fato yourself in a certain way, you act differently." He and Yee
de não poder fazer turnês mais longas em virtude de sua call it "the Proteus effect," after the shape-changing Greek
filha mais nova. god. The effect of appearance on behavior, they find, carries
d) planeja continuar tocando mesmo quando estiver um over from the virtual world to the real one, with intriguing
roqueiro “oitentão”. consequences. (…)
e) tem medo de estender sua carreira por tanto tempo que
acabe por morrer durante uma apresentação. Adapted from http://www.newsweek.com.
Acesso em 5/6/2010.
059 | JFS 2012
He's a desk-bound pen pusher who dreams of trekking
through jungles.

– O homem descrito acima:


060 | ITA 2011
Cezanne Becomes Priciest Painting Ever
De acordo com o título e o subtítulo do texto, avatares:

a) proporcionam efeitos terapêuticos e ajudam a prevenir


doenças como a fobia social.
b) são versões imaginárias e mais atraentes de nós mesmos.
c) são mais confiáveis e, por isso, não despertam fobias.
d) têm uma proposta de entretenimento, que torna as
pessoas mais sociáveis.
e) são mais confiáveis do que algumas propostas
terapêuticas disponíveis em nossa sociedade.

061 | ITA 2011


Assinale a opção CORRETA. The Bridgeman Art Library / Getty Images

a) Os estudiosos da Universidade de Stanford não Not so fast, David Choe — you can no longer call
consideraram, em seus experimentos, a descoberta your Facebook murals the most expensive work of art,
realizada em 1988, cuja explicação ainda é controversa. because Qatar has come to the rescue. The oil-rich country
b) Psicólogos ainda questionam se, de fato, a aparência afeta has bought Paul Cezanne's painting The Card Players for
o humor e opinião das pessoas. more than $250 million, making it by far the highest price
c) Jeremy Bailenson e Nick Yee afirmam que a aparência ever paid for a work of art. Up until this point, the most
transforma o modo como as pessoas nos tratam e disso expensive painting ever sold was a Jackson Pollock for $140
depende a maneira como percebemos a nós mesmos. million in the frenzied pre-recession year of 2006, and in
d) A aparência física afeta o comportamento das pessoas e recent days the graffiti artist David Choe was reportedly
traz consequências para o mundo real e não apenas para o given Facebook stock options for decorating the company's
virtual. murals in 2005 that cost mere thousands then but will now
e) O foco dos estudiosos está no figurino dos avatares e no be worth some $200 million after the social-network giant
modo como isso afeta a agressividade dos jogadores. goes public. But Cezanne's small, quietly somber work beats
them all — there are four other Card Players in the world,
062 | ITA 2011 and they are at the Metropolitan Museum of Art in New
Assinale a opção em que o referente do pronome está York, the Musée d’Orsay in Paris, the Courtauld in London,
INCORRETO. and the Barnes Foundation in Philadelphia.

a) they (linha 10) em they found that – raiders Adapted from http://www.thedailybeast.com/
b) they (linha 11) em they wore black – raiders
c) those (linha 19) em than those in, say, baby blue? – 064 | JFS 2012
players De acordo com o texto:
d) them (linha 21) em cause them to play – players
e) Their (linha 27) em their quarry – Bailenson and Yee’s a) os quadros do artista David Choe eram considerados os
mais valiosos do mundo, até a venda de um Cézanne por
063 | ITA 2011 250 milhões de dólares para um magnata do Qatar.
A measure of aggression (linha 11), encroachment (linha b) o autor do texto, ao comentar que “o Qatar veio ao
13), roughing the kicker (linha 13) e digital representations resgate”, demonstra sua aversão às obras de David Choe.
of players (linhas 27/28) têm, respectivamente, valor c) antes do quadro The Card Players ser vendido por 250
semântico de: milhões de dólares, a obra mais cara da história era um
Pollock, pintado em 2006.
a) exemplificação – explicação – exemplificação – explicação d) o quadro The Card Players é tão apreciado pela
b) explicação – exemplificação – explicação – explicação comunidade artística que possui réplicas para exposição
c) explicação – exemplificação – exemplificação – permanente, espalhadas por museus nos Estados Unidos e
exemplificação na Europa.
d) explicação – exemplificação – exemplificação – explicação e) o artista David Choe foi pago pelos seus serviços ao
e) exemplificação – explicação – explicação – exemplificação Facebook em ações, hoje avaliadas em 200 milhões de
dólares.
065 | JFS 2012
068 | JFS 2012
Qual dos seguintes pares de adjetivos pode formar o grau
Qual dos seguintes verbos, retirados do texto, pode ser
superlativo da mesma forma que the most expensive e the
sinônimo de to stroll?
highest, respectivamente?
a) To take.
a) lavish – handsome b) To approach.
b) active – useful c) To wander.
c) famous – boring d) To zip.
d) tired – acid e) To explore.
e) eager – complex

A collaboration Google and 17 of the


world's top art galleries and museums, including the
National Gallery and Tate Britain in the UK, the Google Art
Project takes the Street View approach into the gallery.
With Google Art Project, users can wander around
17 of the world's top galleries and museums and view 1,061
artworks. There are also 17 special gigapixel images – one
for each participating institution's most treasured piece,
allowing viewers to zoom right in to brush-stroke levels of
detail.
Over the past 18 months, a Google team has been
zipping around the likes of the Rijksmuseum in Amsterdam
and the Palace of Versailles using trolley mounted cameras
to photograph corridors and galleries. Users can explore
each gallery from room to room or create their own
collections of masterpieces.

Adapted from http://www.creativereview.co.uk/

066 | JFS 2012


A lacuna presente no 1º parágrafo deve ser preenchida por:

a) between.
b) among. Adapted from http://cyberextazy.files.wordpress.com/
c) through.
d) over. 069 | JFS 2012
e) amidst. Analise as seguintes asserções:

067 | JFS 2012 I. O computador analisou o passado estudantil do rapaz sem


Sobre o texto, qual dos itens abaixo pode ser considerado curiosidade pessoal alguma.
correto? II. O gosto musical do rapaz é considerado de baixa
qualidade pelo computador.
a) A Google, a National Gallery e o Tate Britain estão à frente III. O computador utilizou-se de avançadas equações
de um projeto que visa divulgar, via internet, o acervo de 17 matemáticas para deduzir em que época o rapaz havia se
dos principais museus do mundo. formado.
b) O Google Art Project permite que o usuário veja fotos, de IV. O rapaz tem vergonha de certas bandas que costumava
extrema qualidade, de mais de mil obras-de-arte. ouvir nos anos 90.
c) Através do GAP, é possível ver detalhes de determinadas
obras, como as pinceladas de um quadro, por exemplo. – Agora, assinale a alternativa correta:
d) O GAP, que levou um ano e meio para ser concluído,
permite que o usuário visite os principais museus de cada a) Há apenas um item certo.
região do mundo. b) Há apenas dois itens certos.
e) As câmeras utilizadas pela equipe do GAP eram tão c) Todos os itens estão certos.
pesadas que precisaram ser carregadas por carrinhos. d) Todos os itens estão errados.
e) Dos itens ímpares, apenas um está certo.
THURSDAY, DECEMBER 16, 2010.
I’m not saying that the programs are not a
good idea. I imagine that they have a lot of benefits and
Newsweek Article: Bullying and Empathy
could especially help kids who would not necessarily be
(Kate Altman, M.S.)
bullies themselves, but may have quietly stood by while
witnessing bullying, to become more confident about
standing up to/reporting bullies. However, to truly reduce
bullying, society and schools need to find ways to identify
and work with aggressive children and their families from a
young age — to troubleshoot factors (from not having basic
needs met, to divorce, to models of aggression in the home,
etc.) that contribute to triggering aggressive behavior. Such
an approach would be expensive and time-consuming and
would command a lot of schools’ resources, but it is hard to
imagine a more lightweight approach being nearly as
effective.

070 | ITA 2012


Newsweek offers an article on how schools are Para estar adequada ao contexto em que aparece, a palavra
using empathy-training programs in an effort to reduce effective (linha 1), deve ser acrescida de:
bullying in schools: http://www.newsweek.com/2010/12
/15/can-schools-teach-kids-not-to-bully.html a) fully.
b) by.
The effective of such programs is c) ness.
unclear at this point, and experts are divided on whether it d) ful.
makes more sense to offer the programs to young children e) lessly.
(elementary school age) or older children (middle school
071 | ITA 2012
age) (both, is probably the answer). High school kids are
A opção que descreve a palavra empathy é:
simply difficult to reach logistically, since they all have
different schedules all day. Unsurprisingly, some experts a) essentially compassion.
have found that the most important component to empathy b) walking in another’s shoes.
training is to include the parents. c) an admirable trait.
In assessing these programs and the broader issues d) a band-aid on more systemic problems.
of empathy-training and bullying, there are multiple factors e) societal and educational issues.
to consider and no clear answers. First of all, empathy is one
of the most difficult and least-understood skills we can 072 | ITA 2012
develop – adults and kids alike. Empathy is the process of Segundo o texto:
viewing and understanding the world through another’s
experience, and it is often confused with sympathy, which a) é difícil incluir os alunos de ensino médio nos empathy
is, essentially, compassion and lacks the “walking in training programs, por serem mais velhos e demandarem
another’s shoes component” (which is not to say it is not uma abordagem diferenciada.
an admirable trait, it’s just different from empathy). b) o componente mais importante na empatia é a relação
Developmentally, children may not be able to truly familiar.
understand and practice empathy until they are closer to the c) simpatia está estritamente relacionada à empatia.
pre-teen years, but introducing the concept early and often d) é inútil trabalhar o conceito de empatia anteriormente à
is a good primer for its later development. pré-adolescência.
Another big question to consider: are programs e) há, provavelmente, fatores mais determinantes para a
focused on empathy simply band-aids on much larger, more prática de bullying do que a falta de empatia.
systemic problems? Why are kids bullying other kids in the
first place? What family issues, societal issues, educational 073 | JFS 2012
issues, are contributing to the need/urge to humiliate and “All right, everyone, we're not here to talk shop. Let's have a
attack other children for some sort of personal gain and good time.” We are not here to:
satisfaction? My guess is that for many kids, participating in
a brief (or even a few brief) empathy-skills seminars simply a) fight.
is not enough, and will not get at the root(s) of the b) make a scene.
problem(s), no matter how young they are when the c) waste time.
programs begin. d) talk about business.
e) listen to tall stories.
Australia's hybrid shark reveals evolution in action
074 | JFS 2012
By John Roach De acordo com o texto:

a) cientistas australianos obtiveram êxito em sua pesquisa


sobre o cruzamento de tubarões de espécies diferentes.
b) após o sucesso obtido com a mistura de diferentes
espécies de tubarões, os cientistas australianos esperam
conseguir repetir o experimento com outros tipos de
animais.
c) apesar de não estar diretamente envolvido na pesquisa
mencionada no texto, Bob Hueter teve um papel
fundamental para o sucesso dos estudos.
d) foi preciso a pesquisa com os tubarões híbridos ter sido
produzida no laboratório de Bob Hueter para que o estudo
obtivesse credibilidade junto à comunidade científica.
e) é possível que a hibridização dos tubarões tenha ocorrido
em virtude da interferência humana no ecossistema dos
Hybrid sharks have been discovered swimming in animais.
the waters off Australia's east coast. The finding may be
driven by climate change, a research team says, suggesting 075 | JFS 2012
such discoveries could be more common in the future. Qual das referências a seguir está incorreta:
The hybridization is between the Australian black
tip shark which favors tropical waters and the larger, a) the finding (3º parágrafo) refere-se a hybrid sharks.
common black tip shark, which favors sub-tropical and b) me (4º parágrafo) refere-se a John Roach.
temperate waters. c) his (5º parágrafo) refere-se a Bob Hueter.
While the distribution for the genetically distinct d) which (5º parágrafo) refere-se a his lab.
species overlaps along the northern and eastern Australian e) it (10º parágrafo) refere-se a the hybridization.
coastline, the finding that they mated and produced
offspring is unprecedented, according to the discovery team 076 | JFS 2012
from the University of Queensland. Assinale a opção em que a tradução do termo da coluna I
"To actually find something like this and prove it não corresponde ao termo da coluna II.
genetically is unprecedented," Bob Hueter, director of the
Center for Shark Research at the Mote Marine Laboratory in Coluna I Coluna II
Sarasota, Florida, told me Tuesday. a) driven (1º parágrafo) estimulada
Hueter was not involved with the research, though b) favors (2º parágrafo) prefere
one of the scientists responsible for the discovery used to c) overlaps (3º parágrafo) ultrapassa
work in his lab, which he said lends the finding credibility. d) offspring (3º parágrafo) filhotes
The finding is based on genetic testing and body e) though (5º parágrafo) embora
measurements and reported December 2011 in the journal
Conservation Genetics. 077 | JFS 2012
The team identified 57 of the hybrids from five Julgue os itens a seguir:
locations spanning 1,250 miles along the Australian coast.
I. A preposição along (3º parágrafo) pode ser substituída por
"Wild hybrids are usually hard to find, so detecting
between.
hybrids and their offspring is extraordinary," Jennifer
II. Os vocábulos measurements (5º parágrafo) e measures
Ovenden, an expert in genetics of fisheries species and team
são equivalentes.
member, said in a news release.
III. O vocábulo spanning (6º parágrafo) pode ser traduzido
The hybridization could be an adaptation to climate
change, the team noted, allowing the tropical Australian como “abrangendo”.
black tip shark to live in the cooler, sub-tropical waters. IV. O vocábulo news (7º parágrafo) equivale a press.
It could also be a technique to survive in over-fished
– Agora, marque a opção correta:
waters, speculated Hueter. As fisheries are depleted,
hybridization is a way to keep reproducing.
a) Há apenas um item correto.
"In a sense, it is catching evolution in action," he
b) Há apenas dois itens corretos.
told me.
c) Apenas os itens ímpares estão corretos.
Adapted from http://futureoftech.msnbc.msn.com/ d) Apenas o item I está incorreto.
e) Apenas o item IV está incorreto.
How warming is changing the wild kingdom
, she believes, it will be the relatively
small things that people do that will have the biggest impact:
By Ker Than
"Hummer sales, thank heaven, are dropping since gas prices
have gone up, and hybrid [car] sales have gone up. It's that
The planet is warming, humans are mostly to
type of stuff."
blame and plants and animals are going to dramatic lengths
to cope. That's the consensus of a number of recent studies Adapted from http://www.msnbc.msn.com/
that used wildlife to gauge the extent of global warming and – with small alterations
its effects.
While the topic of climate change is contentious – 078 | JFS 2012
including whether the planet is actually heating up – a Consoante o texto, a questão da mudança climática é um
growing number of documented shifts in traits and assunto:
behaviors in the wild kingdom is leading many scientists to
conclude the world is changing in unnatural ways. a) controverso.
Among the changes: Marmots end their b) polêmico.
hibernations about three weeks earlier now compared to 30 c) delicado.
years ago. Polar bears today are thinner and less healthy d) ultrapassado.
than those of 20 years ago. Many fish species are moving e) cansativo.
northward in search of cooler waters. A fruitfly gene
normally associated with hot, dry conditions has spread to 079 | JFS 2012
populations living in traditionally cooler southern regions. Os vocábulos em negrito no texto podem ser substituídos
Over the past century, Earth's average respectivamente por:
temperature has risen by about 1 degree Fahrenheit and a) endure – reckon – warning – terrific
many scientists believe greenhouse gases and carbon b) survive – rate – enforcing – gruesome
dioxide emissions from human activities are to blame. Left c) manage – figure out – menacing – serious
unattended, they warn, temperatures may rise by an d) thrive – estimate – scaring – grave
additional 2-10 degrees by the end of the century. In the e) deal – survey – urging – sudden
leading computer models, it follows that polar ice will melt
and seas would rise drastically, threatening coastal 080 | JFS 2012
communities around the globe. (…) A lacuna presente no último parágrafo do texto deve ser
Ecosystems and wildlife aren't the only things that corretamente preenchida por:
increasing temperatures will affect.
"Global warming is going to be a big stress to all a) Afterwards
animals, including Homo sapiens," said Terry Root, an b) Moreover
environmental science and policy professor at Stanford c) Although
University. d) At the end
A recent report issued by the Pew Center for Global e) In the end
Climate Change, a Virginia-based nonprofit organization,
warned that rising temperatures could exacerbate health 081 | JFS 2012
risks such as asthma for the elderly, the infirm and the poor, Após a leitura do texto, é correto afirmar que:
and especially for those in poor countries.
Even if all pollution were stopped today, the climate a) a quantidade de espécies de animais que precisaram
will warm at least another degree by the year 2100 and seas mudar de seu habitat natural em busca de condições
will rise 4 inches (11 centimeters), according to one recent melhores de vida é imensurável.
study. Another report says warming is unstoppable through b) existem muitos documentos que afirmam que as
the year 2400. Despite the dire warnings, many scientists mudanças comportamentais de certas espécies de animais
believe it may not be too late to reverse the trend. são sem precedentes.
The Pew report suggests creating transitional c) a cada cem anos, a temperatura média do planeta Terra
habitats that link natural areas as a way to help migrating aumenta em torno de 1%, e as regiões que mais podem
species. Also, alleviating other environmental stressors like sofrer as consequências desse aumento são as áreas
habitat destruction could help reduce their combined effects litorâneas.
with global warming. d) a raça humana também poderá sofrer as consequências
Root is encouraged by the fact that many cities are do aquecimento global, em especial os habitantes das áreas
following higher environmental standards, even if state and mais pobres do planeta.
national governments are dragging their feet. e) seria necessário eliminar a poluição imediatamente para
que a temperatura da Terra não aumentasse acima das
previsões nos próximos 100 anos.
From the Amazon to the Himalayas, ten of the
083 | JFS 2009
world’s greatest natural wonders face destruction if the
Complete as lacunas presentes no último parágrafo do texto
climate continues to warm at the current rate, warns WWF.
utilizando (ou omitindo) o artigo definido.
Released ahead of the International Panel on
Climate Change’s (IPCC’s) Second Working Group Report, a
a) the – the – * – the
WWF briefing – Saving the world's natural wonders from
b) the – * – * – the
climate change – reports on how the devastating impacts of
c) * – * – the – the
global warming are damaging some of the world’s greatest
d) * – the – * – *
natural wonders.
e) * – the – * – the
They include the: Amazon; Great Barrier Reef and
other coral reefs; Chihuahua Desert in Mexico and the US;
084 | JFS 2012
hawksbill turtles in the Caribbean; Valdivian temperate
No excerto “if the climate continues to warm at the current
rainforests in Chile; tigers and people in the Indian
rate, warns WWF”, extraído do 1º parágrafo, os vocábulos
Sundarbans; Upper Yangtze River in China; wild salmon in
warm e warn têm entre si uma relação de:
the Bering Sea; melting glaciers in the Himalayas; and East
African coastal forests.
a) sinonímia.
“While we continue to pressure governments to
b) antonímia.
make meaningful cuts in heat-trapping greenhouse gas
c) polissemia.
emissions, we are also working on adaptation strategies to
d) paronímia.
offer protection to some of the world’s natural wonders as
e) hiperonímia.
well as the livelihoods of the people who live there,” said Dr.
Lara Hansen, Chief Scientist of WWF’s Global Climate
085 | IME 2012
Change Programme.
When I qualified as a military engineer, the wise old colonel
“We are trying to buy people and nature time, as
who gave me my insignias said: “When you get to the front
actions to stop the root cause of climate change are taken.”
line, you will feel fear, but remember this: never fear the
Faced with water shortages along the Yangtze River,
enemy, never fear the danger, only fear letting down those
WWF is working in China with the government and local
who have gone before you.”
authorities to help communities best adapt to climate
change impacts. This includes developing a climate witness
– Which of the following comments could follow the
project in the Yangtze River basin so that people affected by
previous scene?
climate change can speak for themselves.
In the Valdivian forests of Chile and Argentina, the
a) And that is what leaders call loneliness: its first aspect is
global conservation organization is working with local
that it is something that one really feels in times of crisis,
partners to reduce forest fires and adjust conservation plans
when the sky is falling in around you and you are the only
to ensure that resistant forests – where 3,000-year-old trees
one who can make a decision as long as you are the first in
are found – can be protected.
command.
“From [I] turtles to [II] tigers, from [III] desert of
b) And that is what leaders call humor: it is one of the things
Chihuahua to [IV] great Amazon – all these wonders of
that I set great store by. A little bit of humor at a time of
nature are at risk from warming temperatures,” stressed Dr.
crisis often just lifts the mood and gets you focused on
Hansen. “While adaptation to changing climate can save
something else.
some, only drastic action by governments to reduce
c) And that is what leaders call heritage: the sense of not just
emissions can hope to stop their complete destruction.”
doing a job in the here and now, you belonging to something
that’s got a fantastic foundation, and you feel responsible for
Adapted from http://www.panda.org/
its future.
d) And that is what leaders call adaptability: it takes very
082 | JFS 2009
different qualities to command a platoon at war, which is a
Marque o título que melhor contempla o tema do texto.
tactical activity, and run a major change program in a large
organization, which has to do with a more strategic
a) Natural Wonders Feel the Heat
leadership.
b) How WWF Can Save the World
e) And that is what leaders call delegation: delegators have a
c) Is It Hot? It Will Even Get Hotter and Hotter
huge responsibility in terms of judging the capability and
d) How to Cope with the Global Warming?
competence of the people that they are giving the jobs to.
e) The Main Consequences of the Global Warming
Torrential rain broke briefly on January 23 rd 2010.
Phosphorus is a central component of the energy-
Nevertheless, 33 stalwarts of the Military Institute of
Engineering kept on gathering and reminiscing. carrying molecule in all cells (adenosine triphosphate) and
also the phospholipids that form all cell membranes.
086 | IME 2012 Arsenic, which is chemically similar to phosphorus, is
poisonous for most life on Earth. Arsenic disrupts metabolic
What can be understood by the passage?
pathways chemically it behaves similarly to
phosphate.
a) In spite of the tempest, the gathering took place as
"We know that some microbes can breathe arsenic,
planned.
b) The torrential rain expected happened to pour lightly. but what we've found is a microbe doing something new –
c) Due to torrential rain just 33 people showed up. building parts of itself out of arsenic," said Felisa Wolfe-
d) Heavy rains disturbed the meeting. Simon, a NASA Astrobiology Research Fellow in residence at
the U.S. Geological Survey in Menlo Park, Calif., and the
e) The purpose of the gathering was observing the rain while
research team's lead scientist. "If something here on Earth
telling of past experiences.
can do something so unexpected, what else can life do that
we haven't seen yet?"
NASA-Funded Research Discovers Life
Built with Toxic Chemical The newly discovered microbe, strain GFAJ-1, is a
member of a common group of bacteria, the
Gammaproteobacteria. In the laboratory, the researchers
successfully grew microbes from the lake on a diet that was
very lean on phosphorus, but included generous helpings of
arsenic. When researchers removed the phosphorus and
replaced it with arsenic the microbes continued to grow.
Subsequent analyses indicated that the arsenic was being
used to produce the building blocks of new GFAJ-1 cells.
The key issue the researchers investigated was
when the microbe was grown on arsenic did the arsenic
actually became incorporated into the organisms' vital
biochemical machinery, such as DNA, proteins and the cell
membranes. A variety of sophisticated laboratory
techniques was used to determine where the arsenic was
incorporated.
NASA-funded astrobiology research has changed The team chose to explore Mono Lake because of
the fundamental knowledge about what comprises all its unusual chemistry, especially its high salinity, high
known life on Earth. alkalinity, and high levels of arsenic. This chemistry is in part
Researchers conducting tests in the harsh a result of Mono Lake's isolation from its sources of fresh
environment of Mono Lake in California have discovered the water for 50 years.
first known microorganism on Earth able to thrive and The results of this study will inform ongoing
reproduce using the toxic chemical arsenic. The research in many areas, including the study of Earth's
microorganism substitutes arsenic for phosphorus in its cell evolution, organic chemistry, biogeochemical cycles, disease
components. mitigation and Earth system research. These findings also
"The definition of life has just expanded," said Ed will open up new frontiers in microbiology and other areas
Weiler, NASA's associate administrator for the Science of research.
Mission Directorate at the agency's Headquarters in "The idea of alternative biochemistries for life is
Washington. "As we pursue our efforts to seek signs of life in common in science fiction," said Carl Pilcher, director of the
the solar system, we have to think more broadly, more NASA Astrobiology Institute at the agency's Ames Research
diversely and consider life as we do not know it." Center in Moffett Field, Calif. "Until now a life form using
This finding of an alternative biochemistry makeup arsenic as a building block was only theoretical, but now we
will alter biology textbooks and expand the scope of the know such life exists in Mono Lake." (…)
search for life beyond Earth. The research is published in this
week's edition of Science Express. Adapted from http://www.nasa.gov/
Carbon, hydrogen, nitrogen, oxygen, phosphorus
and sulfur are the six basic building blocks of all known
forms of life on Earth. Phosphorus is part of the chemical
backbone of DNA and RNA, the structures that carry genetic
instructions for life, and is considered an essential element
for all living cells.
087 | JFS 2011
091 | JFS 2011
It can be inferred from the text that:
In the sentence “Researchers conducting tests in the harsh
a) a new form of life from outer space was discovered in environment of Mono Lake in California have discovered the
Mono Lake, California, by NASA researchers. first known microorganism on Earth able to thrive and
b) despite arsenic is poisonous, it is perfectly common some reproduce using the toxic chemical arsenic”, extracted from
living organisms using it to reproduce themselves. the 2nd paragraph, the words in bold can be replaced,
c) the new microorganism has stretched the definition about respectively, by:
what encompasses life on Earth.
d) the concept of life has changed widely after the discovery a) realizing – smooth – get on
of the new form of life. b) carrying out – strict – develop
e) in order to keep searching life in the solar system, c) attaining – sharp – rise
scientists must look for it first on Earth. d) leading – rough – wear out
e) handling – mild – grow
088 | JFS 2011
th 092 | JFS 2011
Which of the following options fills in the gap in the 6 e) Only items I and III are correct.
paragraph of the text?

a) because
b) despite
c) whether
d) hence
e) thus

089 | JFS 2011


Judge the following items:

I. The new microorganism is the only microbe on Earth that


is able to cope with arsenic.
II. It is possible that life can do things scientists have not
contemplated yet.
III. GFAJ-1, a breed of the Gammaproteobacteria, is capable
of incorporating the element arsenic in its proteins.

– Now, Mark the correct option:

a) Only item I is correct.


b) Only item II is correct.
c) Only item III is correct.
d) Only items I and II are correct.
e) Only items II and III are correct.

090 | JFS 2011


Judge the following items:

I. The scientists are still investigating the very moment


arsenic was incorporated by the microorganism.
II. Mono Lake’s salinity, alkalinity and arsenic levels
increased 50 years ago.
III. Not only microbiology but also other scientific fields shall
be benefited with the discovery.

– Now, Mark the correct option:

a) Only item I is correct.


b) Only item II is correct.
c) Only item III is correct.
d) Only items I and II are correct.
Which of the following words can be turned into the plural
form in the same way as bacteria?

a) Campus
b) Encyclopedia.
c) Criterion.
d) Radius.
e) Stadium.

093 | JFS 2011


In the sentence “Arsenic, which is chemically similar to
phosphorus, is poisonous for most life on Earth”,
extracted from the 6th paragraph, the Relative Pronoun
which:

a) is correct, but can be replaced by that.


b) is correct and cannot be replaced.
c) is correct, but can be replaced by what.
d) is incorrect and must be replaced by that.
e) is incorrect and must be replaced by what.

094 | JFS 2011


In the sentence “the researchers successfully grew
microbes from the lake on a diet that was very lean on
phosphorus”, also taken from the 8 th paragraph, the word
lean can be replaced by:

a) rich.
b) fat.
c) plump.
d) bare.
e) flat.

095 | JFS 2011


In the sentence “the arsenic was being used to produce
the building blocks of new GFAJ-1 cells”, extracted from
the 8th paragraph, the words in bold are, respectively:

a) adjective – adjective
b) verb – adjective
c) adjective – noun
d) verb – noun
e) adverb – adjective
Sticky Fingers
a) worked with the police to develop the use of fingerprints
to solve a murder.
By Olivia Judson b) found the fingerprints of a shopkeeper and his wife
In 1905, two brothers, Alfred murdered near London.
and Albert Stratton, were found guilty c) admitted that they had murdered a shopkeeper and his
of murdering a shopkeeper and his wife near London.
wife in Deptford, a town outside d) were the first people to be convicted of murder because
London. The evidence? A thumbprint of fingerprint evidence.
at the scene of the crime. The e) were the first people to use fingerprints to prove that a
brothers were hanged. murder had been committed.
The Stratton trial was the first time in Western
jurisprudence that fingerprint evidence had been presented 097 | FGV 2009
in a murder case. As such, it was a triumph for Charles Which of the following probably best explains the
Darwin's cousin Francis Galton. Galton had spent years importance of Francis Galton?
collecting fingerprints, studying and classifying their patterns
a) He was the first person to collect and use fingerprint
of loops, arches, and whorls. It was he who had not just
samples on a systematic basis.
speculated, but demonstrated that fingerprints are a reliable
b) He was the first person to show that each person’s
way of telling one person from another, and persuaded the
fingerprints are unique and therefore can be used to help
police that they could be used to solve crimes.
solve crimes.
Up to that point, fingerprints had been used not as
c) He was the first person to use fingerprints as a reliable
a means to identify criminals, but as a way for you to prove
means of identification.
that you were you and not someone else. The ancient
d) He was the person who found the thumbprint that the
Babylonians sometimes impressed fingerprints on clay
police used to convict Alfred and Albert Stratton of murder.
tablets that recorded business transactions, and centuries
e) In helping to investigate a murder near London, he proved
ago the Chinese made use of thumbprints on clay seals. In
that a thumbprint at the scene of the crime belonged to one
India in the nineteenth century, a fingerprint took the place
of the Stratton brothers.
of a signature for people who were illiterate and could not,
therefore, sign their names. The first use of fingerprints by 098 | FGV 2009
"officialdom" didn't come until the 1860s, when William According to the information in the article:
Herschel, a magistrate for the British colonial administration
in India, realized that fingerprints could be used as a means a) in India in the nineteenth century, an illiterate person
of identification when people came to collect their pensions. could give his fingerprint instead of his signature.
The person collecting the pension would give a print, which b) in the nineteenth century, India’s knowledge of
would be compared to a print on file; in that way, fingerprints was more advanced than China’s.
fingerprints could be used to prevent identity fraud. c) to facilitate commerce between their countries in the
In instituting this, Herschel made the assumption nineteenth century, the Indians and the Chinese used
that individuals have unique fingerprints; the fact that it was fingerprints to register cross-border business transactions.
actually so remained to be proved. That proof was provided d) the Indian intelligentsia in the nineteenth century used
by Galton, who demonstrated statistically that the odds of fingerprints instead of signatures to identify themselves.
two people having the same fingerprints are vanishingly e) for Indians in the nineteenth century, giving your
remote. He also – using prints sent to him by Herschel – fingerprint instead of signing your name was considered a
confirmed Herschel's observation that fingerprints do not disgrace.
change with age, a crucial feature if they were to be a
reliable form of identification. And Galton began to develop 099 | FGV 2009
a method for cataloging fingerprints, so that police could file According to the article, in the 1860s William Herschel:
fingerprints by type and quickly compare any two sets. (A
a) became chief magistrate for the British colonial pension
full-fledged cataloging system, based on Galton's, was
administration in India.
subsequently developed by Edward Richard Henry, who had
b) set up a fingerprint-identification system in colonial India
served as inspector general of police in Bengal; the finger
to prevent fraud when people received their retirement
print classification system came to be known as the "Henry
money.
System.") In short, Galton laid the groundwork for the police
c) decided that Indians must leave a signature when they
to begin to build a usable fingerprint database.
collected their pensions from the British colonial
administration.
096 | FGV 2009
d) restructured India’s colonial pension system.
According to the information in the article, Alfred and Albert
e) made it impossible for illiterate Indians to defraud the
Stratton:
British colonial administration in India.
100 | FGV 2009
a) there are two different verb tenses.
In paragraph 4, this in the phrase “In instituting this…” most b) there is no auxiliary verb.
likely refers to:
c) there are only regular verbs.
a) a pension system for British civil servants in colonial India. e) there are only irregular verb.
b) the use of fingerprints in identifying known criminals. d) there is only one verb tense.
c) are structuring of fingerprint-identification techniques.
d) fingerprint identification to prevent dishonesty in the Assessing the Afghan war: Guess what? We aren't winning
pension system.
e) a strengthening of pension laws in British colonial India.

101 | FGV 2009


According to the information in the article:

a) William Herschel believed that no fingerprint is the same


as any other fingerprint, but he never proved it.
b) William Herschel developed modern police fingerprinting
techniques.
c) William Herschel was unaware of the great importance of
fingerprints in solving crimes. So, America, remember the Vietnam War?
d) William Herschel’s work in India proved that each Because the war in Afghanistan just gave me a bad case of
fingerprint is unique. déjà vu.
e) William Herschel’s use of fingerprint identification On Wednesday, The Times' Ken Dilanian and
eliminated fraud in the British colonial administration in David S. Cloud reported:
India.
The U.S. intelligence community says in a secret
102 | FGV 2009 new assessment that the war in Afghanistan is mired in
Which of the following probably best describes the “crucial stalemate, and warns that security gains from an increase in
feature” mentioned in paragraph 4? American troops have been undercut by pervasive
corruption, incompetent governance and Taliban fighters
a) Fingerprints can be an important part of police operating from neighboring Pakistan, according to U.S.
investigations. officials.
b) Each person’s fingerprints are unique. Could someone please tell me how in the world
c) As the years pass, a person’s fingerprints remain the we've let this happen? Why is it that the best and the
same. brightest keep getting Americans killed for nothing?
d) It is possible to collect and catalog fingerprints.
The British got run out of Afghanistan in the 19th
e) Only identical twins can have identical fingerprints.
century. The Soviets got run out of Afghanistan in the 20th
century. Yet we've allowed ourselves to get stuck there?
103 | FGV 2009
And according to the classified National Intelligence
With respect to the information in the article, which of the
Estimate completed last month and cited by The Times’
following is not true about Francis Galton?
reporters, things aren't likely to improve: In a section looking
a) He was related to Charles Darwin. at future scenarios, the NIE also asserts that the Afghan
b) His fingerprint research took years. government in Kabul may not be able to survive as the U.S.
c) In his fingerprint work he received help from William steadily pulls out its troops and reduces military and civilian
Herschel. assistance.
d) He verified the existence of important fingerprint The costs? Cover your eyes: Some in Congress and
characteristics. the Obama administration are concerned that the bleak
e) His fingerprint cataloging system is an important part of assessment suggests little progress was made in the last
police work around the world. year. During that time, the U.S. has suffered more than 400
military fatalities and spent more than $100 billion. As of
104 | JFS 2010 Wednesday, 1,873 Americans had been killed in Afghanistan
President Barack Obama has approved a significant troop since U.S. forces invaded in late 2001, according to the
increase for Afghanistan, Pentagon officials said Tuesday. website icasualties.org.
The new troop deployment is expected to include 8,000 In 2001 and 2002, when the George W. Bush
Marines from Camp Lejeune, North Carolina, as well as administration launched the war in retaliation for the 9/11
4,000 additional Army troops from Fort Lewis, Washington. attacks, 51 U.S. soldiers died. Imagine if we'd had the good
sense to declare victory then and get out?
Instead, we've doubled down on a bad bet. The
107 | JFS 2012
result? Steadily rising casualties, with 499 killed in 2010 and
Assinale a opção que preenche corretamente a lacuna (II)
418 last year.
presente no 12º parágrafo do texto.
Now you might say that's not so bad, really,
compared with the Vietnam War, in (I) more than 58,000 a) during
Americans were killed. b) off
Sure. Try telling that to the families of the dead. c) by
Tell that to the family of Lance Cpl. Donald Hogan. d) of
The San Clemente native and Tesoro High School graduate is e) from
to be awarded the Navy Cross next week. What did he do? In
August 2009, while on patrol in Helmand province, he 108 | JFS 2012
spotted an explosive device and hurled himself into the body Assinale a opção correta.
of a fellow Marine to protect him and others (II) the blast.
This California hero died saving others. I'm sure his a) A sensação de déjà vu, mencionada pelo autor no 1º
family is proud. I'm also sure they'd rather have him home, parágrafo do texto, dá-se em virtude da guerra do
safe. Some will say that if we leave now, the sacrifices of Afeganistão desenrolar-se de maneira bastante semelhante
Hogan and others will have been in vain. But I say we can't à do Vietnã.
afford any more such tragic sacrifices for a lost cause. b) O 3º parágrafo, extraído da revista Time, apresenta um
We can't do much about Afghanistan now. parecer oficial sobre as causas e consequências da guerra do
President Obama says we'll be out by 2014. Good. Hopefully Afeganistão.
he sticks to that plan. And hopefully a Republican hawk c) É possível que com o início do processo de retirada das
doesn't become president. tropas do Afeganistão, o governo local não consiga
But we need to make sure there aren't any more minimizar os problemas provocados pela guerra naquele
Afghanistans. And to do that, everyone in this country needs país.
to have some skin in this game. d) O autor afirma que apesar do número de mortes na
We need a military draft. We need to make sure guerra do Vietnã ter sido muito maior ao da guerra do
that all of America's sons and daughters are subject to Afeganistão, esta causou um impacto mais significativo nas
combat duty. famílias norte-americanas que perderam seus filhos em
That way, the next time war fever hits, we'll be sure combate do que aquela.
that everyone has caught cold before we go into battle. e) Apesar de orgulhosa, a família do anspeçada Donald
It's time to put a stop to politicians doing the Hogan preferiria tê-lo vivo a tê-lo enterrado como herói de
deciding while only a few do the dying. guerra.

Adapted from http://opinion.latimes.com/ 109 | JFS 2012


As perguntas feitas pelo autor ao longo do texto:
105 | JFS 2012
Qual dos vocábulos a seguir, extraídos do texto, não pode a) indicam que ele possui muitas dúvidas sobre o desenrolar
ser considerado um falso cognato? dos eventos da guerra do Afeganistão.
b) levam o leitor a refletir sobre qual o desfecho mais
a) reported (2º parágrafo) provável para a guerra do Afeganistão.
b) assessment (3º parágrafo) c) foram utilizadas como recurso para levar o leitor a uma
c) officials (3º parágrafo) reflexão mais específica sobre determinados aspectos da
d) classified (6º parágrafo) guerra do Afeganistão.
e) casualties (9º parágrafo) d) representam as principais dúvidas da população
estadunidense, de um modo geral, acerca dos conflitos no
106 | JFS 2012 Afeganistão.
Assinale a opção que preenche corretamente a lacuna (I) e) induzem o leitor a uma interpretação mais ampla sobre o
presente no 10º parágrafo do texto. desenrolar da guerra do Afeganistão.

110 | JFS 2012


a) what
A sentença Yet we've allowed ourselves to get stuck there?,
b) which
c) that extraída do 5º parágrafo, apresenta uma idéia de:
d) whose a) causa.
e) where b) consequência.
c) condição.
d) contraste.
e) concessão.
Adapted from http://4.bp.blogspot.com/

111 | JFS 2012


Consoante a imagem:

a) o Presidente dos Estados Unidos, Barack Obama, dedicou


uma linha direta para que os soldados em combate
pudessem ter acesso a ele no caso de uma eventual
necessidade.
b) o fato de o Presidente Barack Obama ter visitado um país
normalmente tão neutro como a Dinamarca pode indicar
que ele está em busca de mais apoio para os projetos dos
Estados Unidos junto à comunidade externa.
c) a visita de Barack Obama à Dinamarca foi realizada com o Adapted from Time, July 11, 2011.
intuito de se conseguir reforços para a guerra do
Afeganistão. 112 | FUVEST 2012
d) os soldados aguardam instruções diretas da Casa Branca A pesquisa descrita no texto mostrou que a maioria dos
para saber quais os procedimentos que devem ser realizados norte-americanos:
em combate.
e) apesar de indisponível no momento da ligação, o a) está satisfeita com as respostas dos EUA aos ataques de
Presidente Obama tem o costume de comunicar-se 11 de setembro de 2001.
diretamente com os soldados norte-americanos em b) avalia a última década nos EUA de forma desfavorável.
combate. c) pede ao governo ações mais efetivas de combate ao
terrorismo.
JUST 10 YEARS INTO A NEW CENTURY, MORE THAN d) acredita que, desde os ataques de 11 de setembro de
TWO-thirds of the country sees the past decade as a period 2001, o governo conseguiu melhorar sua imagem.
of decline for the U.S., according to a new TIME/Aspen Ideas e) espera que o país supere, completamente, o trauma dos
Festival poll that probed Americans on the decade since the ataques de 11 de setembro de 2001.
tragic events of Sept. 11, 2001. Osama bin Laden is dead and
al-Qaeda seriously weakened, but the impact of the 9/11 113 | FUVEST 2012
attacks and the decisions that followed have, in the view of A sequência “most Americans think another terrorist attack
most Americans, put the U.S. in a tailspin that the country in the U.S. is likely” significa que, para a maioria dos norte-
has been unable to shake during two administrations and americanos, outro ataque terrorista nos EUA é:
almost 10 years of trying.
ACCORDING TO THE POLL, ONLY 6% OF MORE a) iminente.
THAN 2,000 Americans believe the country has completely b) muito temido.
recovered from the events of 9/11. Some of this pessimism c) impensável.
can be tied to fears of more terrorist attacks. Despite the d) provável.
death of bin Laden, most Americans think another terrorist e) uma incógnita.
attack in the U.S. is likely.
114 | FUVEST 2012
116 | FUVEST 2012
Com base nos gráficos que acompanham o texto, é correto
De acordo com o texto, o especialista Gary Bradski afirma
afirmar que, para os norte-americanos: que:
a) o evento de 11 de setembro de 2001 é mais significativo
a) a sua empresa projetou um robô com capacidade de
que outros eventos ocorridos na última década.
percepção.
b) a morte de Osama bin Laden reduz o receio de novos b) os robôs já estão bem mais desenvolvidos, atualmente.
ataques terroristas contra os EUA.
c) a construção de robôs que reproduzam capacidades
c) o governo de Obama é avaliado com pessimismo e biológicas é difícil.
descrédito, hoje.
d) as pessoas podem ser beneficiadas por robôs com
d) o risco de um ataque praticado por terroristas internos é
capacidade de planejamento.
maior que o de um ataque praticado por terroristas
e) a habilidade das pessoas em operar robôs sofisticados é
externos.
surpreendente.
e) a recessão econômica tem relação com os ataques e as
ameaças sofridos pelos EUA.
Gorillaz give away their new album made on an iPad
Although robots have made
great strides in manufacturing,
where tasks are repetitive, they are
still no match for humans, who can
grasp things and move about
effortlessly in the physical world.
Designing a robot to mimic
the basic capabilities of motion and
perception would be revolutionary,
researchers say, with applications
stretching from care for the elderly
to returning overseas manufacturing Gorillaz, the cartoon-styled supergroup renowned
operations to the United States for breaking new ground in the worlds of music and
(albeit with fewer workers). technology, have notched up another claim to fame.
Yet the challenges remain immense, far higher than The band that headlined this year's Glastonbury
artificial intelligence obstacles like speaking and hearing. “All festival today released The Fall, an album that can be
these problems where you want to duplicate something downloaded free and was largely produced using only an
biology does, such as perception, touch, planning or iPad. "I've never been someone who's embraced technology
grasping, turn out to be hard in fundamental ways,” said particularly," frontman Damon Albarn told a New Zealand TV
Gary Bradski, a vision specialist at Willow Garage, a robot station this month, shortly after announcing that Gorillaz
development company based in Silicon Valley. “It’s always would stop playing live in their current format. "I've always
surprising, because humans can do so much effortlessly.” tried to keep true to my roots, which was just a four-track
and a guitar, but I got given an iPad and I suddenly found
Adapted from http://www.nytimes.com, July 11, 2011. myself in a position where I could make quite a sonically
sophisticated record in my hotel room."
115 | FUVEST 2012 Albarn, who led the Britpop revolution with Blur,
Segundo o texto, um grande desafio da robótica é: has called The Fall a love letter to America. "I used to be very
baffled by this place, and I guess I still am in some ways. But
a) não desistir da criação de robôs que falem e entendam o right now, with all that's going on, this is a good place to be."
que ouvem. The album was made available to fans who
b) melhorar a capacidade dos robôs para a execução de opened the final door of a virtual Advent calendar on the
tarefas repetitivas. band's website. It can also be accessed as a stream on
c) não tentar igualar as habilidades dos robôs às dos seres Gorillaz.com, while a physical release of the record is
humanos. planned for the new year.
d) voltar a fabricar robôs que possam ser comercializados While the 42-year-old Albarn made use of 20
pela indústria norte-americana. applications on his new tablet device to create the album,
e) projetar um robô que imite as habilidades básicas de traditionalists will be reassured to learn that a handful of
movimento e percepção dos seres humanos. conventional instruments were also employed. Mick Jones
contributes guitar on one track, while Paul Simonon – his
erstwhile bandmate in the Clash, now a Gorillaz stalwart –
chips in with bass on another.
Gorillaz are not the first major band to release an
120 | JFS 2011
album free via the internet. Radiohead put out In Rainbows
Assinale a opção CORRETA.
as a digital download in 2007 employing an "honesty box"
scheme. But experts suggest Gorillaz are at the vanguard in a) Músicos tradicionalistas criticaram a maneira como o
recognising how the business model of popular music is novo álbum do Gorillaz foi produzido.
changing, with fans able to play games, become an exclusive b) A forma como o Gorillaz disponibilizou o download do
member of its fan club and buy limited-edition artwork all on álbum The Fall segue o mesmo modelo criado pelo
the website. (…) Radiohead.
c) As vendas do álbum In Rainbows foram muito melhores
Adapted from http://www.guardian.co.uk/
do que as do The Fall.
117 | JFS 2011 d) Mick Jones e Paul Simonon já haviam tocado juntos antes
O texto informa que Damon Albarn: em outra banda antes de participarem do Gorillaz.
e) Através do Gorillaz.com, o usuário que se tornar membro
a) foi de encontro às suas influências musicais ao produzir do fã clube da banda poderá divertir-se com jogos on-line e
The Fall em um iPad. comprar conteúdos exclusivos.
b) fez parte de uma banda britânica chamada Blur.
c) produziu um álbum inteiro do Gorillaz usando apenas um 121 | JFS 2011
iPad. Qual das sentenças a seguir, extraídas do texto, não se
d) apesar de ser britânico, dedicou o novo álbum do Gorillaz encontra na voz passiva?
aos Estados Unidos.
e) produziu o álbum The Fall no quarto do hotel onde estava a) …an album that can be downloaded free and was largely
hospedado. produced using only an iPad.
b) … but I got given an iPad and I suddenly found myself in a
118 | JFS 2011 position where I could make quite a sonically sophisticated
Analise as asserções a seguir: record in my hotel room.
c) I used to be very baffled by this place, and I guess I still am
I. O Gorillaz foi a atração principal do festival de Glastonbury in some ways.
de 2010. d) The album was made available to fans who opened the
II. O Gorillaz adotará um novo formato antes de voltar a se final door of a virtual Advent calendar on the band's
apresentar ao vivo. website.
III. O Gorillaz disponibilizou o álbum The Fall para download e) It can also be accessed as a stream on Gorillaz.com, while
gratuito no site oficial da banda. a physical release of the record is planned for the new year.

– Agora, assinale a opção certa: 122 | JFS 2011


Read the following lyrics.
a) apenas a I está correta.
b) apenas a II está correta. Our whole universe was in a hot, dense state
c) apenas a III está correta. Then nearly 14 billion years ago expansion started... Wait!
d) apenas a I e a II estão corretas. The Earth began to cool
e) apenas a I e a III estão corretas The autotrophs began to drool,
Neanderthals developed tools
119 | JFS 2011 We built the Wall
Analise as asserções a seguir: We built the pyramids
Math, Science, History, unraveling the mystery
I. No 1º parágrafo, notch up equivale semanticamente a
That all started with a big bang
achieve.
Bang!
II. No 5º parágrafo, erstwhile significa o mesmo que former.
III. Ainda no 5º parágrafo, stalwart pode ser substituído por – It is correct to say that:
partisan.
a) the Simple Past, predominantly used along the text, can
– Agora, assinale a opção certa:
be replaced by the Past Perfect without changing the
a) apenas a I está correta. context.
b) apenas a I e a II estão corretas. b) unraveling (line 8) and disclosing are interchangeable.
c) apenas a I e a III estão corretas. c) the verb to drool (line 4) means the same as to pour.
d) apenas a II e a III estão corretas. d) the relative pronoun that (line 9) can be substituted by
e) todas estão corretas. which.
e) the words whole (line 1) and all (line 9) are synonyms.
Blow your diet? Blame your brain
Study author Delores Albarracin, a professor of
Low-fat labels and encouraging exercise can backfire psychiatry at the university, suspects that the exercise
posters simply inspired the students to do something — and
By Linda Carroll | msnbc.com contributor
because food was available, eating became the thing to do.
What this means, she says, is that we need to be careful
about when and where we encourage people to work out.
We shouldn’t be showing ads touting the benefits of
exercise when people are sitting in front of the TV with a bag
of chips in their hands.
The study brings up the intriguing possibility that
these ads could be doing more harm than good if they’re not
targeted correctly, says Dr. Louis Aronne, clinical professor
of medicine and director of the Comprehensive Weight
Control Program at the New York-Presbyterian
Hospital/Weill Cornell Medical Center. (…)

Adapted from http://www.msnbc.msn.com/id/30702871/

123 | JFS 2010


The study described in the text shows that:

a) the more a person eats, the fatter he/she becomes.


b) workout ads can lead a person to eat more.
c) diet and exercise make the brain work harder.
Getty Images stock d) the most famous diets show no side effects.
e) no diet can make a person get slimmer in short term.
Ever make a resolution to go out and exercise and
end up grabbing a gooey chocolate cupcake instead? 124 | JFS 2011
No matter how good our intentions are, sometimes The college students who participated in the study:
it seems like our stomachs are out to sabotage us. Scientists
are now starting to understand why this happens. a) ate more raisins after analyzing the posters that promoted
As it turns out, the issue is often not insatiable exercise.
stomachs, but diet-undermining brain chemistry. At labs b) ate more raisins after analyzing the posters that
around the country, researchers are finding that our brains promoted goals such as joining a group.
behave in just the opposite way we would expect them to c) ate more raisins after analyzing the posters that promoted
when it comes to diet and exercise. togetherness.
Researchers recently discovered that public service d) stopped eating raisins after the results of the research
announcements exhorting the fat and flabby among us to were released.
get more exercise might have an unfortunate and e) stopped working out after the results of the research
unexpected side effect: They can inspire people to eat more, were released.
according to a study published in the journal Obesity.
To learn a little more about the impact of campaigns
designed to get couch potatoes moving, scientists from the “Life is something that happens
University of Illinois rounded up 53 college students and when you can't get to sleep.”
asked them to judge a series of posters promoting exercise. Fran Lebowitz
After they rated the exercise posters, the students were then
asked to evaluate some raisins. They were told they could
eat as many raisins as they needed to make the evaluation.
The researchers then ran the same experiment but
substituted posters that promoted goals such as joining a
group or togetherness for the exercise posters. Again the
students were asked to rate some raisins after scoring the
posters. The students scarfed down more raisins after
scrutinizing posters that promoted exercise than after
looking over the other set of posters.
3RD Part | Translations 8. I told my sister I'd lend her my new shirt if she let me
borrow her jacket, but she didn't take the bait.
Mark the correct translations to the words/expressions in a) oferta
bold. b) isca
c) engodo
1. The film tells anew the story of his rise to fame and d) negócio
power. e) empréstimo
a) novamente
b) parcialmente 9. Poverty begets hunger, and hunger begets crime.
c) tendenciosamente a) aumenta
d) aleatoriamente b) acentua
e) sem novidades c) piora
d) gera
2. In good condition, dolls from this period sell for £500 e) exacerba
apiece.
a) por cada pedaço 10. Stay a little longer, I beseech you!
b) um pedaço a) solicito
c) cada b) imploro
d) à vista c) peço
e) parcelado d) exijo
e) ordeno
3. We tried to persuade her not to resign, but to no avail.
a) sem chance 11. The George Cross is a decoration that is bestowed on
b) sem necessidade British civilians for acts of great bravery.
c) não foi fácil a) apreciada
d) em vão b) merecida
e) sem compromisso c) usada
d) vendida
4. He avowed that he regretted what he had done. e) concedida
a) negou
b) confessou 12. His theory has produced a blizzard of statistics on the
c) jurou global dimming phenomenon.
d) salientou a) grande quantidade
e) insinuou b) pequena quantidade
c) média
5. You can't help but stand in awe of powerful people. d) aumento
a) medo e) diminuição
b) espanto
c) pânico 13. The breadth of her knowledge is amazing.
d) respeito a) alcance
e) desprezo b) largura
c) extensão
6. There followed an awkward silence while we all tried to d) limite
think of something to say. e) fronteira
a) sepulcral
b) demorado 14. I've tried persuading her, but she won't budge.
c) estranho a) discutir
d) longo b) vacilar
e) embaraçoso c) mudar
d) conversar
7. They come from a privileged background. e) tentar
a) vizinhança
b) origem
c) formação
d) arredores
e) residência
15. All the rooms have built-in cupboards and wardrobes.
22. This is a daring new film by one of our most original
a) espaçosos
modern directors.
b) na medida certa
a) inteligente
c) reformados
b) astuto
d) embutidos
c) temerário
e) adaptados d) arriscado
e) audacioso
16. The men were caked in layers of filth and grime.
a) atolados
23. Stop dawdling! You'll be late for school!
b) sujos
a) perambular
c) enfurnados
b) dormir
d) soterrados
c) perder tempo
e) cobertos d) parar à toa
e) titubear
17. The caucus was held to decide which candidate the
party will support in the next election.
24. The race ended in a dead heat.
a) votação
a) acidente trágico
b) reunião
b) morte
c) pesquisa c) vitória acachapante
d) sufrágio d) empate
e) análise
e) tragédia
18. All my attempts to apologize with my friends were very 25. When the newspapers published the full story, all his
clumsy. earlier deceits were revealed.
a) desastrosas a) escândalos
b) bem sucedidas b) fraudes
c) bem elaboradas
c) segredos
d) aceitas
d) sonhos
e) recusadas e) ambições
19. It was said that the police concealed vital evidence 26. She is the new dean of the Faculty of Engineering.
during the investigations.
a) reitora
a) revelou
b) professora
b) descobriu
c) diretora
c) escondeu
d) coordenadora
d) investigou
e) supervisora
e) negligenciou
27. Anyone not paying the registration fee by 31 March will
20. They started washing up, so that was our cue to leave be deemed to have withdrawn from the scheme.
the party.
a) exigido
a) sinal b) cobrado
b) pista
c) considerado
c) sugestão
d) solicitado
d) idéia e) requisitado
e) afirmativa
28. The company is reconsidering the way in which it
21. She was a small, dainty child, unlike her sister who was
deploys its resources.
large and had big feet.
a) economize
a) magra
b) melhore
b) levada
c) aumente
c) delicada d) majore
d) gentil
e) aplique
e) agradável
29. A digest of the research findings is now available.
36. He was a very earnest young man.
a) coletânea
a) esforçado
b) gráfico
b) talentoso
c) relatório
c) sério
d) resumo
d) rico
e) versão e) forte
30. You shouldn't be so diffident about your achievements.
37. The whole country had tried to efface the memory of
a) hesitante
the old dictatorship.
b) orgulhoso
a) apagar
c) pedante
b) relembrar
d) presunçoso
c) homenagear
e) indeciso d) prestar continência
e) saudar
31. That bank is in dire straits.
a) processo de falência
38. He had the eerie feeling that he had met this stranger
b) situação terrível before.
c) situação muito boa
a) forte
d) moratória
b) engraçado
e) concordata c) estranho
d) sinistro
32. The normally dour Mr James was photographed smiling
e) intenso
and joking with friends.
a) presunçoso
39. He managed to eke out a living one summer by selling
b) arrogante drinks on a beach.
c) despretensioso
a) ficar rico
d) mal-humorado b) ganhar um bom dinheiro
e) sisudo c) fazer uma pequena fortuna
d) viver com pouco dinheiro
33. The community has dwindled to a tenth of its former e) abrir um empreendimento
size in the last two years.
a) dividiu-se 40. Naturally, I embroidered the tale a little to make it
b) dobrou more interesting.
c) aumentou a) enfeitei
d) diminuiu b) fantasiei
e) desmembrou-se c) melhorei
d) reduzi
34. They crowded round the spokesperson, eager for any e) maquiei
news.
a) eufóricos
41. The festival is to encompass everything from music,
b) ansiosos theatre and ballet to literature, cinema and the visual arts.
c) impacientes a) relacionar
d) irritados
b) misturar
e) indóceis c) combinar
d) enfatizar
35. Five billion dollars of this year's budget is already
e) abranger
earmarked for hospital improvements.
a) investido 42. He went to France as a United Nations special envoy.
b) arrecadado a) encarregado
c) gasto
b) responsável
d) reservado c) diretor
e) empreendido d) enviado
e) nomeado
43. He eschewed publicity and avoided nightclubs.
50. It's a competent enough piece of writing but it lacks
a) frequentava
flair.
b) ficava longe de
a) um bom enredo
c) adorava
b) suspense
d) odiava
c) romance
e) aproveitava-se da d) empolgação
e) talento
44. In this economically depressed area, evictions are
common.
51. He is always making flattering remarks about me and
a) calotes my projects.
b) empréstimos a) desagradáveis
c) turbulências b) lisonjeiras
d) crises c) importantes
e) despejos d) inteligentes
e) inesperadas
45. The dinner party conversation faltered for a moment.
a) intensificou-se 52. I think she just thought I was being flippant.
b) cessou a) elegante
c) esquentou b) exibido
d) perdeu o sentido c) desrespeitoso
e) fugiu do foco principal
d) agradável
e) brincalhão
46. He bid us both a fond farewell.
a) felicitação
53. She spent the day fretting about what she'd said to
b) saudação Nicky.
c) despedida a) preocupando-se
d) recepção b) lembrando
e) atendimento c) analisando
d) considerando
47. This is a far-fetched idea. e) ponderando
a) muito boa
b) horrível 54. A group of suntanned children were frolicking on the
c) mirabolante beach.
d) infalível a) nadando
e) inalcançável b) brincando
c) correndo
48. Hundreds of prisoners began a fast in protest about d) andando
prison conditions. e) descansando
a) jejum
b) corrida 55. She frowned at me, clearly annoyed due to my report
c) revolta card.
d) debate a) brigou comigo
e) rebelião b) gritou comigo
c) franziu a testa pra mim
49. She slept fitfully throughout the night and arose before d) chamou minha atenção
dawn.
e) sorriu para mim
a) muito bem
b) confortavelmente 56. The government continues to fudge the issue by
c) relaxadamente refusing to give exact figures.
d) irregularmente
a) evitando
e) aconchegadamente b) mentindo sobre
c) debatendo
d) adiando
e) amenizando
57. She started gabbling away at me in Spanish and I didn't
64. The aircraft's landing gear grazed the treetops as it
understand a word.
landed.
a) explicar
a) chocou-se contra
b) puxar conversa
b) desviou
c) gritar c) quase atingiu
d) xingar d) roçou
e) falar depressa
e) enrolou-se com
58. He bought a pair of garish Bermuda shorts.
65. They are in a financial gridlock due to high interest
a) belas rates.
b) caras
a) deficiência
c) baratas b) débito
d) espalhafatosas c) crise
e) floridas
d) impasse
e) colapso
59. I thought her outfit was ghastly.
a) fantástico 66. The men who arrived in the guise of drug dealers were
b) exclusivo actually undercover police officers.
c) importado a) procura
d) horrível b) disfarce
e) chamativo
c) desculpa
d) perseguição
60. Should I add a scarf to this jacket or would it be gilding e) investigação
the lily?
a) útil 67. There are any number of miracle cures on the market
b) exagero for people gullible enough to buy them.
c) quente demais a) ricas
d) adequado
b) necessitadas
e) inconveniente c) doentes
d) leigas
61. Detectives have given the flat a thorough going-over.
e) influenciáveis
a) prisão
b) apreensão 68. "If you must," came the gruff reply.
c) batida a) imediata
d) desarrumada
b) doce
e) averiguação c) áspera
d) autoritária
62. The management tried unsuccessfully to graft new e) infame
working methods onto the existing ways of doing things.
a) adotar
69. He eventually won the match after five gruelling sets.
b) inserir a) maçantes
c) adicionar
b) estonteantes
d) investir
c) emocionantes
e) criar d) exaustivos
e) longos
63. The children grappled for the ball.
a) lutaram 70. She's a real guzzler!
b) jogaram a) beberrona
c) optaram
b) fanfarrona
d) lançaram
c) comilona
e) furaram d) cafajeste
e) mau-caráter
71. He'd been drinking the night before and was looking a
78. In their heyday, they sold as many records as all the
bit haggard.
other groups in the country put together.
a) de ressaca
a) estréia
b) doente
b) auge
c) abatido c) separação
d) enjoado d) declínio
e) sonolento
e) início
72. He spoke quietly, in halting English.
79. Half these factories now stand idle.
a) hesitante
a) falidas
b) perfeito
b) prósperas
c) indefectível c) ociosas
d) formal d) em greve
e) informal
e) em expansão
73. What did you hanker after most when you were in
80. The worst of her criticism was reserved for journalists,
prison? photographers and others of their ilk.
a) receava
a) tipo
b) sentia falta
b) profissão
c) fazia c) cargo
d) ansiava
d) laia
e) evitava
e) área
74. There is a great deal of interest in harnessing wind and
81. Inasmuch as you are their commanding officer, you are
waves as new sources of power. responsible for the behaviour of these men.
a) promover
a) além de
b) enfatizar b) em vez de
c) ressaltar c) embora
d) aumentar
d) apesar de
e) fazer uso de
e) já que
75. My grandfather always says to me: “Make hay while 82. The inmates are in the cafeteria now.
the sun shines”.
a) novatos
a) Antes tarde do que nunca. b) detentos
b) Nunca deixe para o amanhã o que você pode fazer hoje. c) alunos
c) Deus ajuda a quem cedo madruga.
d) funcionários
d) Aproveite enquanto é tempo. e) inquilinos
e) A pressa é inimiga da perfeição.
83. You have to pay the installment today.
76. A few angry locals started heckling the speaker.
a) multa
a) interromper b) fiança
b) espancar c) pensão
c) agredir
d) prestação
d) maltratar e) indenização
e) interrogar
84. The office will be issuing permits on Tuesday and
77. Heedless destruction of the rainforests is contributing Thursday mornings.
to global warming.
a) publicando
a) enorme b) emitindo
b) avassaladora
c) divulgando
c) despretensiosa d) aplicando
d) precipitada e) restringindo
e) imprudente
85. Invoices must be submitted by the 24th of every
93. Different ethnic groups have different systems of
month.
kinship.
a) faturas
a) parentesco
b) memorandos
b) afinidade
c) atas c) liderança
d) solicitações d) governo
e) portarias
e) casamento
86. Perhaps some caviar can tempt your jaded palate.
94. There's a knack to using this corkscrew.
a) aguçado
a) talento
b) refinado
b) jeito
c) cansado
c) regra
d) exigente
d) lei
e) apurado
e) norma
87. He’s been living a jarring experience.
a) fascinante 95. Being an actor has a certain amount of kudos attached
to it.
b) desagradável
c) nova a) prestígio
d) inesperada b) contratempos
e) súbita c) transtornos
d) importância
88. I only said it in jest – you're obviously not fat. e) valor
a) sem pensar
b) de propósito 96. The food was nice enough but the service was rather
c) sem querer lackadaisical.
d) de brincadeira a) incivil
e) para irritar b) atabalhoado
c) indelicado
89. He works as a juggler. d) desastrado
a) percussionista e) displicente
b) dançarino
c) equilibrista 97. He is always lashed out by the reporters.
d) malabarista a) elogiado
e) trapezista b) entrevistado
c) atacado
90. He jutted his jaw out defiantly. d) citado
a) tremia e) enaltecido
b) projetava
c) roçava 98. He bought lead pipes.
d) mordia a) plástico
e) balançava b) borracha
c) cerâmica
91. She's a keen tennis player. d) chumbo
a) brilhante e) silicone
b) péssimo
c) regular 99. The business has liabilities of £2 million.
d) esforçada a) ativo
e) iniciante b) passivo
c) patrimônio
92. Her imagination was kindled by the exciting stories her d) investimentos
grandmother told her. e) lucro
a) envolvida
b) desestimulada
c) aguçada
d) anestesiada
e) invadida
100. That farm is his livelihood.
108. There's no point in sitting at home and moping.
a) principal patrimônio
a) chorando
b) herança
b) lamentando-se
c) meio de vida
c) reclamando
d) riqueza
d) enfurnado
e) legado e) escondendo-se
101. I'm loath to spend it all at once.
109. He always says mordant remarks.
a) relutante
a) sarcásticas
b) a favor
b) imprudentes
c) contra
c) desagradáveis
d) disposto a
d) inteligentes
e) decidido a
e) intrigantes
102. From an early age the brothers have loathed each
110. She managed to muster the courage to ask him to the
other.
cinema.
a) detestaram a) reunir
b) amaram
b) juntar
c) ajudaram
c) coletar
d) sustentaram d) fazer
e) ensinaram
e) criar
103. The motion of the car almost lulled her to sleep. 111. I got sick of her constant nagging.
a) levou a) reclamações
b) acalentou b) espirros
c) fez c) vaciladas
d) não deixou d) traições
e) tornou impraticável e) descuidos

104. My sweater got mangled in the washing machine. 112. After our busy day we both sat and nodded off in front
a) manchado of the TV.
b) desfigurado a) jantamos
c) mutilado b) relaxamos
d) lavado c) cochilamos
e) acabado d) conversamos
e) continuamos trabalhando
105. I like mellow flavours.
a) adocicados 113. Christine was completely nonplussed by his reply.
b) fortes a) confusa
c) suaves b) perplexa
d) apimentados c) ofendida
e) exóticos d) revoltada
e) humilhada
106. She seemed so very meek and mild.
a) submissa e conformada 114. Some of his colleagues say that he's obnoxious.
b) suave e frágil a) competente
c) pacífica e sutil b) preguiçoso
d) dócil e calma c) displicente
e) calma e perspicaz d) radical
e) insuportável
107. She has a mischievous sense of humour.
a) estranho 115. Fifty odd people came to the party.
b) maléfico a) estranhas
c) perspicaz b) desacompanhadas
d) malicioso c) penetras
e) sarcástico d) e poucas
e) estrangeiras
116. The engine had been making an ominous sound all the
123. She asked in a pleading tone of voice.
way from London.
a) arrogante
a) estranho
b) áspero
b) sinistro
c) rouco
c) ameaçador d) suplicante
d) perigoso e) desafiador
e) espantoso
124. I spilled the juice while I was pouring it.
117. Unions are fighting a plan by universities to outsource
a) bebendo
all non-academic services.
b) preparando
a) reduzir
c) adoçando
b) cortar d) esfriando
c) aumentar e) servindo
d) terceirizar
e) restringir 125. The idea is preposterous!
a) fantástica
118. Neither side in the conflict seems willing to make peace
b) absurda
overtures.
c) plausível
a) acordos d) aceitável
b) propostas e) ruim
c) convenções
d) concessões 126. I've always been prone to accidents.
e) comunicados a) propenso
b) medroso
119. Student grants these days are paltry.
c) disposto
a) insignificantes d) receoso
b) fartas e) preparado
c) abrangentes
d) extensas 127. "I don't approve of that kind of language," she said,
e) restritas pursing her lips.
a) lambendo
120. You have to treat your patrons well.
b) cerrando
a) patrões c) fechando
b) clientes d) mordendo
c) fornecedores
e) franzindo
d) garçons
e) funcionários 128. At the end of the match, the pitch was a real quagmire.
a) bagunça
121. It has been alleged that he received a payoff from an
b) pântano
arms dealer.
c) lamaçal
a) ameaça d) atoleiro
b) prazo e) arruaça
c) aviso
d) propina 129. His conviction was quashed in March 1986 after his
e) chantagem counsel argued that the police evidence was a tissue of lies.
a) aumentada
122. Let’s go for a pint. b) rejeitada
a) festa c) revogada
b) cigarro
d) negada
c) cerveja e) adiada
d) passeio
e) viagem
130. I started to feel queasy as soon as the boat left the
137. The team were rather ragged in the first half of the
harbour.
match, but improved in the second half.
a) mal
a) indisciplinado
b) com saudade
b) desatento
c) com medo c) irregular
d) enjoado d) violento
e) empolgado
e) distraído
131. Are you in the queue for tickets?
138. She's always raking up that old quarrel.
a) espera
a) chorando por causa de
b) ânsia
b) remoendo-se por conta de
c) fila c) desenterrando
d) procura d) reclamando de
e) expectativa
e) esbravejando devido a
132. We'll have to quicken the pace if we want to keep up
139. Rampant inflation means that our wage increases soon
with him. become worth nothing.
a) manter
a) grande
b) estabilizar
b) desenfreada
c) diminuir c) em decadência
d) ultrapassar
d) selvagem
e) acelerar
e) causticante
133. When asked earlier why he seemed to be so relaxed, 140. She has an excellent rapport with her staff.
Mr McCarthy quipped: "It's the drugs".
a) avaliação
a) gracejou
b) rendimento
b) afirmou c) desempenho
c) replicou
d) entrosamento
d) admitiu
e) receita
e) ratificou
141. She looked ravishing!
134. There's no point quibbling over a couple of dollars.
a) encantadora
a) cobrar
b) com raiva
b) não pagar c) cansada
c) pechinchar
d) indisposta
d) criar caso
e) faminta
e) cobrar juros
142. She was widely reckoned the best actress of her
135. Lennie's bottom lip quivered and tears started in his generation.
eyes.
a) estimada
a) franziu b) aclamada
b) tremeu c) proclamada
c) ressecou
d) considerada
d) umedeceu e) julgada
e) foi ferido
143. On her way to her interview she silently rehearsed
136. Even at the end, when cancer racked his body, he was
what she would say.
calm and cheerful.
a) arguia
a) torturava b) questionava
b) corroia
c) declarava
c) deteriorava d) declamava
d) acabava e) ensaiava
e) definhava
144. I was told the news first and then I relayed it to the
151. I baked a huge cake this morning, and the kids scoffed
others.
the lot.
a) escondi
a) confeitaram
b) omiti
b) jogaram fora
c) declarei c) deixaram cair
d) comuniquei d) desperdiçaram
e) repeti
e) devoraram
145. Don't let her rile you.
152. We would now like to broaden the scope of the enquiry
a) irritar and look at more general matters.
b) mentir para a) extensão
c) falar mal de b) influência
d) pensar bobagens sobre c) repercussão
e) enganar d) divulgação
e) sigilo
146. The speaker attempted to rouse the crowd with a cry
for action.
153. The poor dog – it's shivering!
a) acalmar a) com fome
b) apaziguar
b) com medo
c) incitar c) tremendo
d) ludibriar
d) doente
e) comover
e) com frio
147. He was the runner-up of the race.
154. The American space shuttle can be used many times to
a) campeão put payloads in space.
b) bi-campeão
a) nave espacial
c) desclassificado b) ônibus espacial
d) segundo colocado c) estação espacial
e) terceiro colocado
d) foguete espacial
e) satélite
148. Some people believe that to succeed in this world you
have to be ruthless.
155. I don't think Rick will ever manage to slake his lust for
a) ambicioso power.
b) impiedoso
a) saciar
c) desonesto b) diminuir
d) implacável
c) aumentar
e) insensível
d) estimular
e) conter
149. He nodded his head sagely.
a) sabiamente
156. You may sneer, but a lot of people like this kind of
b) negativamente
music.
c) positivamente a) ironizar
d) agitadamente
b) detestar
e) pacificamente
c) execrar
d) ser contra
150. I’m already sated.
e) injuriar-se
a) conformado
b) cansado 157. She soothed the crying baby.
c) extenuado a) perdeu a paciência com
d) saciado
b) agrediu
e) sobrecarregado
c) se irritou com
d) amamentou
e) acalmou
158. His reputation was permanently tainted by the financial
165. You need to trawl through a lot of data to get results
scandal.
that are valid.
a) agredida
a) pesquisar
b) atormentada
b) coletar
c) agitada c) reunir
d) manchada d) interligar
e) afligida
e) anexar
159. He gave me a good telling-off for forgetting the 166. Tuition fees will increase next year.
meeting.
a) crédito
a) conselho
b) moradia
b) lembrete
c) saúde
c) sugestão d) transporte
d) bronca
e) educação
e) ressalva
167. The Swedes are not alone in finding their language
160. Allow the meat to thaw properly before cooking it. under pressure from the ubiquitous spread of English.
a) ferver
a) amplo
b) esfriar
b) onipresente
c) aquecer c) avançado
d) derreter
d) desenfreado
e) degelar
e) exagerado
161. The mayor's political future has been hanging by a 168. She gave an unbiased opinion about the issue.
thread since the fraud scandal.
a) errada
a) está um caos
b) sincera
b) tornou-se obscuro c) tendenciosa
c) está por um fio
d) imparcial
d) foi decidido
e) confusa
e) acabou
169. Such a high increase will impose an undue burden on
162. His business thrived in the years before the war. the local tax payer.
a) faliu
a) moderado
b) prosperou b) aceitável
c) estagnou c) necessário
d) desandou
d) exagerado
e) minguou e) imprescindível
163. Several insurance companies are now touting their 170. His behaviour was unseemly.
wares on local radio.
a) impecável
a) anunciando b) indefectível
b) vendendo c) inadequado
c) demonstrando
d) irrepreensível
d) exibindo e) normal
e) lançando
171. The upshot of the discussions is that there will be no
164. Arrogance is a very unattractive personality trait. redundancies.
a) defeito a) votação
b) qualidade b) debate
c) traço
c) resultado
d) falta de educação d) proposta
e) modo e) alvitre
172. She sat through the whole meeting without uttering a
179. This film is x-rated.
word.
a) fantástico
a) proferir
b) obsceno
b) entender
c) violento
c) anotar d) agradável
d) comentar e) sem graça
e) explicar
180. This was once a Roman road in days of yore.
173. The vanquished army surrendered their weapons.
a) guerra
a) abalado
b) romance
b) derrotado
c) outrora
c) covarde d) peleja
d) inteiro e) fome
e) estremecido

174. She entered the vault with an armed guard.


“Aos colegas professores e estimados alunos de todo o Brasil
a) caixa-forte que utilizam esta insana compilação de questões:
b) sala
Este material é gratuito e sempre será.”
c) recinto Jefferson Celestino
d) prisão
e) ala

175. This issue looks likely to continue to vex the


government.
a) assombrar
b) perseguir
c) acuar
d) irritar
e) acalmar

176. The two older children tend to vie with the younger
one for their mother's attention.
a) chorar
b) agredir
c) unir-se
d) competir
e) atracar-se

177. As a medical examiner I can vouch from experience that


his death was accidental.
a) imaginar
b) supor
c) conjeturar
d) estimar
e) garantir

178. The women gathered around the coffin and began to


wail.
a) lamentar
b) rezar
c) cantar
d) acenar
e) aplaudir
Professor Jefferson Celestino da Costa

Answers
Adjectives and Adverbs
0 0 0 0 0 0 0 0 0 0
0 0 0 0 0 0 0 0 0 1
1 2 3 4 5 6 7 8 9 0
D C B E B A C E E D
0 0 0 0 0 0 0 0 0 0
1 1 1 1 1 1 1 1 1 2
1 2 3 4 5 6 7 8 9 0
A E A B B D C C A B
0 0 0 0 0 0 0 0 0 0
2 2 2 2 2 2 2 2 2 3
1 2 3 4 5 6 7 8 9 0
D E B D C D B D C C
0 0 0 0 0 0 0 0 0 0
3 3 3 3 3 3 3 3 3 4
1 2 3 4 5 6 7 8 9 0
A D D D C C A B E D
0 0 0 0 0 0 0 0 0 0
4 4 4 4 4 4 4 4 4 5
1 2 3 4 5 6 7 8 9 0
A D C A A E A B E C
0 0 0 0 0 0 0 0 0 0
5 5 5 5 5 5 5 5 5 6
1 2 3 4 5 6 7 8 9 0
D A A B B D B A B D
0 0 0 0 0 0 0 0 0 0
6 6 6 6 6 6 6 6 6 7
1 2 3 4 5 6 7 8 9 0
B E E E E C A A A D
0 0 0 0 0 0 0 0 0 0
7 7 7 7 7 7 7 7 7 8
1 2 3 4 5 6 7 8 9 0
D A A D C D A E B D
0 0 0 0 0 0 0 0 0 0
8 8 8 8 8 8 8 8 8 9
1 2 3 4 5 6 7 8 9 0
A C E E A C D C D A
0 0 0 0 0 0 0 0 0 1
9 9 9 9 9 9 9 9 9 0
1 2 3 4 5 6 7 8 9 0
E E D A B D B E B A
1 1 1 1 1 1 1 1 1 1
0 0 0 0 0 0 0 0 0 1
1 2 3 4 5 6 7 8 9 0
C E D D C D C C B A
1 1 1 1 1 1 1 1 1 1
1 1 1 1 1 1 1 1 1 2
1 2 3 4 5 6 7 8 9 0
B A E C E B E D C D
Pronouns
0 0 0 0 0 0 0 0 0 0
0 0 0 0 0 0 0 0 0 1
1 2 3 4 5 6 7 8 9 0
C B D D C D E E B E
Professor Jefferson Celestino da Costa
0 0 0 0 0 0 0 0 0 0
1 1 1 1 1 1 1 1 1 2
1 2 3 4 5 6 7 8 9 0
B B A C D C B C C A
0 0 0 0 0 0 0 0 0 0
2 2 2 2 2 2 2 2 2 3
1 2 3 4 5 6 7 8 9 0
B C E A E A D E A E
0 0 0 0 0 0 0 0 0 0
3 3 3 3 3 3 3 3 3 4
1 2 3 4 5 6 7 8 9 0
E C C A C C C A E A
0 0 0 0 0 0 0 0 0 0
4 4 4 4 4 4 4 4 4 5
1 2 3 4 5 6 7 8 9 0
E E C A E A C C B C
0 0 0 0 0 0 0 0 0 0
5 5 5 5 5 5 5 5 5 6
1 2 3 4 5 6 7 8 9 0
E D D C C A D D C B
0 0 0 0 0 0 0 0 0 0
6 6 6 6 6 6 6 6 6 7
1 2 3 4 5 6 7 8 9 0
B C C A D B A E A C
0 0 0 0 0 0 0 0 0 0
7 7 7 7 7 7 7 7 7 8
1 2 3 4 5 6 7 8 9 0
C D B C C D C E D B
0 0 0 0 0 0 0 0 0 0
8 8 8 8 8 8 8 8 8 9
1 2 3 4 5 6 7 8 9 0
E E C E B B E D D D
0 0 0 0 0 0 0 0 0 1
9 9 9 9 9 9 9 9 9 0
1 2 3 4 5 6 7 8 9 0
C D E E C E B A A B
1 1 1 1 1 1 1 1 1 1
0 0 0 0 0 0 0 0 0 1
1 2 3 4 5 6 7 8 9 0
C A C B D C E C A C
1 1 1 1 1 1 1 1 1 1
1 1 1 1 1 1 1 1 1 2
1 2 3 4 5 6 7 8 9 0
C D D A A B C C C C
1 1 1 1 1 1 1 1 1 1
2 2 2 2 2 2 2 2 2 3
1 2 3 4 5 6 7 8 9 0
C B A A A D D B C C
1 1 1 1 1 1 1 1 1 1
3 3 3 3 3 3 3 3 3 4
1 2 3 4 5 6 7 8 9 0
E A B B A B D D C A
1 1 1 1 1 1 1 1 1 1
4 4 4 4 4 4 4 4 4 5
1 2 3 4 5 6 7 8 9 0
B C D A D B D D E A
Quantifiers ad Intensifiers
0 0 0 0 0 0 0 0 0 0
0 0 0 0 0 0 0 0 0 1
1 2 3 4 5 6 7 8 9 0
A D A B C E B D A A
0 0 0 0 0 0 0 0 0 0
1 1 1 1 1 1 1 1 1 2
1 2 3 4 5 6 7 8 9 0
E D B C C A B A D E
0 0 0 0 0 0 0 0 0 0
2 2 2 2 2 2 2 2 2 3
1 2 3 4 5 6 7 8 9 0
A D B E D D A B B B
0 0 0 0 0 - - - - -
3 3 3 3 3
1 2 3 4 5
A B B E E - - - - -
Verbs
0 0 0 0 0 0 0 0 0 0
0 0 0 0 0 0 0 0 0 1
1 2 3 4 5 6 7 8 9 0
B E C B A C E D E E
0 0 0 0 0 0 0 0 0 0
1 1 1 1 1 1 1 1 1 2
1 2 3 4 5 6 7 8 9 0
D E D E A C B E A E
0 0 0 0 0 0 0 0 0 0
2 2 2 2 2 2 2 2 2 3
1 2 3 4 5 6 7 8 9 0
B E A A C A E D B A
0 0 0 0 0 0 0 0 0 0
3 3 3 3 3 3 3 3 3 4
1 2 3 4 5 6 7 8 9 0
B E A D A A B A D A
0 0 0 0 0 0 0 0 0 0
4 4 4 4 4 4 4 4 4 5
1 2 3 4 5 6 7 8 9 0
A E D A A B C C C E
1 1 1 1 1 1 1 1 1 1
5 5 5 5 5 5 5 5 5 6
1 2 3 4 5 6 7 8 9 0
E A E B E B E C B C
1 1 1 1 1 1 1 1 1 1
6 6 6 6 6 6 6 6 6 7
1 2 3 4 5 6 7 8 9 0
B E C D A B D D B E
1 1 1 1 1 1 1 1 1 1
7 7 7 7 7 7 7 7 7 8
1 2 3 4 5 6 7 8 9 0
A B A C E D B E A C
Modal Auxiliaries
0 0 0 0 0 0 0 0 0 0
0 0 0 0 0 0 0 0 0 1
1 2 3 4 5 6 7 8 9 0
C B B B D E C E B C
0 0 0 0 0 0 0 0 0 0
1 1 1 1 1 1 1 1 1 2
1 2 3 4 5 6 7 8 9 0
C B B D B B D C C A
0 0 0 0 0 0 0 0 0 0
2 2 2 2 2 2 2 2 2 3
1 2 3 4 5 6 7 8 9 0
D A D B A A A B B B
0 0 0 0 0 0 0 0 0 0
3 3 3 3 3 3 3 3 3 4
1 2 3 4 5 6 7 8 9 0
D C B C A D D D C C
0 0 0 0 0 0 0 0 0 0
4 4 4 4 4 4 4 4 4 5
1 2 3 4 5 6 7 8 9 0
A B E D E D A B A E
0 0 0 0 0 0 0 0 0 0
5 5 5 5 5 5 5 5 5 6
1 2 3 4 5 6 7 8 9 0
A B A B C E B C E C
0 0 0 0 0 0 0 0 0 0
6 6 6 6 6 6 6 6 6 7
1 2 3 4 5 6 7 8 9 0
C D A E A E E C C B
0 0 0 0 0 - - - - -
7 7 7 7 7
1 2 3 4 5
A D C C E - - - - -
Active and Passive Voice
0 0 0 0 0 0 0 0 0 0
0 0 0 0 0 0 0 0 0 1
1 2 3 4 5 6 7 8 9 0
A A C E E A C C A C
0 0 0 0 0 0 0 0 0 0
1 1 1 1 1 1 1 1 1 2
1 2 3 4 5 6 7 8 9 0
D C B A E E D A A E
0 0 0 0 0 0 0 0 0 0
2 2 2 2 2 2 2 2 2 3
1 2 3 4 5 6 7 8 9 0
E D B A D C D E A B
0 0 0 0 0 0 0 0 0 0
3 3 3 3 3 3 3 3 3 4
1 2 3 4 5 6 7 8 9 0
C E D A A C D B A C
0 0 0 0 0 0 0 0 0 0
4 4 4 4 4 4 4 4 4 5
1 2 3 4 5 6 7 8 9 0
B B B A C B E E E E
0 0 0 0 0 0 0 0 0 0
5 5 5 5 5 5 5 5 5 6
1 2 3 4 5 6 7 8 9 0
A B C B D A D B A B
0 0 0 0 0 0 0 0 0 0
6 6 6 6 6 6 6 6 6 7
1 2 3 4 5 6 7 8 9 0
B B B D D A A E D B
0 0 0 0 0 0 0 0 0 0
7 7 7 7 7 7 7 7 7 8
1 2 3 4 5 6 7 8 9 0
D D A D B B B C D A
0 0 0 0 0 0 0 0 0 0
8 8 8 8 8 8 8 8 8 9
1 2 3 4 5 6 7 8 9 0
B C B A A D C C C D
0 0 0 0 0 0 0 0 0 1
9 9 9 9 9 9 9 9 9 0
1 2 3 4 5 6 7 8 9 0
E D C A B A A E C D
Direct and Indirect Speech
0 0 0 0 0 0 0 0 0 0
0 0 0 0 0 0 0 0 0 1
1 2 3 4 5 6 7 8 9 0
C D D D A A D C A C
0 0 0 0 0 0 0 0 0 0
1 1 1 1 1 1 1 1 1 2
1 2 3 4 5 6 7 8 9 0
B A C D C D A E D A
0 0 0 0 0 0 0 0 0 0
2 2 2 2 2 2 2 2 2 3
1 2 3 4 5 6 7 8 9 0
D B B C C A E D A A
0 0 0 0 0 - - - - -
3 3 3 3 3
1 2 3 4 5
B E D D A - - - - -
Conditionals
0 0 0 0 0 0 0 0 0 0
0 0 0 0 0 0 0 0 0 1
1 2 3 4 5 6 7 8 9 0
A A C D D B A E A A
0 0 0 0 0 0 0 0 0 0
1 1 1 1 1 1 1 1 1 2
1 2 3 4 5 6 7 8 9 0
D D E E D A C C C E
0 0 0 0 0 0 0 0 0 0
2 2 2 2 2 2 2 2 2 3
1 2 3 4 5 6 7 8 9 0
E D C B E A D E E C
0 0 0 0 0 0 0 0 0 0
3 3 3 3 3 3 3 3 3 4
1 2 3 4 5 6 7 8 9 0
B C B C A B D C A C
0 0 0 0 - - - - - -
4 4 4 4
1 2 3 4
D D C D - - - - - -
Question Tags
0 0 0 0 0 0 0 0 0 0
0 0 0 0 0 0 0 0 0 1
1 2 3 4 5 6 7 8 9 0
D D B B A D A D B A
0 0 0 0 0 0 0 0 0 0
1 1 1 1 1 1 1 1 1 2
1 2 3 4 5 6 7 8 9 0
C A D B D B E B D C
Rejoinders
0 0 0 0 0 0 - - - -
0 0 0 0 0 0
1 2 3 4 5 6
E E A C A B - - - -
Articles
0 0 0 0 0 0 0 0 0 0
0 0 0 0 0 0 0 0 0 1
1 2 3 4 5 6 7 8 9 0
C E B E C B D D D B
0 0 0 0 0 0 0 0 0 0
1 1 1 1 1 1 1 1 1 2
1 2 3 4 5 6 7 8 9 0
E D B A C A D B D B
0 0 0 0 - - - - - -
2 2 2 2
1 2 3 4
E C D C - - - - - -
Plural of the Nouns
0 0 0 0 0 0 0 0 0 0
0 0 0 0 0 0 0 0 0 1
1 2 3 4 5 6 7 8 9 0
B D C C B A B C A D
0 0 0 0 0 0 0 0 0 0
1 1 1 1 1 1 1 1 1 2
1 2 3 4 5 6 7 8 9 0
A C D C D E E B E D
Genitive Case
0 0 0 0 0 0 0 0 0 0
0 0 0 0 0 0 0 0 0 1
1 2 3 4 5 6 7 8 9 0
D B B D C B E D A B
0 0 0 0 0 0 0 0 - -
1 1 1 1 1 1 1 1
1 2 3 4 5 6 7 8
A A C D B A B D - -
Numbers
0 0 0 0 0 0 0 0 - -
0 0 0 0 0 0 0 0
1 2 3 4 5 6 7 8
D E E D C B A D - -
Prepositions
0 0 0 0 0 0 0 0 0 0
0 0 0 0 0 0 0 0 0 1
1 2 3 4 5 6 7 8 9 0
D A E C B D D A D E
0 0 0 0 0 0 0 0 0 0
1 1 1 1 1 1 1 1 1 2
1 2 3 4 5 6 7 8 9 0
C E E B D D B E D C
0 0 0 0 0 0 0 0 0 0
2 2 2 2 2 2 2 2 2 3
1 2 3 4 5 6 7 8 9 0
D A E E B A E A D D
0 0 0 0 0 0 0 0 0 0
3 3 3 3 3 3 3 3 3 4
1 2 3 4 5 6 7 8 9 0
B A C D B E A D E B
0 0 0 0 0 0 0 0 0 0
4 4 4 4 4 4 4 4 4 5
1 2 3 4 5 6 7 8 9 0
B E D E D D A E A D
0 0 0 0 0 0 0 0 0 0
5 5 5 5 5 5 5 5 5 6
1 2 3 4 5 6 7 8 9 0
A E A C B B B B E D
0 0 0 0 0 0 0 0 0 0
6 6 6 6 6 6 6 6 6 7
1 2 3 4 5 6 7 8 9 0
A C E A B B D E A D
0 0 0 0 0 0 0 0 0 0
7 7 7 7 7 7 7 7 7 8
1 2 3 4 5 6 7 8 9 0
B C A B B E C D B D
0 0 0 0 0 0 0 0 0 0
8 8 8 8 8 8 8 8 8 9
1 2 3 4 5 6 7 8 9 0
B E D C B B E E A D
Conjunctions
0 0 0 0 0 0 0 0 0 0
0 0 0 0 0 0 0 0 0 1
1 2 3 4 5 6 7 8 9 0
C C E A E B A E C E
0 0 0 0 0 0 0 0 0 0
1 1 1 1 1 1 1 1 1 2
1 2 3 4 5 6 7 8 9 0
A D E A A B E B C A
0 0 0 0 0 0 0 0 0 0
2 2 2 2 2 2 2 2 2 3
1 2 3 4 5 6 7 8 9 0
E E D B C E D C E D
0 0 0 0 0 0 0 0 0 0
3 3 3 3 3 3 3 3 3 4
1 2 3 4 5 6 7 8 9 0
B E D C E E C A A D
0 0 0 0 0 0 0 0 0 0
4 4 4 4 4 4 4 4 4 5
1 2 3 4 5 6 7 8 9 0
B B D C D D E D D C
0 0 0 0 0 0 0 0 0 0
5 5 5 5 5 5 5 5 5 6
1 2 3 4 5 6 7 8 9 0
C C A A B E A B B C
0 0 0 0 0 0 0 0 0 0
6 6 6 6 6 6 6 6 6 7
1 2 3 4 5 6 7 8 9 0
D A E C B D E C E B
0 0 0 0 0 0 0 0 0 0
7 7 7 7 7 7 7 7 7 8
1 2 3 4 5 6 7 8 9 0
A C E D A B B D D C
0 0 0 0 0 0 0 0 0 0
8 8 8 8 8 8 8 8 8 9
1 2 3 4 5 6 7 8 9 0
D D B D E C C E E D
0 0 0 0 0 0 0 0 0 1
9 9 9 9 9 9 9 9 9 0
1 2 3 4 5 6 7 8 9 0
C D D E D E B E A B
1 1 1 1 1 1 1 1 1 1
0 0 0 0 0 0 0 0 0 1
1 2 3 4 5 6 7 8 9 0
E D C A C C B B D E
1 1 1 1 1 1 1 1 1 1
1 1 1 1 1 1 1 1 1 2
1 2 3 4 5 6 7 8 9 0
C C B E E B B A B C
1 1 1 1 1 - - - - -
2 2 2 2 2
1 2 3 4 5
C D D A B - - - - -
Subjunctive, Imperative, Infinitive and Gerund
0 0 0 0 0 0 0 0 0 0
0 0 0 0 0 0 0 0 0 1
1 2 3 4 5 6 7 8 9 0
D D B E D D D C C A
0 0 0 0 0 0 0 0 0 0
1 1 1 1 1 1 1 1 1 2
1 2 3 4 5 6 7 8 9 0
C A E C C D A D D C
Phrasal Verbs
0 0 0 0 0 0 0 0 0 0
0 0 0 0 0 0 0 0 0 1
1 2 3 4 5 6 7 8 9 0
E D D E A A B C D A
0 0 0 0 0 0 0 0 - -
1 1 1 1 1 1 1 1
1 2 3 4 5 6 7 8
A A C B A B E A - -
False Cognate Words
0 0 0 0 0 0 0 0 0 0
0 0 0 0 0 0 0 0 0 1
1 2 3 4 5 6 7 8 9 0
B A B A D C D E C C
0 0 0 0 0 0 0 0 - -
1 1 1 1 1 1 1 1
1 2 3 4 5 6 7 8
B C C B A C A B - -
Mixed Topics
0 0 0 0 0 0 0 0 0 0
0 0 0 0 0 0 0 0 0 1
1 2 3 4 5 6 7 8 9 0
B D C A E D A C B C
0 0 0 0 0 0 0 0 0 0
1 1 1 1 1 1 1 1 1 2
1 2 3 4 5 6 7 8 9 0
A E D C C A A A E C
0 0 0 0 0 - - - - -
2 2 2 2 2
1 2 3 4 5
A B E C D - - - - -
Idioms and Vocabulary
0 0 0 0 0 0 0 0 0 0
0 0 0 0 0 0 0 0 0 1
1 2 3 4 5 6 7 8 9 0
A D C D D A C A A C
0 0 0 0 0 0 0 0 0 0
1 1 1 1 1 1 1 1 1 2
1 2 3 4 5 6 7 8 9 0
B D E E E A B C E C
0 0 0 0 0 0 0 0 0 0
2 2 2 2 2 2 2 2 2 3
1 2 3 4 5 6 7 8 9 0
D B A B B A B D A D
Synonyms and Antonyms
0 0 0 0 0 0 0 0 0 0
0 0 0 0 0 0 0 0 0 1
1 2 3 4 5 6 7 8 9 0
D A B D B B E E E C
0 0 0 0 0 0 0 0 0 0
1 1 1 1 1 1 1 1 1 2
1 2 3 4 5 6 7 8 9 0
C C A B D C D C A D
0 0 0 0 0 0 0 0 0 0
2 2 2 2 2 2 2 2 2 3
1 2 3 4 5 6 7 8 9 0
C D D A A B C D D C
0 0 0 0 0 - - - - -
3 3 3 3 3
1 2 3 4 5
C D D B D - - - - -
Reading Skills and General Review
0 0 0 0 0 0 0 0 0 0
0 0 0 0 0 0 0 0 0 1
1 2 3 4 5 6 7 8 9 0
E D C D E B C B B A
0 0 0 0 0 0 0 0 0 0
1 1 1 1 1 1 1 1 1 2
1 2 3 4 5 6 7 8 9 0
B E D A B A E A C E
0 0 0 0 0 0 0 0 0 0
2 2 2 2 2 2 2 2 2 3
1 2 3 4 5 6 7 8 9 0
B D C C D E B B B C
0 0 0 0 0 0 0 0 0 0
3 3 3 3 3 3 3 3 3 4
1 2 3 4 5 6 7 8 9 0
D C B C B B D E A B
0 0 0 0 0 0 0 0 0 0
4 4 4 4 4 4 4 4 4 5
1 2 3 4 5 6 7 8 9 0
A C D E A C B E B D
0 0 0 0 0 0 0 0 0 0
5 5 5 5 5 5 5 5 5 6
1 2 3 4 5 6 7 8 9 0
C C C C B C D B E B
0 0 0 0 0 0 0 0 0 0
6 6 6 6 6 6 6 6 6 7
1 2 3 4 5 6 7 8 9 0
D A D E A A C C A C
0 0 0 0 0 0 0 0 0 0
7 7 7 7 7 7 7 7 7 8
1 2 3 4 5 6 7 8 9 0
B E D E E C B A C E
0 0 0 0 0 0 0 0 0 0
8 8 8 8 8 8 8 8 8 9
1 2 3 4 5 6 7 8 9 0
D A C D C A C A E C
0 0 0 0 0 0 0 0 0 1
9 9 9 9 9 9 9 9 9 0
1 2 3 4 5 6 7 8 9 0
B C B D A D B A B D
1 1 1 1 1 1 1 1 1 1
0 0 0 0 0 0 0 0 0 1
1 2 3 4 5 6 7 8 9 0
A C E A A B E E C D
1 1 1 1 1 1 1 1 1 1
1 1 1 1 1 1 1 1 1 2
1 2 3 4 5 6 7 8 9 0
B B D A E C B A B D
1 1 1 1 - - - - - -
2 2 2 2
1 2 3 4
C B B A - - - - - -
Translations
0 0 0 0 0 0 0 0 0 0
0 0 0 0 0 0 0 0 0 1
1 2 3 4 5 6 7 8 9 0
A C D B B E C A D B
0 0 0 0 0 0 0 0 0 0
1 1 1 1 1 1 1 1 1 2
1 2 3 4 5 6 7 8 9 0
E A C C D E B A C A
0 0 0 0 0 0 0 0 0 0
2 2 2 2 2 2 2 2 2 3
1 2 3 4 5 6 7 8 9 0
C E C D B A C E D A
0 0 0 0 0 0 0 0 0 0
3 3 3 3 3 3 3 3 3 4
1 2 3 4 5 6 7 8 9 0
B E D B D C A C D A
0 0 0 0 0 0 0 0 0 0
4 4 4 4 4 4 4 4 4 5
1 2 3 4 5 6 7 8 9 0
E D B E B C C A D E
0 0 0 0 0 0 0 0 0 0
5 5 5 5 5 5 5 5 5 6
1 2 3 4 5 6 7 8 9 0
B C A B C A E D D B
0 0 0 0 0 0 0 0 0 0
6 6 6 6 6 6 6 6 6 7
1 2 3 4 5 6 7 8 9 0
E C A D D B E C D A
0 0 0 0 0 0 0 0 0 0
7 7 7 7 7 7 7 7 7 8
1 2 3 4 5 6 7 8 9 0
C A D E D A E B C A
0 0 0 0 0 0 0 0 0 0
8 8 8 8 8 8 8 8 8 9
1 2 3 4 5 6 7 8 9 0
E B D B A C B D D B
0 0 0 0 0 0 0 0 0 1
9 9 9 9 9 9 9 9 9 0
1 2 3 4 5 6 7 8 9 0
A C A B A E C D B C
1 1 1 1 1 1 1 1 1 1
0 0 0 0 0 0 0 0 0 1
1 2 3 4 5 6 7 8 9 0
A A B E C D D B A E
1 1 1 1 1 1 1 1 1 1
1 1 1 1 1 1 1 1 1 2
1 2 3 4 5 6 7 8 9 0
A C B E D A D B A B
1 1 1 1 1 1 1 1 1 1
2 2 2 2 2 2 2 2 2 3
1 2 3 4 5 6 7 8 9 0
D C D E B A E C C C
1 1 1 1 1 1 1 1 1 1
3 3 3 3 3 3 3 3 3 4
1 2 3 4 5 6 7 8 9 0
C E A D B A C C B D
1 1 1 1 1 1 1 1 1 1
4 4 4 4 4 4 4 4 4 5
1 2 3 4 5 6 7 8 9 0
A D E D A C D B A D
1 1 1 1 1 1 1 1 1 1
5 5 5 5 5 5 5 5 5 6
1 2 3 4 5 6 7 8 9 0
E A C B A A E D D E
1 1 1 1 1 1 1 1 1 1
6 6 6 6 6 6 6 6 6 7
1 2 3 4 5 6 7 8 9 0
C B A C A E B D D C
1 1 1 1 1 1 1 1 1 1
7 7 7 7 7 7 7 7 7 8
1 2 3 4 5 6 7 8 9 0
C A B A D D E A B C
Did you find any error? Send me a feedback, please!
Professor Jefferson Celestino da Costa | Histórico Profissional

Belém-Pa

1996 a 1998 Sociedade Civil Integrada MADRE CELESTE (Sede Icoaraci)


1998 a 2004 Colégio TITULAR – Sistema ELITE de Ensino | Preparatório IME-ITA 2000
a 2002 Grupo Educacional IDEAL (Sede Batista Campos) | Preparatório IME-ITA
2000 a 2004 Centro de Ensino Pleno IDEAL (Sede São Francisco)
2000 Colégio Marista NOSSA SENHORA DE NAZARÉ
2001 a 2002 EFOMM – Escola de Formação de Oficiais da Marinha Mercante/CIABA – Centro de Instrução Almirante Braz de Aguiar
2003 a 2004 Grupo Educacional OLIMPUS (Sede Alcindo Cacela)

Fortaleza-Ce

2005 a 2009 Colégio ZÊNITE


2005 a 2011 CEM – Curso para Escolas Militares | Preparatório AFA-EFOMM
2008 a 2011 Colégio DÁULIA BRINGEL
2010 a 2011 Colégio MAXIMUS
2011 Colégio SANTA CECÍLIA

Desde 2005 Colégio MASTER (Sede Bezerra de Menezes) | Preparatório IME-ITA


Desde 2006 Colégio ARI DE SÁ CAVALCANTE (Sedes Hildete de Sá Cavalcante, Duque de Caxias e Aldeota) | Preparatório IME-ITA
Desde 2006 SAS – Sistema Ari de Sá (Professor-autor do Material Didático de Língua Inglesa)
Desde 2008 Colégio ANTARES (Sedes Jardins, 6 Bocas, Praia de Iracema e Fátima) | Preparatório IME-ITA
Desde 2009 Organização Educacional X DA QUESTÃO | Preparatório AFA-EFOMM
Desde 2010 Curso DIPLOMATA | Preparatório IRBr
Desde 2012 Colégio 7 DE SETEMBRO (Sedes Diplomata Ednildo Gomes de Soárez e Edilson Brasil Soárez | Preparatório IME-ITA

Você também pode gostar